Sunteți pe pagina 1din 99

Question

Number Unit Label


1 Statics

Choice A
Charge

Choice B
Energy
It is uniformly
It is concentrated distributed
at the center of the throughout the
sphere.
sphere.

Choice C
Impulse
Its density
decreases radially
outward from the
center.

5 Statics

A parallelplate capacitor is charged by connection to a battery. If the battery


is disconnected and the separation between the plates is increased, what will
happen to the charge on the capacitor and the voltage across it?
E
A point P is 0.50 meter from a point charge of 5.0 10^(8) coulomb. The
intensity of the electric field at point P is most nearly:
D
A point P is 0.50 meter from a point charge of 5.0 10^(8) coulomb. The
electric potential at point P is most nearly:
C

Both remain fixed. Both increase


2.5 10^1
2.5 10^(8) N/C N/C
2.5 10^1
2.5 10^(8) N/C N/C

Both decrease
9.0 10^2
N/C
9.0 10^2
N/C

6 Statics

One joule of work is needed to move one coulomb of charge from one point
to another with no change in
velocity. Which of the following is true between the two points?

The resistance is
one ohm.

The potential
The current is one difference is one
ampere.
volt.

The charge
increases and the
voltage decreases.
1.8 10^3
N/C
1.8 10^3
N/C
The electric field
strength is one
newton per
coulomb.

7 Statics

Two positive charges of magnitude q are each a distance d from the origin A
of a coordinate system as shown
above. At which of the following points is the electric field least in magnitude? A

8 Statics

Two positive charges of magnitude q are each a distance d from the origin A
of a coordinate system as shown
above. At which of the following points is the electric potential greatest in
magnitude?

9 Statics

A parallelplate capacitor has a capacitance C. A second parallelplate


capacitor has plates with twice the area and twice the separation. The
capacitance of the second capacitor is most nearly:

2C

4C

10 Statics

Two identical conducting spheres are charged to +2Q and Q. respectively,


and are separated by a distance d
(much greater than the radii of the spheres) as shown above. The magnitude
of the force of attraction on the left
sphere is F1. After the two spheres are made to touch and then are
reseparated by distance d, the magnitude of the force on the left sphere is F2.
Which of the following relationships is correct?
E

2F1=F2

F1=F2

F1=2F2

11 Statics

The distance
The charge on
between the plates each plate

An electron volt is a measure of


An electron is accelerated from rest for a time of 10^-9 second by a uniform
electric field that exerts a force of
8.0 x 1015 newton on the electron. What is the magnitude of the electric
field?
An electron is accelerated from rest for a time of 10^-9 second by a uniform
electric field that exerts a force of
8.0 x 1015 newton on the electron. The speed of the electron after it has
accelerated for the 10^9 second is most nearly

energy

electric field

The area of the


plates
electric potential
due to one
electron

F1=4F2
The potential
difference across
the plates

F1=8F2

The capacitance of a parallelplate capacitor can be increased by increasing


which of the following?
C

8.0 10^24 N/C

9.1 10^22 N/C

10^1 m/s

2 Statics

3 Statics
4 Statics

12 Statics

13 Statics

14 Statics

Question Text
The electron volt is a measure of

Answer
B

A solid conducting sphere is given a positive charge Q. How is the charge Q


distributed in or on the sphere?

16 Statics

A hollow metal sphere of radius R is positively charged. Of the following


distances from the center of the
sphere, which location will have the greatest electric field strength?
Two isolated charges, + q and 2q, are 2 centimeters apart. If F is the
magnitude of the force acting on charge
2Q, what are the magnitude and direction of the force acting on charge +q?

17 Statics

Charges + Q and 4Q are situated as shown above. The net electric field is
zero nearest which point?
A

15 Statics

Choice D
Momentum
Its density
increases radially
outward from the
center.

Choice E
Velocity
It is uniformly
distributed on the
surface of the
sphere only.

The charge
remains fixed
7.5 10^8
N/C
7.5 10^8
N/C
The electric field
strength is one
joule per electron.

Justification
by definition
Since charge is free to move around
on/in a conductor, excess charges will
repel each other to the
outer surface
When the battery is disconnected, Q
remains constant. Since C decreases
when d increases and
Q = CV, V will increase
E = kQ/r^2
V = kQ/r

W = qV
Since both charges are positive, the
electric field vectors point in opposite
directions at points
between the two. At point A, the
magnitudes of the electric field vectors
are equal and therefore
cancel out, making E = 0 at point A
7.statics.png
V = kQ/r and since both charges are
positive, the largest potential is at the
closest point to the
two charges (it is more mathematically
complex than that, but this reasoning
works for the
choices given)
7.statics.png
C=
0
A/d; if A 2, C 2 and if d 2, C 2 so
the net effect is C is unchanged
The net charge on the two spheres is
+Q so when they touch and separate,
the charge on each
sphere (divided equally) is Q. F
Q1
Q2 so before contact F (2Q)(Q) =
2Q2
and after
contact F ( Q)( Q) = Q2
E
or 1/8 of the original force
10.statics.png

C = 0A/d and changing Q or V has no


None of the above effect on the capacitance

force per unit


electron charge

electric charge

By definition

8.0 10^6 N/C

2.0 10^5 N/C

5.0 10^4 N/C

E = F/q

10^3 m/s

10^5 m/s

10^7 m/s

10^9 m/s

0 (center of the
sphere)

3R/2

5R/4

2R

(1/2) F Toward
charge 2q

2 F Away from
charge 2q

F Toward charge F Away from


2q
charge 2q

Image name

v = at where a = F/m. So we have v


= 10^4 10^9 10^30 = 10^(14 +
9 (30))
Inside the metal sphere E = 0. Once
None of the above outside the sphere E decreases as you
because the field move away so the
is of constant
strongest field will be the closest point
strength
to the outside of the sphere
2F Toward charge
2q
Newtons third law
Where E is zero must be closer to the
smaller charge to make up for the
weaker field. The
vectors point in opposite directions
when outside the two opposite charges.
These two criteria
E
eliminate 4 of the choices.
17.statics.png

18 Statics

19 Statics

20 Statics

21 Statics

22 Statics

A positive charge of 106 coulomb is placed on an insulated solid conducting


sphere. Which of the following is
true?
E
Two large parallel conducting plates P and Q are connected to a battery of
emf E, as shown above.
A test charge is placed successively at points I, II, and III. If edge effects are
negligible, the force on the charge
when it is at point III is
B
Forces between two objects which are inversely proportional to the square of
the distance between the objects
include which of the following? I. Gravitational force between two celestial
bodies
II. Electrostatic force between two electrons
III. Nuclear force between two neutrons
C
The diagram above shows an isolated, positive charge Q. Point (B) is twice
as far away from Q as point A. The
ratio of the electric field strength at point A to the electric field strength at
point B is
B

Which of the following is true about the net force on an uncharged conducting
sphere in a uniform electric
field?
A

24 Statics

Two conducting spheres of different radii, as shown above, each have charge
Q. Which of the following
occurs when the two spheres are connected with a conducting wire?
E
Two parallel conducting plates are connected to a constant voltage source.
The magnitude of the electric field
between the plates is 2,000 N/C. If the voltage is doubled and the distance
between the plates is reduced to 1/5
the original distance, the magnitude of the new electric field is
E

25 Statics

The figure above shows two particles, each with a charge of +Q, that are
located at the opposite corners of a
square of side d. What is the direction of the net electric field at point P ?

26 Statics

The figure above shows two particles, each with a charge of +Q, that are
located at the opposite corners of a
square of side d. What is the potential energy of a particle of charge +q that is
held at point P ?
D

23 Statics

27 Statics
28 Statics

29 Statics

30 Statics

31 Statics

32 Statics

. Two parallel conducting plates, separated by a distance d, are connected to


a battery of emf E. Which of the
following is correct if the plate separation is doubled while the battery remains
connected?
B
A 4 F capacitor is charged to a potential difference of 100 V. The electrical
energy stored in the capacitor is
E
The hollow metal sphere shown above is positively charged. Point C is the
center of the sphere and point P is
any other point within the sphere. Which of the following is true of the electric
field at these points?
A

Charges Q and +Q are located on the x and yaxes, respectively, each at


a distance d from the origin O, as
shown above. What is the direction of the electric field at the origin O ?
D
Charges Q and +Q are located on the x and yaxes, respectively, each at
a distance d from the origin O, as
shown above. What is the magnitude of the electric field at the origin O?
D
An electron e and a proton p are simultaneously released from rest in a
uniform electric field E, as shown above.
Assume that the particles are sufficiently far apart so that the only force acting
on each particle after it is
released is that due to the electric field. At a later time when the particles are
still in the field, the electron and
the proton will have the same
E

The electric field


inside the sphere
The charge
resides uniformly is constant in
magnitude, but not
throughout the
zero.
sphere.
of equal magnitude of equal magnitude
and in the same
and in the same
direction as the
direction as the
force on the
force on the
charge when it is charge when it is
at point I
at point II

An insulated metal
object acquires a
net positive charge
when brought near
to, but not in
contact with, the
sphere
much greater in
equal in magnitude magnitude than
to the force on the the force on the
charge when it is charge when it is
at point I, but in the at point II, but in
opposite direction the same direction

When a second
conducting sphere
is connected by a
conducting wire to
the first sphere,
charge is
transferred until
the electric
potentials of the
two spheres are
equal.
much less in
magnitude than
the force on the
charge when it is
at point II, but in
the same direction

I, II, and III

The electric field in


the region
surrounding the
sphere increases
with increasing
distance from the
sphere.

I only

III only

I and II only

8 to 1

4 to 1

2 to 1

It is zero

No charge flows

II and III only

1 to 1
It produces a
torque on the
It is in the direction sphere about the
It is in the direction opposite to the
direction of the
of the field
field
field.
Negative charge
Negative charge
flows from the
Negative charge
flows from the
larger sphere to
flows from the
smaller sphere to
the smaller sphere larger sphere to
the larger sphere
until the electric
the smaller sphere until the electric
field at the surface until the electric
field at the surface
of
potential of each
of
each sphere is the sphere is the
each sphere is the
same.
same.
same.

1 to 2
It causes the
sphere to oscillate
about an
equilibrium
position.
Negative charge
flows from the
smaller sphere to
the larger sphere
until the electric
potential of each
sphere is the
same.

Charges flow when there is a difference


in potential. Analyzing the other
choices: A is wrong
because the charge resides on the
surface. For B, E = 0 in a charged
conducting sphere. E =
kQ/r^2
eliminates choice C. And for D, charge
separation will occur, but the object will
not
acquire any charge.

E is uniform between charged parallel


plates therefore the force on a charge is
also uniform
between the plates
19.statics.png

Fg = Gm1m2/r2 and FE = kq1q2/r2.


The nuclear force does not have a
similar relationship.
E 1/r2
so if r 2, E 4

21.statics.png

While the charges may separate, the


forces on the opposite charges are in
opposite directions,
canceling out
V = kQ/r so the smaller sphere is at the
lower potential (more negative = lower)
Negative charge
flows from low to high potential so the
charge will flow from the smaller sphere
to the larger.
The flow of charge ceases when there
is no difference in potential.
23.statics.png

E = V/d so if V 2, E 2 and if d 5, E
5 so the net effect is E 10
The electric field vectors from the two
charges point down and to the left
(away from the
charges) so the resultant field points
northeast
east
southeast
south
southwest
down and left
25.statics.png
The potential energy of a particle at a
location is the potential at that location
times the charge.
SQRT(2)qQ/
2SQRT(2)qQ/
In this case, the potential is kQ/d +
zero
(40d)
1qQ/(40d)
2qQ/(40d)
(40d)
kQ/d = (2kQ/d)
25.statics.png
If the battery remains connected, the
The potential
The potential
potential remains constant. C
The electric charge The electric charge difference between difference between
decreases as the separation
on the plates is
on the plates is
the plates is
the plates is
The capacitance is increases soothe charge Q = CV will
doubled.
halved
doubled.
halved
unchanged.
also decrease
800N/C

1,600N/C

2,400N/C

5,000N/C

20,000N/C

2 10^10 J

2 10^6 J
It is zero at C, but
at P it is not zero
and is directed
outward.

2 10^4 J

2 10^2 J

It is zero at both
points.

2 10^8 J
It is zero at C, but
at P it is not zero
and is directed
inward

It is zero at P, but
at C it is not zero.

It is not zero at
either point.

east

west

northwest

southeast

southwest

kQ/(2d^2)

kQ/(2^(1/2)d^2)

kQ/d^2

2^(1/2)kQ/d^2

2kQ/d^2

magnitude of
acceleration

Since the electron and the proton have


equal charge, the forces on them are
equal. Since they
have different masses, the
magnitude of force accelerations, speeds and
acting on them
displacements will not be equal.
32.statics.png

direction of motion speed

displacement

UC = CV^2
In metals under electrostatic conditions,
the electric field is zero everywhere
inside.
29.statics.png
The electric field vector from the +Q
charge points down and from the Q
charge points to the
right so the resultant field points down
and right
30.statics.png
The two vectors, each of magnitude E =
kQ/d2, point at right angles to each
other so the resultant field is 2E
30.statics.png

33 Statics

34 Statics

35 Statics

36 Statics

37 Statics

38 Statics

10 V

8V

6V

4V

2V

The electric field between charged


parallel plates is uniform, which means
the potential changes
uniformly with distance. For a change of
8 V over 4 cm means the change of
potential with
position (and the electric field strength)
is 2 V/cm, which gives the potential 1
cm away from the
2 V plate as 4 V
33.statics.png

800 V/m

600 V/m

400 V/m

200 V/m

100 V/m

E=V/d

1/4 K

1/2 K

4K

W = K = QV (mass doesnt have an


effect on the kinetic energy, just on the
speed in this case)

2K
A small negatively
charged object
The charge on Y is
placed at point X
negative and the
The strength of the The electric field is would tend to
charge on Z is
electric field is the strongest midway move toward the
positive.
same everywhere. between Y and Z. right.

9.0 x 10^3 N/C

1.8 x 10^4 N/C

2.4 x 10^4 N/C

3.6 x 10^4 N/C

1.4 x 10^5 N/C

E = kQ/r2

1/4 Co

1/2 Co

Co

2 Co

4 Co

directed
perpendicular to
the surface

independent of the
directed parallel to surface charge
the surface
density
zero
directly
proportional to the
distance between
R and S
positive
zero

C = 0A/d; 0(2A)/(2d) = 0A/d


Charges arrange themselves on
conductors so there is no electric field
inside, and no electric field
component along the surface

Two large, flat, parallel, conducting plates are 0.04 m apart, as shown above.
The lower plate is at a potential of
2 V with respect to ground. The upper plate is at a potential of 10 V with
respect to ground. Point P is located
0.01 m above the lower plate. The electric potential at point P is
D
Two large, flat, parallel, conducting plates are 0.04 m apart, as shown above.
The lower plate is at a potential of
2 V with respect to ground. The upper plate is at a potential of 10 V with
respect to ground. Point P is located
0.01 m above the lower plate. The magnitude of the electric field at point P is D
A particle of charge Q and mass m is accelerated from rest through a
potential difference V, attaining a kinetic
energy K. What is the kinetic energy of a particle of charge 2Q and mass m/2
that is accelerated from rest
through the same potential difference?
D

The diagram above shows electric field lines in an isolated region of space
containing two small charged
spheres, Y and Z. Which of the following statements is true?
A hollow metal sphere 1.0 m in diameter carries a charge of 4.0 C. The
electric field at a distance of 2.0 m
from the center of the sphere is most nearly
A parallelplate capacitor has a capacitance Co. A second parallelplate
capacitor has plates with twice the area and twice the separation. The
capacitance of the second capacitor is most nearly

39 Statics

The electric field E just outside the surface of a charged conductor is

40 Statics

Points R and S are each the same distance d from two unequal charges, + Q
and +2Q, as shown above. The work required to move a charge -Q from point
R to point S is
D

41 Statics

A rigid insulated rod, with two unequal charges attached to its ends, is placed
in a uniform electric field E as shown above.The rod experiences a
B

42 Statics

The electric field of two long coaxial cylinders is represented by lines of force
as shown above. The charge on
the inner cylinder is +Q. The charge on the outer cylinder is
E

43 Statics

44 Statics

45 Statics

46 Statics

47 Statics

48 Statics

An isolated capacitor with air between its plates has a potential difference Vo
and a charge Qo. After the space between the plates is filled with oil, the
difference in potential is V and the charge is Q. Which of the following pairs of
relationships is correct?
B
Two small spheres have equal charges q and are separated by a distance d.
The force exerted on each sphere by the other has magnitude F. If the charge
on each sphere is doubled and d is halved, the force on each sphere has
magnitude
E

Which of the following statements about conductors under electrostatic


conditions is true?

A charged particle traveling with a velocity v in an electric field E experiences


a force F that must be
D
A positive charge of 3.0 10^ -8 coulomb is placed in an upward directed
uniform electric field of 4.0 10^4 N/C.When the charge is moved 0.5 meter
upward,the work done by the electric force on the charge is
A
The following configurations of electric charges are located at the vertices of
an equilateral triangle.Point P is equidistant from the charges. In which
configuration is the electric field at P equal to zero?
A

dependent on the
path taken from R
to S

Both charges
spheres Y and Z
The field lines point away from Y and
carry the charge of toward Z making Y positive and Z
the same sign.
negative.

infinite

33.statics.png

36.statics.png

By symmetry VR = VS so VRS = 0
and W = qV
40.statics.png
The force on the upper charge is to the
left and twice the magnitude of the
net force to the left
net force to the
force on the bottom charge, which is to
net force to the left and a
net force to the
right and a
the right. This makes the net force to
and a clockwise
counterclockwise right and a
counterclockwise rotation, but no net the left and the torque on the rod to be
rotation
rotation
clockwise rotation rotation
force
counterclockwise.
41.statics.png
Compared to the +Q charge at the
center, the charge on the outer surface
of the outer cylinder has twice the
magnitude and is of opposite sign (so it
is 2Q). There is also an equal and
opposite charge induced on the inner
surface of the outer cylinder making the
total charge on the outer cylinder 2Q +
positive 3Q
positive Q
0 negative Q
negative 3Q
Q
42.statics.png
Since the capacitor is isolated, Q
remains constant. Filling the place with
oil (a dielectric) will increase the
capacitance, causing the potential (V =
Q = Qo and V > Vo Q = Qo and V < Vo Q > Qo and V = Vo Q < Qo and V < Vo Q > Qo and V > Vo Q/C) to decrease
negative

parallel to v

perpendicular to v

perpendicular to v
and E

FE q1q2/r2; if q1 and q2 2; F
4 and if r 2, F 4 making the
net effect F 4 4
Since there is no component of the
The electric field at The surface of a
electric field along a conducting surface
the surface of a
conductor is
under electrostatic conditions, no work
conductor is
always an
is done moving the charge around the
tangent to the
equipotential
surface, meaning no differencesin
surface.
surface.
potential
Regardless of velocity, the force on a
charge in an electric field is parallel to
parallel to E
perpendicular to E the field (F = qE)

6 10^ -4 J

12 10^ 4 J

2 10^ 4

8 x 10^ 4 J

12 x 10^4 J

W = Fd = qEd

E points away from + charges and


toward charges. Use symmetry.

2F
Charge that is
Positive work is
placed on the
required to move a surface of a
positive charge
conductor always
over the surface of spreads evenly
a conductor
over the surface

4F

The electric
potential inside a
conductor is
always zero.

8F

16F

48.statics.png

The following configurations of electric charges are located at the vertices of


an equilateral triangle. Point P is equidistant from the charges. In which
configuration is the electric field at P pointed at the midpoint between two of
the charges?
49 Statics

51 Statics

52 Statics

53 Statics

54 Statics

55 Statics

56 Statics

57 Statics

58 Statics

59 Statics

50 Statics
60 Statics

61 Statics

62 Statics

The capacitance
decreases.

The potential
difference across
the capacitor
decreases.

From X to Y

From Y to X

E = 0, V = kQ/R

E = 0, V = kQ/r

D and E are not symmetric so the field


will not point at the midpoint of any
side. The field in
C
D
E
choice B points at the bottom charge.
48.statics.png
Since the capacitor is isolated, Q
The electric field
remains constant. Filling the place with
The energy of the The charge on the between the
oil (a dielectric) will
capacitor does not capacitor plates
capacitor plates
increase the capacitance, causing the
change.
decreases
increases.
potential (V = Q/C) to decrease.
It cannot be
V = kQ/r so the smaller sphere (Y) is at
It cannot be
determined without the higher potential. Negative charge
There will be no
determined without knowing whether flows from low to
flow of charge in
knowing the
the spheres are
high potential so the charge will flow
the wire.
magnitude of Q.
solid or hollow.
from X to Y.
52.statics.png
Once inside a uniform sphere of
charge, the electric field is zero. Since
E = 0 the potential does
not change within the sphere (meaning
E = 0, V = 0
E = kQ/r^2, V = 0 E = kQ/R^2, V = 0 it is the same value as the surface)

E = kQ/R^2, V =
kQ/R

E = kQ/R, V =
kQ/R

E = kQ/r^2, V =
E = kQ/R, V = kQ/r kQ/r

A sheet of mica is inserted between the plates of an isolated charged


parallelplate capacitor. Which of the
B
Two conducting spheres, X and Y have the same positive charge +Q, but
different radii(rx > ry) as shown above. The spheres are separated so that the
distance between them is large compared with
either radius. If a wire is connected between them, in which direction will
electrons be directed in the wire?
A
A sphere of radius R has positive charge Q uniformly distributed on its
surface. Which of the following represents the magnitude of the electric field
E and the potential V as functions of r, the
distance from the center of the sphere, when r < R ?
A sphere of radius R has positive charge Q uniformly distributed on its
surface. Which of the following represents the magnitude, of the electric field
E and the potential V as functions of r, the
distance from the center of sphere, when r > R ?
From the electric field vector at a point, one can determine which of the
following?
I. The direction of the electrostatic force on a test charge of known sign at that
point
II. The magnitude of the electrostatic force exerted per unit charge on a test
charge at that point
III. The electrostatic charge at that point
A conducting sphere of radius R carries a charge Q. Another conducting
sphere has a radius R/2, but carries the
same charge. The spheres are far apart. The ratio of the electric field near the
surface of the smaller sphere to
the field near the surface of the larger sphere is most nearly
A circular ring made of an insulating material is cut in half. One half is given a
charge q uniformly distributed
along its arc. The other half is given a charge + q also uniformly distributed
along its arc. The two halves are
then rejoined with insulation at the junctions J, as shown above. If there is no
change in the charge
distributions, what is the direction of the net electrostatic force on an electron
located at the center of the circle?
Four positive charges of magnitude q are arranged at the corners of a square,
as shown above. At the center C
of the square, the potential due to one charge alone is Vo and the electric
field due to one charge alone has
magnitude Eo. Which of the following correctly gives the electric potential and
the magnitude of the electric
field at the center of the square due to all four charges?

Two charges, 2Q and +Q, are located on the xaxis, as shown above. Point
P, at a distance of 3D from the
origin O, is one of two points on the positive xaxis at which the electric
potential is zero. How far from the
origin O is the other point?
D
Two concentric, spherical conducting shells have radii r1 and r2 and charges
Q1 and Q2, as shown above. Let r
be the distance from the center of the spheres and consider the region r1 < r
< r2. In this region the electric field
is proportional to
A

Which of the following configurations is most likely to produce these


equipotential lines?

The force on an electron located on the 0 volt potential line is

I only

III only

0.25

Toward the top of


the page

I and II only

0.5

II and III only

Toward the bottom


of the page
To the right

E = kQ/r^2, V =
kQ/r^2

Outside a uniform sphere of charge, it


behaves as a point charge.

I, II, and III

E = F/q. The vector nature of the


equation allows one to find the direction
of F and the equation
itself allows one to find the ratio F/q, but
not q specifically

Outside a uniform sphere of charge, it


4 behaves as a point charge. E = kQ/r2

To the left

Into the page.

By symmetry, the force on an electron


at the center from the top half will be
straight down and
the force from the bottom half will also
be straight down

57.statics.png

E is a vector so all the individual E field


vectors from the four charges will
Electric Potential = Electric Potential = Electric Potential = Electric Potential = Electric Potential = cancel. V is a scalar
zero, Electric Field zero, Electric Field 2Vo, Electric Field 4Vo, Electric Field 4Vo, Electric Field and will add since they are all positive
= zero
= 2Eo
= 4Eo
= zero
= 2Eo
charges.
58.statics.png
The points where V = 0 must lie closer
to the smaller charge. Unlike electric
field vectors which
also require the individual vectors point
in opposite directions, there are a locus
of points (in this
case in a ring surrounding the +Q
charge) where V = 0 as the two
charges are opposite in sign
and V is a scalar. So the other point on
the x axis is between the two charges,
but closer to the
+Q charge. This must be a value
2/3D
D
3/2D
5/3D
2D
between 1.5 D and 2 D
59.statics.png
When inside a uniform shell of charge,
there is an electric field due to the shell.
When outside a
uniform shell of charge, the electric field
Q1/r^2
(Q1 + Q2)/r^2
(Q1 + Q2)/r
Q1/r1 + Q2/r
Q1/r + Q2/r2
is as if the shell was a point charge.
60.statics.png

0N

D
to the right, but its
magnitude cannot
be determined
without knowing
1N, directed to the 1N, directed to the the distance
right
left
between the lines

E
to the left, but its
magnitude cannot
be determined
without knowing
the distance
between the lines

The potential difference between the


plates is 4 V and the right side is the
positive plate. We
need the batteries pointing in the same
direction with the positive terminal on
the right.
62.statics.png

The electron experiences a force


toward the positive plate of magnitude
F = Eq. E = V/d and
cannot be calculated without knowing d. 62.statics.png

63 Statics

Two metal spheres that are initially uncharged are mounted on insulating
stands, as shown above. A negatively
charged rubber rod is brought close to, but does not make contact with,
sphere X. Sphere Y is then brought
close to X on the side opposite to the rubber rod. Y is allowed to touch X and
then is removed some distance
away. The rubber rod is then moved far away from X and Y. What are the
final charges on the spheres?
D

67 Statics

Which of the following capacitors, each of which has plates of area A, would
store the most charge on the top
plate for a given potential difference V?
A parallelplate capacitor has charge +Q on one plate and charge Q on the
other. The plates, each of area A,
are a distance d apart and are separated by a vacuum. A single proton of
charge +e, released from rest at the
surface of the positively charged plate, will arrive at the other plate with
kinetic energy proportional to
Two initially uncharged conductors, 1 and 2, are mounted on insulating
stands and are in contact, as shown
above. A negatively charged rod is brought near but does not touch them.
With the rod held in place, conductor
2 is moved to the right by pushing its stand, so that the conductors are
separated. Which of the following is now
true of conductor 2?
As shown above, two particles, each of charge +Q, are fixed at opposite
corners of a square that lies in the plane
of the page. A positive test charge +q is placed at a third corner. What is the
direction of the force on the test
charge due to the two other charges?

68 Statics

If F is the magnitude of the force on the test charge due to only one of the
other charges, what is the magnitude
of the net force acting on the test charge due to both of these charges?

69 Statics

Two charges are located on the line shown in the figure below, in which the
charge at point I is +3q and the
charge at point III is +2q. Point II is halfway between points I and III....Other
than at infinity, the electric field strength is zero at a point on the line in which
of the following ranges?
C

64 Statics

65 Statics

66 Statics

Sphere X:
Zero Sphere Y:
Zero

Sphere X:
Negative Sphere
Y: Negative

Sphere X:
Sphere X:
Negative Sphere Positive Sphere
Y: Positive
Y: Negative

Sphere X:
Positive Sphere
Y: Positive

While spheres X and Y are in contact,


electrons will repel away from the rod
out of sphere X into
sphere Y.
63.statics.png
The capacitor with the largest
capacitance will store the most charge.
C = 0A/d where glass >
air and Kvacuum
64.statics.png

(edQ)/A

It is uncharged

Q^2/(eAd)

It is positively
charged

eQ^2/Ad

It is negatively
charged

It is charged, but
its sign cannot be
predicted

It is at the same
potential that it
was before the
charged rod was
brough near

While spheres 1 and 2 are in contact,


electrons will repel away from the rod
out of sphere 1 into
sphere 2.

Zero

F/2^(1/2)

[2^(1/2)]F

To the left of I

Between I and II

Between II and III

To the right of III

To the left of I

Between I and II

Between II and III

To the right of III

4104 V

102 V

2.5103 V

2106 V

6106 V

72 Statics

The graph above shows the electric potential V in a region of space as a


function of position along the xaxis.
At which point would a charged particle experience the force of greatest
magnitude?

73 Statics

Suppose that an electron (charge e) could orbit a proton (charge +e) in a


circular orbit of constant radius R. Assuming that the proton is stationary and
only electrostatic forces act on the particles, which of the following represents
the kinetic energy of the twoparticle system?
B

75 Statics

Q/ed

71 Statics

74 Statics

(AeQ)/d

K = qV. To find V we use Q = CV (V


is V in this case) which gives K = e
(Q/C) and if we
use C = 0A/d we have K = e(Qd/0A)

The force vectors from the two +Q


charges point down and to the left
(away from the charges) so
E
the resultant force points down and left
The two vectors, each of magnitude F,
point at right angles to each other so
the resultant field is
2 2F
Where E is zero must be closer to the
smaller charge to make up for the
weaker field. The
vectors point in opposite directions
when between the two like charges.
None; the field is
These two criteria
zero only at infinity eliminate 4 of the choices
Since both charges are positive and V
None; this
is a scalar equal to kQ/r, the potential
potential is never will never be zero
negative.
in the vicinity of these two charges

The electric potential is negative at some points on the line in which of the
following ranges?
E
The work that must be done by an external agent to move a point charge of 2
mC from the origin to a point 3 m away is 5 J. What is the potential difference
between the two points?
C

70 Statics

If the only force acting on an electron is due to a uniform electric field, the
electron moves with constant
A
Two charged particles, each with a charge of +q, are located along the xaxis
at x = 2 and x = 4, as shown
above. Which of the following shows the graph of the magnitude of the
electric field along the xaxis from the
origin to x = 6?
A

66.statics.png

67.statics.png

67.statics.png

69.statics.png

69.statics.png

C
acceleration in a
acceleration in the direction
direction of the
perpendicular to
field
that of the field

acceleration in a
direction opposite
to that of the field

speed in a
direction opposite
to that of the field

speed in the
direction of the
field

W=qV
The electric field (and hence, the
electric force on a charge) is greatest
where the potential changes most
rapidly with position (the greatest
gradient) since E = V/d. On this graph,
this
would be the point where the slope is
the greatest
72.statics.png
FE =FC andq1 =q2 =e so we have
ke2/R2 =mv2/R and we multiply both
sides by 12R so the B right side
becomes 12 mv2 (the kinetic energy).
Choices C and E could have been
eliminated
because they are negative, and kinetic
energy cannot be negative. Choices A
& D are
dimensionally incorrect (D has the units
of a force, not energy, and A has the
units of electric
potential)
73.statics.png
If F is constant and F = ma, the
acceleration is also constant. Negative
charges experience forces A opposite
in direction to electric field lines.

By symmetry, E = 0 at the midpoint and


goes to infinity near each charge (E =
kQ/r2)
75.statics.png

78 Statics

A positive electric charge is moved at a constant speed between two


locations in an electric field, with no work
done by or against the field at any time during the motion. This situation can
occur only if the...
D
The nonconducting hollow sphere of radius R shown above carries a large
charge +Q, which is uniformly
distributed on its surface. There is a small hole in the sphere. A small charge
+q is initially located at point P. a
distance r from the center of the sphere.If k = 1/4o, what is the work that
must be done by an external agent
in moving the charge +q from P through the hole to the center O of the
sphere?
E
A capacitor is constructed of two identical conducting plates parallel to each
other and separated by a distance d. The capacitor is charged to a potential
difference of V0 by a battery, which is then disconnected. If any edge effects
are negligible, what is the magnitude of the electric field between the plates? B

79 Statics

A capacitor is constructed of two identical conducting plates parallel to each


other and separated by a distance d. The capacitor is charged to a potential
difference of V0 by a battery, which is then disconnected. A sheet of
insulating plastic material is inserted between the plates without otherwise
disturbing the system.

76 Statics

77 Statics

charge is moved
charge is moved in opposite to the
the direction of the direction of the
field
field

charge is moved
perpendicular to
an equipotential
line

charge is moved
along an
equipotential line

electric field is
uniform

If no work is done by the field and there


is a field present, the motion must be
perpendicular to the field, along an
equipotential line, making the force
perpendicular to the displacement of
the
charge (a requirement for zero work).
Along an equipotential line, V = 0 and
W = qV.

Inside the sphere, E = 0 which means


the potential does not change with
position and is the same value as the
surface, which is kQ/R. At point P, the
potential is kQ/r. W = qV = q(kQ/R
kQ/r)

Zero

kqQ/r

kqQ/R

kq(Q q)/r

kqQ(1/R 1/r)

V0d

V0/d

d/V0

It causes the
capacitance to
decrease

None; the
capacitance does
not change

V02/d
Nothing can be
said about the
effect without
knowing the
thickness of the
sheet

E=V/d

It causes the
capacitance to
increase

V0/d2
Nothing can be
said about the
effect without
knowing the
dielectric constant
of the plastic

Q only

the charge
distribution on the
sphere only

Q and the charge


distribution on the
sphere

all of the point


charges

9 10^3 J

9 10^3 J

6 10^4 J

2 10^4 J

2 10^7 J

UC = CV^2

1/V

1/V^(1/2)

V^(1/2)

V^2

If the battery is disconnected, Q


remains constant. If a dielectric is
inserted between the plates, the
capacitance increases and since Q =
CV, the potential difference decreases.
Conductors under electrostatic
conditions will arrange their changes so
no electric field exists inside (other
than those created by charges placed
inside the empty cavity). Fields from
external
charges will not penetrate into
conducting enclosures.
80.statics.png

82 Statics

A point charge +Q is inside an uncharged conducting spherical shell that in


turn is near several isolated point
charges, as shown above. The electric field at point P inside the shell
depends on the magnitude of
A
A 20 F parallelplate capacitor is fully charged to 30 V. The energy stored in
the capacitor is most nearly...
B
A potential difference V is maintained between two large, parallel conducting
plates. An electron starts from
rest on the surface of one plate and accelerates toward the other. Its speed
as it reaches the second plate is
proportional to...
C

83 Statics

Particles of charge Q and 4Q are located on the xaxis as shown in the


figure above. Assume the particles are
isolated from all other charges. Which of the following describes the direction
of the electric field at point P?
E

positive x

positive y

negative y

84 Statics

Particles of charge Q and 4Q are located on the xaxis as shown in the


figure above. Assume the particles are isolated from all other charges. At
which of the labeled points on the xaxis is the electric field zero?

W = K = qV and K = mv^2
At point P, the field due to charge Q
points up and to the right and the field
due to charge 4Q is
larger in magnitude and points down
Components in
Components in
and to the right. Due to the asymmetry,
both the x and +y both the +x and y no components will
directions
directions
cancel.
Where E is zero must be closer to the
smaller charge to make up for the
weaker field. The
vectors point in opposite directions
outside the two opposite charges.
These two criteria eliminate
3 of the choices. For the magnitudes of
the electric fields to be zero the ratios
Q/r2
must be equal
giving (in units along the x axis) Q/r2
= 4Q/(r + 4 units)2
A
D
E
giving r = 4 units
Since E = V/d, E represents the slope
of the line on the graph which could be
choice C or D.
since V 1/r the slope is
proportional to V/r = (1/r)/r =
D
E
1/r^2 which is choice C

12^(1/2) / 4 *
Q/R^2

18^(1/2) / 4 *
Q/R^2

3^(1/2) / 2 *
Q/R^2

3 / 2 * Q^2/R

9 / * Q^2/R

80 Statics
81 Statics

85 Statics

86 Statics

87 Statics

88 Statics

A solid metallic sphere of radius R has charge Q uniformly distributed on its


outer surface. A graph of electric
potential V as a function of position ris shown above. Which of the following
graphs best represents the
magnitude of the electric field E as a function of position rfor this sphere?
As shown in the figure above, six particles, each with charge +Q, are held
fixed and ate equally spaced around
the circumference of a circle of radius R. What is the magnitude of the
resultant electric field at the center of the circle?
As shown in the figure above, six particles, each with charge +Q, are held
fixed and ate equally spaced around
the circumference of a circle of radius R. With the six particles held fixed, how
much work would be required to bring a seventh particle of charge + Q
from very far away and place it at the center of the circle?
The diagram above shows equipotential lines produced by an unknown
charge distribution. A, B, C, D, and E
are points in the plane. Which vector below best describes the direction of the
electric field at point A ?

6^(1/2) / 4 *
0 Q/R^2

0 6^(1/2) / 4 * Q/R 3 / 2 * Q^2/R^2

northeast

southwest

southeast

northwest

77.statics.png

By symmetry, all the vectors cancel

83.statics.png

83.statics.png

85.statics.png 85.
staticsquestion.png

86.statics.png

W = qV = +Q(Vcenter V) =
+QVcenter where Vcenter = V = kQ/r
= 6kQ/R
86.statics.png
E points from high potential to low
potential, perpendicular to equipotential
None of these; the lines (the direction of
field is zero.
the force on a positive charge)
88.statics.png

89 Statics

90 Statics

91 Statics

The diagram above shows equipotential lines produced by an unknown


charge distribution. A, B, C, D, and E
are points in the plane. At which point does the electric field have the greatest
magnitude?
B

The diagram above shows equipotential lines produced by an unknown


charge distribution. A, B, C, D, and E
are points in the plane. How much net work must be done by an external
force to move a 1 C point charge from rest at point C to rest
at point E ?
B
The plates of a parallelplate capacitor of cross sectional area A are
separated by a distance d, as shown above.
Between the plates is a dielectric material of constant K. The plates are
connected in series with a variable
resistance R and a power supply of potential difference V. The capacitance C
of this capacitor will increase if
which of the following is decreased?
D

92 Statics

A physics problem starts: "A solid sphere has charge distributed uniformly
throughout. . . " It may be correctly
concluded that the

93 Statics

A uniform spherical charge distribution has radius R.. Which of the following
is true of the electric field
strength due to this charge distribution at a distance r from the center of the
charge?

94 Statics

95 Statics

96 Statics

97 Statics

98 Statics

99 Statics

100 Statics

101 Statics

102 Statics

103 Statics

When a negatively charged rod is brought near, but does not touch, the
initially uncharged electroscope shown
above, the leaves spring apart (I). When the electroscope is then touched
with a finger, the leaves collapse (II).
When next the finger and finally the rod are removed, the leaves spring apart
a second time (III). The charge on
the leaves is
A positively charged conductor attracts a second object. Which of the
following statements could be true?
I. The second object is a conductor with negative net charge.
II. The second object is a conductor with zero net charge.
III. The second object is an insulator with zero net charge..
A point charge of +4.0 C is placed on the negative xaxis 0.20 m to the left
of the origin, as shown in the
accompanying figure. A second point charge q is placed on the positive x
axis 0.30 m to the right of the origin. If the net electric field at the origin is
zero. What is q?
A point charge of +4.0 C is placed on the negative xaxis 0.20 m to the left
of the origin, as shown in the
accompanying figure. A second point charge q is placed on the positive x
axis 0.30 m to the right of the origin. If the net electric potential at the origin is
zero, what is q?
Which of the following statements about solid conductors in electrostatics are
true?
I. The electric field inside the conductor is always zero.
II. The electric potential inside the conductor is always zero.
III. Any net charge is on the surface.
A small object with charge q and weight mg is attached to one end of a string
of length L. The other end is
attached to a stationary support. The system is placed in a uniform horizontal
electric field E, as shown in the
accompanying figure. In the presence of the field, the string makes a constant
angle q with the vertical. What is
the sign and magnitude of q?
An isolated conducting sphere of radius R has positive charge + Q. Which
graph best depictsthe electric
potential as a function of r, the distance from the center of the sphere?
In the figure to the right, equipotential lines are drawn at 0, 20.0 V, and 40.0
V. The total work done in moving
a point charge of + 3.00 mC from position a to position b is:
Two positive point charges repel each other with force 0.36 N when their
separation is 1.5 m. What force do
they exert on each other when their separation is 1.0 m?

An amber rod is given a net negative charge and held at rest. Which of the
following statements is true?

20 J

10 J

10 J

20 J

30 J

E points from high potential to low


potential, perpendicular to equipotential
lines (the direction of
the force on a positive charge)
88.statics.png
VCE = VE VC = 10 V. The amount
of work, W = qV = 1 C 10 V = 10
C. Since the B
external force must push against the
negative charge to keep it from
accelerating and bring it to
rest at point E, the work done by the
external force must be negative.
88.statics.png

C = KoA/d. Only changes in the


geometry of the capacitor will change
the capacitance, not changes to the
battery or resistor.

R
electric field inside
the sphere is the
electric field is zero same as the
everywhere inside electric field
the sphere
outside

It is directly
It is greatest when It is greatest when proportional to r
r = 0.
r = R/2.
when r > R.

positive in both I
and III

I only

9.0 C

negative in both I
and III

II only

K
electric potential
on the surface of
the sphere is not
constant

positive in I,
negative in III

III only

6.0 C

9.0 C

6.0 C

I only

II only

electric potential in
the center of the
sphere is not made For charge to be distributed throughout
sphere is zero
of metal
a material, it must be non-conducting
Advanced question (not exactly in the B
curriculum, but interesting). Like gravity
inside a uniform sphere of mass, the
It is directly
field is directly proportional to r when
proportional to r
It is directly
inside the sphere (and proportional to
when r < R.
proportional to r2. 1/r2 when outside)
93.statics.png
In I, charge separation occurs (negative
charges repel to the leaves). The whole
process describes charging by
induction, where the electrons leave the
electroscope to ground (the finger) and
impossible to
once contact with ground is broken, the
negative in I,
determine in either electroscope is left with a positive
positive in III
I or III
charge (III)

I & II only

0 6.0 C

0 6.0 C

III only

I & III only

I, II & III

Charged objects attract object with an


opposite charge, but also neutral
objects by separation of charges.

9.0 C

If E = 0, the field vectors point in


opposite directions, making q positive.
In magnitude we can find q by (+4 C)/
(0.2 m)2= q/(0.3 m)2
96.statics.png

9.0 C

If V = 0 and V = kQ/r then q must be


negative and (+4 C)/(0.2 m) = q/(0.3
m)

II & III only

by definition of conductors under


electrostatic conditions

positive with
magnitude
(mg)/E

positive with
magnitude
(mg)/(Etan(theta))

negative with
magnitude
(mg)/E

negative with
magnitude
(mg)/(Etan(theta))

negative with
magnitude
(E)/(mgtan(theta))

4.00 mJ

8.00 mJ

12.0 mJ

24.0 mJ

120 mJ

0.81 N

0.54 N

0.36 N

91.statics.png

0.24 N
The amber rod is
surrounded by
both a magnetic
The amber rod is The amber rod is field that circles
The amber rod is surrounded only by surrounded only by the rod and an
surrounded only by an electric field
an electric field
electric field that is
a magnetic field
that is directed out that is directed into directed out from
that circles the rod. from the rod.
the rod.
the rod.

0.16 N
The amber rod is
surrounded by
both a magnetic
field that circles
the rod and an
electric field that is
directed into the
rod.

96.statics.png

Since the electrostatic force pushes the


charge to the right, with the field line, it
is a positive charge. Fy= 0 gives T cos
= mg and Fx= 0 gives T sin = FE=
qE. Divide the two expressions to
eliminate T.
99.statics.png
V is constant inside and on the surface
of a charged conductor and is
proportional to 1/r outside a charged
conducting sphere.
100.statics.png
W = qV (motion along an equipotential
line requires no work so only V
matters, not the path)
102.statics.png
F 1/r2

In a later topic, you will learn magnetic


field are created by moving charges.
Electric field lines
point toward negative charges.

104 Statics

105 Statics
106 Statics

107 Statics

108 Statics

109 Statics

110 Statics

Two isolated conducting spheres (S1 of radius 0.030 m and initial charge +
6.0 nC and S2
of radius 0.040 m and initial charge + 2.0 nC) are connected by a conducting
wire. Charge will flow in the wire until:
E

A point charge +q is placed midway between two point charges +3q and q
separated by a distance 2d. If
Coulombs constant is k, the magnitude of the force on the charge +q is:
How much work is required to move 24 mC of charge 4.0 m parallel to a
uniform 6.0 N/C electric field?
An isolated conducting sphere of radius R has positive charge + Q. Which
graph best depicts the electric field
as a function of r, the distance from the center of the sphere?

both spheres are


equally charged.

the net charge is


zero.

the force of
repulsion between both spheres have both spheres are
the two spheres
the same surface at the same
becomes equal.
charge density.
potential.

2kq^2/d^2

4kq^2/d^2

6kq^2/d^2

8kq^2/d^2

10kq^2/d^2

1.0 mJ

16 mJ

36 mJ

62 mJ

576 mJ

Point charges 1 and 2 have equal magnitude. The diagram to above shows
the electric field lines surrounding
them. Which of the following statements is true?
A
A charged rod is placed between two insulated conducting spheres as shown.
The spheres have no net charge.
Region II has the same polarity as Region
B
A charged rod is placed between two insulated conducting spheres as shown.
The spheres have no net charge.
Region II has the same polarity as Region
B

112 Statics

Two large oppositely charged insulated plates have a uniform electric field
between them. The distance between the plates is increased. Which ofthe
following statements is true? I. The field strength decreases.
II. The field strength increases.
III. The potential difference between the plates increases.
C
When two charged pointlike objects are separated by a distance R, the force
between them is F. If the distance between them is quadrupled, the force
between them is
E

113 Statics

An electroscope is given a positive charge, causing its foil leaves to separate.


When an object is brought near the top plate of the electroscope, the foils
separate even further. We could conclude
A

111 Statics

114 Statics

115 Statics

116 Statics
117 Statics

Four positive point charges are arranged as shown in the accompanying


diagram. The force between charges 1
and 3 is 6.0 N; the force between charges 2 and 3 is 5.0 N; and the force
between charges 3 and 4 is 3.0 N. The
magnitude of the total force on charge 3 is most nearly
A
Two isolated parallel plates are separated by a distance d. They carry
opposite charges Q and each has surface
area A. Which of the following would increase the strength of the electric field
between the plates? I. Increasing Q
II. Increasing A
III. Increasing d
A

Consider the two oppositely charged plates asshown in the diagram. At which
of the marked pointsshown in
the diagram would a positively charged particle have the greatest electrical
potential energy?
E
How much work would be required to move a 4 coulomb charge 6 meters
parallel to a 24 N/Celectric field?
E

118 Statics

. When a positive electrically charged glass rod is brought near a neutral


hollow metal sphere suspended by an
insulating string, the sphere will be attracted to the rod because:

119 Statics

An alpha particle and a proton are placed equal distance between two large
charged metal plates as shown.
Which of the following would best describe the motion of the two particles if
they were free to move?

Charge 1 is
positive, charge 2
is negative.

Charge 1 is
Both charges 1
negative, charge 2 Both charges 1
and 2 are
is positive.
and 2 are positive. negative.

E
Both charges 1
and 2 have the
same sign, but it is
impossible to tell
which.

I only

III only

IV only

I & III only

I & IV only

I only

III only

IV only

I & III only

I & IV only

I only

II only

III only

I and III only

II and III only

16 F

4F

F/4

that the object is


that the object is
that the object is
negatively
positively charged. electrically neutral. charged.

6.3 N

8.0 N

10 N

Charges flow when there is a difference


in potential.
The distance between the +q charge
and each charge is d. The force on the
+q charge from each
charge is in the same direction, making
the net force kq2/d2 + k(3q2)/d2
W = Fd = qEd
E is zero inside a charged conductor
and is proportional to 1/r2 outside a
charged conducting sphere.

108.statics.png

Electric field lines point in the direction


of the force on a positive charge (away
from positive charges and toward
negative charges)
109.statics.png
The rod will attract the same charge
from each sphere to the side closer to
the rod.
110.statics.png
The rod will attract the same charge
from each sphere to the side closer to
the rod.
110.static.png
Since the plates are insulated, the
charge remains constant. If the
distance is increased, the
capacitance will decrease (C A/d)
and since Q = CV, the potential
difference must increase by the
same factor that the distance
increases. This means E = V/d
remains the same.

F 1/r2 ; if r 4, F 16
If the leaves are positive, further
separation means they are becoming
more positive, which
implies electrons are leaving the
leaves, attracted to the top plate of the
only that the object only that the object electroscope. This will
is charged.
is uncharged
occur if the object is positively charged.
Vector addition. Since all the charges
are positive, the forces due to charges
2 and 4 point in
opposite directions, making the
magnitude of the net force along the x
axis 2 N. Combine this
with a net force along the y axis of 6 N
11 N
14 N
using the Pythagorean theorem
114.statics.png.png
F/16

I only

II only

III only

I & III only

II & III only

Q = CV and V = Ed and using C =


0
A/d gives E = Q/
0
A
The greatest potential energy would be
where the charge is at the point with the
greatest potential
since UE
= qV (at a point). This is closest to the
positive plate.
116.statics.png

0J

24 J

96 J

576 J

W = Fd = qEd

the rod is much


larger than the
sphere

the rod removes


electron from the
sphere

The alpha particle


will travel upwards
with twice the
velocity of the
proton.

Both particles will


travel upwards
with the same
velocity.

144 J
the charge on the
the electric charge rod causes a
produces a
separation of
magnetic field to
charge in the
attract the sphere sphere

some of the
protons from the
Charged objects attract neutral objects
rod have been
by separating the charges within the
given to the sphere neutral object.
An alpha particle has twice the charge
and four times the mass of a proton.
The alpha particle
The alpha particle Twice the charge
will accelerate
Both particles will will accelerate
means twice the electric force. This,
upwards with twice accelerate
upwards with half combined with four times the mass
the acceleration of upwards with the the acceleration of gives half the
the proton.
same acceleration. the proton.
acceleration.

119.statics.png

120 Statics

Two parallel metal plates carry opposite electrical charges each with a
magnitude of Q. The plates are separated
by a distance d and each plate has an area A. Consider the following:
I. increasing Q
II. increasing d
III. increasing A
Which of the following would have the effect of reducing the potential
difference between the plates?

121 Statics

A positive point charge of +q and a negative point charge of q are separated


by a distance d. What would be
the magnitude of the electric field midway between the two charges?
E

122 Statics

A positive charge +Q located at the origin produces an electric field E0 at


point P (x = +1, y = 0). A negative
charge 2Q is placed at such a point as to produce a net field of zero at point
P. The second charge will be
placed on the
C

123 Statics

124 Statics

125 Statics

126 Statics

127 Statics

128 Statics

129 Statics

130 Statics

131 Statics

A 300 eV electron is aimed midway between two parallel metal plates with a
potential difference of 400 V. The
electron is deflected upwards and strikes the upper plate as shown. What
would be the kinetic energy of the
electron just before striking the metal plate?
Two small hollow metal spheres hung on insulating threads attract one
another as shown. It is known that a
positively charged rod will attract ball A.
I. Ball A has a positive charge
II. Ball B has a negative charge
III. Ball A and Ball B have opposite charges
Which of the above can be correctly concluded about the charge on the
balls?
A 5 106 coulomb electric charge is placed midway between two parallel
metal plates connected to a 9volt
battery. If the electric charge experiences a force of 1.5 104 newtons,
what is the separation of the metal
plates?
A parallelplate capacitor is connected to a resistanceless circuit with a
battery having emf E until the capacitor
is fully charged. The battery is then disconnected from the circuit and the
plates of the capacitor are moved to
half of their original separation using insulated gloves. Let Vnew be the
potential difference across the capacitor
plates when the plates are moved together. Let Vold be the potential
difference across the capacitor plates when
connected to the battery. Vnew/Vold=

A solid, uncharged conducting sphere of radius 3a contains a hollowed


spherical region of radius a. A point
charge +Q is placed at the common center of the spheres. Taking V = 0 as r
approaches infinity, the potential at
position r = 2 a from the center of the spheres is:
Two identical electrical point charges Q, separated by a distance d produce
an electrical force of F on one
another. If the distance is decreased to a distance of 0.40d, what is the
strength of the resulting force?

Q = CV and C = 0A/d which gives V =


Qd/0A
At a point midway between the charges
E = kq/(d/2)2 from each charge. Since
they are opposite
charges, the field vectors between the
E=0
E=kq/d^2
E=2kq/d^2
E=4kq/d^2
E=8kq/d^2
charges point in the same direction.
121.statics.png
For the E field vectors to point in
opposite directions, point P must lie
outside the two charges.
For the magnitudes of E due to each
charge to cancel, Point P must be
x-axis where 0 < x
closer to the smaller
x-axis where x > 1 < 1
x-axis where x < 0 y-axis where y > 0 y-axis where y < 0 charge
122.statics.png
The extra kinetic energy gained by the
electron is W = K = qV, where V is
the potential
difference between the midway line and
the upper plate, which is 200 V. This
makes the
additional kinetic energy 200 eV.
Kinetic energy is a scalar so the total
KE of the electron is now
360 eV
400 eV
500 eV
700 eV
740 eV
300 eV + 200 eV
123.statics.png
I only

II only

III only

I and III

II and III

I only

II only

III only

all of these

none of these

If a positive rod attracts ball A, it is


either negative or neutral. For ball B to
also attract ball A
means ball B can be charged positive
or negative (if ball A is neutral) or
neutral (if ball A is
positive)

6.75 109 m

2.7 104 m

3.7 103 m

0.30 m

3.3 m

F = Eq and E = V/d giving d = qV/F

The most convincing proof of the fact that electrical charge comes in a
fundamentallysized basic amount was
provided by the work of
E
Four electrical charges are arranged on the corners of a 10 cm square as
shown. What would be the direction of the resulting electric field at the center
point P?
B
A proton is released between the two parallel plates of the fully charged
capacitor shown above. What would
be the resulting acceleration of the proton?
E

0 2kQ/3a

kQ/3a

kQ/a

kQ/2a

Since the battery is removed, the


charge remains constant. If the
distance is decreased, the
capacitance will increase (C A/d)
and since Q = CV, the potential
difference must decrease by
the same factor that the distance
4 decreases.
126.statics.png
Since the spherical shell is conducting,
a charge of Q is induced on the inner
surface. This gives
a charge of +Q on the outer surface
since the spherical shell is neutral. As E
= 0 inside the
conducting shell, the potential inside is
constant and the same as on the
surface, which is kQ/r

16F

6.3F

2.5F

0.40F

0.16F

Crookes

Lorentz

Rutherford

Faraday

Millikan

Right

Up

Left

Down

Northeast

F 1/r2; if r 0.4 then F 0.42


The oil drop experiment not only found
the charge of the electron, but also the
fact that charge
was quantized.
The force vectors from each charge
and their relative magnitude are going
from P to the outer dots.

1.0 107 m/s2

7.3 1013 m/s2

9.6 108 m/s2

6.3 1019 m/s2

3.8 1011 m/s2

Using F=ma=qE and E=V/d gives


a=qV/md

129.statics.png

130.statics.png

131.statics.png

132 Statics

A circular parallelplate capacitor is connected to a battery in a circuit. The


capacitor is fully charged before
the battery is disconnected from the circuit. A uniform material of dielectric
constant is inserted between the
plates of the capacitor, effectively filling the space between the plates. Let
Uold be the energy stored by the
capacitor before the dielectric was inserted, while Unew is the energy stored
after the dielectric was inserted.
The ratio of Unew/Uold is:

133 Statics

A solid uncharged conducting sphere has radius 3a contains a hollowed


spherical region of radius 2a. A point
charge +Q is placed at a position a distance a from the common center of the
spheres. What is the magnitude of
the electric field at the position r = 4a from the center of the spheres as
marked in the figure by P?
B

134 Statics

135 Statics

136 Statics

137 Statics

138 Statics

139 Statics

The potential energy of two like charges


A
A positively charged object is brought near but not in contact with the top of
an uncharged gold leaf
electroscope. The experimenter then briefly touches the electroscope with a
finger. The finger is removed,
followed by the removal of the positively charged object. What happens to the
leaves of the electroscope when
a negative charge is now brought near but not in contact with the top of the
electroscope?
B

143 Statics

For the diagram shown below, what is the ratio of the charges q2/q1 where
the diagram shown has a representation of the field lines in the space near
the charges.

1/k

0 kQ/16a^2

1 k

decreases as the
charges are
separated.

3kQ/16a^2

is proportional to
depends on the
the square of the
sign of the charge. relative speed.

kQ/9a^2
is inversely
proportional to the
square of the
separation.
is repulsive.

they remain
uncharged

they move farther


apart

they move closer


together

they remain
positively charged
but unmoved

-5Q

3Q

-4Q

4Q

Right

Down

Left

Southeast

the electric
potential of the
batteries is
neutralized.

the charge on the


radio is zero.

the sound cannot


travel through the
cage.

the electric field of


the radio waves
cannot penetrate
the cage.

VC =VB <VA

VA <VB <VC
If the potential
energy of the
system is positive,
net positive work is
required to bring
any new charge
not part of the
system in from
infinity to its final
resting location.

VC =VB =VA

If the potential
energy of the
system is negative,
net positive work
by an external
agent is required
to take the charges
in the system back
to infinity.

If the potential
energy of the
system is zero, no
negative charges
are in the
configuration.

If the potential
energy of the
system is negative,
net positive work
by an external
agent was required
to assemble the
system of charges.

4F

2F

F/2

F/4

-3/2

-2/3

A portable radio that is playing is placed inside a screen cage as shown.


When inside the cage the radio stops
playing because
D
A spherical conducting shell has a net charge +Q placed on it. Which of the
following is the correct
relationship for the electric potential at the points labeled A, B, and C? Point A
is at the center of the sphere,
point B is at the surface of the shell, a distance R from point A, and point C is
a distance R from point B outside
the sphere. As r goes to infinity, V = 0.
E

142 Statics

141 Statics

1/(k^2)

UE = kq1q2/r
The process described is charging by
induction which gives the electroscope
in this case a net negative charge.
Bringing a negative charge near the top
of the electroscope will cause electrons
they remain
to repel to the leaves. Since the leaves
negatively charged are already negative, this will cause
but unmoved
them to separate further.
The charge density is Q/area which is
Q/4r2 so for the inner surface it is
Qinner/4a2 and for the outer surface it
is Qouter/16a2. For these to be equal
Qouter must be 4Qinner. Because of
the +Q charge inside, there is a charge
of Q induced on the inner surface,
which means the outer surface must
have charge -4Q. Thus the total charge
-3Q
on the shell is -5Q.
The force on a positive charge is in the
direction of the electric field at that
Up
location.
Conductors under electrostatic
conditions will arrange their changes so
no electric field exists D inside (other
than those created by charges placed
inside the empty cavity). Fields from
none of the above external sources will not penetrate into
reasons.
conducting enclosures.
Once inside a uniform sphere of
charge, the electric field is zero.
Since E = 0 the potential does not
change within the sphere
(meaning it is the same value as
the surface). V 1/r outside the
VC <VB =VA
sphere.
If the potential
energy of the
system is zero,
then there is no
electric force
anywhere in space
on any other
charged particle
not part of the
system.

A solid spherical conducting shell has inner radius a and outer radius 2a. At
the center of the shell is located a
point charge +Q. What must the excess charge of the shell be in order for the
charge density on the inner and
outer surfaces of the shell to be exactly equal?
A
A small positive test charge is placed at point P in the region near two
charges. Which of the following arrows
indicates the direction of the force on the positive test charge?
C

Which statement about a system of point charges that are fixed in space is
necessarily true?
A positive point charge exerts a force of magnitude F on a negative point
charge placed a distance x away. If the distance between the two point
charges is halved, what is the magnitude of the new force that the positive
point charge exerts on the negative point charge?
Two uniformly charged nonconducting spheres on insulating bases are
placed on an air table. Sphere A has a charge +3Q coulombs and sphere B
has a charge +Q coulombs. Which of the following correctly illustrates the
magnitude and direction of the electrostatic force between the spheres when
they are released?

140 Statics

Since the battery is disconnected, Q


remains constant. UC = Q2/2C so
Unew/Uold = Cold/Cnew = (0A/d)/
k^2
(0A/d) = 1/
Since the spherical shell is conducting,
a charge of Q is induced on the inner
surface. This gives a charge of +Q on
the outer surface since the spherical
shell is neutral and the field outside the
shell is as if the shell was a point
None of the above. charge.
133.statics.png

VC <VB <VA

0.666

1.5

136.statics.png

137.statics.png

138.statics.png

139.statics.png

F 1/r^2
. Newtons third law requires the forces
be equal and opposite. This eliminates
choices A, B and
C. Since they both positive, the force is
repulsive.
142.statics.png
q2/q1
= lines on q2/lines on q1 and since the
lines point toward q2 and away from q1
they are B
oppositely charged, making the ratio
1 negative.
143.statics.png

144 Statics

In which Region(s) is there a place on the xaxis (aside from infinity) at which
the electric potential is equal to zero?
E

Only in Region II

Only in Region III

In both Regions I
and II

In both Regions I
and III

In both Regions II
and III

145 Statics

In which Region(s) is there a place on the xaxis (aside from infinity) at which
the electric field is equal to zero?
B

Only in Region II

Only in Region III

In both Regions I
and II

In both Regions I
and III

In both Regions II
and III

146 Statics

A parallelplate capacitor is connected to a battery. Without disconnecting the


capacitor, a student pulls the capacitors plates apart so that the plate
separation doubles. As a result of this action, what happens to the voltage
across the capacitor and the energy stored by the capacitor?
D

the voltage
doubles; the
energy stays the
same

the voltage
the voltage halves; doubles; the
the energy doubles energy halves

147 Statics

A person rubs a neutral comb through their hair and the comb becomes
negatively charged. Which of the following is the best explanation for this
phenomenon?

The hair gains


protons from the
comb.

The hair gains


protons from the
comb while giving
electrons to the
comb.

148 Statics

A charge of +Q is located on the xaxis at x = 1 meter and a charge of 2Q


is held at x = +1 meter, as shown in the diagram above. At what position on
the xaxis will a test charge of +q experience a zero net electrostatic force?

(3 + 8) m

1/3 m

149 Statics

The two plates of a parallel-plate capacitor are a distance d apart and are
mounted on insulating supports. A
battery is connected across the capacitor to charge it and is then
disconnected. The distance between the
insulated plates is then increased to 2d. If fringing of the field is still negligible,
which of the following
quantities is doubled?
D

150 Statics

151 Statics

152 Statics

Four point charges are each brought from infinity into a region of empty space
and are "attached in place" into a
square arrangement of side length a as shown below. The location marked P
is at the center of the square and
has no charge associated with it. Which is a true statement about the
configuration of charges?
E
Two point objects each carrying charge 10Q are separated by a distance d.
The force between them is F. If half
the charge on one object is transferred to the other object while at the same
time the distance between them is
doubled, what is the new force between the two objects?
A
2.A parallel plate capacitor is charged to a voltage V. To double the energy
stored on the capacitor, what would
the voltage between the plates have to become?
C

The capacitance of The total charge


the capacitor
on the capacitor

The net electric


field at point P is
directed to the left

The net electric


field at point P is
directed to the
right.

the voltage stays


the same; the
energy halves

The points where V = 0 must lie closer


to the smaller charge. Unlike electric
field vectors which
also require the individual vectors point
in opposite directions, there are a locus
of points (in this
case in a ring surrounding the Q
charge) where V = 0 as the two
charges are opposite in sign
and V is a scalar.
144,145.statics.png
For E to be zero, the electric field
vectors from each charge must point in
opposite directions and
must therefore occur at a point outside
the charges. For the electric field
vectors from each
charge to be equal in magnitude so
they can cancel, it must also occur at a
point closer to the
smaller charge to make up for the
weaker field.
144,145.statics.png
Since the battery remains
connected, V remains constant. C
decreases as d increases (C 1/d)
and UC = CV
D
2

the voltage stays


the same; the
energy doubles
The comb loses
protons to the
The comb loses
persons hand
The hair loses
protons to the
while also gaining
electrons to the
persons hand
electrons from the Only electrons are transferred in static
comb.
holding the comb. hair.
charging processes.
Any charge will experience a net force
of zero where the electric field is zero.
This must be
where the fields from each charge point
in opposite directions and also closer to
the smaller
charge, which is to the left of the +Q
charge (the answer will be to the left of
1 m). Let the
distances to the +Q and the 2Q
charge be x and (X + 2), respectively.
This gives E1
= E2 and
kQ/x2
= k(2Q)/(x + 2)2
Solve for x and add the extra 1 m to the
0
0.333 (3 + 8) m
origin
148.statics.png
Since the battery is removed, the
charge remains constant. If the
distance is increased, the
capacitance will decrease (C A/d)
The surface
density of the
The intensity of the and since Q = CV, the potential
charge on the
electric field
difference must increase by
plates of the
The energy stored between the plates the same factor that the distance
capacitor
in the capacitor
of the capacitor
increases and Uc= QV
UE = kq1q2/r where we are summing
each pair of charges. Computing UE
gives a total
potential energy that is negative. This
means negative work must have been
done by an outside
The total force on
agent to keep the chargesfrom colliding
each charge from The electric
The outside agent into each other and stop them in their
the other three in potential at the
that assembled the respective
the configuration is point P is (2Q)/
charges did
locations. The net field and the potential
zero.
[(pi)Ea)]
negative net work at the center is zero due to symmetry
150.statics.png

0.19 F

0.25 F

0.75 F

4.0 F

no change in F

0.25 V

0.50 V

1.4 V

2.0 V

4.0 V

F q1q2/r2; the original force F


100Q2/d2. The new charges are
15Q and 5Q making the new force
F 75Q2/(2d)2= 19Q2/d
Assuming C remains constant and UC
= CV2, for UC to double V must
increase by 2

153 Statics

154 Statics

155 Statics

156 Statics

157 Statics

158 Statics

Two identical spheres carry identical electric charges. If the spheres are set a
distance d apart they repel one another with a force F. A third sphere,
identical to the other two but initially uncharged is then touched to one sphere
and then to the other before being removed. What would be the resulting
force between the original two spheres?
Two parallel metal plates 0.04 meters apart are connected to a 1.5 volt
battery. When fully charged, each metal
plate has a charge of magnitude 9.0 x104coulombs. What is the capacitance
of the two plates?
An alpha particle is accelerated to a velocity v in a particle accelerator by a
potential difference of 1200 V.
Which of the following potential differences would be needed to give the
alpha particle twice the velocity?
An electrical charge Q is placed at one vertex of an equilateral triangle. When
an identical charge is placed at
another vertex, each charge feels a force of 15 N. When a third charge
identical to the first two, is placed at the
third vertex, what would be the magnitude of the force on each charge?
Two conducting spheres with the same charge Q are separated by an infinite
distance. Sphere A has a radius of
10 cm while sphere B has a radius of 20 cm. At what distance from the
centers of the spheres would the
magnitude of the electric field be the same?
A large conducting sphere labeled X contains an electrical charge Q. Sphere
X is connected by a metal wire to
a small uncharged conducting sphere labeled Y. The wire is then removed.
How does the electrical field (Ey) at the surface of sphere Y compare to the
electrical field (Ex) at the surface of sphere X?

5/8 F

3/8 F

When one sphere is touched, the


charged divides equally ( Q
each). When this sphere is then
touched to the second sphere, the
net charge (3/2 Q) is divided
equally ( Q each). Since F q1q2,
the original force is proportional to
Q2 and the new force is then
proportional to ( Q)(Q) = 3/8 Q

1.5 102 F

1.2 103 F

3.0 104 F

6.0 104 F

9.0 104 F

Q = CV

7200 V

4800 V

4100 V

2400 V

1700 V

W = K = qV so V v^2 and for


v to double, V must increase by 4
Adding the force vectors shown (each
15 N) with x components that cancel
and y-components that equal 15 N cos
30 gives F= 2 15 N cos 30 = 26 N

15 N

26 N

30 N

42 N

45 N

15 cm from A and
15 cm from B

20 cm from A and
34 cm from B

20 cm from A and
40 cm from B

30 cm from A and
40 cm from B

40 cm from A and
40 cm from B

Ey= 0

Ey= Ex

Ey< Ex

Ey> Ex

Ex= 0

0.17 V

3.7 V

5.9 V

14 V

27 V

Once outside the spheres, they act as


point charges and their difference in
size is irrelevant
157.Statics.png
When connected, the potentials
become equal. This gives kQX/rX =
kQY/rY and since E = kQ/r2, dividing
the potentials by their respective radii
gives kQX/(rX)^2< kQY/(rY)^2
1.5 10^12 excess electrons is a
charge of magnitude (1.5 10^12)
(1.6 10^19) = 2.4 10^7C.Use Q =
CV

points K and N

points L and M

points K and L

points M and N

point K only

The field is zero everywhere inside a


metal sphere.

(A) Ed

(B) E/d

(C) EA/d

(D) Ed/A

(E) EA

(A) 1.4 10^5 N

(B) 1.7 10^5 N

(C) 4.2 10^5 N

(D) 4.6 10^5 N

(A) 1440 N/C

(B) 720 N/C

(C) 360 N/C

(D) 180 N/C

V = Ed
The force on Q2 from Q1 points
downward and the force from Q3 points
at right angles to the left. Compute
each force using F = kq1q2/r2and use
(E) 1.47 10^4 N the Pythagorean theorem.
162.statics.png
The field at the center due to Q1 and
Q3 cancels. The only contribution to the
field then is that due to Q2. E =
(E) 90 N/C
kQ/r^2where r2= 0.3^2+ 0.4^2
162.statics.png

(B) the opposite


sign but 1/2 the
magnitude as
originally on
sphere A

(C) the opposite


sign but 1/4 the
magnitude as
originally on
sphere A

(D) the same sign


but 1/2 the
magnitude as
originally on
sphere C

(E) neutrally
charged

161 Statics

9.What voltage would be required across a 8.9 nF capacitor to accumulate


1.5 10^12
excess electrons on one plate of the capacitor?
E
A hollow metal sphere is uniformly charged with positive charge. Points K and
L are inside the sphere and
points M and N are outside the sphere as shown in the diagram. At which
point would the field be the smallest?
C
Two circular metal plates, each having an area A are placed to one another a
distance d apart. When a potential
difference is applied across the two plates, an electric field E is measured
halfway between the two plates at
their centers. What is the magnitude of the potential difference between the
two plates?
A

162 Statics

Three electric charges (Q1, Q2, and Q3) are arranged at three corners of a
rectangle as shown in the diagram and
each has a charge of 40 nC. What is the magnitude of the net force on Q2?

159 Statics

160 Statics

163 Statics

164 Statics

What would be the magnitude of the total electric field at center point X?
Which of the following graphs would best represent the electric field of a
hollow Van de Graff sphere as a
function of distance from its center when it is charged to a potential of
400,000 volts?

165 Statics

Three metal spheres A, B, and C are mounted on insulating stands. The


spheres are touching one another, as
shown in the diagram below. A strong positively charged object is brought
near sphere A and a strong negative
charge is brought near sphere C. While the charged objects remain near
spheres A and C, sphere B is removed
by means of its insulating stand. After the charged objects are removed,
sphere B is first touched to sphere A
and then to sphere C. The resulting charge on B would be of what relative
amount and sign?

(A) the same sign


but 1/2 the
magnitude as
originally on
sphere A

166 Statics

A charge is uniformly distributed through a volume of radius a. Which of the


graphs below best represents the
magnitude of the electric field as a function of distance from the center of the
sphere?

160.Statics.png

E inside = 0 and outside E 1/r^2 164.statics.png


Initially, when B is removed, A and C
are equally and oppositely charged and
B is neutral.
Touching B to A gives B the charge
of A (split equally). The charge on B is
then that of C
and oppositely charged. When B and C
touch, the total charge between them is
the charge of
C and the same sign as C. Each sphere
then has of the charge of C after
contact is made. This
makes the end result that the charge on
sphere B is the original charge of A
and the same sign
as sphere C, which is opposite that of A 165.statics.png
Advanced question (not exactly in the B
curriculum, but interesting). Like gravity
inside a
uniform sphere of mass, the field is
directly proportional to r when inside the
sphere (and
proportional to 1/r^2 when outside)
166.statics.png

167 Statics

168 Statics

Four point charges are placed at the corners of a square with diagonal 2a as
shown in the diagram. What is the
total electric field at the center of the square?
B
A free electron and a free proton are placed between two oppositely charged
parallel plates. Both are closer to
the positive plate than the negative plate. See the diagram below. Which of
the following statements is true? I. The force on the proton is greater than the
force on the electron.
II. The potential energy of the proton is greater than that of the electron.
III. The potential energy of the proton and the electron is the same.
B

(B) kq/a2 at an
angle 45 below
the x axis.

(C) 3kq/a2 at an
angle 45 above
the x axis.

(D) 3kq/a2 at an
angle 45 below
the +x axis.

(A) I only

(B) II only

(C) III only

(D) I & II only

(B) The charge q


(A) +Q on the inner is spread uniformly
surface, q Q on between the inner
the outer surface. and outer surface.

(C) Q on the
inner surface, q
+ Q on the outer
surface.

(D) Q on the
inner surface, q
on the outer
surface.

(C) kQ/R

(D) k(Q q)/R

The field due to the two +3q charges


cancel. The q in the upper right
counters q from the
lower left, leaving the net contribution to
the field a q from the lower left.
167.statics.png
With equal charge, the forces are the
same. The potential energy of the
charges is equal in magnitude, but
positive for the proton and negative for
the electron. For scalars, positive
numbers are higher than negative
(E) I & III only
numbers.
168.statics.png
The charge Q in the middle will induce
a charge Q on the inner surface of the
shell. For the net charge of the shell to
(E) Zero charge on be q, the outer surface must have the
the inner surface, rest of the charge such that qouter +
q on the outer
qinner = q so qouter= q qinner= q
surface.
(Q) = Q q
169.statics.png
The potential inside the shell is the
same as the potential at the surface of
the shell since E = 0 inside the shell. V
(E) k(Q q)/b
= kqouter/b
(E) 9kq/a2 at an
angle 45 above
the +x axis.

169 Statics

How is the charge q distributed after it has reached equilibrium?

170 Statics

What is the electrostatic potential at a distance R from the center of the shell,
where b < R < a?
E

(A) 0

171 Statics

Conducting sphere X is initially uncharged. Conducting sphere Y has twice


the diameter of sphere X and
initially has charge q. If the spheres are connected by a long thin wire, which
of the following is true once
equilibrium has been reached?

(A) Sphere Y has (B) Spheres X and .(C) Sphere Y has .(D) Sphere Y has .(E) Spheres X and
half the potential of Y have the same twice the potential half the charge of Y have the same Charges flow when there is a difference
sphere X.
potential.
of sphere X
sphere X
charge
in potential.

172 Statics

Four positive charges are fixed at the corners of a square, as shown above.
Three of the charges have magnitude Q, and the fourth charge has a
magnitude 2Q. Point P is at the center of the square at a distance r from each
charge. What is the electric potential at point P?
E

(A) Zero

(B) kQ/r r

173 Statics

What is the magnitude of the electric field at point P ?

(A) Zero

(B) kQ/r2

(A) The
capacitance

(B) The stored


electrostatic
energy

(A) It is greater.

(A) 9U

(A) Mg

(B) Mg + kq/d

(C) Mg kq/d

(D) Mg + kq2/d2

(E) Mg kq2/d2

A small positively charged sphere is lowered by a nonconducting thread into


a grounded metal cup without
touching the inside surface of the cup, as shown above. The grounding wire
attached to the outside surface is
disconnected and the charged sphere is then removed from the cup. Which of
the following best describes the
subsequent distribution of excess charge on the surface of the cup?
B
A helium nucleus (charge +2q and mass 4m) and a lithium nucleus (charge
+3q and mass 7m) are accelerated
through the same electric potential difference, V0. What is the ratio of their
resultant kinetic energies, K Lithium/ K Helium?
E

(A) 2/3

(B) 6/7

(C) 1

(D) 7/6

(E) 3/2

(E)
Noregiononthexaxis

K=qVsoK1/K2=q1/q2
179.statics.png
The field is zero where the fields from
each charge point in opposite directions
and also closer to A
the smaller charge, which is to the left
of the Q charge
181.static.png

40 V/0.25 m
perpendicular to
the plane of the
page

E=V/d
E fields point from high potential to low
potential, perpendicular to the
equipotential lines.

174 Statics

175 Statics

176 Statics

177 Statics

178 Statics

179 Statics

180 Statics
181 Statics

182 Statics

If the separation between the plates of an isolated charged parallel-plate


capacitor is increased slightly, which of
the following also increases?
The two charged metal spheres X and Y shown above are far apart, and each
is isolated from all other charges.
The radius of sphere X is greater than that of sphere Y, and the magnitudes
of the electric fields just outside their
surfaces are the same. How does the charge on sphere X compare with that
on sphere Y?
Two negative point charges are a distance x apart and have potential energy
U. If the distance between the point
charges increases to 3x, what is their new potential energy?
Sphere X of mass M and charge +q hangs from a string as shown above.
Sphere Y has an equal charge +q and is
fixed in place a distance d directly below sphere X. If sphere X is in
equilibrium, the tension in the string is most
nearly

(A) kq/a2 at an
angle 45 above
the +x axis.

A point charge Q is located at the origin, while a second point charge +2Q is
located at x = d on the x-axis, as shown above. A point on the x-axis where
the net electric field is zero is located in which of the following regions?
A
Which of the following expressions best represents the magnitude of the
electric field at point P ?
B

The direction of the electric field at point P is most nearly

(B) kQ/a

(C) 2kQ/r

(D) 4kQ/r

(E) 5kQ/r

(B) It is less.

V = kQ/r
172.statics.png
The electric field cancels from
symmetry all but +Q remaining in the
(C) 2kQ/r2
(D) 4kQ/r2
(E) 5kQ/r2
upper right corner and E = B kQ/r2
173.statics.png
Since the plates are insulated, the
charge remains constant. If the
distance is increased, the B
capacitance will decrease (C A/d)
(E) The magnitude and since Q = CV, the potential
(C) The force of
(D) The magnitude of the electric field difference must increase by
attraction between of the charge on
in the region
the same factor that the distance
the plates
each plate
between the plates increases. UC = 12 QV
174.statics.png
(E) It cannot be
D) It cannot be
determined without
determined without knowing the actual
knowing the actual value of the
If the E fields are the same, that means
radii of the
electric field just
kQX/rX2 = kQY/rY2, or QX/QY =
(C) It is the same. spheres.
outside the sphere rX2/rY2

(B) 3U

(C) U

(D) 1/3 U

(E) 1/9 U

UE1/r

F=0sowehaveT+k(q)(q)/d2
Mg=0givingT=Mgkq2/d2
177.statics.png
When lowered inside, the charged
sphere induces a negative charge on
(E) Negative
the inner surface of the B cup. The
(A) Negative
(B) Negative
(C) Positive charge (D) Positive charge charge resides on outer surface remains neutral since it is
charge resides on charge resides on resides on the
resides on the
the inside surface, grounded. When the grounding wire is
the inside surface, the outside
inside surface, and outside surface,
and positive
removed, the cup has a net negative
and no charge
surface, and no
no charge resides and no charge
charge resides on charge, which when the sphere is
resides on the
charge resides on on the outside
resides on the
the outside
removed, will move to the
outside surface.
the inside surface. surface.
inside surface.
surface.
outer surface of the cup.
178.statics.png

(A)-<x<0

(B)0< x<d/2

(C)d/2<x<d

(D)d<x<

10 V/0.14 m

10 V/0.04 m

25 V/0.14 m

25 V/0.04 m

toward the right

toward the bottom toward the top of


of the page
the page

toward the left

182.183.static.png

182.183.static.png

183 Statics

The electric field E0 and potential V0 at the surface of each drop is given by
which of the following?

00

kQ/R2 kQ/R^2

kQ/R^2 kQ/R

0 kQ/R

kQ/R 0

184 Statics

If two droplets happen to c2ombine into a single larger droplet, the new
potential V at the surface of the larger droplet is most nearly equal to

3V

2V

2/3square root2 V

3square root 2 V

2ke2/a

3ke2/2a

ke2/2a

3ke2/2a

3ke2/a

zero

450 V/m

900 V/m

4,500 V/m

90,000 V/m

I and II

I and IV

II and III

II and IV

III and IV

1V

2V

3V

4V

6V

increase the
ammeter reading

decrease the
ammeter reading

increase the
voltmeter reading

decrease the
voltmeter reading

produce no
change in either
meter reading

E = kQ/r2
The resistances are as follows: I: 2 ,
II: 4 , III: 1 , IV: 2
1.circuits.png
The total resistance of the 3 and 6
in parallel is 2 making the total circuit
resistance 6 B
andthetotalcurrentE/R=1A.
This1Awilldivideintheratioof2:
1throughthe3and6 respectively so
the 3 resistor receives 2/3 A making
the potential difference IR = (2/3 A)(3
)
= 2 V.
2.circuits.png
Adding resistors in parallel decreases
the total circuit resistance, this
increasing the total current A in the
circuit.
3.circuit.png

R = L/A. Greatest resistance is the


longest, narrowest resistor.

4/9.

2/3.

In parallel V1 = V2. Q1 = C1V1 and Q2


= C2V2 so Q1/Q2 = C1/C2 = 1.5
5.circuits.png
For steady power dissipation, the circuit
must allow current to slow indefinitely.
For the greatest
power, the total resistance should be
the smallest value. These criteria are
met with the resistors
in parallel.
6-7.circuits.png
To retain energy, there must be a
capacitor that will not discharge through
a resistor. Capacitors
in circuits C and E will discharge
through the resistors in parallel with
them.
6-7.circuits.png

0.025 V

.67 V

2.5 V

6.25 V

40 V

P = IE

IRt

I^2Rt

IR^2t

IRt^2

IR/t

W = Pt = I2Rt
The resistance of the two 2 resistors
in parallel is 1 . Added to the 2
resistor in series with
the pair gives 3

1 circuits

Two protons and an electron are assembled along a line, as shown above.
The distance between the electron and each proton is a. What is the work
done by an external force in assembling this configuration of charges?
B
A conducting sphere with a radius of 0.10 meter has 1.0 109 coulomb of
charge deposited on it. The electric field just outside the surface of the sphere
is
C
Which two arrangements of resistors shown above have the same resistance
between the terminals?
B

2 circuits

In the circuit shown above, what is the value of the potential difference
between points X and Y if the 6volt battery has no internal resistance?

185 Statics

186 statics

5 circuits

A lamp, a voltmeter V, an ammeter A, and a battery with zero internal


resistance are connected as shown above. Connecting another lamp in
parallel with the first lamp as shown by the dashed lines would
The five resistors shown below have the lengths and crosssectional areas
indicated and are made of material with the same resistivity. Which has the
greatest resistance?
Two capacitors are connected in parallel as shown above. A voltage V is
applied to the pair. What is the ratio of charge stored on C1 to the charge
stored on C2, when C1 = 1.5C2 ?

6 circuits

The five incomplete circuits below are composed of resistors R, all of equal
resistance, and capacitors C, all of
equal capacitance. A battery that can be used to complete any of the circuits
is available. Into which circuit should the battery be connected to obtain the
greatest steady power dissipation?

3 circuits

4 circuits

7 circuits

8 circuits

9 circuits

The five incomplete circuits below are composed of resistors R, all of equal
resistance, and capacitors C, all of
equal capacitance. A battery that can be used to complete any of the circuits
is available. Which circuit will retain stored energy if the battery is connected
to it and then disconnected?
The circuit shown above left is made up of a variable resistor and a battery
with negligible internal resistance. A
graph of the power P dissipated in the resistor as a function of the current I
supplied by the battery is given
above right. What is the emf of the battery?
An immersion heater of resistance R converts electrical energy into thermal
energy that is transferred to the
liquid in which the heater is immersed. If the current in the heater is I, the
thermal energy transferred to the
liquid in time t is

11 circuits

The total equivalent resistance between points X and Y in the circuit shown
above is
The five resistors shown below have the lengths and crosssectional areas
indicated and are made of material
with the same resistivity. Which resistor has the least resistance?

12 circuits

In the circuit shown above, the value of r for which the current I is 0.5 ampere
is
E

10 circuits

1 3/2.

9/4.

1
Increasing the
pressure applied to
the material
beyond a certain
threshold pressure

10

20

Cooling the
material below a
certain threshold
temperature

Stretching the
material to a wire
of sufficiently small
diameter

Placing the
material in a
sufficiently large
magnetic field

Conservation of
energy

Ampere's law

Faraday's law

Ohm's law

13 circuits

Which of the following will cause the electrical resistance of certain materials
known as superconductors to
suddenly decrease to essentially zero?

0
Increasing the
voltage applied to
the material
beyond a certain
threshold voltage

14 circuits

Kirchhoffs loop rule for circuit analysis is an expression of which of the


following?

Conservation of
charge

Forasphere,E=kQ/r2andV=kQ/r
184.185.static.png
Combining two droplets doubles the
charge. The volume is doubled, which
means the radius is multiplied by 3
square root 2
184.185.static.png
The work to assemble the charges is
the potential energy of the system,
which is the sum of the potential
energies of each pair of charges UE =
ke2/a ke2/a + ke2/2a
186.static.png

4.circuits.png

10.circuits.png

R = L/A. Least resistance is the


widest, shortest resistor
11.circuits.png
The resistance of the two resistors in
parallel is r/2. The total circuit
resistance is then 10 + 12 E r, which
is equivalent to E/I = (10 V)/(0.5 A) = 20
= 10 + r/2
12.circuits.png
Resistance varies directly with
temperature. Superconductors have a
resistance that quickly goes C to zero
once the temperature lowers beyond a
certain threshold
The loop rule involves the potential and
energy supplied by the battery and its
use around a
circuit loop.

15 circuits

The equivalent capacitance for this network is most nearly

16 circuits

The charge stored in the 5microfarad capacitor is most nearly

17 circuits

What is the emf of the battery?

18 circuits
19 circuits

What is the potential difference across the terminals X and Y of the battery?
What power is dissipated by the 4ohm internal resistance of the battery?

C
A

20 circuits

In the diagrams above, resistors R1 and R2 are shown in two different


connections to the same source of emf
that has no internal resistance. How does the power dissipated by the
resistors in these two cases compare?

21 circuits

The product (2 amperes 2 volts 2 seconds) is equal to

22 circuits

The electrical resistance of the part of the circuit shown between point X and
point Y is

23 circuits

24 circuits

25 circuits

26 circuits

When there is a steady current in the circuit, the amount of charge passing a
point per unit of time is
A certain coffeepot draws 4.0 A of current when it is operated on 120 V
household lines. If electrical energy costs 10 cents per kilowatthour, how
much does it cost to operate the coffeepot for 2 hours?

A battery having emf E and internal resistance r is connected to a load


consisting of two parallel resistors each having resistance R. At what value of
R will the power dissipated in the load be a maximum?
D
Two concentric circular loops of radii b and 2b, made of the same type of
wire, lie in the plane of the page, as shown above. The total resistance of the
wire loop of radius b is R. What is the resistance of the wire loop of radius
2b?
D

28 circuits

The total capacitance of several capacitors in parallel is the sum of the


individual capacitances for which of the following reasons?
A
A wire of length L and radius r has a resistance R. What is the resistance of a
second wire made from the same material that has a length L/2 and a radius
r/2?
B

29 circuits

The operating efficiency of a 0.5 A, 120 V electric motor that lifts a 9 kg mass
against gravity at an average velocity of 0.5 m/s is most nearly
E

30 circuits

What is the current I1?

31 circuits

How do the currents I1, I2, and 13 compare?


A
When lighted, a 100watt light bulb operating on a 110volt household circuit
has a resistance closest to
E

27 circuits

32 circuits

The capacitance of the 4 F and 2F in


parallel is 6 F. Combined with the 3F
in series gives 2
F for the right branch. Added to the 5
10/7 F
3/2 F
7/3 F
7 F
14 F
F in parallel gives a total of 7 F
Since the 5 F capacitor is in parallel
with the battery, the potential difference
across it is 100 V.
360 C
500 C
710 C
1,100 C
1,800 C
Q = CV
Total circuit resistance (including
internal resistance) = 40 ; total current
1.2 V
6.0 V
10.8 V
12.0 V
13.2 V
= 0.3 A. E = IR
VXY = E Ir where r is the internal
1.2 V
6.0 V
10.8 V
12.0 V
13.2 V
resistance
0.36 W
1.2 W
3.2 W
3.6 W
4.8 W
P = I2r
With more current drawn from the
battery for the parallel connection, more
It is different for
It is different for
power is dissipated in
each connection, each connection, this connection. While the resistors in
but one must know but one must know series share the voltage of the battery,
It is greater for the
the values of R1
the value of to
the resistors in
It is greater for the parallel
It is the same for
and R2 to know
know which is
parallel have the full potential difference
series connection. connection.
both connections. which is greater.
greater.
of the battery across them.
Amperes = I (current); Volts = V
(potential difference); Seconds = t
8 coulombs
8 newtons
8 joules
8 calories
8 newtonamperes (time): IVt = energy
Resistance of the 1 and 3 in series
= 4 . This, in parallel with the 2
resistor gives (2 4)
/(2 + 4) = 8/6 . Also notice the
equivalent resistance must be less than
2 (the 2 resistor is
4/3
2
2.75
4
6
in parallel and the total resistance in
The upper branch, with twice the
the same
greater in the 1 greater in the 2
greater in the 1 resistance of the lower branch, will
everywhere in the resistor than in the resistor than in the greater at point X resistor than in the have the current of the
circuit
2 resistor
3 resistor
than at point Y
3 resistor
lower branch.

2.4 cents

4.8 cents

0 r/2

15.circuits.png

17.circuits.png
17.circuits.png
17.circuits.png

20.circuits.png

22.circuits.png

22.circuits.png

8.0 cents

9.6 cents

16 cents

2r

4r

Power = IV = 480 W = 0.48 kW. Energy


= Pt = (0.48 kW)(2 hours) = 0.96 kW-h 24.circuits.png
Total circuit resistance of the load =
R/2. Total circuit resistance including
the internal resistance D = r + R/2. The
current is then E/(r + R/2) and the total
power dissipated in the load is P =
I2Rload
= (E2R/2)/(r + R/2)2. Using calculus
max/min methods or plotting this on a
graph gives the value
of R for which this equation is
maximized of R = 2r. This max/min
problem is not part of the B curriculum
but you should be able to set up the
equation to be maximized.
25.circuits.png

R/4
The charge on
each capacitor
depends on its
capacitance, but
the potential
difference across
each is the same.

R/2
The charge is the
same on each
capacitor, but the
potential difference
across each
capacitor depends
on its capacitance.

2R

4R

The larger loop, with twice the radius,


has twice the circumference (length)
and R = L/A

Equivalent
capacitance is
always greater
than the largest
capacitance.

Capacitors in a
circuit always
combine like
resistors in series.

The parallel
combination
increases the
effective
separation of the
plates.

By process of elimination, A is the only


possible true statement.

4R

2R

R/2

R/4

7%

15.circuits.png

13%

25%

53%

0.8mA

1.0mA

2.0mA

3.0mA

6.0mA

I1 > I2 > I3

I1 > I3 > I2

I2 > I1 > I3

I3 > I1 > I2

I3 > I2 > I1

102

101

10

100

R=L/A. IfL2,
R2andisr2thenA4andR4makingtheneteffectR24
ThemotorusesP=IV=60WofpowerbutonlydeliversP=Fv=mgv=45Wofpower.
The E efficiency is what you get
75% what you are paying for = 45/60
Resistance of the 2000 and 6000
in parallel = 1500 , adding the 2500
in series gives a total circuit resistance
of 4000 . Itotal = I1 = E/Rtotal
30.circuits.png
I1 is the main branch current and is the
largest. It will split into I2 and I3and
since I2 moves through the smaller
resistor, it will be larger than I3.
30.circuits.png
P=V2/R

32.circuits.png

33
34
35
36
37
38
39
40
41
42
43

circuits
circuits
circuits
circuits
circuits
circuits
circuits
circuits
circuits
circuits
circuits

44 circuits

In the circuit shown above, what is the resistance R?

12

18

The current through R is found using


the junction rule at the top junction,
where 1 A + 2 A enter giving I = 3 A.
Now utilize Kirchhoffs loop rule through
the left or right loops: (left side) + 16
V (1 A)(4 ) (3 A)R = 0 giving R = 4

33.circuits.png

When the switch is closed, the circuit


behaves as if the capacitor were just a
wire and all the
potential of the battery is across the
resistor. As the capacitor charges, the
voltage changes over
to the capacitor over time, eventually
making the current (and the potential
difference across the
resistor) zero and the potential
difference across the capacitor equal to
the emf of the battery
44.circuits.png

Which graph best represents the voltage across the capacitor versus time?
Three 6microfarad capacitors are connected in series with a 6volt battery.
The equivalent capacitance of the set of capacitors is

0.5 F

2 F

3 F

9 F

18 F

4 J

6 J

12 J

18 J

36 J

l only

d only

l and only

d and only

l, d, and

P = I^2R and R = L/A giving P


L/d^2

P/9

P/3

3P

6P

P = I^2 R

49 circuits

The energy stored in each capacitor is


The power dissipated in a wire carrying a constant electric current I may be
written as a function of the length l
of the wire, the diameter d of the wire, and the resistivity of the material in
the wire. In this expression, the
power dissipated is directly proportional to which of the following?
A wire of resistance R dissipates power P when a current I passes through it.
The wire is replaced by another
wire with resistance 3R. The power dissipated by the new wire when the
same current passes through it is
Two resistors of the same length, both made of the same material, are
connected in a series to a battery as
shown above. Resistor II has a greater cross. sectional area than resistor I.
Which of the following quantities
has the same value for each resistor?

in series (1/Ct)= E(1/c)


There are several ways to do this
problem. We can find the total energy
stored and divide it into
the three capacitors: UC = CV2=
(2 F)(6 V)2= 36 J 3 = 12 J each

Potential
Electric field
difference between strength within the
the two ends
resistor
Resistance

Current per unit


area

Current

50 circuits

Below is a system of six 2microfarad capacitors. The equivalent capacitance


of the system of capacitors is
C

2/3F

4/3 F

3 F

6 F

12 F

51 circuits

What potential difference must be applied between points X and Y so that the
charge on each plate of each
capacitor will have magnitude 6 microcoulombs?
C

1.5 V

3V

6V

9V

18 V

52 circuits

In the circuit above, the emf's and the resistances have the values shown.
The current I in the circuit is 2 amperes.
The resistance R is

53 circuits

The potential difference between points X and Y is

1.2 V

6.0 V

8.4 V

10.8 V

12.2 V

54 circuits

How much energy is dissipated by the 1.5ohm resistor in 60 seconds?

6J

180 J

360 J

720 J

1440 J

55 circuits

Immediately after the switch is closed, the current supplied by the battery is

V/(R1+ R2)

V/R1

V/R2

V(R1+R2)/R1R2

zero

56 circuits

A long time after the switch has been closed, the current supplied by the
battery is

V/(R1+R2)

V/R1

V/R2

V(R1+R2)/R1R2

zero

45 circuits

46 circuits

47 circuits

48 circuits

Since these resistors are in series, they


must have the same current.
Each branch, with two capacitors in
series, has an equivalent capacitance
of 2 F 2 = 1 F.
The three branches in parallel have an
equivalent capacitance of 1 F + 1 F +
1 F = 3 F
For each capacitor to have 6 C, each
branch will have 6 C since the two
capacitors in series in
each branch has the same charge. The
total charge for the three branches is
then 18 C. Q = CV
gives 18 C = (3 F)V
Utilizing Kirchhoffs loop rule starting at
the upper left and moving clockwise:
(2 A)(0.3 ) +
12 V 6 V (2 A)(0.2 ) (2A)(R)
(2A)(1.5 ) = 0
Summing the potential differences: 6
V (2 A)(0.2 ) (2A)(1 ) = 8.4 V
Energy = Pt = I2
Rt
When the switch is closed, the circuit
behaves as if the capacitor were just a
wire, shorting out the resistor on the
right.
When the capacitor is fully charged, the
branch with the capacitor is closed to
current, effectively removing it from the
circuit for current analysis.

49.circuits.png

50.circuits.png

51.circuits.png

52.circuits.png
53.circuits.png
54.circuits.png

55.circuits.png

56.circuits.png

57 circuits

58 circuits

59 circuits

A 30ohm resistor and a 60ohm resistor are connected as shown above to a


battery of emf 20 volts and internal resistance r. The current in the circuit is
0.8 ampere. What is the value of r?
C
A variable resistor is connected across a constant voltage source. Which of
the following graphs represents the power P dissipated by the resistor as a
function of its resistance R?
A

0.22

4.5

16

70

1.5

V0+ QC I2R = 0

V0 Q/C IR = 0

V02-Q2/2C-I2R=0 V0-CI-I2R=0

Q/C IR = 0

61 circuits

If the ammeter in the circuit above reads zero, what is the resistance R ?
E
A resistor R and a capacitor C are connected in series to a battery of terminal
voltage V0. Which following equations relating the current I in the circuit and
the charge Q on the capacitor describes this circuit?
B
Which of the following combinations of 4 resistors would dissipate 24 W
when connected to a 12 Volt
battery?
E

62 circuits

A narrow beam of protons produces a current of 1.6 103 A. There are 109
protons in each meter along the beam. Of the following, which is best
estimate of the average speed of the protons in the beam?
D

10^15 m/s

10^12 m/s

10^7 m/s

10^7 m/s

10^12 m/s

63 circuits

The equivalent capacitance between points X and Z is

1.0 F

2.0 F

4.5 F

6.0 F

9.0 F

64 circuits

Three identical capacitors, each of capacitance 3.0 F, are connected in a


circuit with a 12 V battery as shown above.The potential difference between
points Y and Z is

Zero

3V

4V

8V

9V

Bulb 1:Goes out


Bulb 2:Gets
brighter

Bulb 1:Gets
brighter Bulb 2:
Goes out

Bulb 1:Gets
brighter Bulb 2:
Gets slightly
dimmer

Bulb 1:Gets
slightly dimmer
Bulb 2:Gets
brighter

Bulb 1:Nothing
Bulb 2:Goes out

0.1 F
0.1

1 F
10

2/3 F
12

12 F
120

1/6 F
1440

60 circuits

66 circuits
67 circuits

The circuit in the figure above contains two identical lightbulbs in series with a
battery. At first both bulbs glow with equal brightness. When switch S is
closed, which of the following occurs to the bulbs?
B
Three 1/2 F capacitors are connected in series as shown in the diagram
above. The capacitance of the combination is
E
A hair dryer is rated as 1200 W, 120 V. Its effective internal resistance is
C

68 circuits

When the switch S is open in the circuit shown above, the reading on the
ammeter A is 2.0 A. When the switch is closed, the reading on the ammeter is B

doubled

69 circuits

Two conducting cylindrical wires are made out of the same material. Wire X
has twice the length and twice the diameter of wire Y. What is the ratio Rx/Ry
of their resistances?
B

"1/4"

65 circuits

71 circuits

You are given three 1.0 resistors. Which of the following equivalent
resistances CANNOT be produced using all three resistors?
C
The figures above show parts of two circuits, each containing a battery of emf
and internal resistance r. The current in each battery is 1 A, but the direction
of the current in one battery is opposite to that in the other. If the potential
differences across the batteries' terminals are 10 V and 20 V as shown, what
are the values of and r ?
C

72 circuits

In the circuit shown above, the equivalent resistance of the three resistors is

10.5

73 circuits

What is the current through the 6.0 resistor shown in the accompanying
circuit diagram? Assume all batteries have negligible resistance.

0A

70 circuits

1/3

E = 5 V, r = 15

Total resistance = E/I = 25 .


Resistance of the 30 and 60
resistors in parallel =20 adding the
internal resistance in series with the
external circuit gives Rtotal= 20 + r =
25
57.circuits.png
P = V2/R and if V is constant P
58.circuits.png
1/R
For the ammeter to read zero means
the junctions at the ends of the
ammeter have the same potential. For
this to be true, the potential drops
across the 1 and the 2 resistor
must be equal, which means the
current through the 1 resistor must
be twice that of the 2 resistor. This
means the resistance of the upper
branch (1 and 3 ) must be that of
the lower branch (2 and R) giving 1
+ 3 = (2 + R)
59.circuits.png
Kirchhoffs loop rule (V = Q/C for a
capacitor)
To dissipate 24 W means R = V2/P = 6
. The resistances, in order, are: 8 ,
4/3 , 8/3 , 12 and 6
Dimensional analysis: 1.6 103
A = 1.6 103 C/s 1.6 1019
C/proton = 1016 protons/sec 109
protons/meter = 107 m/s
The equivalent capacitance of the two 3
F capacitors in parallel is 6 F,
combined with the 3 F in series gives
Ctotal = 2 F
The equivalent capacitance between X
and Y is twice the capacitance between
Y and Z. This means the voltage
between X and Y is 12 the voltage
between Y and Z. For a total of 12 V,
this gives 4 V between X and Y and 8 V
between Y and Z.
Closing the switch short circuits Bulb 2
causing no current to flow to it. Since
the bulbs were
originally in series, this decreases the
total resistance and increases the total
current, making bulb
1 brighter.

In series 1/CT = = 1/C


P = V^2/R
Closing the switch reduces the
resistance in the right side from 20 to
15 , making the total
circuit resistance decrease from 35 to
30 , a slight decrease, causing a
slight increase in
increased slightly
decreased slightly
current. For the current to double, the
but not doubled
the same
but not halved
halved
total resistance must be cut in half.
R = L/A L/d^2 where d is the
diameter. Rx/Ry = Lx/dx^2
Ly/dy^2 = (2Ly)d^2/[Ly(2dy)^2]
12
1
2
4 =
Using all three in series = 3 , all three
in parallel = 1/3 . One in parallel with
two in series =
2/3 , one in series with two in parallel
2/3
1.0
1.5
3.0
= 3/2
The values cannot Summing the potential differences from
be computed
bottom to top:
unless the
left circuit: (1 A)r + E = 10 V
complete circuits
right circuit: + (1 A)r + E = 20 V, solve
E =10 V, r = 100 E = 15 V, r = 5
E = 20 V, r = 10 are shown.
simultaneous equations
The equivalent resistance of the 20
and the 60 in parallel is 15 , added
to the 35 resistor
15
20
50
115
in series gives 15 + 35 = 50
If you perform Kirchhoffs loop rule for
the highlighted loop, you get a current
of 0 A through
0.40 A
0.50 A
1.3 A
1.5 A
the 6 resistor.

61.circuits.png

63.circuits.png

64.circuits.png

65.circuits.png
66.circuits.png

68.circuits.png

71.circuits.png

72.circuits.png

73.circuits.png

74 circuits

Four identical light bulbs K, L, M, and N are connected in the electrical circuit
shown. Rank the current through the bulbs.
D

K>L>M>N

L=M>K=N

L>M>K>N

N>K>L=M

N>L=M>K

75 circuits

Four identical light bulbs K, L, M, and N are connected in the electrical circuit
shown. In order of decreasing brightness (starting with the brightest), the
bulbs are:
D

K=L>M>N

K=L=M>N

K>L=M>N

N>K>L=M

N>K=L=M

76 circuits

Four identical light bulbs K, L, M, and N are connected in the electrical circuit
shown. Bulb K burns out. Which of the following statements is true?
E

All the light bulbs


go out.

Only bulb N goes


out.

Bulb N becomes
brighter

The brightness of Bulb N becomes


bulb N remains the dimmer but does
same
not go out.

N is in the main branch, with the most


current. The current then divides into
the two branches,
with K receiving twice the current as L
and M. The L/M branch has twice the
resistance of the K
branch. L and M in series have the
same current.
Current is related to brightness (P =
(I^2)R) ("N is in the main branch, with
the most current. The current then
divides into the two branches,
with K receiving twice the current as L
and M. The L/M branch has twice the
resistance of the K branch. L and M in
series have the same current.")
If K burns out, the circuit becomes a
series circuit with the three resistors, N,
M and L all in
series, reducing the current through
bulb N.
If M burns out, the circuit becomes a
series circuit with the two resistors, N
and K in series, with
bulb L going out as well since it is in
series with bulb M.
Using Kirchhoffs loop rule around the
circuit going through either V or R since
they are in
parallel and will have the same
potential drop gives: V (1.00 mA)(25
) + 5.00 V (1.00
mA)(975 ) = 0

74.circuits

75.circuits

76.circuits

77 circuits

Four identical light bulbs K, L, M, and N are connected in the electrical circuit
shown. Bulb M burns out. Which of the following statements is true?
E

All the light bulbs


go out.

Only bulb M goes


out.

Bulb N goes out


but at least one
The brightness of Bulb N becomes
other bulb remains bulb N remains the dimmer but does
lit
same
not go out.

78 circuits

The voltmeter in the accompanying circuit diagram has internal resistance


10.0 k and the ammeter has internal
resistance 25.0 . The ammeter reading is 1.00 mA. The voltmeter reading is
most nearly:
D

1.0 V

2.0 V

3.0 V

79 circuits

When two resistors, having resistance R1 and R2, are connected in parallel,
the equivalent resistance of the
combination is 5 . Which of the following statements about the resistances
is correct?

Both R1 and R2
are greater than 5
.

Both R1 and R2
are equal to 5 .

Both R1 and R2
are less than 5 .

zero

0.020 A

0.025 A

5.0 V
78.circuits
One of the
resistances is
greater than 5 ,
one of the
The sum of R1 and resistances is less The equivalent resistance in parallel is
R2 is 5 .
than 5 .
smaller than the smallest resistance.
When the capacitor is fully charged, the
branch on the right has no current,
effectively making
the circuit a series circuit with the 100
and 300 resistors. Rtotal
C
0.033 A
0.100 A
= 400 , E = 10 V = IR
80.circuits

1.0 V

2.0 V

3.0 V

6.0 V

12 V

(n^2)P

nP

P/n

P/(n^2)

I only

II only

I and II only

I and III only

I, II, and III

charge on

current through

power from

resistance of

voltage across

R/8

R/4

R/2

2R

10.0 A

12.0 A

14.1 A

0.100 A

0.141 A

10 V

18 V

24 V

30 V

42 V

80 circuits

81 circuits

82 circuits

83 circuits
84 circuits

See the accompanying figure. What is the current through the 300 resistor
when the capacitor is fully
charged?
C
Three resistors R1, R2, and R3 are connected in series to a battery.
Suppose R1 carries a current of 2.0 A, R2
has a resistance of 3.0 , and R3 dissipates 6.0 W of power. What is the
voltage across R3?
C
When a single resistor is connected to a battery, a total power P is dissipated
in the circuit. How much total
power is dissipated in a circuit if n identical resistors are connected in series
using the same battery? Assume the internal resistance is zero.
D

Consider the compound circuit shown above. The three bulbs 1, 2, and 3
represented as resistors in the
diagram are identical. Which of the following statements are true? I. Bulb 3
is brighter than bulb 1 or 2.
II. Bulb 3 has more current passing through it than bulb 1 or 2.
III. Bulb 3 has a greater voltage drop across it than bulb 1 or 2.
When any four resistors are connected in parallel, the _______ each resistor
is the same.

86 circuits

Wire I and wire II are made of the same material. Wire II has twice the
diameter and twice the length of wire I. If wire I has resistance R, wire II has
resistance
A heating coil is rated 1200 watts and 120 volts. What is the maximum value
of the current under these conditions?

87 circuits

In the accompanying circuit diagram, the current through the 6.0 resistor is
1.0 A. What is the power supply voltage V?
D

85 circuits

4.0 V

77.circuits

In series, they all have the same


current, 2 A. P3
= I3V3
P = (E^2)/R. Total resistance of n
resistors in series is nR making the
power P = (E^2)/nR = P/n
The current through bulb 3 is twice the
current through 1 and 2 since the
branch with bulb 3 is
half the resistance of the upper branch.
The potential difference is the same
across each branch,
but bulbs 1 and 2 must divide the
potential difference between them.
83.circuits
by definition of a parallel circuit
R = L/A L/d^2 where d is the
diameter. RII/RI = LII/dII^2
LI/dI^2 = (2LI)dI^2/[LI(2dI)^2] =

P = IV
If the current in the 6 resistor is 1 A,
then by ratios, the currents in the 2
and 3 resistor are 3 A and 2 A
respectively (since they have 1/3 and
1/2 the resistance). This makes the
total current 6 A and the potential drop
across the 4 resistor 24 V. Now use
Kirchhoffs loop rule for any branch.

87.circuits.png

88 circuits
89 circuits

In the circuit diagrammed above, the 3.00F capacitor is fully charged at


18.0 C. What is the value of the power supply voltage V?
What is the resistance of a 60 watt light bulb designed to operate at 120
volts?

93 circuits

Given the simple electrical circuit above, if the current in all three resistors is
equal, which of the following statements must be true?
Wire Y is made of the same material but has twice the diameter and half the
length of wire X. If wire X has a resistance of R then wire Y would have a
resistance of
The diagram above represents a simple electric circuit composed of 5
identical light bulbs and 2 flashlight cells. Which bulb (or bulbs) would you
expect to be the brightest?
Three different resistors R1, R2 and R3 are connected in parallel to a battery.
Suppose R1 has 2 V across it, R2 = 4 , and R3 dissipates 6 W. What is the
current in R3?

94 circuits

Which of the following statements is NOT true concerning the simple circuit
shown where resistors R1, R2 and R3 all have equal resistances?

90 circuits

91 circuits

92 circuits

95 circuits

96 circuits

97 circuits

98 circuits

99 circuits

100 circuits

101 circuits
102 circuits
103 circuits

104 circuits

If all of the resistors in the above simple circuit have the same resistance,
which would dissipate the greatest
power?
The following diagram represents an electrical circuit containing two uniform
resistance wires connected to a
single flashlight cell. Both wires have the same length, but the thickness of
wire X is twice that of wire Y.
Which of the following would best represent the dependence of electric
potential on position along the length of
the two wires?
Each member of a family of six owns a computer rated at 500 watts in a 120
V circuit. If all computers are
plugged into a single circuit protected by a 20 ampere fuse, what is the
maximum number of the computers can
be operating at the same time?

4.40 V

6.00 V

8.00 V

10.4 V

11.0 V

The voltage across the capacitor is 6 V


(Q = CV) and since the capacitor is in
parallel with the 300 resistor, the
voltage across the 300 resistor is
also 6 V. The 200 resistor is not
considered since the capacitor is
charged and no current flows through
that branch. The 100 resistor in
series with the 300 resistor has 1/3
the voltage (2 V) since it is 1/3 the
resistance. Kirchhoffs loop rule for the
left loop gives E = 8 V.
88.circuits.png

0.5

7200

P = V^2/R

X and Y have
equal resistance

60
X and Y added
together have the
same resistance
as Z

240

X, Y, and Z all
have equal
resistance

R/8

R/2

V only

V and W only

V and Z only

X and Y each have


For the currents in the branches to be
more resistance
none of the above equal, each branch must have the
than Z
must be true
same resistance.
90.circuits.png
R L/A = L/d^2. If d 2, R 4
and if L 2, R 2 making the net
2R
8R
effect R 8
all five bulbs are
Bulbs in the main branch have the most
the same
current through them and are the
V, W and Z only
brightness
brightest.
92.circuits.png

0.33 A

0.5 A

the largest current the voltage


will pass through
across R2
R1
is 5 volts

2A
the power
dissipated in
R3 could be 5
watts

3A
if R2 were to burn
out, current would
still flow through
both R1 and R3

Resistor A

Resistor C

Resistor D

The steady current in the above circuit would be closest to which of the
following values?

When the switch is closed, what would be the current in the circuit?
How much current flows through a 4 ohm resistor that is dissipating 36 watts
of power?

106 circuits

How would the current through the 2 ohm resistor compare to the current
through the 4 ohm resistor?

0.33 C

0.67 C

the net resistance


of the circuit is less If the resistances are equal, they will all
than R1
draw the same current.
94.circuits.png
Resistor D is in a branch by itself while
They would all
resistors A, B and C are in series,
dissipate the same drawing less current
power
than resistor D.
95.circuits.png

1.0 C

In parallel, all the resistors have the


same voltage (2 V). P3 = I3V3

4 5 or more

1.5 C

3.0 C

Even though the wires have different


resistances and currents, the potential
drop across each is
1.56 V and will vary by the same
gradient, dropping all 1.56 V along the
same length.

96.circuits.png

Each computer draws I = P/V = 4.17 A.


4 computers will draw 16.7 A, while 5
will draw over 20
A.
The capacitance of the two capacitors
in parallel is 2C. Combined with a
capacitor in series
gives C=(Cx2C)/(C+2C)=2C/3
98.circuits.png

0.2 amp

0.37 amp

0.5 amp

2.0 amp

only bulb D

only bulb E

only bulbs A or E

only bulbs C or D

P = IV = 1.56 kW. Energy = Pt = 1.56


$1.55 kW 8 h = 12.48 kW-h
Resistance of bulbs B & C = 20
combined with D in parallel gives 6.7
for the right side.
Combined with A & E in series gives a
5.0 amp
total resistance of 26.7 . E = IR
100-101.circuits.png
A and E failing in the main branch
would cause the entire circuit to fail. B
and C would affect
bulbs B, C, or D
each other.
100-101.circuits.png

30 V
30 V

40 V
40 V

60 V
60 V

70 V
70 V

110 V
110 V

1.1 A

1.7 A

2.0 A

2.3 A

3.0 A

2.25 amps

3.0 amps

4.24 amps

9.0 amps

144 amps

oneforth as large onehalf as large

Which bulb (or bulbs) could burn out without causing other bulbs in the circuit
to also go out?
A
With the switch open, what would be the potential difference across the 15
ohm resistor?
A
With the switch open, what must be the voltage supplied by the battery?
D

105 circuits

Three identical capacitors each with a capacitance of C are connected as


shown in the following diagram. What
would be the total equivalent capacitance of the circuit?
B
An electric heater draws 13 amperes of current when connected to 120 volts.
If the price of electricity is
$0.10/kWh, what would be the approximate cost of running the heater for 8
hours?
D

Resistor B

12 A

$0.19

$0.29

$0.75

$1.25

four times as large twice as large

equally as large

V = IR
E = IRtotal where Rtotal = 35
With the switch closed, the resistance
of the 15 and the 30 in parallel is
10 , making the
total circuit resistance 30 and E = IR

102-104.circuits.png
102-104.circuits.png

104.circuits.png

P = I^(2)R
The equivalent resistance through path
ACD is equal to the equivalent
resistance through path
ABD, making the current through the
two branches equal
106.circuits.png

107 circuits

108 circuits

What would be the potential at point B with respect to point C in the above
circuit?
A cylindrical resistor has length L and radius r. This piece of material is then
drawn so that it is a cylinder with
new length 2L. What happens to the resistance of this material because of
this process?

"+7 V"

"+3 V"

"0 V"

"-3 V"

the resistance is
quartered.

the resistance is
halved.

the resistance is
unchanged.

the resistance is
doubled.

Bulb #1 only

Bulb #2 only

Bulbs #2 and #3
only

Bulbs #1, #2, and


#3

110 circuits

A circuit is connected as shown. All light bulbs are identical. When the switch
in the circuit is closed
illuminating bulb #4, which other bulb(s) also become brighter?
A
A cylindrical graphite resistor has length L and crosssectional area A. It is to
be placed into a circuit, but it
first must be cut in half so that the new length is L. What is the ratio of the
new resistivity to the old
resistivity of the cylindrical resistor?
C

111 circuits
112 circuits

Through which resistor(s) would there be the greatest current?


Which resistor(s) have the greatest rate of energy dissipation?

113 circuits

The circuit shown has an ideal ammeter with zero resistance and four
identical resistance light bulbs which are
initially illuminated. A person removes the bulb R4 from its socket thereby
permanently breaking the electrical
circuit at that point. Which statement is true of the circuit after removing the
bulb?

114 circuits

A current through the thin filament wire of a light bulb causes the filament to
become white hot, while the
larger wires connected to the light bulb remain much cooler. This happens
because

115 circuits

In the circuit above the voltmeter V draws negligible current and the internal
resistance of the battery is 1.0 ohm. The reading of the voltmeter is

109 circuits

The resistance in each of the two paths


is 9 , making the current in each
branch 1 A. From
point A, the potential drop across the 7
resistor is then 7 V and across the 4
resistor is 4 V,
"-7 V"
making point B 3 V lower than point C 107.circuits.png
Since the volume of material drawn into
a new shape in unchanged, when the
the resistance is
length is doubled,
quadrupled.
the area is halved. R = L/A
Closing the switch reduces the total
resistance of the circuit, increasing the
current in the main
None of the bulbs. branch containing bulb 1
109.circuits.png

J only
J only

M only
M only

N only
N only

The power
supplied by the
battery increases
the thin filament
has more
resistance than the
larger connecting
wires.

The voltage
accross R1
increases

The voltage from B


to C increases
the larger
connecting wires
have more
resistance than the
filament.

10.5 V

12.0 V

10.8 V

13.0 V

Resistivity is dependent on the material.


0.25 Not to be confused with resistance
Resistors J and N are in the main
branch and therefore receive the
K&L only
largest current
K&L only
P = I^(2)R
Breaking the circuit in the lower branch
lowers the total current in the circuit,
decreasing the
voltage across R1. Looking at the upper
loop, this means R2
A
The bulb R2
now has a larger share of the battery
maintains the
voltage and the voltage across AD is
same brightness
the same as the voltage across BC
the current in the
filament is less
than that through
the connecting
In series circuits, larger resistors
wires.
develop more power
With a total resistance of 10 , the total
current is 1.2 A. The terminal voltage
11.6 V
VT = E Ir
Most rapid heating requires the largest
power dissipation. This occurs with the
resistors in
E
parallel.

0.2 A

2A

4A

5A

7.2 A

D
D

0.5

J&N only
J&N only

The ammeter
reading is
unchanged
the current in the
filament is greater
than that through
the filament wire is the connecting
not insulated.
wires.

117 circuits

Suppose you are given a constant voltage source V0 and three resistors R1,
R2, and R3 with R1 > R2 > R3. If you wish to heat water in a pail which of the
following combinations of resistors will give the most rapid heating?
E
A household iron used to press clothes is marked 120 volt, 600 watt. In
normal use, the current in it is
D

118 circuits

For the circuit shown, a shorting wire of negligible resistance is added to the
circuit between points A and B.
When this shorting wire is added, bulb #3 goes out. Which bulbs (all identical)
in the circuit brighten?
C

Only Bulb 2

Only Bulb 4

Only Bulbs 1 and 4 Only Bulbs 2 and 4 Bulbs 1, 2 and 4

119 circuits

For the configuration of capacitors shown, both switches are closed


simultaneously. After equilibrium is
established, what is the charge on the top plate of the 5 F capacitor?

100 C

50 C

30 C

116 circuits

25 C

121 circuits

How many coulombs will pass through the identified resistor in 5 seconds
once the circuit was closed?
E
A junior Thomas Edison wants to make a brighter light bulb. He decides to
modify the filament. How should
the filament of a light bulb be modified in order to make the light bulb produce
more light at a given voltage?
B

Increase the
resistivity only.

Increase the
diameter only.

Decrease the
diameter only.

Decrease the
diameter and
increase the
resistivity.

122 circuits

In the circuit diagram above, all of the bulbs are identical. Which bulb will be
the brightest?

120 circuits

1.2

12

2.4

10 C

111.circuits.png
112.circuits.png

113.circuits.png

115.circuts.png

116.circuts.png

P = IV
Shorting bulb 3 decreases the
resistance in the right branch,
increasing the current through bulb 4
and decreasing the total circuit
resistance. This increases the total
current in the main branch
containing bulb 1.
118.circuts.png
The total charge to be distributed is
+100 C 50 C = + 50 C. In parallel,
the capacitors must
have the same voltage so the 20 F
capacitor has four times the charge of
the 5 F capacitor.
This gives Q20 = 4Q5 and Q20 + Q5 =
4Q5 + Q5 = 5Q5 = 50 C, or Q5

= 10 C
119.circuts.png
The equivalent resistance of the two 4
resistors on the right is 2 making
the total circuit
resistance 10 and the total current
2.4 A. The 2.4 A will divide equally
between the two
branches on the right. Q = It = (1.2 A)(5
24
6 s) = 6 C
120.circuts.png
For more light at a given voltage, more
current is required, which requires less
Increase the length resistance. R =
only.
L/A
The bulbs all have
the same
Bulb C in the main branch receiving the
brightness.
total current will be the brightest
122.circuts.png

123 circuits

For the circuit shown, the ammeter reading is initially I. The switch in the
circuit then is closed. Consequently:

124 circuits

Approximately how much would it cost to keep a 100 W light bulb lit
continuously for 1 year at a rate of $0.10
per kW hr?

126 circuits

.In the circuit shown above, the potential difference between points a and b is
zero for a value of capacitance C
of
A
The equivalent resistance of the circuit shown to the right with resistances R1
= 4.00 , R2 = 3.00 , and R3
B

127 circuits

For the circuit shown, when a shorting wire (no resistance) connects the
points labeled A and B, which of the
numbered light bulbs become brighter? Assume that all four bulbs are
identical and have resistance R .

128 circuits

In terms of the seven fundamental SI units in the MKS system, the Ohm is
written as

125 circuits

1/3 microfarad
0.111

0.923

Bulb 1 only

Bulb 2 only

(kgm^2)/
(A^2s^3)

(kgm^2s)/C^2

130 circuits

.Consider a simple circuit containing a battery and three light bulbs. Bulb A is
wired in parallel with bulb B and
this combination is wired in series with bulb C. What would happen to the
brightness of the other two bulbs if
bulb A were to burn out?
C
For the RC circuit shown, the resistance is R = 10.0 , the capacitance is C =
5.0 F and the battery has voltage
= 12 volts . The capacitor is initially uncharged when the switch S is closed at
time t = 0. At some time later, the
current in the circuit is 0.50 A. What is the magnitude of the voltage across
the capacitor at that moment?
D

131 circuits

132 circuits

129 circuits

Wire CD shorts out bulb #3 so it will


never light. Closing the switch merely
adds bulb #2 in
The power
parallel to bulb #1, which does not
supplied by the
change the potential difference across
battery decreases. bulb #1.
1 year = 365 days 24 hours/day =
8760 hours. W (energy) = Pt = 0.1 kW
8760 hours = 867
$1
$10
$100
$1000
$100000 kW-h $0.10 per kW-h = $ 86.7
For points a and b to be at the same
potential, the potential drop across the
3 resistor must be
equal to the potential drop across
capacitor C. The potential drop across
the 3 resistor is three
times the drop across the 1 resistor.
For the potential drop across capacitor
C to be three times
the crop across the 1 F capacitor, C
2/3 microfarad
2 microfarads
3 microfarads
9 microfarads
must be 1/3 the capacitance, or 1/3 F

The potential
The potential
difference between
The ammeter
difference between E and F stays the Bulb #3 lights up
reading decreases. E and F increases. same.
more brightly.

There would be no
change in the
brightness of either Both would get
bulb B or bulb C.
brighter.

1.08

3.00

In parallel
126.circuits.PNG
Shorting bulb 4 decreases the
resistance in the right branch,
increasing the current through bulb 3
and in the main branch containing bulb
Bulb 3 only
Bulbs 1 and 3 only Bulbs 1, 2, and 3 1.
127.circuits.PNG
R = V/I where V = W/Q and Q = It
giving R = W/I2
t and W = joules = kg m2
/s
kgm/Cs
(kgm^2)/As^2
2
If A were to burn out, the total
resistance of the parallel part of the
circuit increases, causing less
current from the battery and less
current through bulb A. However, A and
B split the voltage
from the battery in a loop and with less
current through bulb A, A will have a
smaller share of
Bulb B would get Bulb B would get
voltage, increasing the potential
brighter and bulb C dimmer and bulb C Only bulb B would difference (and the current) through
would get dimmer. would get brighter. get brighter
bulb B.

5 volts

6 volts

7 volts

12 volts

In the circuit shown above, the 10 F capacitor is initially uncharged. After the
switch S has been closed for a
long time, how much energy is stored in the capacitor?
D

0 J

100 J

250 J

500 J

1000 J

.In the circuit shown above, a constant current device is connected to some
identical light bulbs. After the switch
S in the circuit is closed, which statement is correct about the circuit?

Bulb #2 becomes
brighter.

Bulb #1 becomes
dimmer.

All three bulbs


become equally
brighter.

The voltage
between points C
and D is
decreased.

The power from


the current device
is increased.

120 V

80 V

60 V

40 V

30 V

134 circuits

Two 1000 resistors are connected in series to a 120volt electrical source.


A voltmeter with a resistance of
1000 is connected across the last resistor as shown. What would be the
reading on the voltmeter?
D
Two resistors, one with resistance R and the second with resistance 4R are
placed in a circuit with a voltage V. If resistance R dissipates power P, what
would be the power dissipated by the 4R resistance?
A

4P

2P

.5P

.25P

135 circuits

Which resistor has the greatest electric current through it?

133 circuits

125.circuits.PNG

9.00

0 volts

123.circuts.png

When the current is 0.5 A, the voltage


across the resistor is V = IR = 5 V.
According to the loop
rule, the remaining 7 V must be across
the capacitor.
When the switch has been closed a
long time, the voltage across the
capacitor is 10 V as the
current has stopped and the resistor
has no potential drop across it. UC =
CV
D
2
Since there is constant current, bulb 1
remains unchanged and bulbs 2 and
three must now split
the current. With half the current
through bulb 2, the potential difference
between A and B is
also halved.
The voltmeter is essentially another
resistor. The voltmeter in parallel with
the 100 resistor
acts as a 500 resistor, which will half
the voltage of the 100 resistor on
the left. Thus the
120 V will split into 80 V for the 1000
resistor and 40 V for the voltmeter
combination.

130.circuits.PNG

131.circuits.PNG

132.circuits.PNG

133.circuits.PNG

P = I^2 * R and the current is the same


through each resistor.
134.circuits.PNG
The greatest current is in the main
branch.
135.circuits.png

136 circuits

Which resistor has the greatest potential difference across it?

137 circuits

A battery, an ammeter, three resistors, and a switch are connected to form


the simple circuit shown above. When the switch is closed what would
happen to the potential difference across the 15 ohm resistor?

2
it would be twice
the potential
difference across
the 30 ohm
resistor

it would equal the


potential difference
across the 20 ohm
resistor

it would equal the


potential difference
across the 30 ohm
resistor

138 circuits

What would be the current at point E in the circuit?

2 amp

4 amp

5 amp

139 circuits

What would be the potential at point B with respect to point D?

positive 2 V

positive 4 V

positive 5 V

140 circuits

Two resistors and a capacitor are connected with a 10 volt battery, a switch
and an ideal ammeter to form the simple electrical circuit shown. After the
switch is closed and the current in the circuit reaches a constant value, what
is the reading on the ammeter in the circuit?

9.2 102A

8.1 102A

7.5 102A

141 circuits

When the switch is closed, what would be the current in the circuit shown in
the diagram above if the two batteries are opposing one another?

1.25 A

.75 A

.5 A

142 circuits

Four resistors, R1, R2, R3, and R4, are connected in the circuit diagram
above. When the switch is closed, current flows in the circuit. If no current
flows through the ammeter when it is connected as shown, what would be the
value of R3?
D

it would be half the


potential difference
The 15 resistor would be in parallel
across the 30 ohm
with the 30 resistor when the switch
resistor
none of the above is closed.
ACD = 9 , ABD = 9 so the total
resistance is 4.5 making the total
7 amp
9 amp
current E/R = 2 A.
The 2 A will divide equally between the
two branches with 1 A going through
each branch.
From B to D we have (1 A)(2 ) = 2
positive 7 V
positive 9 V
V, with B at the higher potential
When the capacitor is charged, the
branch is effectively removed from the
circuit, making it a
simple parallel circuit. The total
6.9 102A
Zero
resistance is 133.3 and V = IR
In a simple series circuit with two
batteries opposing one another the
voltages subtract from one
another. The total effective voltage for
this circuit is then 4 V. With a total
resistance of 20
.3 A
.2 A
the total current is (4 V)/(20 )
For no current to flow, the potential
drop across R1 must equal the potential
drop across R2. For
this to occur I1R1 = I2R2. Since the two
branches also have the same potential
difference as a
whole (they are in parallel) we also
have I1(R1 + R3) = I2(R2 + R4). Solve
(R1*R4)/R2
R1
for R3
When the capacitor is charged, the
branch is effectively removed from the
circuit, making the
circuit a 10 resistor in series with two
10 resistors in parallel. The lone 10
resistor has
twice the voltage of the two 10
resistors in parallel with an effective
resistance of 5 . The 10
volts will then divide with 3.3 V going to
the parallel combination and 6.7 V
going to the single
10 resistor. The capacitor is in
parallel with the single 10 resistor. Q
100 C
150 C
= CV
The resistances are, respectively, 4/3
D
E
R, 2/5 R, R, and 5/3 R
Closing the switch adds another parallel
branch, increasing the total current
delivered by the battery. Bulb 3 will get
brighter. Bulb 2, in its own loop with
It cannot be
bulb 3 and the battery will then lose
determined without some of its share of the potential
knowingthe emf of difference from the battery and will get
Both bulbs
the battery.
dimmer
For the 3 capacitors in series on the
right CTC= C/3. Adding to the
capacitor in parallel gives C + C/3 =
3C
4C
4C/3
Superconductors have a property
where the resistance goes to zero
D
E
below a certain threshold temperature

(R1+R4)/[(R1+R2) [(R1+R2)*(R4)]/
*(R3+R4)]
(R2+R4)

(R1+R2)/R4

23 C

40 C

67 C

144 circuits

The diagram above shows an electrical circuit composed of 3 resistors and 1


capacitor. If each resistor has a
resistance of 10 and the capacitor has a value of 10 F, what would be the
charge stored in the capacitor
when an EMF of 10 V is maintained in the circuit for a sufficient time to fully
charge the capacitor?
C
Given 4 identical resistors of resistance R, which of the following circuits
would have an equivalent resistance of 4/3 R?
A

145 circuits

The three lightbulbs in the circuit above are identical, and the battery has zero
internalresistance. When switch S is closed to causebulb 1 to light, which of
the other two bulbs increase(s) in brightness?
C

Neither bulb

Bulb 2 only

Bulb 3 only

146 circuits

What would be the equivalent capacitance of the circuit shown if each


capacitor has a capacitance of C?

4/3 C

147 circuits

Which of the following graphs would best represent the resistance versus
temperature relationship for a superconductor?

143 circuits

Let the current through the 1 be x.


The potential difference across the 1
resistor is then x
volts. The current will divide between
the upper branch (5 ) and the lower
branch (9 ) with
(using the current divider ratio method)
9/(9 + 5) = 9/14 x in the upper branch
and 5/14 x in the
lower branch. The potential differences
are then IR giving for the 2, 3, 4, 5
resistors,
respectively 18/14 x, 27/14 x, 20/14 x
and 25/14 x volts.
135.circuits.png

137.circuits.png

138.circuits.png

138.circuits.png

140.circuits.png

141.circuits.png

142.circuits.png

143.circuits.png
144.statics.png

145.statics.png

146.statics.png

147.statics.png

148 circuits

What would be the total current being supplied by the battery in the circuit
shown above?

3.0 amperes

2.25 amperes

2.0 amperes

1.5 amperes

1.0 amperes

51V

42V

36V

24V

21V

150 circuits

In the electric circuit shown above, the current through the 2.0 resistor is
3.0 A. Approximately what is the emf of the battery?
Which of the following wiring diagrams could be used to experimentally
determine R using Ohm's Law? Assume an ideal voltmeter and an ideal
ammeter.

151 circuits

If B2 were to burn out, opening the circuit, which voltmeter(s) would read zero
volts?
B

none would read


zero

Only V2

152 circuits

If B2 were to burn out, opening the circuit, what would happen to the reading
of V1? Let V be its original reading when all bulbs are functioning and let V
(underline) be its reading when B2 has burnt out.

V > 2V

2V> V> V

153 circuits

Closing which of the switches will produce the greatest power dissipation in
R2?

S1 only

S2 Only

154 circuits

Closing which of the switches will produce the greatest reading on the
ammeter?

S1 only

S2 only

155 circuits

Closing which of the switches will produce the greatest voltage across R3?

S1 only

S2 only

156 circuits

Two cables can be used to wire a circuit. Cable A has a lower resistivity, a
larger diameter, and a different
length than cable B. Which cable should be used to minimize heat loss if the
same current is maintained in
either cable?

Cable A

Cable B

157 circuits

What would be the reading on a voltmeter connected across points A and C ? A

12 V

6V

158 circuits

What would be the reading on an ammeter inserted at point B ?

9A

6A

159 circuits

The equivalent capacitance for the circuit is

1/11 F

11/18 F

Q3 < Q6 V3 = V6

Q3 = Q6 V3 < V6

149 circuits

160 circuits

161 circuits

162 circuits

How do the charge Q3 stored in the 3 F capacitor and the voltage V3 across
it compare with those of the 6 F capacitor?
C
A length of wire of resistance R is connected across a battery with zero
internal resistance. The wire is then cut
in half and the two halves are connected in parallel. When the combination is
reconnected across the battery,
what happens to the resultant power dissipated and the current drawn from
the battery?
E
A fixed voltage is applied across the length of a tungsten wire. An increase in
the power dissipated by the wire
would result if which of the following could be increased?
B

No change No
change
The resistivity of
the tungsten

Doubles Doubles
The crosssectional area of
the wire

On the right, the 6 and 3 resistor in


parallel have an equivalent resistance
of 2 . Added to the 4 resistance in
the middle branch which is in series
with the pair gives 6 across the
middle. This is in parallel with the 3
resistor at the top giving an equivalent
resistance of 2 . Lastly add the 4
resistor in the main branch giving a
total circuit resistance of 6 . V = IR
148.statics.png
Using ratios, the currents in the 6and
3 resistors are 1 A and 2 A. They
have three times and 3/2 times the
resistance of the 2 resistor so they will
have 1/3 and 2/3 the current. The total
current is then 6 A giving a potential
drop of 36 V across the 6 resistor in
the main branch and adding any one of
the branches below with the loop rule
gives 36 V + 6 V = 42 V for the battery 149.statics.png

Voltmeters must be placed in parallel


and ammeters must be placed in series
Even though B2 burns out, the circuit is
only V2, V4, and
The would all read still operating elsewhere as there are
only V3 and V4
V5
zero
still closed paths
With B2 burning out, the total
resistance of the circuit increases as it
is now a series circuit. This decreases
the current in the main branch,
decreasing V1. For V1 to be halved,
the current must be halved which
means the total resistance must be
doubled, which by inspection did not
happen in this case (total before = 5/3
V= V
V> V>V/2
V/2 > V
R, total after = 3 R)
S1 must be closed to have any current.
Closing S2 will allow current in R2 but
S1 and S2 only
S1 and S3 only
S1, S2, and S3
closing R3 would short circuit R2.
S1 must be closed to have any current.
Closing S3 will short circuit R3, leaving
only resistor R1, E
S3 only
S1 and S2
S1 and S3
which is the lowest possible resistance.
S1 must be closed to have any current.
The greatest voltage will occur with the
greatest current
through R3 but closing S2 or S3 will
S1 and S2 only
S1 and S3 only
S1, S2, and S3
draw current away from R3.
It cannot be
determined without
knowing the
It cannot be
materials
determined without contained in each
The heat loss is
knowing the length cable
the same for both. of each cable.
123.
R = L/A
0 V, since the fuse Starting at A and summing potential
would break the
differences counterclockwise to point C
3V
2V
circuit
gives 12 V
The branch with two 2 resistors has a
0 A, since the fuse total resistance of 4 and a potential
would break the
difference of 12 V.
3A
2A
circuit
V = IR
For the 6 F and 3 F capacitor in
series, the equivalent capacitance is 2
F. Adding the 2 F in
parallel gives a total capacitance of 4
1 F
4 F
11 F
F
In series the capacitors have the same
charge, but the smaller capacitor will
have the larger
potential difference (to force the same
Q3 = Q6 V3 > V6 Q3 > Q6 V3 = V6 Q3 > Q6 V3 > V6 charge on a smaller area)

Quadruples
Doubles
The length of the
wire

Doubles
Quadruples

Before cutting the resistance is R. After


cutting we have two wires of resistance
R which in
parallel is an equivalent resistance of
R. P = V2/R and I = V/R

Quadruples
Quadruples
The temperature of
The temperature of the wires
P = V2/R and R = L/A giving P =
the wire
surroundings
V2A/L

150.statics.png

151.statics.png

152.statics.png

153.statics.png

154.circuits.png

155.circuits.png

157.cicuits.png

158.circuits.png

159.circuits.png

163 circuits

In a 30-minute interval, one kilowatt-hour of electrical energy is dissipated in a


resistance of 20 ohms by a
current of
A

Four infinitely long wires are arranged as shown in the accompanying figure
endon view. All four wires are perpendicular to the plane of the page and
have the same magnitude of current I. The conventional current in the wire in
the upper righthand corner is directed into the plane of the page. The other
conventional currents are out of the plan of the page. Point P is a distance a
1 magnetostatics from all four wires. What is the total magnetic field at point P?
C

The conventional current I in a long straight wire flows in the upward direction
as shown in the figure. (Electron flow is downward.) At the instant a proton of
charge +e is a distance R from the wire and heading directly toward it, the
2 magnetostatics force on the proton is:
E
A charged particle with constant speed enters a uniform magnetic field whose
direction is perpendicular to the
3 magnetostatics particles velocity. The particle will:
E

A long straight wire conductor is placed below a compass as shown in the top
view figure.
When a large conventional current flows in the conductor as shown, the N
pole of the
4 magnetostatics compass:
D

A proton of mass M and kinetic energy K passes undeflected through a


region with electric and magnetic fields
perpendicular to each other. The electric field has magnitude E. The
5 magnetostatics magnitude of the magnetic field B is
An electric current flows through a horizontal wire as shown.
Which option best represents the direction of the magnetic field
6 magnetostatics at point P

Two bar magnets are to be cut in half along the dotted lines shown. None
7 magnetostatics of the pieces are rotated. After the cut:
An ion with charge q, mass m, and speed V enters a magnetic field B and is
deflected into a path with a radius
of curvature R. If a second ion has speed 2V, while m, q, and B are
unchanged, what will be the radius of the
8 magnetostatics second ions path?

A wire moves through a magnetic field directed into the


page. The wire experiences an induced charge separation
9 magnetostatics as shown. Which way is the wire moving?
A charged particle with constant velocity enters a uniform magnetic field
whose direction is parallel to the
10 magnetostatics particles velocity. The particle will

10 amp.

20 amp.

14.1 amp.

36 amp.

1 kW-h = 1000 W 60 min = 60,000 Wmin = I2Rt = I2(20 )(30 min)


Each wire contributes a B field given by
oI / 2a in a direction found using
RHRcurl. The
direction of each B field is as follows,
(1)Top right wire: B up&left, (2)Top left
wire: B
up&right, (3)Bottom left wire: B up&left,
(4)Bottom right wire: B down&left.
Forces from
1 and 4 cancel leaving both 3 and 4 B
fields acting up and left and adding
0 together.
1.magnetostatics.png
The B field at the location of the charge
+e is created by the wire next to it and
given by
B = oI / 2R. Based on RHRcurl the
direction of that B field is into the page
at that
location. Then the force on that charge
is given by Fb=qvB, with q=e and B
ev(oI)/(2R)
from before
downward (in the so Fb = ev(oI / 2R). Using the
opposite direction RHRflat for the + charge, the force
as I)
comes out as down.
2.magnetostatics.png
18 amp.

o I / 2a toward o I / 2a toward 2(o I / 2a)


the upper left hand the lower left hand toward the upper
corner
corner
left hand corner

2(o I / 2a)
toward the lower
left hand corner

(o/2)I^2 toward
the wire

(oI^2L)/(2R)
upward (in the
same direction as
I)

(oI^2L)/(2R)
downward (in the
opposite direction
as I)

ev(oI)/(2R)
upward (in the
same direction as
I)

Speed up

Slow down

Experience no
change in velocity

Follow a parabolic Follow a circular


arc
arc

remains
undeflected

points to the south points to the west

points to the east

has its polarity


reversed

ME^2/K

ME/2K

2ME^2/K

ME^2/2K

ME^2/K^2

out of the page

to the right of the


page

toward the top of


the page

into the page

None of the halves The two halves of


will attract any
each magnet will
other
attract each other

The two halves of


each magnet will
repel each other

Charges moving through magnetic


fields move in circles
The compass is ABOVE the wire. Using
RHRcurl on the wire, the B field points
towards the
right at the location above the wire.
Since compasses follow B field lines,
the compass will
also point right, which is east.
4.magnetostatics.png
To be undeflected, the electric and
magnetic forces must balance.
Fe = Fb Eq = qvB B = E / v With v
related to K by K = mv2
gives B = E / (2K/m) which is
equivalent to choice D

toward the bottom Focus on + charge direction and use


of the page
RHRcurl and you get into page
6.magnetostatics.png
When cutting a magnet, you must end
up with two new magnets having 2
poles each. For the top
magnet the current N and S must stay
as is, so the left of center part becomes
a S and the
right of center part becomes a N. There
are now two opposite poles that attract.
For the
bottom magnet, by slicing it down the
center you now have two magnets on
The two halves of The two halves of top of each
the top magnet will the top magnet will other. The poles would not change their
repel, the two
attract, the two
current locations so you have two north
halves of the
halves of the
and two
bottom magnet will bottom magnet will south poles near each other on top and
attract
repel
bottom which makes repulsion.
7.magnetistatics.png

4R

2R

R/2

to the right

to the left

out of the page

toward the top of


the page

For this scenario, The circular motion is


provided by the magnetic force. So that
Fnet(C) = mv2/r
R/4
qvB = mv2/r qBr = mv 2 x V --> 2 x r
Focus on a single + charge in the wire
that gets pushed to the right. So this +
charge is moving in
a magnetic field pointing into the page
with a force directed right, based on
toward the bottom RHRflat, the
of the page
charge must be moving down.
9.magnetostatics.png

speed up

slow down

experience no
change in velocity

follow a parabolic
arc

follow a circular
arc

T and Y

T and Z

X and Y

X and Z

S, T, and Z

The diagram to the right depicts iron filings sprinkled around three permanent
11 magnetostatics magnets. Pole R is the same pole as
D

When moving parallel to magnetic


fields, no forces are experienced.
Assume R is north. Based on the lines,
T would have to be north and so would
Y.
11.magnetostatics.
This makes X and Z south and S north. png

If conventional electric current flows from left to right in a wire as shown, what
12 magnetostatics is the direction of the magnetic field at point P?
C

towards the bottom


of the paper
into the paper

out of the paper


to the right
Using RHRcurl, we get into the page
both wires will
the force on the
experience a
the wires will
the wires will
right hand wire will torque until they
experience a force experience a force cancel the force on are at right angles
Parallel current wires with same
of attraction
of repulsion
the left hand wire to each other
none of the above direction current attract.
Focus on a single + charge in the wire
that gets pushed to the right. So this +
charge is moving
down with a force directed right, based
on RHRflat, the magnetic field must
towards the bottom
towards the top of point into the
into the page
of the page
towards the right
out of the page
the page
page.
By definition, E fields exert forces on +
charges in the same direction as the E
field. So the force
from the E field must be UP. To
maintain a constant velocity, this
upwards force must be
counterbalanced by a downwards force,
which in this case it is to be provided by
the
magnetic field. With a + charge moving
right, and a magnetic force down,
into the plane of
out of the plane of
up the plane of the RHRflat gives a
the page
the page
to the right
to the left
page
magnetic field pointing out of the page.
A coil of wire (solenoid) like this
becomes an electromagnet when the
current runs through it.
Use the RHRsolenoid to determine
that the right side of this electromagnet
becomes the
north side. Now pretend that the
electromagnet is simply a regular
magnet with a N pole on
the right and a S pole on the left and
draw the field lines. In doing so, the
lines end up
not move since the not move since the pointing to the left at the location of the
point toward the
magnetic field of
magnetic field of
compass. Since compasses follow
point toward the
point toward the
bottom of the
the coil is into the the coil is out of
magnetic field
left
right
paper
paper
the paper
lines, the compass will also point left.
Due to action reaction the forces must
be the same. Another way to look at it
is that wire A
creates the field that wire B is sitting in
based on its current I, Ba
= oI
a
/ 2R. The force on
wire B is dependent on the field from A,
and also the current in wire B itself and
is given by
Fb
=Ba
I
b
L Fb
= (oI
a
/ 2R) Ib
L. So since both currents from A and B
affect each
FB on A = 4 FA on FB on A = FA on FB on A = 2 FA on FB on A = FA on
respective force, they should share the
B
B
B
B
FB on A = FA on B same force.
Think about this as if you are looking
down at a table top with the + particle
on it. An E field is
pointed down into the table so an
electric force acts down into the table
also. The electric
force pushing down will not move the
charge. A magnetic field comes up out
It cannot be
The relative sizes of the table,
determined without of the electric and but since the charge is at rest, the
knowing the exact magnetic fields are magnetic field exerts zero force on it.
value of the charge needed to answer So Fe
FE > FB
FE<FB
FE<FB
of the particle
this question
> Fb

towards the north


pole

Two light wires are hung vertically. With electrical current in both wires
13 magnetostatics directed upwards

A wire moves with a velocity v through a magnetic field and


experiences an induced charge separation as shown. What is the
14 magnetostatics direction of the magnetic field?

A positively charged particle moves to the right. It enters a


region of space in which there is an electric field directed up the
plane of the paper as shown. In which direction does the
magnetic field have to point in this region so that the particle
15 magnetostatics maintains a constant velocity?

A compass is placed near a coil of wire. A conventional electrical


current is then run through the coil from left to right as shown. This
16 magnetostatics will cause the North pole of the compass to:

Two parallel wires are carrying different electric current in the same direction
as shown. How does the
17 magnetostatics magnitude of the force of A from B compare to the force of B from A

A positively charged particle of mass M is at rest on a table. A nonzero


electric field E is directed into the
plane of the table. A nonzero magnetic field B is directed out of the plane of
the table. What is true about the
magnitude of the electric force on the particle FE compared to the magnetic
18 magnetostatics force on the particle FB?
A positive electric charge of negligible weight is released from rest between
the
poles of a horseshoe magnet as shown. What would be the direction of the
19 magnetostatics acceleration of the charge caused by the magnetic field?

towards the top of


the paper

towards the south


pole

upwards

downwards

As described above, a charge not


moving will not experience a magnetic
none of the above force

12.magnetostatics.
png

14.magnetostatics.
png

15.magnetostatics.
png

16.magnetostatics.
png

17.magnetostatics.
png

19.magnetostatics.
png

Two very long currentcarrying wires are shown end on in the figure. The
wire on the left has a 4A current
going into the plane of the paper and the wire on the right has a 3A current
coming out of the paper.
Disregarding the case of x --> , in which region(s) could the magnetic field
from these two wires add to zero
20 magnetostatics on the xaxis.

21 magnetostatics The magnetic field line passing through point P inside the solenoid is directed D
The diagram below shows a straight wire carrying a
current i in a uniform magnetic field. An arrow
indicates the magnetic force F on the wire. Of the
following possibilities, the direction of the magnetic
22 magnetostatics field must be
A

For the four identical current-carrying wires shown (with conventional


current coming out of the plane of the page), the wire on the right is
labeled P. What is the direction of the magnetic force on the wire labeled
23 magnetostatics P from the other wires?

A wire has a conventional current I directed to the right. At the instant shown
in the figure, an
electron has a velocity directed to the left. The magnetic force on the electron
24 magnetostatics at this instant is
E

Region I only

Region II only

Region III only

to the right

to the left

downward toward
the bottom of the
page

First of all we should state that a larger


current makes a bigger B field and the
further from the
wire the less the B field. Using
RHRcurl, the 4A wire has decreasing
magnitude B fields
pointing down in regions II and III on
the axis and upwards on region I. The
3A wire has B
fields pointing upwards in region III and
downwards in regions II and I. To
cancel, fields
would have to oppose each other.
Region I is a possibility but since the
distance from the 4A
wire is smaller at every point and it also
has a larger current it will always have
a larger B
field so there is no way to cancel.
Region II has fields in the same
direction and cannot
cancel. Region III has opposing fields.
Since the 4A wire has a larger current
but also a
larger distance away from any point in
Region III and the 3A wire has a
smaller current but a
closer distance to any point in Region
III it is possible that these two factors
compensate to
Regions I and III
make equal B fields that oppose and
only
none
could cancel out.
Using RHRsolenoid the top of the loop
is N and the bottom is S. Drawing a
field line out of the
top and looping outside down to the
bottom, you have to continue up
through the solenoid to
complete the field line so the direction
is UP. (Note: this may seem
counterintuitive because
the field line points from the south to
the north which is opposite of what you
might think but
in no direction
this is INSIDE the solenoid (magnet).
upward toward the since the magnetic Only outside, do lines come out of N
top of the page
field is zero
and into S.)

out of the page

into the page

to the right

up the plane of the down the plane of


page
the page

To the left

zero.

Use RHRflat
We first need to determine the direction
of the B field at P due to the other wires
using RHRcurl.
The top wire creates a B field pointing
up&right, the bottom wire creates a B
field pointing
up&left. The left and right parts of these
cancel out making a field only up from
these two
wires. The wire on the left also
produces a field only up so the net B
field points up at
Up the plane of the Down the plane of
location P. Now using RHRflat for the
To the right
page
the page
There is no force. right wire, the force is left.
First determine the B field direction
created by the current wire at the
location above the wire
using RHRcurl. This gives B out of
directed toward the page. Then use LHRflat for the
directed out of the directed into the
directed toward the bottom of the
negative charge to get
plane of the page. plane of the page. top of the page.
page.
force acting down.

20.magnetostatics.
png

21.magnetostatics.
PNG

22.magnetostatics.
PNG

23.magnetostatics.
PNG

24.magnetostatics.
PNG

An electron moves in the plane of the page through two


regions of space along the dotted-line trajectory shown in
the figure. There is a uniform electric field in Region I
directed into the plane of the page (as shown). There is no
electric field in Region II. What is a necessary direction of
the magnetic field in regions I and II? Ignore gravitational
25 magnetostatics forces.

A proton moves straight up the plane of this page into a region that has a
magnetic field directed
to the right. If the particle is undeflected as it passes through this region, in
what direction must
26 magnetostatics there be a component of electric field? Ignore gravity.

To the left

Into the page

Out of the page

Down the page

To the right

For the figure shown, the variable resistance in the circuit is increased at a
constant rate. What is the direction of the magnetic field at the point P at the
27 magnetostatics center of the circuit

In region I, the electric field pushes the


negative electron with a force opposite
the direction of
the E field (out of the page). For the
charge to not be pushed out, the
magnetic field must
create a force into the page to resist
this. Based on LHRflat the B field must
point up. Then
in region II based on how the charge
gets pushed, its magnetic force is up
initially. Using
LHRflat again in region II gives B field
direction out of the page.
Based on RHRflat the magnetic force
is directed into the page. To be
undeflected, the E field
must create a force out of the page to
resist this, and since its a + charge the
E field points
out.
This is a loop. Current flows clockwise
around the loop. Using the RHR
solenoid for the single
loop the B field in the center is pointing
into the page.
Charges moving without energy loss
have to maintain a constant radius
circle. For the circle to
decrease in radius, energy would be
radiated out from it. Since its an
electron we use
LHRflat to get a force pointing down
making it follow path D.

directed into the


page

directed out of the directed to the


page
right

directed to the left

zero

Use RHRflat

to the left

to the right

into the page

out of the page

Use RHRcurl
Using RHRcurl we find the direction of
the magnetic field from each wire. To
the right of the
leftmost wire, its field points down along
the axis with a decreasing magnitude
as you move
away from it. For the rightmost wire its
field also points down when you move
left of it.
Since both fields point down between
the wires, they will add and cannot
cancel. On the far
right side of the arrangement, the
leftmost wire makes a field down and
the rightmost wire
makes a field up but since the
distances to any location are different
from each wire the
magnitude of the fields would be
different so no way to cancel. The same
would happen on
31.magnetostatics.
the far left of the wires.
PNG

Which of the paths represents the path of an electron traveling


without any loss of energy through a uniform magnetic field directed
28 magnetostatics into the page?
A wire in the plane of the page carries a current directed toward the
top of the page as shown. If the wire is located in a uniform
magnetic field B directed out of the page, the force on the wire
29 magnetostatics resulting from the magnetic field is
The direction of the magnetic field at point R caused by the current I in
30 magnetostatics the wire shown is

Two long, parallel wires are separated by a distance d, as shown. One wire
carries a steady current I into the
plane of the page while the other wire
carries a steady current I out of the page.
At what points in the plane of the page and
outside the wires, besides points at infinity,
is the magnetic field due to the currents
zero?
31 magnetostatics (A) Only
An electron is in a uniform magnetic field B that is directed out of the
plane of the page, as shown. When the electron is moving in the plane of
the page in the direction indicated by the arrow, the force on the electron is
32 magnetostatics directed

toward the wire

(B) At all points on


(A) Only at point P the line SS'
two wires

A) toward the right B) out of the page

A) Changing the
magnitude of the
magnetic field

B) Increasing the
diameter of the
circle by
strenching the
spring

C) Rotating the
spring about a
diameter

Of the following, which is the best estimate of the work done by the magnetic
34 magnetostatics field on the protons during one complete orbit of the circle?
A

0J

10^-22 J

10^-5 J

Of the following, which is the best estimate of the speed of a proton in the
35 magnetostatics beam as it moves in the circle?

10^-2 m/s

10^3 m/s

10^6 m/s

A metal spring has its ends attached so that it forms a


circle. It is placed in a uniform magnetic field, as
shown. Which of the following will not cause a current
33 magnetostatics to be induced in the spring?

C) into the page

(D) At all points on


a circle of radius
2d centered on
point P
(E) At no points

(D) toward the top E) toward the


of the page
bottom of the page Use LHRflat
To induce a current, the flux through
the spring loop must change. When
moving the spring
parallel to the magnetic field, the same
B field and the same area is enclosed
D) Moving the
E) Moving the
in the loop so
spring parallel to
spring in and out of the flux stays constant and there is no
the magnetic field the magnetic field induced current.
When moving in a circle at constant
velocity, no work is done as explained
10^2 J
10^20 J
in previous answers.
Choose 1 proton moving in the circle.
For this proton. Fnet(C) = mv2/r Fb =
mv2/r
qvB = mv2/r v = qBr/m = 1.6x1019
(0.1)(0.1) / (1.67x1027) ~ 1021 / 10
10^8 m/s
10^15 m/s
27

25.magnetostatics.
png

27.magnetostatics.
png

28.magnetostatics.
PNG

29.magnetostatics.
PNG
30.magnetostatics.
PNG

32.magnetostatics.
PNG

33.magnetostatics.
PNG

Two parallel wires, each carrying a current I, repel each other with a force F.
36 magnetostatics If both currents are doubled, the force of repulsion is

An electron e and a proton p are simultaneously released from


rest in a uniform electric field E, as shown. Assume that the
particles are sufficiently far apart so that the only force acting
on each particle after it is released is that due to the electric
field. At a later time when the particles are still in the field,
37 magnetostatics the electron and the proton will have the same
As shown, a positively charged particle
moves to the right without deflection
through a pair of charged plates. Between
the plates are a uniform electric field E of
magnitude 6.0 N/C and a uniform
magnetic field B of magnitude 2.0 T,
directed as shown in the figure. The speed
38 magnetostatics of the particle is most nearly
Two long, parallel wires, fixed in space, carry currents I1 and I2
. The force of attraction has magnitude F. What
39 magnetostatics currents will give an attractive force of magnitude 4F?

2F

direction of motion Speed

0.33 m/s

0.66 m/s

I2 & 2I

I1 and I2

1 induction

The rate of change of flux has which of the following units

2 induction

For the solenoids shown in the diagram (which are assumed to be close to
each other), the resistance of the left-hand circuit is slowly increased. In
which direction does the ammeter needle (indicating the direction of
conventional current) in the right-hand circuit deflect in response to this
change?

3 induction

A strong bar magnet is held very close to the opening of a solenoid as shown
in the diagram. As the magnet is moved away from the solenoid at constant
speed, what is the direction of conventional current through the resistor
shown and what is the direction of the force on the magnet because of the
induced current?
A

A magnet is dropped through a vertical copper pipe slightly larger than the
magnet. Relative to the speed it would fall in air, the magnet in the pipe falls.

more slowly
because it is
attracted by the
innate magnetic
field of the pipe

4 induction

22 F

Farads

The needle
deflects to the left

4F

Displacement

3.0 m/s

As described in question 17, the force


on either wire is Fb = (oIa / 2R) Ib
A
L. So doubling both
42 F
8F
Is in the equation gives 4x the force.
Not a magnetism question, but lets
review. Since the charge magnitude is
the same, they will
experience the same forces based on
Fe=Eq, but move in opposite directions.
Since the
masses are different, the same forces
will affect each object differently so that
the smaller
mass electron accelerates more, thus
gains more speed and covers more
distance in equal
magnitude of force time periods. So only the force is the
37.magnetostatics.
Magnitude of Accel acting on them
same.
PNG

12 m/s

18 m/s

To be undeflected, the electric and


magnetic forces must balance.
Fe = Fb Eq = qvB v = E / B = 6 / 2

38.magnetostatics.
PNG

the force on either wire is Fb = (oIa /


2R) Ib
Since the particle is moving parallel to
the field it does not cut across lines and
Joules
Volts
m/s
Webers
has no force.
We first need to determine the direction
of the B field at P due to the other wires
using RHRcurl. The top wire creates a
B field pointing up&right, the bottom
wire creates a B field pointing up&left.
The left and right parts of these cancel
out making a field only up from these
two wires. The wire on the left also
The needle rotates
produces a field only up so the net B
The needle
in
field points up at location P. Now using
The needle
oscillates back and counterclockwise The needle never RHRflat for the right wire, the force is
deflects to the right forth
circles
moves
left.
2.induction.png
A complex problem. On the left
diagram, the battery shows how +
current flows. Based on this it flows left
through the resistor and then down on
the front side wires of the solenoid.
Using the RHRsolenoid, the right side
of the solenoid is the North pole. So
field lines from the left solenoid are
pointing to the right plunging into the
solenoid core of the right side circuit. As
the resistance in the left side increases,
less current flows, which makes the
magnetic field lines created decrease in
value. Based on Lenz law, the right side
solenoid wants to preserve the field
lines so current flows to generate field
lines to the right in order to maintain the
flux. Using the RHRsolenoid for the
right hand solenoid, current has to flow
down on the front side wires to create
the required B field. Based on this,
current would then flow down the
resistor and to the left through the
B
C
D
E
ammeter.
3.induction.png
Similar to the problem above. The field
lines from the bar magnet are directed
to the left through the solenoid. As the
magnet is moved away, the magnitude
of the field lines directed left in the
solenoid decrease so by Lenz law the
solenoid makes additional leftward field
to maintain the flux. Based on RHR
more slowly
more quickly
solenoid, the current would flow up the
because the
more quickly
because the
front side wires of the solenoid and then
currents induced in
because it is
currents induced in to the right across the resistor. This
the pipe produce
attracted by the
the pipe produce
also means that the left side of the
an opposing
innate magnetic
an opposing
solenoid is a N pole so it attracts the S
magnetic field
at the same rate
field of the pipe
magnetic field
pole of the nearby magnet.
I1 and I2

2I1 and 2I2

4I1 and 4I2

6 induction

A 0.20 m long copper rod has constant velocity 0.30 m/s traveling through a
uniform magnetic field of 0.060 T. The rod, velocity, and magnetic field are all
mutually perpendicular. What is the potential difference induced across the
rods length?
B
When a wire moving through a magnetic field has a voltage induced between
the wires ends, that voltage is
I. directly proportional to the strength of the magnetic field
II. directly proportional to the velocity of the wire
III. directly proportional to the diameter of the wire
A

7 induction

Lenzs law concerning the direction of an induced current in a conductor by a


magnetic field could be a restatement of
D

5 induction

8 induction

9 induction

A square loop is placed in a uniform magnetic field perpendicular to the plane


of the loop as shown. The loop is 0.50 meters on a side and the magnetic
field B has a strength of 2 T. If the loop is rotated through an angle of 90 in
0.1 second what would be the average induced EMF in the loop?
D
The figure shows a bar moving to the right on two conducting rails. To make
an induced current i in the direction indicated, in what direction would the
magnetic field be in the area contained within the conducting rails?
C

0.0036 V

0.040 V

0.090 V

1.0 V

25 V

As the magnet falls down towards the


pipe, which is a looped conductor, the
magnetic field lines plunging into that
conductor increase in magnitude.
Based on Lenzs law, current flows in
the conductor to oppose the gain in
field and maintain the flux. The copper
loop will create a B field upwards to
maintain flux and this upwards B field
will be opposite from the magnets B
field which will make it slow.

I only

II only

III only

II and III only

Plug into =BLv

Amperes law

Ohms Law

Teslas Law

I and II only
The Law of
Conservation of
Energy

0.025 C

0.40 V

out of the page

into the page

10 induction

There is a counterclockwise current I in a circular loop of wire situated in an


external magnetic field directed out of the page as shown. The effect of the
forces that act on this current is to make the loop

expand in size

contract in size

11 induction

The figure shows a rectangular loop of wire of width l and resistance R. One
end of the loop is in a uniform magnetic field of strength B at right angles to
the plane of the loop. The loop is pulled to the right at a constant speed v.
What are the magnitude and direction of the induced current in the loop?

1x10^-4 V

2.5x10^-3 V

rotating the
secondary coil
about the z-axis

rotating the
secondary coil
about a diameter

13 induction

In each of the following situations, a bar magnet is aligned along the axis of a
conducting loop. The magnet and
the loop move with the indicated velocities. In which situation will the bar
magnet NOT induce a current in the
conducting loop?
C
A square loop of copper wire is initially placed perpendicular to the lines of a
constant magnetic field of
5 x 10-3 tesla. The area enclosed by the loop is 0.2 square meter. The loop is
then turned through an angle of
90 so that the plane of the loop is parallel to the field lines. The turn takes
0.1 second. The average emf
induced in the loop during the turn is
E

14 induction

Two circular coils are situated perpendicular to the z-axis as shown.


There is a current in the primary coil. All of the following procedures
will induce a current in the secondary coil EXCEPT

12 induction

none of these

Based on =BLv
This is a fact. It is best thought about
through example and thinking about
how nonconservative forces are at
play. Lenz law says opposing fields are
induced for moving magnets, this slows
them if the opposite was true you
would get accelerated systems where
5V
10 V
80 V
energy would not be conserved
Use = / t
= ( BAf BAi ) / t
to the right
to the left
It is impossible
= (0(2)(0.5x0.5))/0.1
The rail makes a loop of wire as shown
by the current flow. Using Lenz law, as
the loop expands with the motion of the
bar, it is gaining flux lines in whatever
direction the B field is and the loop
current flows in a direction to oppose
that gain. Using RHRsolenoid for the
single loop, the B field induced is
rotate about an
rotate about an
directed out of the page so it must be
axis perpendicular axis in the plane of accelerate into the opposing the gain of B field that is
to the page
the page
page
already there going into the page.
Take a small section of wire on the loop
at the top, bottom, right and left hand
sides and find the forces on them. For
example, the section of wire on the top
has current pointing left and B pointing
out using RHRflat for that piece
gives a force pointing up. At all of the
positions, the force acts in a manner to
C
D
E
pull the loop outwards and expand it.
The induced emf occurs in the left side
vertical wire as that is where the charge
separation
happens. Looking at that wire, the
induced emf is given by =BLv. This
emf then causes a
current I to flow in the loop based on
V=IR, so I is given as BLv / R. The
direction of that
current is found with Lenz law as there
is a loss of flux into the page, RHR
solenoid shows
current must flow CW to add back flux
C
D
E
into the page and maintain it.
As long as the flux inside the loop is
changing, there will be an induced
current. Since choice E
has both objects moving in the same
direction, the flux through the loop
remains constant so
0.01 V
100 V
400 V
no need to induce a current.
moving the
decreasing the
secondary coil
cross-sectional
closer to the
varying the current area of the
Same as question 35, different
primary coil
in the primary coil secondary coil
numbers

8.induction.png

9.induction.png

10.induction.png

11.induction.png

12.induction.png

14.induction.png

15 induction

16 induction

17 induction

A magnetic field B that is decreasing with time is directed out of the page
and passes through a loop of wire in the plane of the page, as shown. Which
of the following is true of the induced current in the wire loop?
A wire of constant length is moving in a constant magnetic field, as shown.
The wire and the velocity vector are perpendicular to each other and are
both perpendicular to the field. Which of the following graphs best
represents the potential difference E between the ends of the wire as a
function of velocity?
A square loop of wire of resistance R and side a is oriented with its plane
perpendicular to a magnetic field B, as shown. What must be the rate of
change of
the magnetic field in order to produce a current I in the loop?

21 induction

A rectangular wire loop is at rest in a uniform magnetic field B of


magnitude 2 T that is directed out of the page. The loop measures
5 cm by 8 cm, and the plane of the loop is perpendicular to the field, as
shown. The total magnetic flux through the loop is
A single circular loop of wire in the plane of the page is perpendicular to
a uniform magnetic field B directed out of the page, as shown. If the
magnitude of the magnetic field is decreasing, then the induced current in
A uniform magnetic field B that is perpendicular to the plane of the page
now passes through the loops, as shown. The field is confined to a region
of radius a, where a < b, and is changing at a constant rate. The induced
emf in the wire loop of radius b is . What is the induced emf in the
wire loop of radius 2b ?
Two conducting wire loops move near a very
long, straight conducting wire that carries a
current I. When the loops are in the positions
shown, they are moving in the direction shown
with the same constant speed v. Assume that
the loops are far enough apart that they do not
affect each other. Which of the following is
true about the induced electric currents, if any,
in the loops

22 induction

A wire loop is rotated in a uniform magnetic field about an axis perpendicular


to the field, as shown. How many times is the induced current in the loop
reversed if the loop makes 3 complete revolutions from the position shown?

18 induction

19 induction

20 induction

Looking at the primary coil, current


flows CCW around it so based on
RHRsolenoid the
magnetic field lines from that coil are
pointing to the left and they extend into
the secondary
coil. To induce a current in the
secondary coil, the flux through the
secondary coil needs to
be changed so an induced current will
flow based on Lenz law. Choice A
means spinning
the coil in place like a hulahoop or a
spinning top and this will not cause a
change in flux.
15.induction.png
Based on Lenz law, as the flux pointing
up decreases, current flows in the loop
to add back that
lost flux and maintain it. Based on
RHRsolenoid, current would have to
flow CCW
16.induction.png

It is
counterclockwise
in direction.

It is clockwise in
direction.

It is directed into
the page.

It is directed out of It is zero in


the page.
magnitude.

IR/a2

Ia2
/R

Ia/R

Ra/I

IRa

zero

2 x 10-3 T-m2

8 x 10-3 T-m2

2 x 10-1 T-m

8 x 10-1 T-m

directed out of the directed into the


paper
paper

clockwise around
the loop

counterclockwise
around the loop

Zero

zero (no current is


induced)
= BA = (2)(0.05)(0.08)
19.induction.png
From Lenz law, as the flux decreases
the loop induces current to add back
that declining field.
Based on RHRsolenoid, current flows
4
CCW to add field coming out of page.
20.induction.png

/2

One

Two

Three

Six

Twelve

Based on =BLv, its a linear variation 17.induction.png


We are looking to find rate of change of
magnetic field B/t so we need to
arrange equations to
find that quantity. Using induced emf for
a loop we have. = / t = BA/t, and
substituting V=IR, and area = a2 we
have IR = B (a2) / t isolate B/t
to get answer
18.induction.png

Since both loops contain the same


value of BA and it is changing the same
for both of them, the
quantity BA/t is the same for both so
both have the same induced emf.
21.induction.png
Above the wire is a B field which is
directed into the page based on
RHRcurl. That B field has a decreasing
magnitude as you move away from the
wire. Loop 1 is pulled up and therefore
is
loosing flux lines into the page. By Lenz
Law current flows to maintain those
lines into the
page and by RHRsolenoid current
would have to flow CW to add lines into
the page and
maintain the flux. Loop 2 is moving in a
direction so that the magnitude of flux
lines is not changing and therefore
there is no induced current
22.induction.png

23 induction

If the bar is pushed northward on the rails, the electromotive force induced in
the bar as a result of the magnetic field will
D

Be directed
upwards

Be zero

Produce a
westward current

24 induction

A battery is connected between the rails and causes the electrons in the bar
to drift to the east. The resulting magnetic force on the bar is directed

North

South

East

25 induction

A battery is connected between the rails and causes the electrons in the bar
to drift to the east. The resulting magnetic force on the bar is directedvelocity
v to the right through a region of space
where there is a uniform magnetic field B
directed into the page, as shown. The induced
current is as follows

Directed CW both
entering and
leaving
REGION II

Directed CCW
both entering and
leaving
REGION II

Directed CW
entering REGION
II and
CCW leaving
REGION II

26 induction

A square loop of wire of side 0.5 meter and resistance 10-2 ohm is located in
a uniform magnetic field of intensity 0.4 tesla directed out of the page as
shown. The magnitude of the field is decreased to zero at a constant rate in 2
seconds. As the field is decreased, what are the magnitude and direction of
the current in the loop?
B

Zero

5 A,
counterclockwise

5 A, clockwise

27 induction

After the switch S is closed, the initial current through resistor R2 is

from point X to
point Y

from point Y to
point X

zero at all times

28 induction

After the switch S has been closed for a very long time, the currents in the
two circuits are

zero in both
circuits

zero in circuit 1
V/R1 in circuit 1
and V/R2 in circuit and zero in circuit
2
2

This is best done holding a small


circular object like a small plate and
rotating it towards you keeping track of
the current flow. Grab the top of the
plate and pull it towards you out of the
page and move down at the same time
to rotate it. This will increase the flux
lines into the
loop as you rotate and cause a current
to flow to fight the increase until it
becomes flat and
you have moved 90 degrees in
relations to the rotation you are making.
Then as you pass this point and begin
pushing the part of the loop you are
holding down and into the page away
from you, you start to lose field lines
and current will flow the other way to try
and maintain the flux lines until your
hand has moved what was once the top
of the loop all the way to the bottom. At
this point you are 180 degrees through
the rotation and have changed direction
once. As you pass through 180, you will
notice that the current flows the same
way to maintain the zero flux you get at
the 180 location (even though you
might think there should be a change
here, this is where the physical object
helps). Then as you move up the back
and do the same thing on the reverse
side to return the part of the loop you
are holding to the top you will undergo
another direction change at 270
degrees so you have 2 direction
changes total in one revolution. Do it
Produce an
Stop the motion of two more times and you get 6
eastward current
the bar
reversals.
Since the bar is not cutting across field
lines and has no component in a
perpendicular direction to the field line
West
Vertically
there will be no induced emf.
As you enter region II, flux into the page
is gained. To counteract that, current
flows to create a field out of the page to
maintain flux. Based on RHRsolenoid,
Directed CCW
that current is CCW. When
entering REGION
leaving the region, the flux into the
II and
page is decreasing so current flows to
CW leaving
add to that field
REGION II
zero at all times
which gives CW.
Firstuse=/t =(BAfBAi)/t =(0(0.4)
(0.5x0.5))/2 =0.05V Then use V=IR
0.05V = I(.01) I = 5A
Direction is found with Lenz law. As the
field out decreases, the current flows to
add
20 A,
outward field to maintain flux. Based on
counterclockwise 20 A, clockwise
RHRsolenoid, current flows CCW.
Loop 2 initially has zero flux. When the
circuit is turned on, current flows
through loop 1 in a CW direction, and
using RHRsolenoid it generates a B
field down towards loop 2. As the
field lines begin to enter loop 2, loop 2
has current begin to flow based on lenz
law to try and maintain the initial zero
flux so it makes a field upwards. Based
on RHRsolenoid for
Oscillating
Impossible to
loop 2, current would have to flow CCW
between point X
determine its
around that loop which makes it go
and Y
direction
from X to Y.
After a long time, the flux in loop 2
becomes constant and no emf is
induced so no current flows. In circuit 1,
V/R1 in circuit I
oscillating with
the loop simply acts as a wire and the
and V/R2 in circuit constant amplitude current is set by the resistance and
2
in both circuits
V=IR

23.induction.png

25.induction.png

26.induction.png

27.induction.png

27.induction.png

29 induction

30 induction

31 induction

32 induction

1 Quantum

always clockwise
with increasing
magnitude

always clockwise
with decreasing
magnitude

always
counterclockwise
with increasing
magnitude

always
counterclockwise
with decreasing
magnitude

first
counterclockwise,
then clockwise

Top End: Pos


Bottom End: Neg

Top End: Neg


Bottom End: Pos

Top End: Neg


Bottom End: Zero

Top End: Zero


Bottom End: Neg

Top End: Zero


Bottom End: Zero

0.2 V

0.6 V

1.2 V

1.5 V

2.8 V

frequency of light
is decreased

frequency of light
is increased

intensity of light is
decreased

intensity of light is
increased

velocity of light is
increased

gamma

infrared

radio

visible

x-rays

(1/9)

(1/3)

As the magnet moves down, flux


increase in the down direction. Based
on Lenz law, current in the loop would
flow to create a field upwards to cancel
the increasing downwards field.
Using RHRsolenoid, the current would
flow CCW. Then, when the magnet is
pulled
upwards, you have downward flux lines
that are decreasing in magnitude so
current flows to
add more downward field to maintain
flux. Using RHRsolenoid you now get
CW.
29.induction.png
Since the wire is not cutting across the
field lines, there is no force and no
charge separation
30.induction.png
As the loop is pulled to the right, it loses
flux lines right so current is generated
by Lenz law to add more flux lines right.
This newly created field to the right
from the loop is in the same
direction as the magnetic field so
makes an attractive force pulling the
magnet right also.
31.induction.png
Use a 1 second time period, the field
would decrease to 2.5 T in that time.
Then apply = / t
= ( BAf BAi ) / t = A (Bf Bi)
= (0.4)(3 2.5) / 1
Standard photoelectric effect question.
If the frequency does not cause
emission, it is below the
threshold and will not be able to cause
emission. The only way to cause
emission is the increase
the frequency above the threshold.
E=hf. Energy is directly related to
frequency. The higher frequency means
more energy.
de Broglie wavelength is given by, p = h
/ mv = h / = h / mv 3x m =
1/3

6.0 eV

From K = hf y = mx + b the
work function is the y intercept, extend
the line.

In the figure, the north pole of the magnet is first moved down toward the loop
of wire, then withdrawn upward. As viewed from above, the induced current in
the loop is
E
A vertical length of copper wire moves to the right with a steady velocity v in
the direction of a constant horizontal magnetic field B as shown. Which of the
following describes the induced charges on the ends of the wire?
E

A conducting loop of wire that is initially around a magnet is pulled away from
the magnet to the right, as indicated in the figure, inducing a current in the
loop. What is the direction of the force on the magnet and the direction of the
magnetic field at the center of the loop due to the induced current?
A
A uniform magnetic field B is directed out of the page, as shown above.
A loop of wire of area 0.40 m^2 is in the plane of the page. At a certain
instant the field has a magnitude of 3.0 T and is decreasing at the rate
of 0.50 T/s. The magnitude of the induced emf in the wire loop at this
instant is most nearly
A

Light of a single frequency falls on a photoelectric material but no electrons


are emitted. Electrons may be
emitted if the

5 Quantum

Which of the following types of electromagnetic radiation has the least energy
per photon
An atomic particle of mass m moving at speed v is found to have wavelength
. What is the wavelength of a
second particle with a speed 3v and the same mass
A student performs the photoelectric effect experiment and obtains the data
depicted in the accompanying graph
of Ekm (max kinetic energy) of photoelectrons vs the frequency of the
photons. What is the approximate work
function of this material?
According to the Bohr theory of the hydrogen atom, electrons starting in the
4th energy level and eventually
ending up in the ground state, could produce a total of how many lines in the
hydrogen spectra?

6 Quantum

In Rutherfords famous gold foil scattering experiment, he found that most


alpha particles would pass through the
foil undeflected. Which of the following nuclear properties can be inferred
from this observation.

7 Quantum

Which of the following is best explained only by the wave theory of light

2 Quantum

3 Quantum

4 Quantum

1.5 eV

2.0 eV

The nucleus must


have a positive
charge
blackbody
radiation

2.7 eV

The nucleus
Most of the mass contains both
of an atom is in the protons and
nucleus
neutrons
the photoelectric
the Compton effect effect

4.0 eV

43,32,21 or 42, 21 or 43,


31 or 41 this is a total of 6
5
4
3 different transitions.
While Rutherfords experiment did show
most of these, looking at the single fact
The diameter of
provided (that
the nucleus is
most particles pass straight through)
small compared to
only meant that most of the atom was
the diameter of the
empty space and that
atom
none of the above. the nucleus must be small.
pairproduction

diffraction

Diffraction is a unique wave effect.

4.quantum.png

8 Quantum

In the photoelectric effect experiment, a stopping potential of Vstop is needed


when light of frequency f shines on
the electron-emitting metal surface. If the metal surface on which the light
shines is replaced with a new
material that has half the work function, what is the new stopping potential,
Vnew for light of frequency shining
on it?
E

9 Quantum

The diagram to the right shows the lowest four energy levels for an electron in
a hypothetical atom. The electron is excited to the 1 eV level of the atom
and
transitions to the lowest energy state by emitting only two photons. Which of
the following energies could not belong to either of the photons?
B

10 Quantum

11 Quantum

Monochromatic light falling on the surface of an active metal causes electrons


to be ejected from the metallic
surface with a maximum kinetic energy of E. What would happen to the
maximum energy of the ejected
electrons if the frequency of the light were doubled?
E
If the electrons in an electron microscope are traveling with a velocity of 1.6
x107 m/s, what would be the
effective wavelength of the electrons?
C

14 Quantum

A very slow proton has its kinetic energy doubled. What happens to the
protons corresponding deBroglie
wavelength
The diagram shows light being emitted due to
a transition from the n=3 to the n=2 level of a
hydrogen atom in the Bohr model. If the
transition were from the n=3 to the n=1 level
instead, the light emitted would have
Which color of light emitted from an atom would be associated with the
greatest change in energy of the atom?

15 Quantum

Which graph best shows the maximum kinetic energy K of the photoelectrons
as a function of the frequency of
incident light?
A

12 Quantum

13 Quantum

E
D

Stopping potential is given by.


K=Vstopq ... combined with K = hf ,
we see the stopping
potential is related to the incoming light
energy minus the work function.
However, none of the
choices give a proper result. The
answer depends on what that actual
incoming energy hf and
work function are. Here is an example.
Lets say hf was 3eV and the was 2
eV initially. From
Vq=hf the stopping potential for
an electron (1e) would be equal to 1eV.
Now if we were
to half the work function, the new
stopping potential would be 3eV 1eV
= 2 eV so it appears
that the stopping potential doubled. But
that result only works for those sample
numbers. Lets
assume instead that hf = 10eV and =
2eV. Now initially the Vstop = 8eV.
Then we again half
the work function 10eV 1 eV and
we get a stopping potential of 9V .. not
nearly doubled
this time. The results depend on the
actual numbers used because of the
It is indeterminate minus sign in the
Vstop < Vnew <
with the given
equation and not a simple multiplier
Vnew > 2Vstop
Vnew = 2 Vstop
2Vstop
Vnew = Vstop
information
effect.
To transition to the 12eV state with
only two photon emissions, the only
options are for the
electron to make the following
transitions: 1eV 3eV 12eV
giving us photons of energy
2eV and 9eV or 1eV 7 eV
12eV giving photons of energy 6eV and
5eV . This means
that the 4 eV photon is not possible with
2 eV
4 eV
5 eV
6 eV
9 eV
only two transitions.
9.quantum.png
K = hf now double hf Knew =
2hf
(now sub in the first equation
rearranged for hf, into the second
equation)
Knew = 2(K+ ) = 2K + 2
the maximum
the maximum
the maximum
the maximum
the maximum
Knew = 2K +
energy of the
energy of the
energy of the
energy of the
energy of the
So the new energy is increased by
electrons would be electrons would be electrons would be electrons would be electrons would be double the old energy + the work
less than E
E
(2) E
2E
greater than 2E
function value
p = h / mv = h / = h / mv =
(6.63x104) / (9.11x101)(1.6x107) =
1.2x108 m
6.6x109 m
4.5x1011 m
2.6x1011 m
8.6x1017 m
4.5x101 m
From above = h / mv Since K =
mv2 , 2x K means 2x v. So when we
plug in the new
the wavelength is
the wavelength is
velocity of 2v, the wavelength
decreased by a
the wavelength is there is no change increased by a
the wavelength is decreases by this factor since we
factor of 2
halved
in the wavelength factor of 2
doubled.
divide.
3 to 1 would be a higher energy
emission. More energy means more
frequency, and less but
these are not choices. From p = h / ,
lower frequency
less energy
longer wavelength greater speed
greater momentum less means more momentum.
13.quantum.PNG
From E = hf, more frequency = more
Blue
Green
Red
Violet
Yellow
energy.
Below a threshold frequency, there
would be no emissions and thus zero K
for everything below
that point. Above that threshold, more
frequency means more K based on K =
hf , with h as
the constant slope. Graph A has all
A
B
C
D
E
these properties.
15.quantum.PNG

16 Quantum

Which graph best shows the maximum kinetic energy K of a photoelectron as


a function of the intensity of
incident light?
E

17 Quantum

Electrons that have been accelerated from rest through a potential difference
of 150 volts have a de Broglie
wavelength of approximately 1 Angstrom (10l0 meter). In order to obtain
electrons whose de Broglie
wavelength is 0.5 Angstrom (5 x 10lI meter), what accelerating potential is
required?
D

37.5 V

75 V

300 V

18 Quantum

According to the Bohr model of the atom, electrons orbit the nucleus in
definite orbits. According to the laws of
classical physics, this model would be impossible because

the positively
charged nucleus
attracts the
electrons

Coulomb's law
applies

accelerating
electrons radiate
energy

19 Quantum

The energy level diagram is for a hypothetical atom. A


gas of these atoms initially in the ground state is
irradiated with photons having a continuous range of
energies between 7 and 10 electron volts. One would
expect photons of which of the following energies to be
emitted from the gas?

1, 3, 5, and 10 eV
1, 2, and 3 eV only 4, 5, and 9 eV only only

Intensity has no effect on the energy of


a single given photoelectron. Each
photoelectrons
energy is simply based on K = hf .
More intensity means a larger total
number of
photoelectrons and would result in
more total energy, but the energy of
each photoelectron is the
same for all levels of the overall
D
E
intensity.
16.quantum.PNG
The following formulas apply. K = Vq
K = mv2 p = mv p=h/
To get half the , the p must be doubled

To double the momentum, the velocity


must be doubled
When the velocity is doubled, the
Kinetic energy is 4x as much
To get 4x the K, we need 4x the
600 V
22,500 V
potential.
According to classical physics, when
charges accelerate in circles, they
necessarily radiate energy
in the form of light. This would cause
them to spiral into the nucleus as they
radiate continuous
there is a
angular
spectrums of color. This does not
centripetal force on momentum is
happen though, which is a flaw in the
the electrons
conserved
Rutherford model.
The ground state is at 14eV. The next
excited state is 4eV higher (at 10eV)
which cant be
reached since we are only putting in a
range of 710. So we try the next jump
to the 5eV state.
This would require and input of 9 eV
and this is possible since it falls in the
range. The next state
3eV is not possible since it would
require 11 eV input. So the only excited
state we can be
brought to with this energy input is the
5eV state. From this state we will now
go through
emissions as the electrons fall back
Since the original down to the ground state. This can be
done through three
photons have a
range of energies, possible jumps:
one would expect 5eV10eV, then 10eV14eV or it
a range of emitted could go directly from 5eV14eV.
1, 5, 7, and 10 eV photons with no
In these three scenarios, the emissions
only
particular energies. possible are 5, 4 and 9.
19.quantum.PNG

20 Quantum

21 Quantum

All of the following are properties of x-rays EXCEPT:


C
Which of the following graphs best represents the de Broglie wavelength of
a particle as a function of the
linear momentum p of the particle?
D

They penetrate
light materials.

They are deflected They discharge


They ionize gases. by magnetic fields. electrified bodies.

Xrays do not have a charge so would


not be deflected by a magnetic field. All
of the rest of the
listed properties are true however. a) x
rays clearly pass through light materials
as evidenced
from their use in the medical field. b)
From Bohrs energy level diagram for
hydrogren we can
conclude this is true. The differences
between levels on the diagram
represent energies needed to
jump levels, and these energies
correspond to visible and UV light
energies. The energy listed
for each level is the ionization energy,
which is the energy needed to remove
an electron. Any
energy larger than or equal to the
ionization energy for a level will do.
Since Xrays have such
high energy, they clearly will be able to
ionize any level in hydrogen gas c) not
true. d) The
Compton effect shows this ability to
strip electrons. e) An x ray is an EM
wave and like all waves
should diffract. Since the wavelength is
so small, they would have to be
diffracted by very small
They are diffracted openings such as crystal structures in
by crystals.
atoms.v

the fraction of
particles scattered
through large
angles was too
large to be
explained by
no particles
the most common
previous models of passed through
scattering angle
the atom
the foil undeflected was about 90

the most common


scattering angle
was about 180

The production of
x-rays by electrons The scattering of
striking a metal
photons by
target
electrons at rest

From p=h/ , they are inverses.


21.Quantum.PNG
A) Not true. When particles came near
the nucleus, most of them were
deflected up or down
through angles less than 90. A few of
them, were deflected back at angles
larger than 90.
B) TRUE Previous models could not
accounts for the particles that got
scattered through large
angles. These large angle scattering
events prompted Rutherford to
conclude a concentrated +
nucleus to produce this result.
Electron jumps could happen as follows
43,32,21 or 42, 21 or 43,
31 or 41.
In these emissions from the differences
in the energy level graph, we can make
all of the energy
difference choices except 4 eV.
23.Quantum.PNG
Big means the least energy based on
E=hc/ . The least energy corresponds
to the smallest
energy level jump which is 4.
24.Quantum.PNG
The photoelectric effect is the main
proof of lights particle nature. All of the
other choices are
related to the proof of wave natures.
The DavissonGermer experiment
involves the diffraction of electron
particles through a nickel
crystal. Since these particles diffracted,
this demonstrated the wave nature of
particles.

II and III only

This is explained in question 16. K is


based on the work function (which is
based on the nature
of the surface) and K is also based on
the frequency of the incoming light.

22 Quantum

The scattering of alpha particles by a thin gold foil was measured by Geiger
and Marsden. The Rutherford model of the atom was proposed in order to
explain why

more particles
scattered through
angles greater
than 90 than
through angles
less than 90

23 Quantum

Which of the following photon energies could NOT be


found in the emission spectra of this atom after it has been
excited to the n = 4 state?

1 eV

2 eV

3 eV

4 eV

5 eV

24 Quantum

Which of the following transitions will produce the photon with the longest
wavelength?

n = 2 to n = 1

n = 3 to n = 1

n = 3 to n = 2

n = 4 to n = 1

n = 4 to n = 3

25 Quantum

Of the following phenomena, which provides the best evidence that light can
have particle properties?

Interference of
light in thin films

Electromagnetic
radiation

Photoelectric effect Electron diffraction X-ray diffraction

The absorption of
photons by
electrons in an
atom

The alpha-decay
of radioactive
nuclei

The interference
pattern produced
by neutrons
incident on a
crystal

26 Quantum

27 Quantum

Of the following phenomena, which provides the best evidence that particles
can have wave properties?
In the photoelectric effect, the maximum speed of the electrons emitted by a
metal surface when it is illuminated
by light depends on which of the following?
I. Intensity of the light
II. Frequency of the light
III. Nature of the photoelectric surface

I only

III only

I and II only

I, II, and III

28 Quantum

In the Bohr model of the atom, the postulate stating that the orbital angular
momentum of the electron is
quantized can be interpreted in which of the following ways?

An integral number
of electron
wavelengths must
fit into the
electron's circular
orbit.

the inner electron


shells

31 Quantum

Quantum transitions that result in the characteristic sharp lines of the X-ray
spectrum always involve
Which of the following experiments provided evidence that electrons exhibit
wave properties?
I. Millikan oil-drop experiment
II. Davisson-Germer electron-diffraction experiment
III. J. J. Thomson's measurement of the charge-to-mass ratio of electrons
If the momentum of an electron doubles, its de Broglie wavelength is
multiplied by a factor of

32 Quantum

Quantum concepts are critical in explaining all of the following EXCEPT

33 Quantum

If photons of light of frequency f have momentum p, photons of light of


frequency 2f will have a momentum of

34 Quantum

In an experiment, light of a particular wavelength is incident on a metal


surface, and electrons are emitted from
the surface as a result. To produce more electrons per unit time but with less
kinetic energy per electron, the
experimenter should do which of the following?
B

29 Quantum

30 Quantum

36 Quantum

Which of the following imposes a limit on the number of electrons in an


energy state of an atom?
Which graph shows the maximum kinetic energy of the emitted electrons
versus the frequency of the light?

37 Quantum

Which graph shows the total photoelectric current versus the intensity of the
light for a fixed frequency above
the cutoff frequency?

35 Quantum

An electron has a
spin of 1/2.

The atom is
composed of a
small, positively
charged nucleus
orbited by
electrons.

emission of beta
particles from the
nucleus

neutrons within the protons within the


nucleus
nucleus

An obscure fact. Since the emission of


Xray photons are high energy, they
must involve

The DavissionGermer experiment is


discussed in question 26. The other two
choices have
I only
II only
I and III only
II and III only
I, II, and III
nothing to do with matterwaves.
From p=h/ , 2x p means the
0.25
0.5
1
2
4 wavelength.
Rutherfords experiment was not a
quantum concept; it was on the atomic
level and led to a
model of the atom. All of the other
Rutherford's
choices involve a quantization effect or
scattering
Bohr's theory of
Compton
the blackbody
the photoelectric
particle nature of
experiments
the hydrogen atom scattering
spectrum
effect
light which are quantum concepts.
From p=h/ , and c=f p = hf/c.
There is a direct relationship between p
2p
(2)^1/2p
p
p/(2)^1/2
p
and f.
The K of each photoelectron is given
by. K = hf . To reduce the energy of
each photon, we
Increase the
Decrease the
need less f (which means more ) for
intensity and
Increase the
Decrease the
intensity and
the incoming light. Since intensity is
decrease the
intensity and the
intensity and the
increase the
None of the above directly related to the
wavelength of the wavelength of the wavelength of the wavelength of the would produce the number of photoelectrons emitted we
light.
light.
light.
light.
desired result.
want to increase the intensity.
A fact. The Pauli exclusion principle
involves the filling of orbitals by
The law of
electrons and how many
The Heisenberg
conservation of
electrons fill each orbital. This is related
uncertainty
The Pauli
The Bohr model of The theory of
energy
to the quantum state of the electrons in
principle
exclusion principle the hydrogen atom relativity
(C)
each level.

10^8

10^22

10^30

10^40

10^56

/h

h/

/ hc

hc /

Same, same

Greater for red,


less for red

Greater for red,


greater for red

Less for red, less


for red

Less for red,


greater for red

B
B

39 Quantum

A 50,000 W radio station transmits waves of wavelength 4 m. Which of the


following is the best estimate of the
number of photons it emits per second?
C
The work function for a metal is . What is the threshold frequency of incident
light required for the emission of
photoelectrons from a cathode made of that metal?
A

40 Quantum

Two monochromatic light beams, one red and one green, have the same
intensity and the same cross sectional
area. How does the energy of each photon and the number of photons
crossing a unit area per second in the red
beam compare with those of the green beam?

38 Quantum

Only one electron


can exist in each
possible electron
state.
electron energy
levels that have
the same principal
quantum number

The quantization of energy levels is


from de Broglie and the relationship of
momentum to
wavelength through matterwaves. de
An incident photon Broglie theorized that electrons have
is completely
wavelike properties
absorbed when it and must exist in whole number
causes an electron multiples of wavelengths around an
to move to a
orbit to so they interfere
higher energy
constructively and do not get knocked
state.
out.

Same as question 15.


36.quantum.png
The photoelectric current is directly
related to the number of photoelectrons
emitted; the more
photoelectrons the more the current.
Also, the # of photoelectrons is directly
related to the
intensity. This means that photoelectric
current and intensity also have a direct
relationship.
When we are above the threshold
frequency, 0 intensity would correspond
to 0 current, but as
intensity increases, the current
increases proportionally.
37.quantum.png
We find the total energy produced in 1
second and then use the energy of 1
photon to determine
how many photons would be emitted.
Total energy = W = Pt = 50000 (1 sec)
= 50000 J = 5x10^4
Energy of 1 photon E = hc / = 2x10^25/ 4 = 0.5x10^-25 = 5x10^-26
Total Energy / Energy of 1 photon = #
photons released.
5x10^4 / 5x10^-26 = 1030

From the equation. = hfo f = /h


Energy of a photon is related to
frequency. The red light has a lower
frequency and thus less energy per
photon. Intensity is the total energy of
the beam. To have the same intensity,
there would need to be more of the
lower energy red photons.

In an x-ray tube, electrons striking a target are brought to rest, causing x-rays
to be emitted. In a particular x-ray
tube, the maximum frequency of the emitted continuum x-ray spectrum is
fo. If the voltage across the tube is
doubled, the maximum frequency is:
E

fo /2

fo /2

fo

2fo

2fo

1 Nuclear

An atomic mass unit is approximately equal to the mass of a(n):

alpha particle

electron

photon

positron

proton

2 Nuclear

A radioactive oxygen 15O8 nucleus emits a positron and becomes

14N7

15N7

15O8

14F9

15F9

3 nuclear

A radon 220Rn86 nucleus emits an alpha particle becomes a:

216Po84

220At85

220Rn86

220Fr87

224Ra88

4 Nuclear
5 nuclear

A potassium 40K19 nucleus emits a B+ and becomes:


A photon with frequency f behaves as if it had a mass equal to:

E
B

36Cl17
hfc2

44Sc21
hf/c2

40Ar18
c2 /hf

40K19
fc2 /h

40Ca20
h/fc2

6 Nuclear
7 nuclear

What does the ? represent in the nuclear reaction 2H1 + 2H1 -> 3He2
What does the ? represent in the nuclear reaction 6Li3 + ? ->7Li3

D
D

an alpha
an alpha particle

a beta
a deuteron

a gamma
an electron

a proton
a proton

8 Nuclear

An alpha particle is the same as:

a helium nucleus

a positron

an electron

a neutron
a neutron
a high energy
photon

9 nuclear

The following equation is an example of what kind of nuclear reaction

Fission

Fusion

Alpha decay

Beta decay

Positron decay

Fission

Fusion

Alpha decay

Beta decay

Positron decay

11 nuclear

The following equation is an example of what kind of nuclear reaction


During a particular kind of radioactive decay, a particle is emitted from the
nucleus of an atom and the atoms atomic number increases by one. This
decay necessarily involves the emission of _________ from the nucleus

An alpha particle

A beta particle

A gamma ray

A proton

A neutron

12 Nuclear

A nucleus of U (atomic mass 235, atomic number 92) disintegrates to Pb


(atomic mass 207, atomic number 82) in about a billion years by emitting 7
alpha particles and x beta particles, where x is

13 nuclear

The following nuclear reaction occurs:

A proton

This is possible
because Einsteins
equation says that
mass and energy
are equivalent... it
is just very
difficult to achieve
with electrons

A.

41 Quantum

10 Nuclear

14 Nuclear

15 nuclear

A scientist claims to have perfected a technique in which he can


spontaneously convert an electron completely into energy in the laboratory
without any other material required. What is the conclusion about this claim
from our current understanding of physics?
A new element, named Physonium (symbol Phys) is discovered to undergo
double alpha decay and beta decay simultaneously. Amazingly, this causes
the material to decay into an element called Awsomeonium (symbol Oo).
What is the correct representation of the (Oo)?

An electron

a deuteron

The energy of the electrons is the


kinetic energy given by W = Vq = K.
Doubling the voltage doubles the
energy of the electrons. The emitted x
ray energy coming from the electron
energy
is given by E=hf and with double the
energy there should be doube the
frequency.
1 u = 1.66x10 is 1/12 of carbon 12 and
is approximately the same as a proton
mass.
or a positron to be emitted, the oxygen
must have undergone beta+ decay,
which is the opposite of beta decay. In
beta+ decay a proton turns into a
neutron + the emitted beta particle. The
mass number stays the same (P+N still
the same), but the atomic number goes
down by 1 since
there is 1 less proton.
An alpha particle is 4He2 so reduce the
atomic mass by 4 and the atomic
number by 2.
In beta decay, a neutron turns into a
proton. Atomic mass same, mass
number +1.
Equate E=hf to E=mc2 ... m = hf/c2.
For everything to add up properly, we
need ... 1/0... which is a neutron.
Same as above.
Definition of alpha particle.
Uranium split by a neutron is called
fission.
Merging together two elements (He
usually being one of them) is called
fusion.

9.nuclear.png

10.nuclear.png

This is the definition of Beta decay.


The mass number has changed by 28,
the atomic number has changed by 10.
7 alpha particles (4He2)*7 equates to a
loss of 28 for atomic mass and 14 for
atomic number. If
only the alpha particles were emitted, 4
protons would be missing. Those
protons must have
come from the conversion of neutrons
into protons which would happen with
the release of 4
7 beta particles.
To balance the nuclear reaction, the
sum of the values across the top and
across the bottom must match... That
is, we have 4 + 9 = 12 + A A = 1 and
2 + 4 = 6 + Z Z = 0 . This gives us
a particle with 1 nucleon, but 0 protons.
This is a neutron.
13.nuclear.png

An alpha particle

A neutron

This is possible
and it is done all
the time in the
high-energy
physics labs.

A positron
The scientist is
almost correct...
except that in
converting the
electron to energy,
an electrons antiparticle
is produced in the
process as well.

The scientist is
almost correct...
except that in
converting the
electron to energy,
a proton is
produced in the
process as well.

Conservation of charge is required.


Eliminating an electron by itself violates
this. However, when and electron ()
meets a positron (+) the matter and
This is not possible antimatter can annihilate to produce
because charge
energy and the + and charges can
conservation
neutralize to conserve charge .. just as
would be violated. a side note.

B.

C.

D.

E.

Simply balances the numbers across


the top and bottom arrives at choice E.

15.nuclear.png

17 nuclear

The most common isotope of Uranium, U (atomic mass 238, atomic number
92), radioactively decays into lead, Pb (atomic mass 206, atomic number 82),
by a means of a series of alpha and beta particle emissions. How many of
each particle must be emitted.
D
Rutherford was the first person to artificially transmute one element into
another (nitrogen to oxygen). A nuclear equation for his reaction could be
written as follows (see image). The unknown particle in the above equation is A

18 Nuclear

When a radioactive nucleus emits a gamma ray the number of

19 nuclear

A nucleus of polonium218 (atomic mass 218, atomic number 84) emits an


alpha particle (atomic mass 4, atomic number 2).The next two elements in
radioactive decay
chain each emit a beta particle (atomic mass 0, atomic number -1). What
would be the resulting nucleus after these three decays have
occurred?

20 Nuclear

The additional product of the nuclear fission reaction shown above is

21 nuclear

The nuclide 214 Pb 82 emits an electron and becomes nuclide X. Which of


the following gives the mass number
and atomic number of nuclide X?

22 Nuclear

The nuclear reaction X Y + Z occurs spontaneously. If M(x), M(y), and M


(z) are the masses of the three
particles. which of the following relationships is true?

23 nuclear

The equation above is an illustration of

24 Nuclear

A proton collides with a nucleus of N (atomic mass 14, atomic number 7). If
this collision produces a nucleus of C (atomic mass 11, atomic number 6) and
one other particle,
that particle is
D

25 nuclear

A nucleus of tritium contains 2 neutrons and 1 proton. If the nucleus


undergoes beta decay, emitting an electron,
the nucleus is transmuted into

26 Nuclear

Which of the following statements is true of a beta particle?

16 Nuclear

27 nuclear

28 Nuclear

An electron and a positron, each of mass 9.1 x 10^(31) kilogram, are in the
same general vicinity and have very small
initial speeds. They then annihilate each other, producing two photons. What
is the approximate energy of each emerging photon?
An electron and a positron, each of mass 9.1 x 10^(31) kilogram, are in the
same general vicinity and have very small
initial speeds. They then annihilate each other, producing two photons. What
is the angle between the paths of the emerging photons?

32 alphas, 10
betas

16 alphas, 16
betas

A proton
Protons increases
by one while the
number of
neutrons
decreases by one.

A neutron
Protons decrease
by one while the
number of
neutrons increases
by one.

16 alphas, 8 betas 8 alphas, 6 betas

This is the similar to problem 12. Beta


particles do not change the atomic
mass number since there is simply a
conversion between nucleons, so the
only way to reduce the mass number is
by
emitting alpha particles. The mass
number goes down by 32 and each
alpha particle reduces it by
4 so 8 alpha particles are needed. 8
alpha particles by themselves would
also reduce the atomic #
by 16, but it only ends up reduced by
10 so there are 6 protons needed.
These 6 protons come from the beta
decay where 6 neutrons turn into
4 alphas, 18 betas protons and release 6 beta particles.

An electron

A gamma ray

An alpha particle

Protons and
neutrons each
decrease by two

Protons and
neutrons each
increase by two

Protons and
neutrons remain
unchanged

For everything to add up properly, we


need ... 1 which is a proton.

Gamma emission is pure energy so no


particles change.
First in the alpha decay, the atomic
mass goes down by 4 and the atomic
number goes down by 2
leaving X(atomic mass 214, atomic
number 82) then in the two beta
decays, a neutron turns into a proton
each time increasing the atomic
number by two leaving X (atomic
A
B
C
D
E
mass 214, atomic number 84).
Simply make sure everything adds up
A
B
C
D
E
to get the missing piece.
Pb (atomic mass 214, atomic number
82) X + e (atomic mass 0, atomic
number -1) For everything to add up,
we need X (atomic mass 214, atomic
A
B
C
D
E
number 83)
In a nuclear reaction, the total mass
before must be larger than the total
mass after since some of
the mass will be missing afterwards
(mass defect) in the form of released
A
B
C
D
E
energy.
In this reaction, two light elements are
artificially
fusing together and producing a heavier
produced
naturally occurring nuclear
element. This is
radioactive decay radioactive decay disintegration
nuclear fission
nuclear fusion
fusion.
The reaction is as follows p (atomic
mass 1, atomic number 1) + N (atomic
mass 14, atomic number 7) C
(atomic mass 11, atomic number 6)
+ X to make it all add up X must be
a proton
a neutron
a deuteron
an particle
a particle
an alpha.
We start with 2 neutrons and 1 proton.
In beta decay with the emission of an
electron, the process involves a neutron
turning into a proton. The resulting
nucleus would have 1 neutron and 2
protons. An atomic number of 2 is
defined as He. It is He (atomic mass 3,
the nucleus of an the nucleus of an
atomic number 2) which is an isotope of
isotope of helium isotope of lithium an alpha particle
a triton
a deuteron
He (atomic mass 4, atomic number 2).
A beta particle, like all matter, can
B. It has a charge C. It is more
exhibit wave properties. Since the
A. Its speed in a
equal and opposite penetrating than a D. It has a mass of
particle is so small, it can
vacuum is 3 x 10^8 to that of an alpha gamma ray of the about 1,840 times E. It can exhibit
more readily show these wave
m/s.
particle.
same energy.
that of a proton
wave properties.
properties than normal size matter.
Using E=mc^2 with twice the mass
since two particles are destroyed =
it cannot be
(2*9.1x10^(31))(3x10^8)^2 = 1.64x10^
determined unless (13) J 2.63x10^(13) J * (1 eV / 1.6
the frequency of
x10^(19) J) = 1.02x10^6 eV. This is
the photon is
the total energy released, and since
. 0.51 MeV
2.0 MeV
4.0 MeV
6.6 MeV
known.
there are two photons we split it in half.

30

45

90

180

To conserve momentum, the photons


must move in opposite directions.

17.nuclear.png

19.nuclear.png
20.nuclear.png

21.nuclear.png

22.nuclear.png

23.nuclear.png

29 nuclear

The total number of free neutrons in the products of this reaction is

30 Nuclear

Which of the following statements is always true for neutron-induced fission


reactions involving Uranium (atomic mass=235, atomic number 92)?

For everything to add up properly, 3


6 neutrons are needed.
I. is Not True, for the following reason:
In fission, and U235 nucleus is broken
into fragments that make smaller
elements + neutrons
+ energy. The fragments created are
not always the same and there is a
statistical probability
of which fragments can be created. The
reaction provided in this problem is the
most probable
but other elements can be formed such
as the following U235 fission reaction:
U-235 + n Zr-94 + Te-139 + 3n +
energy. There are actually many
combinations of
fragments that can be released. Small
amounts of mass are missing as
released energy but
adding the whole numbers of the
reaction will always balance the
equation for a given reaction.
II. is TRUE. As explained above, as
small amount of the mass will be
missing in the form of
energy after the reaction completes.
This is necessary to produce the
energy from the reaction.
III. is Not True. Again as explained in
the first paragraph. There will be a
small amount of mass
missing but adding the whole numbers
before and after will always result in the
same numbers
of particles for a fission reaction.
30.nuclear.PNG
Fg
=Gmm/r2
Fe
= kqq/r2
The electric and gravitational forces
are inverse squared as
shown from the equations here.
Nuclear is not. This is fact and we dont
know why. It was one
of Einsteins last puzzles and he
considered it a great failure of his to not
solve this. It is called
grand unification theory that attempts to
combine all of the four fundamental
forces into one
unified force. It is a hot topic in modern
physics that is as of yet unsolved.
This is the definition of an isotope.
Same atomic number so same number
of protons. Different
numbers of neutrons make it an
isotope. Also a baseball team on The
Simpsons.
Some reactions conserve all of these,
others do not. Clearly the numbers of
protons is not
conserved as evidenced by beta decay.
The number of nuclei is more often
conserved but in
some reactions such as annihilation the
nuclei are disintegrated and converted
into energy. This
agrees with the law of conservation of
matter and energy, but when looking at
the total numbers
of particles before and the total
numbers of particles after, you would
say that number is not
conserved. Charge is a fundamental
conservation law and it always
conserved. Even in the
annihilation example, the net charge
before was zero and is zero after.

31 nuclear

Forces between two objects which are inversely proportional to the square of
the distance between the objects
include which of the following?
I. Gravitational force between two celestial bodies
II. Electrostatic force between two electrons
III. Nuclear force between two neutrons
C

I only

III only

I and II only

II and III only

I, II, and III

32 Nuclear

Atoms of isotopes of the same element contain the same number of

protons but a
electrons but a
neutrons but a
different number of different number of different number of neutrons as
neutrons
protons
protons
electrons

protons as
neutrons

I only
Hg (atomic
mass=210, atomic
number=80)

I, II, and III


Po (atomic
mass=218, atomic
number=84)

33 nuclear

34 Nuclear

Quantities that are conserved in all nuclear reactions include which of the
following?
I. Electric charge
II. Number of nuclei
III. Number of protons
A negative beta particle and a gamma ray are emitted during the radioactive
decay of a nucleus of Pb (atomic mass=214, atomic number=82). Which of
the following is the resulting nucleus?

II only
Tl (atomic
mass=214, atomic
number=81)

I and III only


Bi (atomic
mass=213, atomic
number=83)

II and III only


Bi (atomic
mass=214, atomic
number=83)

a proton

Heavy stable
nuclei tend to have All light stable
Z < N.
nuclei have Z< N
energy equivalent
of the mass
decrease in the
energy equivalent reaction, minus the
of the mass
kinetic energy of
total energy of the decrease in the
the incident
incident neutron
reaction
neutron
a helium nucleus
(He(atomic
mass=4, atomic
a neutron
an electron
number=2))

I only

III only

I and II only

II and III only

I, II, and III

These are all true statements about


binding energy.

II only

I and II only

I and III only

II and III only

I, II and III

Based on the formula v = (F/(m/L))^.5

35 nuclear

Which of the following statements about the number of protons Z and the
number of neutrons N in stable nuclei is true?

36 Nuclear

When B (atomic mass=10) is bombarded by neutrons, a neutron can be


absorbed and an alpha particle (
He (atomic mass=4)) emitted. If the B(atomic mass=10)
target is stationary, the kinetic energy of the reaction products is equal to the. C

37 nuclear

38 Nuclear

Waves and
1 Sound

Waves and
2 Sound
Waves and
3 Sound
Waves and
4 Sound

Ra (atomic mass=286, aatomic number=88) decays into Rn (atomic


mass=222, atomic number=86) plus
Correct statements about the binding energy of a nucleus include which of
the following?
I. It is the energy needed to separate the nucleus into its individual protons
and neutrons.
II. It is the energy liberated when the nucleus isformed from the original
nucleons.
III. It is the energy equivalent of the apparent loss of mass of its nucleon
constituents.
Which of the following statements about the speed of waves on a string are
true?
I. The speed depends on the tension in the string
II. The speed depends on the frequency
III. The speed depends on the mass per unit length of the string.
A string is firmly attached at both ends. When a frequency of
60 Hz is applied, the string vibrates in the standing wave pattern shown.
Assume the tension in the string and its mass per unit length do not change.
Which of the following frequencies could NOT also produce a standing wave
pattern in the string?

All stable nuclei


have Z = N

kinetic energy of
the incident
neutron

Only heavy stable


nuclei have Z = N

All light stable


nuclei have Z > N.
energy equivalent
of the mass
decrease in the
reaction, plus the
kinetic energy of
the incident
neutron

This is a mass defect question. The


energy released in the reaction is equal
the equivalence of
the missing mass comparing the
products and reactants.

a deuteron ( H
(atomic mass=2,
For everything to add up we need a
atomic number=1)) helium nucleus (alpha particle).

30 Hz

40 Hz

80 Hz

100 Hz

180 Hz

Which is not associated with a sound wave?

Amplitude

Period

Polarization

Velocity

Wavelength

The given diagram is the 3rd harmonic


at 60 Hz. That means the fundamental
is 20Hz. The other A possible standing 2.wavesandsound.
waves should be multiples of 20
png
A fact, sound cannot be polarized since
its longitudinal

A wave has a frequency of 50 Hz. The period of the wave is:

.2 s
doubles and the
speed remains
unchanged

7s

20 s

.02 s

UseT=1/f

is unchanged and is unchanged and


the speed doubles the speed halves

halves and the


speed halves

Frequency and wavelength are


inverses

.25 Hz

1 Hz

2 Hz

4 Hz

8 Hz

From diagram, wavelength = 0.5 m.


Find the frequency with v = f

6.wavesandsound.
png

After waves interfere they move along


as if they never met

7.wavesandsound.
png

.625 m

.75 m

1.25 m

2.5 m

3.75 m

For an openopen pipe the harmonic


frequency is given by f=nv/(2L) with n=1

Waves and
9 Sound
Waves and
10 Sound

If the frequency of sound is doubled, the wavelength:


The standing wave pattern diagrammed to the right is produced in a string
fixed at both ends. The speed of waves in the string is 2 m/s. What is the
frequency of the standing wave pattern?
Two waves pulses approach each other as seen in the figure. The wave
pulses overlap at point P. Which diagram best represents the appearance of
the wave pulses as they leave point P?
If the speed of sound in air is 340 m/s, the length of the organ pipe, open at
both ends, that can resonate at the fundamental frequency of 136 Hz, would
be:
String L and string H have the same tension and length. String L has mass m
and string H has mass 4m. If the speed of the waves in string L is v, the
speed of the waves in string H is
An observer hears a sound with frequency 400 Hz. Its wavelength is
approximately

.01 s
halves and the
speed remains
unchanged

V/2

1.4v

2v

4v

Based on v = (F/(m/L))^.5. 4x the mass


gives 12 the velocity

.85 m

1.2 m

2.75 m

13.6 m

44 m

Waves and
11 Sound

As sound travels from steel into air, both its speed and its:

wavelength
increase

wavelength
decrease

frequency
frequency increase decrease

frequency remain
unchanged

120, 125

115, 120

120, 120

115, 115

120, 115

60 Hz

80 Hz

120 Hz

480 Hz

720 Hz

The speed of the


waves doubles

the wavelength do the speed of the


the waves doubles waves halves

the wavelength of
the waves halves

0.34 m

0.68 m

1.36 m

Waves and
5 Sound
Waves and
6 Sound
Waves and
7 Sound
Waves and
8 Sound

Waves and
12 Sound
Waves and
13 Sound
Waves and
14 Sound

Waves and
15 Sound

When a train is at rest, both a passenger on the train and a ticket seller at the
station agree that the trains whistle
produces sound at a frequency of 120 Hz. When the train is moving away
from the station at 15 m/s, the
passenger hears a frequency of ___ Hz and the ticket seller hears a
frequency of ____ Hz.
E
A pipe that is closed at one end and open at the other resonates at a
fundamental frequency of 240 Hz. The next
lowest/highest frequency it resonates at is most nearly.
E
Assume that waves are propagating in a uniform medium. If the frequency of
the wave source doubles then
D
A natural horn (trumpet with no valves) is similar to a pipe open at both ends.
A musician plans
to create a fundamental frequency of 256 Hz (middle C) using the horn. If
sound travels at 350 m/s, what must be the length of this horn?
B

0.78 m

Speed of sound is 340,use v=f


When sound travels into less dense
medium, its speed decreases (unlike
light) however, like
all waves when traveling between two
mediums, the frequency remains
constant. Based on
v = f , if the speed decreases and the
frequency is constant then the must
decrease also.
Doppler effect. Since the passenger is
on the train moving with the sound, the
frequency is
unaltered, but since the sound is
moving away from the station, people at
the station should hear
a lower frequency.

Openclosed pipes only have odd


multiples of harmonic so next f is 3x f1
For a given medium, speed is constant.
Doubling the frequency halves the
none of the above wavelength
Using
L
f nv n 2
1.46 m
= with n = 1, solve for L

Waves and
16 Sound

A natural horn (trumpet with no valves) is similar to a pipe open at both ends.
A musician plans
to create a fundamental frequency of 256 Hz (middle C) using the horn. A
talented musician can produce a number of overtones on this natural horn.
What would be the frequency of
the fourth overtone produce when the musician is playing a middle C
fundamental?
D

512 Hz

768 Hz

1024 Hz

1280 Hz

Waves and
17 Sound

One stereo loudspeaker produces a sound with a wavelength of 0.68 meters


while the other speaker produces
sound with a wavelength of 0.65 m. What would be the resulting beat
frequency?

3 Hz

23 Hz

66.5 Hz

500 Hz

up then down

down then up

up, down, up

down, up, down

0.5 m

0.75 m

1m

2m

Cy

Cz

Ox

Oy

The fourth overtone is the fifth


harmonic, 5 x the fundamental.
Determine each separate frequency
using the speed of sound as 340 and v
= f . Then subtract
the two frequencies to get the beat
11333 Hz
frequency.
Step the two pulses through each other
a little bit at a time and use
superposition to see how the
amplitudes add. At first the amplitude
jumps up rapidly, then the amplitude
moves down as the
rightmost negative pulse continues to
propagate. At the very end of their
passing the amplitude
there would be no would be all the wave down and then
motion, the pulses once they pass the point will jump back
cancel one another up to equilibrium
Cx is only of a wavelength. To make
a full wavelength you would need 4x
4m
the current length
Wavelengths of each are (dist/cycle)
4L, 4/3 L, 4/5 L, L, 2/3 L
Frequencies are f = v/ .
v/4L, 3v/4L, 5v/4L, v/L, 3v/2L Oy is
Cy , Cz, Ox, Oy
2x Cy

0.0039s

0.020s

2.55Hz

50Hz

256Hz

f = cycles / seconds

Pitch

Volume

Frequency

Wavelength

Length of tube

To use v = f , you also need the

200s

141s

0.007s

0.002s

Waves and
18 Sound
Waves and
19 Sound

Waves and
20 Sound
Waves and
21 Sound
Waves and
22 Sound
Waves and
23 Sound

The diagram shows two transverse pulses moving along a string. One pulse
is moving to the right and the
second is moving to the left.
Both pulses reach point x at the
same instant. What would be
the resulting motion of point x
as the two pulses pass each
other?
C
The diagrams below represent 5 different standing sound waves set up inside
of a set of organ pipes 1 m long. What is the length of the longest wavelength
shown?
E

The diagrams below represent 5 different standing sound waves set up inside
of a set of organ pipes 1 m long. Which organ pipe(s) shows a standing wave
which has twice the frequency of one of the other waves shown?
D
The frequency of the tuning fork is (approximately)
E
In order to calculate the speed of sound from the graph, you would also need
to know
D
A metal bar is vibrating with a frequency of 200 Hz. The resulting period of
oscillation would be
E

Waves and
24 Sound

A tube is open at both ends with the air oscillating in the 4th harmonic. How
many displacement nodes are located within the tube?

Waves and
25 Sound

Two separate strings of the same thickness are stretched so that they
experience the same tension. String B is twice as dense as String A. String A,
of length L, is vibrated at the fundamental frequency. How long is String B if it
has the same fundamental frequency as String A?
B

1/2 L

L/(sqrt(2))

(Sqrt(2))*L

1536 Hz

None of the above T = 1/f


To produce pipe harmonics, the ends
are always antinodes. The first
(fundamental) harmonic is
when there are two antinodes on the
end and one node in-between. To move
to each next
harmonic, add another node in the
middle and fill in the necessary
antinodes. (ex, 2nd harmonic
is ANANA So the 4th
C
harmonic would be ANANANANA and
have four nodes. Alternative
solution if you know what the
harmonics look like you can draw them
and manually count the
5
6 nodes.
The Fundamental is given by
L
nv f
n
2
= (n=1), and the velocity is given by
2L
v=sqrt(Ft/(m/t))

18.wavesandsound.
png
19.wavesandsound.
png

20.wavesandsound.
png
21.wavesandsound.
png
21.wavesandsound.
png
21.wavesandsound.
png

Waves and
26 Sound
Waves and
27 Sound

Waves and
28 Sound

Waves and
29 Sound

Waves and
30 Sound
Waves and
31 Sound

Waves and
32 Sound

Waves and
33 Sound

Waves and
34 Sound
Waves and
35 Sound

This is an openclosed pipe. Look at


the harmonics shown below. Since the
same tuning fork is
used in each case, the frequency for all
of them is the same, and since they all
travel at the same
speed with equal frequencies, the
wavelength of each wave is also the
same (though each has a
different number of wavelengths fitting
in the tube, the wavelength of each is
same). But, the
lengths of the tubes vary using the
water to make each successive
harmonic and fit the necessary
numbers of wavelengths in each tube.
Looking at the diagrams, we can see
that each
harmonic is a wavelength longer than
the next,
regardless of which ones we are
looking at. We
dont have to know which harmonics we
are
looking at if we know the difference
between any
two of them because each harmonic
has the same
difference of . So the difference in
length
between any two consecutive
harmonics is equal to
the wavelength of the wave. L =
. Using
this relationship, we have:
(65 cm 39 cm) =
26.wavesandsound.
= 52 cm
png

A resonance occurs with a tuning fork and an air column of size 39 cm. The
next highest resonance occurs with an air column of 65 cm. What is the
frequency of the tuning fork? Assume that the speed of sound is 343 m/s.

329.8Hz

527.7Hz

659.6Hz

879.5Hz

A place of zero displacement on a standing wave is called

an antinode

a node

the amplitude

the wavenumber

doubles and the


wavelength is
unchanged

doubles and the


wavelength
doubled

doubles while the


wavelength is
halved

4L1

2L1

L1

the harmonic
Definition of a node
Since the medium stays the same the
is unchanged while is unchanged while speed remains constant. Based on v = f
the wavelength is the wavelength is , for constant
doubled
halved
speed, f and change as inverses
Similar to problem 26, we should look
at the harmonic shapes openopen vs
openclosed. Comparing the
fundamental harmonic of the open
open pipe to the
closedopen pipe. The closedopen
pipe should be half as long asthe
openopen pipe in order to fit the
proper number of wavelengths of the
same waveform to produce the given
29.wavesandsound.
1/2L1
1/4L1
harmonic in each.
png

120Hz

60Hz

40Hz

30Hz

20Hz

Two antinodes by definition will be


apart. So 20 cm = , and the = 40
cm. Then using

L, L, L

L, L, 1/6 L

2L, L, L

4L, 2L, L

4L, 4/3 L, 4/5 L

v = f we have 1200 = f (40)

A person vibrates the end of a string sending transverse waves down the
string. If the person then doubles the rate at which he vibrates the string while
maintaining the same tension , the speed of the waves
E

A tube of length L1 is open at both ends. A second tube of length L2 is closed


at one end and open at the other end. This second tube resonates at the
same fundamental frequency as the first tube. What is the value of L2 ?
For a standing wave mode on a string fixed at both ends, adjacent antinodes
are separated by a distance of 20cm. Waves travel on this string at a speed
of 1200 cm/s. At what frequency is the string vibrated to produce this standing
wave?
What would be the wavelength of the fundamental and first two overtones
produced by an organ pipe of length L that is closed at one end and open at
the other?
A small vibrating object S moves across the surface of a ripple tank producing
the wave fronts shown above.
The wave fronts move with speed v. The object is traveling in what direction
and with what speed relative to
the speed of the wave fronts produced?
A cord of fixed length and uniform density, when held between two fixed
points under tension T, vibrates with
a fundamental frequency f. If the tension is doubled, the fundamental
frequency is

1319Hz

(A)

(B)

(C)

(D)

(E)

2f

(2f)

f/(2)

one wavelength

about 340 meters

a distance directly
proportional to the
frequency of the
vibration

a distance directly
proportional to the
square root of the
air density

f/2
a distance
inversely
proportional to the
square root of the The time to make 1 cycle, is also the
pressure
time it takes the wave to travel 1

(A)

(B)

(C)

(D)

(E)

A vibrating tuning fork sends sound waves into the air surrounding it. During
the time in which the tuning fork
makes one complete vibration, the emitted wave travels
A
Two wave pulses, each of wavelength , are traveling toward each other
along a rope as shown. When both pulses are in the region between points X
and Y, which are a distance apart, the shape of the rope is
B

Doppler effect
Doubling the tension, changes the
speed to 2 v comparatively, and
makes the frequency increase by the
same amount.

Superpose the two waves on top of


each other to get the answer

32.
waves_and_sound.
png

35.
waves_and_sound.
png

Waves and
36 Sound

A train whistle has a frequency of 100 hertz as heard by the engineer on the
train. Assume that the velocity of
sound in air is 330 meters per second. If the train is approaching a stationary
listener on a windless day at a
velocity of 30 meters per second, the whistle frequency that the listener hears
is most nearly
B

90 Hz

110 Hz

120 Hz

240 Hz

300 Hz

Waves and
37 Sound

Two sinusoidal functions of time are combined to obtain the result shown in
the figure above. Which of the
following can best be explained by using this figure?

Beats

Doppler Effect

Diffraction

Polarization

Simple harmonic
motion

Waves and
38 Sound

The speed at which waves propagate on the string is

0.4 m/s

2.5 m/s

5 m/s

10 m/s

20 m/s

The fundamental frequency of vibration of the string is

1 Hz

2.5 Hz

5 Hz

7.5 Hz

10 Hz

Longitudinal

Transverse

Torsional

Electromagnetic

Polarized

I only

III only

I and II only

II and III only

I, II, and III

(A)

(B)

(C)

(D)

(E)

I only

III only

I and II only

II and III only

I, II, and III

f/3

f/2

2f

Waves and
39 Sound
Waves and
40 Sound
Waves and
41 Sound
Waves and
42 Sound

Sound in air can best be described as which of the following types of waves? A
In the Doppler effect for sound waves, factors that affect the frequency that
the observer hears include which of the following?
The figure above shows two wave pulses that are approaching each other.
Which of the following best shows the shape of the resultant pulse when the
centers of the pulses, points P and Q coincide?

Doppler equation
This diagram is associated with beats.
The increasing and decreasing
amplitude caused by interference
oscillates the volume up and down
which createsthe beat sound.

v=f
The diagram shows the second
harmonic in the string. Since harmonics 39.
are multiples, the first harmonic would waves_and_sound.
be half of this.
png
Fact
A fact about the Doppler effect. Can
also be seen from the Doppler
equation.

Waves and
45 Sound

Waves and
46 Sound

Two fire trucks have sirens that emit waves of the same frequency. As the fire
trucks approach a person, the person hears a higher frequency from truck X
than from truck Y. Which of the following statements about truck X can be
correctly inferred from this information:
A

I only

III only

I and II only

II and III only

It will be the same Clearly at point C the waves are


at all four points.
compressed so are more frequent.
Harmonics are multiples of the
fundamental so the fundamental must
3f
be f/2.
Based on the Doppler effect, only
speed matters. The faster a vehicle is
moving, the closer together the sound
waves get compressed and the higher
the frequency. Take the case of a very
fast vehicle traveling at the speed of
sound; the compressions are all right
on top of each other. So faster speed
means closer compressions and higher
frequencies. Choice I must be true
because X is a higher frequency so
must be going faster. Distance to the
person affects the volume but not the
pitch so choice II is wrong. III seems
correct but its not. It doesnt matter
whether you are speeding up or slowing
down, it only matters who is going
faster. For example, suppose truck X
was going 5 mph and speeding up
while truck Y was going 50 mph and
slowing down, this is an example of
choice III but would not meet the
requirement that X has a higher
frequency because only actual speed
matters, not what is happening to that
I, II, and III
speed.

Waves and
47 Sound

What is the amplitude of the wave?

4 cm

5 cm

8 cm

10 cm

16 cm

Waves and
48 Sound

What is the speed of the wave?

4 cm/s

25 cm/s

50 cm/s

100 cm/s

200 cm/s

Waves and
49 Sound

A standing wave pattern is created on a guitar string as a person tunes the


guitar by changing the tension in the string. Which of the following properties
of the waves on the string will change as a result of adjusting only the tension
in the string? I. Speed of the traveling wave that creates the pattern II.
Frequency of the standing wave III. Wavelength of the standing wave
C

I only

II only

I and II only

II and III only

I, II, and III

Waves and
44 Sound

41.
waves_and_sound.
png
42.
waves_and_sound.
png

Use superposition and overlap the


waves to see the resultant.
When hitting a fixed boundary, some of
the wave is absorbed, some is reflected
inverted. The reflected wave has less
amplitude since some of the wave is
absorbed, but since the string has not 43.
changed its properties the speed of the waves_and_sound.
wave should remain unchanged.
png

One end of a horizontal string is fixed to a wall. A transverse wave pulse is


generated at the other end, moves toward the wall as shown and is reflected
at the wall. Properties of the reflected pulse include which of the following?
B
A small vibrating object on the surface of a ripple tank is the source of waves
of frequency 20 Hz and speed 60 cm/s. If the source S is moving to the
right, as shown, with speed 20 cm/s, at which of the labeled points will the
frequency measured by a stationary observer be greatest?
C
The frequencies of the first two overtones (second and third harmonics) of a
vibrating string are f and 3f/2 .What is the fundamental frequency of this
string?
B

Waves and
43 Sound

37.
waves_and_sound.
png
38.
waves_and_sound.
png

By inspection.

44.
waves_and_sound.
png

46.
waves_and_sound.
png
47.
waves_and_sound.
png
47.
waves_and_sound.
png

By inspection, the is 10 cm. f = 1 / T


= 5, Then use v = f .
Based on v = (Ftension/(m/l)), the
tension changes the speed. Then
based on f = nv/2L, this resulting speed
change will effect the frequency also.
But since the frequency increases in
49.
direct proportion to the speed, and v = f waves_and_sound.
, the should remain unchanged.
png

Waves and
50 Sound

A tuning fork is used to create standing waves in a tube open at the top and
partially filled with water. A resonance is heard when the water level is at a
certain height. The next resonance is heard when the water level has been
lowered by 0.5 m. If the speed of sound is equal to 340 m/s, the frequency of
the tuning fork is
C

Physical
2 Optics

In Youngs double slit experiment, the second order bright band of one light
source overlaps the third order
band of another light source. If the first light source has a wavelength of 660
nm, what is the wavelength of the
second light source?
D
A student performs an experiment similar to Youngs Double Slit Experiment.
Coherent light passes through
two narrow slits and produces a pattern of alternating bright and dark lines on
a screen. Which of the following
would cause the bright lines on the screen to be further apart?
I. Increasing the distance between the slits
II. Decreasing the distance between the slits
III. Decreasing the wavelength of the light
B

Physical
3 Optics

A diffraction grating of 1000 lines/cm has red light of wavelength 700 nm pass
through it. The distance
between the first and third principal bright spots on a screen 2 m away is
B

Physical
1 Optics

Physical
4 Optics
Physical
5 Optics

Physical
6 Optics
Physical
7 Optics
Physical
8 Optics

Physical
9 Optics

Monochromatic light with a wavelength of 6x107 meters falls upon a single


slit. After passing through the slit,
it forms a diffraction pattern on a screen 1 m away. The distance between the
center maximum and the first
maximum away from the center is 3 mm. What is the thickness of the slit?
C
In a Young's double-slit experiment, the slit separation is doubled. To
maintain the same fringe spacing on the
screen, the screen-to-slit distance D must be changed to
D
Monochromatic light falls on a single slit 0.01 cm wide and develops a first
order minimum (dark band) 0.59
cm from the center of the central bright band on a screen that is one meter
away. Determine the wavelength of
the light.
D
Which station broadcasts with 3.27 m radio waves?
Pioneering radio station KFKA started broadcasting 78 years ago at 1310
(1.31 MHz) on the AM dial. What is
the approximate length of its radio wave?
The length of the most effective transmitting antenna is equal to onefourth
the wavelength of the broadcast
wave. If a radio station has an antenna 4.5 meters long then what is the
broadcast frequency of the radio station?

170 Hz

226 Hz

340 Hz

680 Hz

2450 Hz

1320 nm

990 nm

495 nm

440 nm

330 nm

I only

II only

III only

I & III only

II & III only

14 cm

28 cm

42 cm

140 cm

280 cm

0.1 mm

0.2 mm

0.3 mm

0.4 mm

0.5 mm

D/2

D/(2)^1/2

D(2)^1/2

2D

4D

This is the same as question 26. The


length change corresponds to a
increase in length. L = ... 0.5 =
... = 1 ... then use v = f and
solve for f.
Using m = d sin , the value of sin is
the same for both sources since the
location of the spot
is the same, but the first source is at
m=2, and the second source is at m=3.
Equating d sin for
each gives m1 1 = m2 2 (2)(660) =
3 (2) 2 = 440 nm.

Based on m = dx / L we want to
increase x. Only II does this.
1000 lines/cm gives a line spacing d =
1/1000 cm/line = 1x105 m/line. =
7x107
m=1. m = d sin (1)(7x10
m.
With diffraction gratings, we usually
assume the small angle approximation
does not work, so we
find then use the geometry with tan
or another trig function to find Y. Do this
for each spot.
7) = (1x105) sin = 4.01 tan
= o/a Y1 = 0.14 m
Repeat for m=3 Y3 = 0.43 m.
Subtract Y3 Y1 to find the distance
between = 0.29 m
Note: Since the angle here actually
came out to be small, the x/L small
angle approximation
could be used and the spacing x
between spots could be assumed to be
equal as well, so you
could simply find x for the first spot and
double it to find the spacing 1 to 3.
Single slit. With the given values, we
can see the angle is small so we can
use the small angle
approximation and apply m = dx / L.
Recall for single slits, the first maximum
off center is at
x=1.5 unlike double slits.

1.18 x10^(2) cm

5.90 x10(3) cm

1.18 x10(4) cm

5.90 x10(5) cm

1.18 x10(6) cm

91.7 MHz

92.5 MHz

98.5 MHz

102.5 MHz

106.3 MHz

From m = dx / L, d x2 needs L x2 also.


Single Slit. Again based on the given
values we can see the angle is small so
we can use
m = dx / L dark spot at m=1. Note:
use L=100 cm to get an answer in cm.
Radio wave is EM and travels at light
speed. Use c = f and solve.

23m

230 m

2300 m

23000 m

3x10^(8) m

Radio wave is EM and travels at light


speed. Use c = f and solve.

1.4 x 10^(8) Hz

6.0 x 10^(8) Hz

1.7 x 10(7) Hz

6.7 x 10(7) Hz

3.0 x 10^(8) Hz

1.9 x 10^(10)

3.9 x 10^(10) km

7.9 x 10^(10) km

1.9 x 10^(11) km

5.4 x 1011 km

Physical
10 Optics

A radio signal with a wavelength of 1.2 x 10^(-4)m is sent to a distance


asteroid, is reflected, and returns to Earth
72 hours and 48 minutes later. How far from Earth is the asteroid?

Physical
11 Optics

In the electromagnetic spectrum, rank the following electromagnetic waves in


terms of increasing wavelength.
E

= 4*4.5, Radio wave is EM and


travels at light speed. Use c = f and
solve.
It travels at light speed and takes half
the total time to travel the distance one
way. Use v=d/t.
Convert the time to seconds, find the
distance in meters, then convert that to
km.
changes the opposite of frequencies
(high freq = low ) based on this and
knowledge of the
EM spectrum, the answer is E.
11.physicaloptcs.png

Physical
12 Optics

Physical
13 Optics

Physical
14 Optics

Physical
15 Optics

Physical
16 Optics

Physical
17 Optics

Two sources, in phase and a distance d apart, each emit a wave of


wavelength . See figure below. Which of the
choices for the path difference L = L1 L2 will always produce destructive
interference at point P?
Waves are produced by two point sources S and S vibrating in phase. See
the
accompanying diagram. X represents the location of the 2nd
interference minima.
The path difference SX SX is 4.5 cm. The wavelength of the waves is
approximately
A transmission diffraction grating is ruled with 5000 lines per cm. Through
what angle will the first order
maxima be deflected when light with a wavelength of 4.5 x 107 m strikes the
grating?
In an experiment to measure the wavelength of light using a double slit
apparatus, it is found that the bright
fringes are too close together to easily count them. To increase only the
spacing between the bright fringes, one
could
Two point sources in a ripple tank radiate waves in phase with a constant
wavelength of 0.02 meter. The
first-order interference maximum appears at 6 (use sin 6 = 0.1). The
separation of the sources is most nearly

By definition, when the path difference


equals or any odd multiple of s
for sources of
the same , there will be destructive
interference.
12.physicaloptics.png

d sin

x/L1

(x/L2)d

/2

1.5 cm

1.8 cm

2.3 cm

3.0 cm

4.5 cm

5.2

6.4

13

27

34

Using path diff = m , with m=1.5 for the


2nd min, we have 4.5 cm = (1.5) .
13.physicaloptics.png
5000 lines/cm gives a line spacing d =
1/5000 cm/line = 2x106 m/line.
Then use m = d sin , with m = 1 for
the first max.

increase the slit


width

decrease the slit


width

increase the slit


separation

decrease the slit


separation

none of these

Based on m = dx / L we want to
increase x. d is separation of slits and
less d means more x

0.001 m

0.002 m

0.06 m

0.1 m

It has equally
spaced fringes of
equal intensity.

It has a relatively
strong central
maximum.

It can be produced
only if the slit width
is less than one
wavelength.

It can be produced
only if the slit width
is exactly one
wavelength.

Decreases
Decreases
Decreases

Decreases
Increases
Increases
Increases
Decreases
Decreases
Remains the same Remains the same Increases

10^(10) Hz

10^(6) Hz

10^(8) Hz

3 x 10^(8) Hz

0.07 m

0.09 m

0.16 m

0.20 m

3.0x10^-3m

1.0m

3.0m

3.3m

The fringe pattern


disappears.

The bright lines


are brighter and
the dark lines are
darker.

The bright lines


and the dark lines
are all darker.

The bright lines


and the dark lines
are all brighter.

Physical
18 Optics
Physical
19 Optics

Which of the following is true of a single-slit diffraction pattern?


B
For the five types of electromagnetic radiation listed above, which of the
following correctly describes the way
in which wavelength, frequency and speed, change as one goes from the left
to right on the list?
B
A radar operates at a wavelength of 3 centimeters. The frequency of these
waves is
E

Physical
20 Optics
Physical
21 Optics

Plane sound waves of wavelength 0.12 m are


incident on two narrow slits in a box with
nonreflecting walls, as shown. At a distance of
5.0 m from the center of the slits, a first-order
maximum occurs at point P, which is 3.0 m from
the central maximum. The distance between the
slits is most nearly
A radio station broadcasts on a carrier frequency of 100 MHz. The
wavelength of this radio wave is most nearly

Physical
22 Optics

If one of the two slits in a Youngs double-slit demonstration of the


interference of light is covered with a thin filter that transmits only half the light
intensity, which of the following occurs?
E

0.2 m
Using m = d sin (1)(0.02) = d (0.1)
It can be produced
only if the slit width
is an integral
number of
wavelengths.
Fact about single slits.
Increases
Increases
Increases

Known facts about the EM spectrum.


18.physicaloptics.png
Radar wave is EM and travels at light
10^(10) Hz
speed. Use c = f and solve.
Since the slits are narrow, we can use
m = d sin , but since is clearly
large we cannot use the
x/L small angle approximation. From
the given diagram, the geometry shows
sin = o/h = 3/5 ..
rather than finding , we will just use
this value for sin and plug in
0.24 m
m =d sin (1) (0.12) = d (3/5)
20.physicaloptics.png
Radio waves are EM and travels at light
3.0x10^6m
speed. Use c = f and solve.
This is still a double slit pattern because
there is still light making it through both
slits. One of
the light sources has reduced its
amplitude; which means when it meets
the second light source it
will cause less interference than it
originally did. This means less
constructive interference and
The dark lines are less destructive interference also. So
brighter and the
bright spots become less bright, and
bright lines are
dark spots become
darker.
brighter.

Geometric
2 Optics

A diffraction grating is illuminated by light of wavelength 600 nm. On a screen


100 cm away is a series of bright spots spaced 10 cm apart. If the screen is
now placed 30 cm from the diffraction grating, the new spacingbetween
adjacent bright spots on the screen is most nearly
C
The critical angle of a material is the angle of incidence for which the angle of
refraction is:
D
An object is located 0.20 meters from a converging lens which has a focal
length of 0.15 meters. Relative to the object, the image formed by the lens will
be:
C

Geometric
3 Optics

A plane mirror produces an image that is

Geometric
4 Optics

The principle underlying fiber optics is:

Geometric
5 Optics
Geometric
6 Optics

A diverging lens produces an image of a real object that is:


Light that has a wavelength of 500nm in air has a wavelength 400nm in a
transparent material. What is the index of refraction of the material?

Geometric
7 Optics

A beam of light passes from medium 1 to medium 2 to medium 3 as shown in


the accompanying figure. What is true about the respective indices of
refraction (n1, n2, and n3)
D

Geometric
8 Optics

A laser is embedded in a material of index of refraction n. The


laser beam emerges from the material and hits a target. See the
accompanying figure for the position parameters of the laser and
target. The value of n is:

Geometric
9 Optics

A beam of light is directed toward point P on a boundary as shown to the


right. Which segment best represents the refracted ray?

Physical
23 Optics
Geometric
1 Optics

Geometric
10 Optics

Geometric
11 Optics

Geometric
12 Optics
Geometric
13 Optics

Which of the following is NOT possible for the images formed by the lens in
the
accompanying figure?
A narrow beam of monochromatic light enters a lens parallel to
the optic axis, as shown in the accompanying diagram. Which
arrow best represents the direction of the light after leaving the
lens?
The accompanying diagram shows the path that a light ray takes passing
through three transparent materials. The indices of refraction in materials 1,
2, and 3 are n1, n2, and n3, respectively. Which of the following best
describes the relation between the indices of refraction?
Which diagram best represents what happens to a ray of light entering air
from water? Air is at the top in all
diagrams.

10cm

3cm

1cm

3mm

From the first scenario given. We can


determine the angle of the first spot
using tan = o /a.
tan = 10 / 100 = 5.71. The problem
says the remaining spots are spaced
equally, which is a
rough approximation. The angles are
relatively small, but if we wanted to get
accurate results we
should find each spacing with d sin ,
but, this is just an approximation for the
first few spots.
When the screen is moved closer, the
angle of the light leaving the grating will
not change, but
the spacing on the screen will
decrease. Think about a diagram of this
setup and it is clear that
this must be true. Based on tan = o /a
with the same angle but the adjacent
side changed to 30
cm, we get a new location of the first
spot at 3 cm, so the other spots will
also approximately be
located 3 cm apart as well for relatively
small angles.

30

45

90

180

Definition of critical angle.

real, erect, smaller.


real, inverted and
larger than the
object.

real, inverted,
smaller.
real, upright, and
the same size of
the object.

real, inverted,
larger.
real, upright, and
smaller than the
object.

virtual, erect,
larger.
virtual, inverted,
and smaller than
the object.

virtual, inverted,
smaller.
virtual, upright, and
the same size as
the object.

Using the math, 1/f = 1/do + 1/di, and M


= di / do

diffraction
real, inverted and
larger than the
object.

dispersion
real, upright, and
the same size as
the object.

interference
virtual, inverted,
and smaller than
the object.

polarization
virtual, upright, and
larger than the
object.

total internal
reflection
virtual, upright, and
smaller than the
object.
Fact about diverging lens.

1.56 Use n1 1 = n2 2
MoreLess dense bend away, Less
More dense bend towards. The more
the bend, the bigger the difference in
n1 > n2 > n3
n1 > n3 > n2
n2 > n3 > n1
n2 > n1 > n3
n3 > n1 > n2
n's.
If you look carefully you can see these
are both 345 triangles and are also
the same triangle
flipped. The hypotenuse of each is 1.5
m. Using the sides of the triangles, we
have
sin 1 = o/h = 0.8/1.5 for the bottom
triangle, and sin 2 = o/h = 1.2/1.5 for
the top triangle. Now
use n1 sin 1 = n2 sin 2 n1 (0.8/1.
5) = (1) (1.2/1.5) n1 = 1.2/0.8 = 3/2
1.4
1.5
2.1
3.5
5 =1.5
LessMore bend towards. But it cant
be E because that would only happen if
the incoming
PA
PB
PC
PD
PE
angle was also 0.
The lens shown has thick in the center
and thin on the outside which makes a
converging lens. In
converging lenses, all of the real
images are inverted and can be any
size, but the virtual images
are formed in a magnifying lens
real and smaller in real and larger in
virtual and smaller scenario and are always larger and
real and inverted
size
size
virtual and erect
in size
upright.

n1>n2>n3

n1>n3>n2

n2>n1>n3

n2 > n3 > n1

n3 > n1 > n2

0.64

0.8

Plane mirrors always makes virtual,


same size, upright images.
Fiber optics involves reflecting light in a
fiber strand as the light carries the
signal along the fiber.

1.25

A horizontal beam approaching a


converging lens bends and converges
through the focal point.
MoreLess dense bend away, Less
More dense bend towards. The more
the bend, the bigger the
difference in ns.
Assuming total internal reflection didnt
happen, MoreLess dense bend away

7.geometricoptics.
png

8.geometricoptics.
png

9.geometricoptics.
png

10.geometricoptics.
png

11.geometricoptics.
png

12.geometricoptics.
png
13.geometricoptics.
png

Geometric
14 Optics
Geometric
15 Optics

Geometric
16 Optics

Geometric
17 Optics

Geometric
18 Optics
Geometric
19 Optics

In order to produce and enlarged, upright image of an object, you could use a B
The critical angle in a transparent substance surrounded by air is 30. The
speed of light in the substance (in
multiples of 108 m/s) is most nearly
B

A beam of light traveling in glass (ng = 1.5) strikes a boundary


with air (na = 1.0) at point P. The angle of incidence is 60 as
shown in the diagram. Which ray would best indicate the beams
path after point P?
A small light bulb is placed 20 cm to the right of a converging lens of focal
length 10 cm. Which of the
following statements is NOT true about the image of the bulb formed by the
lens?

An image is formed on a screen by a convergent lens. If the top half of the


lens is then covered what will
happen to the image?
A wave moves from one medium to as second medium with a different index
of refraction. Which of the
following wave properties would NEVER change?

converging lens
more than one
focal length from
the object.

converging lens
less than one focal
length from the
object

diverging lens
more than one
focal length from
the object.

1.5

diverging lens
exactly one focal
length from the
object.

diverging lens less


than one focal
length from the
Need a magnifying glass which is
object
choice B.

It is virtual

It is inverted

It is the same size It is 20 cm to the


as the bulb
left of the lens

the image is
dimmer but
otherwise
unchanged

the image
becomes half as
big

frequency

wavelength

speed

angle
a boundary
between low index
of refraction and
high index of
refraction materials

Use ni sin c = nr sin (90), nr=1 ni =


6 2, then n = c / v to find v.
Generally when we go from moreless
we should always check the critical
angle first rather than
assuming the ray will refract and bend
away. Choice D might be correct, but
not until we first
check the critical angle for total internal
reflection. Use ni sin c = nr sin (90),
ni=1.5, nr=1
c = 41.8. Since our incoming angle
(60) is larger than the critical angle,
total internal reflection
16.geometricoptics.
will occur and you will get choice E.
png

It can be projected Using the math, 1/f = 1/do + 1/di, and M


on a screen
= di / do di +20 M = 1
When light from multiple locations pass
through a given part of a lens to form
an image, only a
small portion of a lens is needed to
form the image. The more of a lens, the
more light rays that
can be bent by it to each image
location. This simply makes the image
brighter. By covering
half the lens, all of the incoming rays
still bend all the same ways but there
are less total rays
being bent to given locations on the
image so it is dimmer. This can easily
be seen by looking at
a lens that has only horizontal rays
the image
approaching it. All of these rays
becomes half as
converge to the focal point;
only the top half of only the bottom
big and is inverted covering a portion of the lens still
the image is
half of the image is from its original
focuses the rays on the focal point, just
produced
produced
position.
less of them.

Geometric
20 Optics

Specular reflection occurs whenever light is incident on

a smooth surface

a rough surface

a boundary
between high
index of refraction
and low index of
refraction materials

Geometric
21 Optics

A beam of light passes from medium 1 to medium 2 to


medium 3 as shown in the diagram. What may be
concluded about the speed of light in each medium?

v3 > v1 > v2

v1 > v2 > v3

v1 > v3 < v2

v2 > v3 > v1

all will change


Fact for refraction problems.
a boundary
between any two
transparent
substances,
regardless of index
of refraction
Fact about specular reflection.
MoreLess dense bend away, Less
More dense bend towards. The more
the bend, the bigger the
difference in ns this shows that n2 >
n1 > n3. More n means less speed so
v2 > v1 > v3
v3 > v1 > v2
Its a diverging lens so light bends away
from what the horizontal path would be
E
without the lens.

0 cm

5 cm

10 cm

20 cm

No image is
formed

0.56 v

0.75 v

1.33 v

1.78 v

3.0 v

Geometric
22 Optics
Geometric
23 Optics

After striking the lens shown in the diagram at right, the


light ray will most likely follow which path?
An object is placed 10 cm in front of the center of a concave curved mirror
with a radius of curvature of 10 cm.
About how far from the mirror will the real image of the object be formed?

Geometric
24 Optics

Light travels from material X with an index of refraction of n=1.5 to material Y


with an index of refraction of
n=2.0. If the speed of light in material Y is v, what is the speed of light in
material X?
C

The focal point is = R/2. Then use the


math 1/f = 1/do + 1/di and di = 10
From n=c/v. n1 = c/v1 1.5 = c / vX
vX
n
= c / 1.5
2 = c/v2 2.0 = c / vY vY
The problem defines v
=c/2
Y=v
So v = c/2, c = 2v then subbing that
into the vx equation we have vX = (2v) /
1.5 = 1.33 v

21.geometricoptics.
png
22.geometricoptics.
png

Geometric
25 Optics

Geometric
26 Optics

Geometric
27 Optics

Geometric
28 Optics

A light ray is incident normal to a thin layer of glass. Given


the figure, what is the minimum thickness of the glass that
gives the reflected light an orangish color
((air) orange light = 600nm)

Two thin lenses each with a focal


length of +10 cm are located 30 cm
apart with their optical axes aligned
as shown. An object is placed 35
cm from the first lens. After the
light has passed through both lenses,
at what distance from the second
lens will the final image be formed?
C
A jeweler can distinguish between a diamond and a piece of glass by
observing the critical angle of light in each
material. Glass with an index of refraction of 1.52 has a critical angle of 41.1
while a diamond with an index
of refraction of 2.42 would have critical angle of:
D

What causes chromatic aberration in a glass lens

Geometric
30 Optics

A converging lens forms a virtual image of a real object that is two times the
objects size. The converging lens
is replaced with a diverging lens having the same size focal length. What is
the magnification of the image
formed by the diverging lens?
A beam of light travels through the air and strikes the surface of
water at an angle of incidence of 45. It continues through the
water and then strikes the bottom of a glass aquarium. Which of
the following would be closest to the angle of refraction after the
beam enters the glass. The index of refraction of water is 1.3 and
that of glass is 1.5

Geometric
31 Optics

Light shines from air into a clear material. When the light makes an angle of
incidence equal to 30, the light
refracts at an angle of 15. If the light is shone from an angle of incidence of
60, what is the angle of refraction?

Geometric
29 Optics

Geometric
32 Optics

Geometric
33 Optics
Geometric
34 Optics

Geometric
35 Optics

50 nm

100 nm

150 nm

200 nm

500 nm

65 cm

35 cm

27 cm

17 cm

14 cm

First find the in the film. nair air =


nfilm film (1)(600) = (1.5) glass
glass = 400 nm
As the light travels through the two
boundaries, you get a phase shift
(flip) at the first
boundary but no shift at the second
boundary. Therefore, you need to make
another of phase
difference total by traveling in the film
thickness to produce constructive
interference to reinforce
the orange wavelength. When the glass
thickness is of the in the glass, the
light will travel
up and down to make the extra
needed. So of the in the glass
gives you 100 nm
thickness needed.
Do the math twice. For the first lens. 1/f
= 1/do + 1/di di = + 14 cm (real). So
this first
preimage is formed 14 cm to the right
of the first lens, which means it is 16 cm
from the
second lens. Now redo the math using
this preimage as the object located
16 cm away from
the second lens. 1/f = 1/do + 1/di di
= + 26.67 cm.

Use ni sin c = nr sin (90), diamond into


air, so nr=1 (2.42) sin c = (1) sin
(90) c = 24.4

65.4

24.4

16.2

38.9
B) Each
wavelength of light
is refracted a
different amount
by the lens

25.8

A) Each
wavelength of light
reflects from the
surface of the lens

C) White light
waves interfere
inside the lens

D) White light
waves diffract
around the edge of
the lens

E) Chromatic
aberration occurs
with mirrors, not
lenses

A) 1

B) 2/5

C) 2/3

D) 3/2

E) 5/2

A) 55

B) 45

C) 38

D) 33

E) 28.

A) 19.5

B) 26.6

C) 30

D) 45

E) 60

An object is in front of a convex lens, at a distance less than the focal length
from the lens. Its image is

A) virtual and
larger than the
object.

C) virtual and
B) real and smaller smaller than the
than the object.
object.

D) real and larger


than the object.

E) virtual and the


same size as the
object.

Light is incident normal to a thin layer of soap. Given the figure,


what is the minimum thickness of the soap film that gives the soap a
bluish color (air(blue) = 500 nm)?

A) 100 nm

B) 200 nm

C) 250 nm

D) 400 nm

E) 500 nm

If the frequency of a periodic wave is doubled, the period of the wave will be

A) halved

B) quartered

C) doubled

(a) The object is


placed at a
position outside
the radius of
curvature for a
converging lens.

(b) The object is


placed at a
position outside
the radius of
curvature for a
diverging lens.

For which of the following does one obtain an image of increased size from a
real object? Take all focus and
radius of curvature values as positive.
D

D) quadrupled
(d) The object is
(c) The object is
placed at a
placed at a
position between
position inside the the focus and
magnitude of the
radius of curvature
focus for a
for a concave
concave lens.
mirror.

E) unchanged
(e) The object is
placed at a
position between
the focus and the
radius of curvature
for a convex
mirror.

25.geometrioptics.
png

26.geometricoptics.
png

Fact about chromatic aberration.


The magnification is M=2. Using M =
di/ do di= 2do Lets assume a value
of do= 10, then di= 20, and from 1/f =
1/do + 1/di, the focal point is 20. Now
redo the math with the focal point for
the diverging lens being negative and
the new di= 6.67, giving a new M=0.67
Use ni sin i = nr sin r, air to water and
find r. That r is the i for the second
water to glass interface. Then do ni sin
i
= nr sin r, water to glass and find r
30.geometricoptics
Use ni sin i = nr sin r, air to material
to find n of the material. Then redo the
problem with r as the unknown and
solve for r
.
A convex lens is a converging lens.
When the object is in front of the focal
point, it acts as a
magnifying glass.
Similar to question 25, except both
boundaries undergo phase shifts, so 1
full extra wavelength is
needed using the soap thickness. This
requires the thickness to be soap
giving the answer.
33.geometricoptics
Frequency are period are inverses.

Draw ray diagrams for each, or make


up numbers and do the math for each
to see which works.

Geometric
36 Optics
Geometric
37 Optics
Geometric
38 Optics
Geometric
39 Optics

A sound wave generated from a tuning fork of single frequency travels from
air (with speed of sound 340 m/s)
into rock (with speed of sound 1500 m/s). Which statement is true about the
wavelength and frequency of the
sound as it passes from air to rock?
When a beam of white light passes through a prism, the exiting light is seen
as a spectrum of visible colors. This
phenomenon is known as
Modern telescopes use mirrors, rather than lenses, to form images. One
advantage of mirrors over lenses is that
the images formed by mirrors are not affected by:

A) The frequency
of the sound
increases and the
wavelength
increases.

B) The frequency
of the sound
increases and the
wavelength is
unchanged.

C) The frequency
of the sound is
unchanged and
the wavelength is
decreased.

D) The frequency
of the sound is
unchanged and
the wavelength is
increased

E) The frequency
of the sound
decreases and the
wavelength is
increased.

When traveling between mediums,


sound behaves opposite from light. As
given in the problem
the sound travels faster in the denser
rock. When the sound speeds up, the
wavelength increases
and the frequency stays the same.

(A) diffraction

(B) dispersion

(C) interference

(D) polarization

(E) reflection

This is a fact.

destructive
interference
virtual, larger than
the object, and
upright.

constructive
interference
virtual, smaller
than the object,
and upright.

chromatic
aberration
virtual, smaller
than the object,
and inverted.

spherical
aberration
real, smaller than
the object, and
inverted.

atmospheric
refraction
real, larger than
the object, and
inverted

The defect in a lens is chromatic


aberration.

Geometric
40 Optics

A diverging lens produces an image of a real object. This image is


B
A light beam passes through the air and strikes the surface of a plastic block.
Which pair of statements correctly
describes the phase changes for the reflected wave and the transmitted
E
wave?

Geometric
41 Optics

The diagram below shows the path taken by a monochromatic light ray
traveling through three media. The
symbols v1, 1, and f1 represent the speed, wavelength, and frequency of the
light in Medium 1, respectively.
Which of the following relationships for the light in the three media is true?
E

Geometric
42 Optics

A real object is located in front of a convex lens at a distance greater than the
focal length of the lens. What type
of image is formed and what is true of the images size compared to that of
the object?
B

Geometric
43 Optics
Geometric
44 Optics

A thin film of thickness t and index of refraction 1.33 coats a glass with index
of
refraction 1.50 as shown to the right. Which of the following thicknesses t will
not
reflect light with wavelength 640 nm in air?
C
Which of the following wave properties cannot be demonstrated by all kinds
of waves?
A

160 nm

240 nm

360 nm

480 nm

640 nm

Polarization

Diffraction

Superposition

Refraction

Geometric
45 Optics

Lenses in fine quality cameras are coated to reduce the reflection from the
lenses. If the coating material has an
index of refraction between that of air and glass, what thickness of coating will
produce the least reflection?
A

onequarter of the onethird of the


wavelength in the wavelength in the
coating
coating

onehalf of the
wavelength in the
coating

Frequency
the amount of
reflection is
independent of the
one wavelength in thickness of the
the coating
coating.

Geometric
46 Optics

A beam of light from the air is incident on a transparent block of material. The
angle of incidence is 49 while
the angle of refraction is 30. What is the velocity of light in the transparent
B
material?

1.8 x 108
m/s

2.0 x 108 m/s

2.3 x 108
m/s

3.0 x 108
m/s

4.5 x 108
m/s

Geometric
47 Optics

Light with a wavelength of 500 nm in a vacuum enters a liquid with an index


of refraction of 1.25 at an angle of
incidence of 40. What would be the wavelength of the light in the liquid?

320 nm

400 nm

500 nm

625 nm

780 nm

Geometric
48 Optics

Light strikes three different thin films, which are in air, as shown. If t denotes
the film thickness and denotes
the wavelength of the light in the film, which films will produce constructive
interference as seen by the
observer?

I only

II only

III only

II and III only

I and III only.

Diverging lens always produces the


same object type no matter what.
The transmitted wave never has a
phase change, but hitting the more
dense block causes the
reflection to flip 180 degrees
MoreLess dense bend away, Less
More dense bend towards. The more
the bend, the bigger the
difference in ns this shows n2
> n1
> n3. More n means less speed, so v3
> v1
> v2 but this
is not a choice. Speed goes with
wavelength, the larger the speed the
more the , so 3
> 1
> 2
Based on various ray diagrams drawn
with the object behind the focal point,
the image is always
real but its size depends on where it is
in location to the focal point
First determine the film. n1
1
= nfilm film (1)(640) = (1.33) film
film = 481 nm.
When the wave reaches each boundary
is undergoes a phase shift at each
boundary so this
essentially cancels out the phase shift.
To not reflect any light, we want to have
destructive
interference. In order to get destructive
interference we need to get a total of
or 1 or
2 phase differences from moving
in the film thickness. These phase
differences require a
thickness equal to film , film , 5/4
film 360 nm thickness matches the
film possibility.

40.geometricoptics.
png

41.geometricoptics.
png

42.geometricoptics.
png

43.geometricoptics.
png

Longitudinal waves cannot be polarized


For airfilmglass of progressively
increasing index, to produce destructive
interference we need
of a wavelength in the coating. See
question 43 for the reason.
First use ni
sin i
= nr
sin r
To find nr
. Then use n = c / v to find v.
Use n1
1
= n2
2 (1)(500) = (1.25) 2
For all three diagrams, there is a
phase shift when entering the film but
no phase shift when
exiting. To produce constructive
interference, a total extra phase
different of from moving in
the film thickness is needed so odd
multiples of will produce
48.geometricoptics.
constructive interference
png

Geometric
49 Optics

Geometric
50 Optics
Geometric
51 Optics
Geometric
52 Optics

The critical angle for a transparent material in air is 30. The index of
refraction of the material is most nearly
E
An object is placed as shown in the figure above. The center of curvature C
and the focal point F of the
reflecting surface are marked. As compared with the object, the image formed
by the reflecting surface is
E

0.33

0.5

1.5

erect and the


same size
converging lens to
produce a real
image of the print

erect and smaller


mirror to produce a
virtual image of the
print

inverted and larger


diverging lens to
produce a real
image of the print

ni
sin c
= nr
sin (90) ni
sin (30) = (1) ni
2 =2

inverted and
smaller
diverging lens to
produce a virtual
image of the print
They are all the
same

When one uses a magnifying glass to read fine print, one uses a

erect and larger


converging lens to
produce a virtual
image of the print

Which color of light is associated with the highestspeed in a vacuum?

Blue

Green

Red

Violet

upright, virtual, and


0.30 meter from
the lens on the
opposite side from
the object
Converting a plane
wave front into a
spherical wave
front

upright, real, and


0.10 meter from
the lens on the
same side as the
object

upright, virtual, and


0.10 meter from
the lens on the
same side as the
object

Forming a virtual
image of a real
object

II only

I and III only

Forming a real
Forming a real
upright image of a inverted image of a Converging lenses make real images
real upright object real upright object but they are always inverted
When in front of the focal point of a
converging lens, it acts as a magnifying
glass. The other
optical instruments can never make
II and III only
I, II and III
larger images.
Using the math, 1/f = 1/do
+ 1/di
, di
= 60, since its virtual, the image is on
the same side as
the object which is why it is in the left.
30 cm to the right (E) 60 cm to the
You would look through this lens from
of the lens
right of the lens
the right side
Light would arrive Light leaving a
at Y in less time by source at Y and
taking a straight
traveling to X
A fact about refraction problems, the
line path from X to would follow the
angles going one way would be the
Y than it does
same path shown same as the angles
taking the path
above, but in
going to other way assuming total
shown above.
reverse.
internal reflection does not occur.
Converging lenses have centers that
are thick and top and bottom parts that
II, III, and IV
II, IV, and V
are thinner.
In flat (plane) mirrors, the image is
simply flipped to the other side of the
D
E
mirror.
Choice I. is true because a soap bubble
is a thin film. The colors produced are
due to the reinforcement of different
colors due to variations in the thickness
of the soap bubble. In order to see
these interference results, the thickness
of the film must be similar in magnitude
to the wavelength of the light. Since the
film is so small, this shows that light has
a very small wavelength. Choice II. also
shows light has a very small
wavelength because a diffraction
grating has very tiny slits in it and to
produce the pattern seen the
wavelength of the light has to be on a
similar scale as the size of the
openings. Choice III. is not true
because all waves regardless of their
wavelength bend and it does not reflect
II and III only
I, II, and III
on their wavelength size.
From practicing ray diagrams, this
should be known. Or a sample could be
done to determine it. Mathematically
this can be shown by using an extreme
example. Suppose do = 1000, and f =
10. Using the lens equation, di = 10.1.
Then decrease do down to 20 and di =
located at a
20. So for the range of values of do
distance between f located at a
larger than 20, the image distance will
and 2f from the
distance more than fall between 1020 which is between f
lens
f from the lens
and 2f.

Geometric
53 Optics

An illuminated object is placed 0.30 meter from a lens whose focal length is
0.15 meter. The image is

Geometric
54 Optics

Which of the following CANNOT be accomplished by a single converging lens


with spherical surfaces?
D

inverted, real, and


0.30 meter from
the lens on the
opposite side from
the object
Converting a
spherical wave
front into a plane
wave front

Geometric
55 Optics

The image of the arrow is larger than the arrow itself in which of the following
cases?
A

I only

Geometric
56 Optics

A postage stamp is placed 30 centimeters to the left of a converging lens of


focal length 60 centimeters. Where is the image of the stamp located?

60 cm to the left of 20 cm to the left of 20 cm to the right


the lens
the lens
of the lens

Geometric
57 Optics

Light leaves a source at X and travels to Y along the path shown above.
Which of the following statements is
correct?

The index of
refraction is the
same for the two
media.

Light travels faster Snell's law breaks


in medium 2 than down at the
in medium 1.
interface.

Geometric
58 Optics

Which three of the glass lenses above, when placed in air, will cause parallel
rays of light to converge?
B

I, II, and III

I, III, and V

I, IV, and V

Geometric
59 Optics

An object is placed near a plane mirror, as shown above. Which of the


labeled points is the position of the image?

Geometric
60 Optics

Observations that indicate that visible light has a wavelength much shorter
than a centimeter include which of
the following?

I only

III only

I and II only

Geometric
61 Optics

If the object distance for a converging thin lens is more than twice the focal
length of the lens, the image is

virtual and erect

larger than the


object

located inside the


focal point

49.geometricoptics.
png

Draw a ray diagram


A magnifying glass is a lens, and is
produced by a converging lens. It is
virtual.
All light waves are EM and travel at
light speed
Using the math, 1/f = 1/do
+ 1/di
, and M = di
/ do
di
= 0.10 m, M = +0.33

55.geometricoptics.
png

57.geometricoptics.
png
58.geometricoptics.
png
59.geometricoptics.
png

60.geometricoptics.
png

Geometric
62 Optics

A concave mirror with a radius of curvature of 1.0 m is used to collect light


from a distant star. The distance between the mirror and the image of the star
is most nearly
B

Geometric
66 Optics

When light passes from air into water, the frequency of the light remains the
same. What happens to the speed and the wavelength of light as it crosses
the boundary in going from air into water?
A physics student places an object 6.0 cm from a converging lens of focal
length 9.0 cm. What is the magnitude of the magnification of the image
produced?
An object is placed at a distance of 1.5f from a converging lens of focal length
f, as shown. What type of image is formed and what is its size relative to the
object?
A light ray passes through substances 1, 2, and 3, as shown. The indices of
refraction for these three substances are n1, n2, and n3, respectively. Ray
segments in 1 and in 3 are parallel. From the directions of the ray, one can
conclude that

Geometric
67 Optics

A beam of white light is incident on a triangular glass prism with an index of


refraction of about 1.5 for visible light, producing a spectrum. Initially, the
prism is in a glass aquarium filled with air, as shown above. If the aquarium is
filled with water with an index of refraction of 1.3, which of the following is
true?
E

Geometric
63 Optics
Geometric
64 Optics
Geometric
65 Optics

Geometric
68 Optics
Geometric
69 Optics
Geometric
70 Optics
Geometric
71 Optics

Geometric
72 Optics

An object, slanted at an angle of 45, is placed in front of a vertical plane


mirror, as shown above. Which of the following shows the apparent position
and orientation of the object's image?
The spherical mirror shown has a center of curvature at point c. Which point
is nearest to the focal point?
An object is placed in front of a converging thin lens at a distance from the
center of the lens equal to half the focal length. Compared to the object, the
image is
Which of the following is characteristic of both sound and light waves?

A thin film with index of refraction n1


separates two materials, each of which has
an index of refraction less than nf. A
monochromatic beam of light is incident
normally on the film, as shown above. If the
light has wavelength within the film,
maximum constructive interference between
the incident beam and the reflected beam
occurs for which of the following film
thicknesses?

0.75 m

1.0 m

2.0 m

0.6

1.5

Using the math, 1/f = 1/do + 1/di, di =


3.6 18 then M = di / do M = 3

n3 must be the
same as n1

n2 must be less
than n1

n2 must be less
than n3

No spectrum is
produced.

A spectrum is
produced, but the
deviation of the
beam is opposite
to that in air.

The positions of
red and violet are
reversed in the
spectrum.

n1 must be equal
to 1.00
The spectrum
produced has
greater separation
between red and
violet than that
produced in air.

all three indices


must be the same
The spectrum
produced has less
separation
between red and
violet than that
produced in air.

A
E

Geometric
75 Optics

Geometric
76 Optics

A large lens is used to focus an image of an object onto a screen. If the left
half of the lens is covered with a dark card, which of the following occurs

Geometric
74 Optics

0.50 m

A light ray R in medium I strikes a sphere of medium II with angle of incidence


, as shown above. The figure shows five possible subsequent paths for the
light ray. Which path is possible if medium I is air and medium II is glass?
E
A light ray R in medium I strikes a sphere of medium II with angle of incidence
, as shown above. The figure shows five possible subsequent paths for the
light ray. Which path is possible if medium I is glass and medium II is air?
A
An object is placed on the axis of a converging thin lens of focal length 2 cm,
at a distance of 8 cm from the lens.The distance between the image and the
lens is most nearly
E

Geometric
73 Optics

0.25 m

Light from a distantstar is assumed to


be all horizontal. Horizontal light hitting
a concave mirror will all converge at the
focal point to form an image of the star
directly on the focal point. With a radius
of curvature = 1m, the focal point is 0.5
m.
When light goes in higher indices of
refraction, itslows down. Since v = f
and f remains constant, when v
63.geometricoptics.
decreases decrease with it.
png

inverted and
upright and larger upright and smaller inverted and larger smaller
They are
They are
They travel with
They can be easily
longitudinal waves. transverse waves. the same velocity. polarized

Draw the ray diagram, or makeup some


numbers and do the math
If the angle in equals the angle out in a
3 tier medium arrangement, then the
substances on the outsides must be the
same.
The larger the difference between ns
the more the rays bend. When the
water is added, the difference between
ns is less so the amount of bending is
less.
In a flat mirror, the image can be found
by flipping the object to the other side,
basically folding it over the mirror onto
the other side.
The focal point is half the center of
curvature.
When an object is placed in front of the
focal point of a converging lens, the
lens acts as a magnifying glass.

65.geometricoptics.
png

66.geometricoptics.
png

67.geometricoptics.
png

68.geometricoptics.
png
69.geometricoptics.
png

inverted and the


same size
They give rise to
interference effects All waves demonstrate interference.

/2

/4

The film has a higher n compared to


both sides, such as soap surrounded by
air. So as the light ray hits the first
boundary it makes a phase flip, but
does not make the flip at the second
boundary. To be constructive, we need
to cover a total of extra phase shift
due to traveling in the film thickness. So
the thickness should be .
72.geometricoptics.
png
Medium I (air) is surrounding the
sphere on both sides. As it enters the
sphere, it goes lessmore so bends
towards the normal line (leaving D or E
as the possibly answers). When the ray
reaches the far edge of the sphere, it
goes from moreless so should bend
73-74.
away from the normal line.
geometricoptics.png

This should be the opposite of the


scenario in the last question.

0.4 cm

0.8 cm

1.6 cm

2.0 cm

2.7 cm

The image
becomes blurred

The image
becomes dimmer

No image is
formed

The right half of


The left half of the the image
image disappears disappears

Using the math, 1/f = 1/do + 1/di, di =


2.67.
When light from multiple locations pass
through a given part of a lens to form
an image, only a small portion of a lens
is needed to form the image. The more
of a lens, the more light rays that can
be bent by it to each image location.
This simply makes the image brighter.
By covering half the lens, all of the
incoming rays still bend all the same
ways but there are less total rays being
bent to given locations on the image so
it is dimmer. This can easily be seen by
looking at a lens that has only
horizontal rays approaching it. All of
these rays converge to the focal point;
covering a portion of the lens still
focuses the rays on the focal point, just
less of them.

73-74.
geometricoptics.png

Geometric
77 Optics

Which of the following statements are true for both sound waves and
electromagnetic waves?

Geometric
78 Optics

As shown, a beam of white light is separated into separate


colors when it passes through a glass prism. Red light is
refracted through a smaller angle than violet light because
red light has a

slower speed in
glass than violet
light

faster speed in
glass than violet
light

slower speed in
the incident beam
than violet light

faster speed in the


incident beam than greater intensity
violet light
than violet light

Geometric
79 Optics

A ray of light in glass that is incident on an interface with ice,


as shown, is partially reflected and partially refracted. The
index of refraction n for each of the two media is given in
the figure. How do the angle of reflection and the angle of
refraction compare with the angle of incidence ?

Geometric
80 Optics

If the focal length of the lens is 0.40 m and point P is


0.30 m to the left of the lens, where is the image of the
object located?

1.2 m to the left of 0.17 m to the left


the lens
of the lens

At the lens

0.17 m to the right 1.2 m to the right


of the lens
of the lens

Geometric
81 Optics

Which of the following characterizes the image when the object is in the
position shown?

Real, inverted, and Real, upright, and


smaller than the
larger than the
object
object

Real, inverted, and Virtual, upright,


Virtual, upright,
larger than the
and larger than the and smaller than
object
object
the object

Geometric
82 Optics

On a day when the speed of sound is 340 m/s, a ship sounds its whistle. The
echo of the sound from the shore is
heard at the ship 6.0 s later. How far is the ship from the shore?
C

56.7 m

1020 m

113 m

1 Kinematics

A ray of light in air is incident on a 30-60-90 prism,


perpendicular to face ab, as shown in the diagram. The ray enters
the prism and strikes face ac at the critical angle. What is the
index of refraction of the prism?
A car travels 30 miles at an average speed of 60 miles per hour and then 30
miles at an average speed of 30
miles per hour. The average speed the car over the 60 miles is

35 m.p.h.

40 m.p.h.

2 Kinematics

Which particle is farthest from the origin at t = 2 seconds

3 Kinematics

Which particle moves with constant non-zero acceleration?

4 Kinematics

Which particle is in its initial position at t = 2 seconds?

Geometric
83 Optics

5 Kinematics

6 Kinematics

The graph above shows the velocity versus time for an object moving in a
straight line. At what time after t = 0 does the object again pass through its
initial position?
C
A body moving in the positive x direction passes the origin at time t =
0. Between t = 0 and t = 1 second, the body has a constant speed of 24
meters per second. At t = 1 second, the body is given a constant
acceleration of 6 meters per second squared in the negative x direction.
The position x of the body at t = 11 seconds is
C

1/2/2013 sqrt(3/2)

2040 m

Between 0 and 1 s 1 s

3 Rationalizing gives us the answer.


Total distance = 60 miles, total time =
1.5 hours; average speed = total
45 m.p.h.
10 m.p.h.
53 m.p.h.
distance/total time
Area bounded by the curve is the
displacement By inspection of particle A
They are the same
the positive area
they are in the
distance from the
between 0 and 1s will be countered by
same location at t origin, but in
It is not possible to an equal negative area between 1 and
= 2 seconds
opposite directions determine
2s.
It is not possible to Constant non-zero acceleration would
both A and B
neither A nor B
determine
be a straight line with a non-zero slope
Area bounded by the curve is the
displacement By inspection of particle A
the positive area
between 0 and 1s will be countered by
It is not possible
an equal negative area between 1 and
both A and B
neither A nor B
to determine
2s.
Area bounded by the curve is the
displacement By inspection the
negative area between 0 and 1s
will be countered by an equal negative
Between 1 and 2 s 2 s
Between 2 and 3 s area sometime between 1 and 2s.

#ERROR!:parse

36 m

#ERROR!:parse

(2sqrt3)/3

4080 m

All waves demonstrate the listed


choices.
Bending of a wave (refraction) is due to
the speed change at an angle. The
more the speed
change, the more the bending. Hence,
the violet bends more so must have a
larger speed change
(more slowing), so the red is faster.
Additionally, we can note that since the
violet slows and
bends more, the index of refraction in
glass for a violet light is higher than the
index for a red
light.
Based on the law of reflection, the
angle of reflection must be the same as
the incoming angle.
When the light enters the ice it is going
moreless so bends away from the
normal. This means
that r is larger than i.
Using the math, 1/f = 1/do
+ 1/di
, di
= 1.2. Its virtual so its on the same
side as the object,
which puts the image on the left side of
the lens.
This is a magnifying glass, which can
be memorized or the math can be done
to prove the
answer.
The time for the sound to travel the one
way distance to the shore is half of the
total time
(6/2 =3 sec). Then use v= d /t to
determine the distance.
From the diagram, the angle at the
bottom of the small top triangle is 30
so when we draw the
normal line on that slanted interface,
the angle of incidence there is 60. We
are told this is the
critical angle which means the angle of
refraction of the scenario is 90. Now
we use
ni
sin c
= nr
sin (90) ni
sin(60) = (1)(1) ni
= 1/ sin 60 ni
1/ (root3 / 2)

75 m

99 m

Between 0 and 1 s; d1= vt; from 1 to 11


seconds; d2= v0t + at2; d = d1+ d2

77.geometricoptics.
png

78.geometricoptics.
png

79.geometricoptics.
png

80-81.
geometricoptics.png

80-81.
geometricoptics.png

83.geometricoptics.
png

2-4.kinematics.png
2-4.kinematics.png

2-4.kinematics.png

5.kinematics.png

8 Kinematics

The displacement, x, of an object moving along the x-axis is shown above as


a function of time, t. The acceleration of this object must be
A
A 2-kilogram block rests at the edge of a platform that is 10 meters above
level ground. The block is launched horizontally from the edge of the platform
with an initial speed of 3 meters per second. Air resistance is negligible. The
time it will take for the block to reach the ground is most nearly
C

9 Kinematics

A diver initially moving horizontally with speed v dives off the edge of a
vertical cliff and lands in the water a
distance d from the base of the cliff. How far from the base of the cliff would
the diver have landed if the diver
initially had been moving horizontally with speed 2v?

(2d)^1/2

2d

4d

10 Kinematics

. A truck traveled 400 meters north in 80 seconds, and then it traveled 300
meters east in 70 seconds. The
magnitude of the average velocity of the truck was most nearly

1.2 m/s

3.3 m/s

4.6 m/s

6.6 m/s

(A)

(B)

(C)

(D)

Since the slope is positive and


constant, so is the velocity, therefore
the acceleration must be zero
7.kinematics.png
For a horizontal projectile, the initial
speed does not affect the time in the
air. Use v0y = 0 with
3.0 s
10 m = gt^2
The time in the air for a horizontal
projectile is dependent on the height
and independent of the
cant be
initial speed. Since the time in the air is
determined without the same at speed v and at speed 2v,
knowing the height the distance (d = vt)
of the cliff
will be twice as much at a speed of 2v
Average velocity = total
displacement/total time; magnitude of
total displacement = 500 m (3-4-5
9.3 m/s
triangle) and total time = 150 seconds 10.kinematics.png
The acceleration is constant and
negative which means the slope of the
velocity time graph must have a
constant negative slope. (Only one
choice has the correct acceleration
(E)
anyway)
11.kinematics.png

1.5 m/s2

3.0 m/s2

6.0 m/s2

10.0 m/s2

12.0 m/s2

(A)

(B)

(C)

(D)

(E)

(A)

(B)

(C)

(D)

(E)

7 Kinematics

11 Kinematics

12 Kinematics

13 Kinematics
14 Kinematics

15 Kinematics

16 Kinematics

17 Kinematics

18 Kinematics

A projectile is fired with initial velocity v at an angle with the horizontal and
follows the trajectory shown
above. Which of the following pairs of graphs best represents the vertical
components of the velocity and
acceleration, v and a, respectively, of the projectile as functions of time t?
An object is released from rest on a planet that has no atmosphere. The
object falls freely for 3.0 meters in the
first second. What is the magnitude of the acceleration due to gravity on the
planet?

How do the speeds of the ball at the three points compare?


Which of the following diagrams best shows the direction of the acceleration
of the ball at point P?
A rock of mass m is thrown horizontally off a building from a height h, as
shown above. The speed of the rock as it leaves the throwers hand at the
edge of the building is v0. How much time does it take the rock to travel from
the edge of the building to the ground?

A ball is thrown straight up in the air. When the ball reaches its highest point,
which of the following is true?
E
The graph above represents position x versus time t for an object being acted
on by a constant force. The average speed during the interval between 1 s
and 2 s is most nearly
D
An object slides off a roof 10 meters above the ground with an initial
horizontal speed of 5 meters per second as shown above. The time between
the object's leaving the roof and hitting the ground is most nearly
C

zero

constant but not


zero

increasing

decreasing

0.3 s

1.0 s

1.4 s

2.0 s

(C)

(D)

It has zero
It is in equilibrium. acceleration.

It has maximum
momentum

It has maximum
kinetic energy.

2 m/s

4 m/s

5 m/s

6 m/s

8 m/s

(A)

(B)

(C)

(D)

(A)

(B)

Car Y is in front of Both cars have the


car X.
same acceleration.

Car Y is behind car Car Y is passing


X.
car X.

Car Y is in front of Both cars have the


car X.
same acceleration.

19 Kinematics

Which of the following is true at time t = 20 seconds?

20 Kinematics

From time t = 0 to time t = 40 seconds, the areas under both curves are
equal. Therefore, which of the following is true at time t = 40 seconds?

21 Kinematics

Which of the following pairs of graphs shows the distance traveled versus
time and the speed versus time for an object uniformly accelerated from rest? E

(A)

(B)

(C)

(D)

22 Kinematics

An object released from rest at time t = 0 slides down a frictionless incline a


distance of 1 meter during the first second. The distance traveled by the
object during the time interval from t = 1 second to t = 2 seconds is

1m

2m

3m

4m

0.05 hr

0.1 hr

1 hr

10 hr

5m

10 m

500 m

1,000 m

23 Kinematics

24 Kinematics

Two people are in a boat that is capable of a maximum speed of 5 kilometers


per hour in still water, and wish to cross a river 1 kilometer wide to a point
directly across from their starting point. If the speed of the water in the river
is 5 kilometers per hour, how much time is required for the crossing?
E
A projectile is fired from the surface of the Earth with a speed of 200 meters
per second at an angle of 30 above the horizontal. If the ground is level,
what is the maximum height reached by the projectile?
C

From rest, h = gt^2


At the top of its path, the vertical
component of the velocity is zero, which
makes the speed at the top a minimum.
With symmetry, the projectile has the
same speed when at the same height,
whether moving up or down.
13.kinematics.png
g points down in projectile motion.
Always.
14.kinematics.png

For a horizontal projectile; h = gt2E


(initial vertical component of velocity is
(E)
zero)
15.kinematics.png
At every point of a projectiles free-fall,
the acceleration is the acceleration due
None of the above to gravity

Car Y is behind car Car Y is passing


X.
car X.

equal to g

Average speed = total distance/total


time = (8 m 2 m)/(1 second)
For a horizontal projectile; h = gt2C
(initial vertical component of velocity is
(E)
zero)
Car X is
The area under the curve is the
accelerating faster displacement. There is more area
than car Y.
under the curve for Car X.
Car X is
Area under the curve is the
accelerating faster displacement. Car Y is moving faster
than car Y.
as they reach the same point.
Uniformly accelerated means the
speed-time graph should be a stright
line with non-zero slope. The
corresponding distance-time graph
should have an increasing slope (curve
(E)
upward)
From the equation d = at2C ,
displacement is proportional to time
squared. Traveling from rest for twice
the time gives 4 times the displacement
(or 4 m). Since the object already
travelled 1 m in the first second, during
the time interval from 1 s to 2 s the
5m
object travelled the remaining 3 m
To travel straight across the river, the
upstream component of the boats
The point directly velocity must cancel the current. Since
across from the
the speed of the current is the same as
starting point
the speed of the boat, the boat must
cannot be reached head directly upstream to cancel the
under these
current, which leaves no component
conditions.
across the river
viy = 200 m/s sin 30 = 100 m/s. At
maximum height vy = 0. Use vy2 =
2,000 m
viy2C + 2gh

17.kinematics.png

18.kinematics.png

19.kinematics.png

20.kinematics.png

21.kinematics.png

25 Kinematics

26 Kinematics

27 Kinematics

28 Kinematics

29 Kinematics

30 Kinematics

31 Kinematics

32 Kinematics

33 Kinematics

34 Kinematics

35 Kinematics

36 Kinematics

37 Kinematics

38 Kinematics

Vectors V1 and V2 shown above have equal magnitudes. The vectors


represent the velocities of an object at
times t1, and t2, respectively. The average acceleration of the object between
time t1 and t2
E
A rock is dropped from the top of a 45-meter tower, and at the same time a
ball is thrown from the top of the
tower in a horizontal direction. Air resistance is negligible. The ball and the
rock hit the level ground a
distance of 30 meters apart. The horizontal velocity of the ball thrown was
most nearly
B

In the absence of air friction, an object dropped near the surface of the Earth
experiences a constant acceleration
of about 9.8 m/s2 This means that the
A 500-kilogram sports car accelerates uniformly from rest, reaching a speed
of 30 meters per second in 6
seconds. During the 6 seconds, the car has traveled a distance of
At a particular instant, a stationary observer on the ground sees a package
falling with speed v1 at an angle to
the vertical. To a pilot flying horizontally at constant speed relative to the
ground, the package appears to be
falling vertically with a speed v2 at that instant. What is the speed of the pilot
relative to the ground?
An object is shot vertically upward into the air with a positive initial velocity.
Which of the following correctly
describes the velocity and acceleration of the object at its maximum
elevation?

A spring-loaded gun can fire a projectile to a height h if it is fired straight up. If


the same gun is pointed at an
angle of 45 from the vertical, what maximum height can now be reached by
the projectile?
A ball is thrown and follows a parabolic path, as shown above. Air friction is
negligible. Point Q is the highest
point on the path. Which of the following best indicates the direction of the
acceleration, if any, of the ball at
point Q?
The velocity of a projectile at launch has a horizontal component vh and a
vertical component vv. Air resistance
is negligible. When the projectile is at the highest point of its trajectory, which
of the following shows the
vertical and horizontal components of its velocity and the vertical component
of its acceleration?
The graph above shows the velocity v as a function of time t for an object
moving in a straight line. Which of
the following graphs shows the corresponding displacement x as a function of
time t for the same time interval?
An object is dropped from rest from the top of a 400 m cliff on Earth. If air
resistance is negligible, what is the
distance the object travels during the first 6 s of its fall?
A target T lies flat on the ground 3 m from the side of a building that is 10 m
tall, as shown above. A student
rolls a ball off the horizontal roof of the building in the direction of the target.
Air resistance is negligible. The
horizontal speed with which the ball must leave the roof if it is to strike the
target is most nearly

(A) zero

(B) directed north

(A) 5 m/s

(B) 10 m/s

(A) speed of the


object increases
9.8 m/s during
each second

(C) directed west

(D) directed north


of east

(E) directed north


of westwas

Acceleration is proportional to v. v =
v2 v1 = v2 + ( v1)
25.kinematics.png

From a height of 45 m (= gt2) it takes


3 seconds to strike the ground. In that
time, the ball thrown traveled 30 m. v =
d/t

(C) 14.1 m/s

(D) 20 m/s

(B) speed of the


object as it falls is
9.8 m/s.

(E) 28.3 m/s


(E) rate of change
of the
displacement with
(C) object falls 9.8 (D) object falls 9.8 respect to time for
meters during
meters during the the object equals 9.8 m/s2 can be thought of as a change
each second
first second only
9.8 m/s2
in speed of 9.8 m/s per second.

15 m

30 m

60 m

90 m

180 m

(A) v1 + v2 (

B) v1 v2

(C) v2 v1

(D) (v1^2 v2^2)


^1/2

(E)( v1^2 +v2^2)


^1/2

vi = 0 m/s; vf = 30 m/s; t = 6 s; v = (vi +


vf)/2 = d/t
velocity of package relative to observer
on ground vpg = v1 =
velocity of package relative to pilot vpp
= v2 =
velocity of pilot relative to ground vpo =
Putting these together into a right
triangle yields vpg^2 + v2^2 = v1^2

h/4

h/(2*2^1/2)

h/2

h/(2^1/2)

While the object momentarily stops at


its peak, it never stops accelerating
downward.
30.kinematics.png
Maximum height of a projectile is found
from vy = 0 at max height and vy^2 =
viy^2 + 2gh and gives hmax = viy^2/2g
= (vi sin)^2/2g. Fired straight up, the
angle = 90 and we have vi = (2gh)^1/2.
Plugging this initial velocity into the
equation for a 45 angle (sin 45 = 1/
(2^1/2)) gives hnew = ( (2gh)^1/2 * 1/
(2^1/2) )^2 /2g = h/2

g points down in projectile motion.


Always.

horizontal velocity vx remains the same


thorughout the flight. g remains the
same as well.
33.kinematics.png

34.kinematics..png

30 m

60 m

120 m

180 m

360 m

The graph above shows velocity v versus time t for an object in linear motion.
Which of the following is a
possible graph of position x versus time t for this object?
A
A student is testing the kinematic equations for uniformly accelerated motion
by measuring the time it takes for
light-weight plastic balls to fall to the floor from a height of 3 m in the lab. The
student predicts the time to fall
using g as 9.80 m/s2 but finds the measured time to be 35% greater. Which
of the following is the most likely
cause of the large percent error?
D

3/10 m/s

2 m/s

3/2 m/s

3 m/s

10 (5/3) m/s

The acceleration
due to gravity is
70% greater than
9.80 m/s2 at this
location.

The acceleration
due to gravity is
70% less than 9.80
m/s2 at this
location.

Air resistance
increases the
downward
acceleration.

The acceleration of The plastic balls


the plastic balls is are not truly
not uniform.
spherical.

32.kinematics.png

For a dropped object: d = gt2


For a horizontal projectile, the initial
speed does not affect the time in the
air. Use v0y = 0 with 10 m = gt2 to
get 2t = For the horzontal part of the
motion; v = d/t
36.kinematics.png
A velocity-time graph represents the
slope of the displacement-time graph.
Analyzing the v-t graph shows a
constant slope, then a decreasing slope
to zero, becoming negative and
increasing, then a constant slope. Note
this is an analysis of the values of v, not
the slope of the graph itself
37.kinematics.png

By process of elimination (A and B are


unrealistic; C is wrong, air resistance
should decrease the acceleration; E is
irrelevant)

41 Kinematics

An object is thrown with velocity v from the edge of a cliff above level ground.
Neglect air resistance. In order
for the object to travel a maximum horizontal distance from the cliff before
hitting the ground, the throw should
be at an angle with respect to the horizontal of
C
Starting from rest at time t = 0, a car moves in a straight line with an
acceleration given by the accompanying
graph. What is the speed of the car at t = 3 s?
D
A flare is dropped from a plane flying over level ground at a velocity of 70 m/s
in the horizontal direction. At the instant the flare is released, the plane begins
to accelerate horizontally at 0.75 m/s2. The flare takes 4.0 s to reach the
ground. Assume air resistance is negligible. Relative to a spot directly under
the flare at release, the flare lands
D

42 Kinematics

As seen by the pilot of the plane (in question #41) and measured relative to a
spot directly under the plane when the flare lands, the flare lands
B

39 Kinematics

40 Kinematics

greater than 60
above the
horizontal

greater than 45
but less than 60
above the
horizontal

greater than zero


but less than 45
above the
horizontal

zero

greater than zero


but less than 45
below the
horizontal

1.0 m/s

2.0 m/s

6.0 m/s

10.5 m/s

12.5 m/s

directly on the
spot.

6.0 m in front of
the spot.

274 m in front of
the spot.

280 m in front of
the spot.

286 m in front of
the spot

286 m behind the


plane.

6.0 m behind the


plane.

directly under the


plane.

12 m in front of the 274 m in front of


plane.
the plane

13.7 m/s

9.8 m/s

6.3 m/s

2.8 m/s

2.4 m/s

45 Kinematics

The graph above is a plot of position versus time. For which labeled region is
the velocity positive and the acceleration negative?
E
A child left her home and started walking at a constant velocity. After a time
she stopped for a while and then continued on with a velocity greater than
she originally had. All of a sudden she turned around and walked very quickly
back home. Which of the following graphs best represents the distance
versus time graph for her walk?
B
In a rescue attempt, a hovering helicopter drops a life preserver to a swimmer
being swept downstream by a
river current of constant velocity v. The helicopter is at a height of 9.8 m. The
swimmer is 6.0 m upstream from
a point directly under the helicopter when the life preserver isreleased. It
lands 2.0 m in front of the swimmer.
How fast is the current flowing? Neglect air resistance.
D

46 Kinematics

A child tosses a ball directly upward. Its total time in the air is T. Its maximum
height is H. What is its height
after it has been in the air a time T/4? Neglect air resistance.
E

H/4

H/3

H/2

2H/3

47 Kinematics

A whiffle ball is tossed straight up, reaches a highest point, and falls back
down. Air resistance is not negligible.
Which of the following statements are true?

I only

II only

I & II only

I & III only

I, II, & III

48 Kinematics

A truck driver travels three-fourths the distance of his run at one velocity (v)
and then completes his run at one
half his original velocity (v). What was the truckers average speed for the
trip?

0.85v

0.80v

0.75v

0.70v

0.65v

49 Kinematics

Above is a graph of the distance vs. time for car moving along a road.
According the graph, at which of the
following times would the automobile have been accelerating positively?

0, 20, 38, & 60


min.

5, 12, 29, & 35


min.

5, 29, & 57 min.

12, 35, & 41 min.

at all times from 0


to 60 min

50 Kinematics

A large beach ball is dropped from the ceiling of a school gymnasium to the
floor about 10 meters below.
Which of the following graphs would best represent its velocity as a function
oftime? (do not neglect air
resistance)

51 Kinematics

At what time would the car be moving with the greatest velocity?

0 sec

2 sec

4 sec

6 sec

8 sec

43 Kinematics

44 Kinematics

3H/4

The 45 angle gives the maximum


horizontal travel to the original
elevation, but the smaller angle causes
the projectile to have a greater
horizontal component of velocity, so
given the additional time of travel allows
such a trajectory to advance a greater
horizontal distance. In other words
given enough time the smaller angle of
launch gives a parabola which will
eventual cross the parabola of the 45
launch.
39.kinematics.png
The area under the curve of an
acceleration-time graph is the change
in speed.
40.kinematics.png

In the 4 seconds to reach the ground,


the flare travelled 70 m/s 4 s = 280 m
horizontally.
In the 4 seconds to reach the ground,
the flare travelled 70 m/s 4 s = 280 m
horizontally. The plane travelled d = vit
+ at2 = (70 m/s)(4 s) + (0.5)(0.75
m/s2)(4 s)2B = 280 m + 6 m, or 6 m
ahead of the flare.
Positive velocity = positive slope.
Negative acceleration = decreasing
slope (or downward curvature
The slope of the line represents her
velocity. Beginning positive and
constant, going to zero, then positive
and larger than the initial, then negative
while the line returns to the time axis
Dropped from a height of 9.8 m, the life
preserver takes (9.8 m = gt2
); t = 1.4 seconds to reach the water. In
that 1.4 seconds the swimmer covered
(6.0 m 2.0 m) = 4.0 m meaning the
water
speed is (4.0 m)/(1.4 s)
The ball takes time T/2 to reach height
H. Using vy= 0 at maximum height and
d/t gives the initial speed as 4H/T. In
addition from the top H = g(T/2)2
=gT2/8. Plugging in a time T/4 gives d=
(4H/T)(T/4) + (g)(T/4)2= H
(gT2/8) = H
While the object momentarily stops at
its peak, it never stops accelerating
downward. Without
air resistance, symmetry dictates time
up = time down. With air resistance
considered, the ball
will have a larger average velocity on
the way up and a lower average
velocity on the way down
since it will land with a smaller speed
than it was thrown, meaning the ball
takes longer to fall.
Total distance = d. Time for first d is
t1
= (d)/v = 3d/4v. Time for second part
is t2
= ( d)/( v) = 2d/4v. Total time is then
t1+ t2= 5d/4v. Average speed = d/
(5d/4v)
Positive acceleration is an increasing
slope (including negative slope
increasing toward zero) or
upward curvature
With air resistance, the acceleration
(the slope of the curve) will decrease
toward zero as the ball
reached terminal velocity. Note: without
air resistance, choice (A) would be
correct
Since for the first 4 seconds, the car is
accelerating positively the entire time,
the car will be
moving fastest just beofre slowing down
after t = 4 seconds.

43.kinematics.png

44.kinematics.PNG

47.kinematics.PNG

48.kinematics.PNG

50.kinematics.PNG

51.kinematics.PNG

v will increase to
v will increase
some critical value v will remain
monotonically with vmax and then
constant,
.
decrease.
independent of .

The area under the curve represents


the change in velocity. The car begins
from rest with an
increasing positive velocity, after 4
seconds the car begins to slow and the
area under the curve
from 4 to 8 seconds couters the
increase in velocity form 0 to 4
seconds, bringing the car to rest.
However, the car never changed
direction and was moving away from its
original starting
6 sec
8 sec
position the entire time.
52.kinematics.PNG
The ball will land with a speed given by
the equaion v2= vi2+ 2gH or v = 2gH.
Rebounding with the speed gives a
new height of vf= 0 = ( 2gH )2+ 2(g)
0.75 m
0.84 m
hnew
The velocity-time graph should
represent the slope of the position-time
graph and the
acceleration-time graph should
represent the slope of the velocity-time
D
E
graph
54.kinematics.PNG
Its a surprising result, but while both
the horizontal and vertical components
v will decrease to
change at a given
some critical value
height with varying launch angle, the
vmin and then
speed (vx2 + vy2)1/2 will be
increase.
None of the above. independent of (try it!)

5.0 m/s2

2.5 m/s2

2.0 m/s2

0.5 m/s2

0.2 m/s2

14 m

20 m

28 m

34 m

56 m

10m

20m

30m

40m

50m

0.1 m/s2

0.2 m/s2

2.0 m/s2

2.8 m/s2

5.6 m/s2

33 m

3(2^3/2) m

3 m

3m

9m

2 m/s

12 m/s

0 m/s

2 m/s

4 m/s

52 Kinematics

At what time would the car be farthest from its original starting position?

0 sec

2 sec

4 sec

53 Kinematics

A ball is dropped 1.0 m to the floor. If the speed of the ball as it rebounds
from the floor is 75% of the speed at
which it struck the floor, how high will the ball rise?

0.28 m

0.35 m

0.56 m

54 Kinematics

. Which of the following sets of graphs might be the corresponding graphs of


Position, Velocity, and
Acceleration vs. Time for a moving particle?

55 Kinematics

56 Kinematics

57 Kinematics

An object is thrown with a fixed initial speed v0 at various angles relative to


the horizon. At some constant height h above the launch point the speed v of
the object is measured as a function of the initial angle . Which of the
following best describes the dependence of v on ? (Assume that the height
h is achieved, and assume that there is no air resistance.)
C
A bird is flying in a straight line initially at 10 m/s. It uniformly increases its
speed to 15 m/s while covering a distance of 25 m. What is the magnitude of
the acceleration of the bird?
B
A person standing on the edge of a fire escape simultaneously launches two
apples, one straight up with a speed of 7 m/s and the other straight down at
the same speed. How far apart are the two apples 2 seconds after they were
thrown, assuming that neither has hit the ground?
C

vf2=vi2+2ad
d1 = (+7 m/s)(2 s) + 12 (10 m/s2)(2 s)
2; d1 = (7 m/s)(2 s) + 12 (10 m/s2)(2
s)2
Range of a projectile R = (vi2 sin 2)/g
and maximum range occurs at = 45o,
which gives vi = Rg . Maximum height
of a projectile is found from vy = 0 at
max height and vy2 = viy2 + 2gh
and gives hmax = viy2/2g = (vi sin )
2/2g. Maximum range occurs at 45o,
which gives h = (Rg)(sin 45)2/2g

61 Kinematics

A certain football quarterback can throw a football a maximum range of 80


meters on level ground. What is the highest point reached by the football if
thrown this maximum range? Ignore air friction.
A bird flying in a straight line, initially at 10 m/s, uniformly increases its speed
to 18 m/s while covering a distance of 40 m. What is the magnitude of the
acceleration of the bird?
A cockroach is crawling along the walls inside a cubical room that has an
edge length of 3 m. If the cockroach starts from the back lower left hand
corner of the cube and finishes in the front upper right hand corner, what is
the magnitude of the displacement of the cockroach?
The position vs. time graph for an object moving in a straight line is shown
below. What is the instantaneous velocity at t = 2 s?

62 Kinematics

Shown below is the velocity vs. time graph for a toy car moving along a
straight line. What is the maximum displacement from start for the toy car?

3m

5m

6.5m

7m

7.5m

63 Kinematics

A cannon fires projectiles on a flat range at a fixed speed but with variable
angle. The maximum range of the cannon is L. What is the range of the
cannon when it fires at an angle /6 above the horizontal? Ignore air.

3/2 L

1/2 L

1/3 L

1/2 L

1/3 L

0.22 m/s2

0.33 m/s2

1.0 m/s2

9.8 m/s2

30 m/s2

2h

3h

4h

h2

vf2=vi2+2ad
The diagonal of a face of the cube is 3
2 m. The diagonal across the cube itself
is the hypoteneuse of this face
diagonal and a cube edge
Instantaneous velocity is the slope of
the line at that point
Displacement is the area under the
curve. Maximum displacement is just
before the car turns D around at 2.5
seconds.
Range of a projectile R = (vi2 sin 2)/g
and maximum range occurs at = 45o,
which gives vi = A Rg .
Using=30ogivesRnew =Rsin60o
advanced question!) The time for one
bounce is found from v = v + (g)t
which gives t = 2v/g. A We are
summing the time for all bounces, while
the velocity (and hence the time)
converge in a geometric series with the
ratio vn+1/vn = r <1 to 1 1r
The acceleration is the slope of the
curve at 90 seconds.
From the equation d = 12 at2,
displacement is proportional to time
squared. Traveling from rest for twice
the time gives 4 times the displacement
(or 4 h). Since the object already
travelled h in
the first second, during the time interval
from 1 s to 2 s the object travelled the
remaining 3h

45m

60m

90m

105m

120m

d=vit+12gt2

58 Kinematics

59 Kinematics

60 Kinematics

64 Kinematics
65 Kinematics

66 Kinematics

67 Kinematics

Every time it rebounds, it loses a proportion of the magnitude of its velocity


due to the inelastic nature of the collision, such that if the speed just before
hitting the ground on a bounce is v, then the speed just after the bounce is rv,
where r < 1 is a constant. Calculate the total length of time that the ball
remains bouncing, assuming that any time associated with the actual contact
of the ball with the ground is negligible.
A
The graph shows velocity as a function of time for a car. What was the
acceleration at time t = 90 seconds?
B

An object is released from rest and falls a distance h during the first second of
time. How far will it fall during the next second of time?
C
A stone is thrown straight downward with a speed of 20 m/s from the top of a
tall building. If the stone strikes the ground 3.0 s later, about how tall is the
building? Assume air resistance is negligible.
D

61.kinematics

62.kinematics

64.kinematics
65.kinematics

68 Kinematics

A coyote can run at a speed of 20 m/s while a prairie dog can manage only
5.5 m/s. If a prairie dog is 45 m in front of a coyote, what is the maximum time
it has to reach its hole without being caught?
B

2.3 s

3.1 s

5.4 s

5.9 s

8.2 s

100 m

510 m

610 m

1020 m

1220 m

The relative speed between the coyote


and the prairie dog is 14.5 m/s. To
cover the 45 m B distance between
them will take t = d/v = (45 m)/(14.5
m/s)
Forthefirstpartofthetrip(thethrust):d1
=vit+12at2 =0m+12(50m/s2)(2s)2
=100m For the second part, we first find
the velocity after the thrust v = at = 100
m/s and at the maximumheightvf =0,
sotofindd2 weusevf2 =vi2 +2ad2
whichgivesd2 =510m
Total displacement west = 1100 m; total
displacement south = 400 m. Use the
Pythagorean theorem
For a horizontal projectile (viy = 0 m/s)
to fall 1 m takes (using 1 m = 12 gt2)
0.45 seconds. To travel 30 m in this
time requires a speed of d/t = (30 m)/
(0.45 s)
Maximum height of a projectile is found
from vy = 0 m/s at max height and (0
m/s)2 = v2 + 2gh and gives h = v2/2g 2
2
Theheightatwhichtheprojectileismovingwithhalfthespeedisfoundfrom(12v)
=v +2(
g)d which gives d = 3v2/8g = 0.75 h
Looking at choices A, D and E
eliminates the possibility of choices B
and C (each ball increases
its speed by 9.8 m/s each second,
negating those choices anyway). Since
ball A is moving faster
than ball B at all times, it will continue to
pull away from ball B (the relative
speed between the
balls separates them).
73.kinematics
Since they all have the same horizontal
component of the shells velocity, the
shell that spends
the longest time in the air will travel the
farthest. That is the shell launched at
the largest angle
(mass is irrelevant).
74.kinematics
This is merely asking for the horizontal
range of a horizontal projectile. The
time in the air is
found from the height using h = gt^2
which gives t =^1/2(2h/g) . The range is
found using d = vt
Flying into the wind the airliners speed
relative to the ground is 500 km/h 100
km/h = 400 km/h
and a 3000 km trip will take t = d/v = 7.5
hours. Flying with the wind the airliners
speed relative
to the ground is 500 km/h + 100 km/h =
600 km/h and a 3000 km trip will take t
= d/v = 5 hours
making the total time 12.5 hours.
The horizontal component of the
velocity is 28.3 m/s cos 60 = 14.15
m/s. If the ball is in the air
for 5 seconds the horizontal
displacement is x = vxt
since (from rest) d = gt^2, distance is
proportional to time squared. An object
falling for twice
the time will fall four times the distance.

70 Kinematics

A model rocket accelerates from rest upwards at 50 m/s2 for 2.0 s before its
engine burns out. The rocket then coasts upward. What is the maximum
height that the rocket reaches? You may assume air resistance is negligible.
A hunter in a forest walks 800 m west. He then turns south and walks 400 m
before turning west again and walking a final 300 m. At the end of the walk,
what is the magnitude of the hunter's displacement from the beginning?

640 m

890 m

1170 m

1390 m

1500 m

71 Kinematics

Robin Hood aims his longbow horizontally at a target's bull's eye 30 m away.
If the arrow strikes the target exactly 1.0 m below the bull's eye, how fast did
the arrow move as it was shot from the bow? Assume air resistance is
negligible.

6 m/s

13 m/s

33 m/s

67 m/s

150 m/s

72 Kinematics

A baseball is thrown vertically into the air with a velocity v, and reaches a
maximum height h. At what height was the baseball moving with one-half its
original velocity? Assume air resistance is negligible.

.25 h

.33 h

.50 h

.67 h

.75 h

73 Kinematics

Two identical bowling balls A and B are each dropped from rest from the top
of a tall tower as shown in the
diagram below. Ball A is dropped 1.0 s before ball B is dropped but both balls
fall for some time before ball A
strikes the ground. Air resistance can be considered negligible during the fall.
After ball B is dropped but before
ball A strikes the ground, which of the following is true?
E

The distance
between the two
balls decreases.

The distance
The velocity of ball The velocity of ball between the two
A increases with
A decreases with balls remains
respect to ball (B) respect to ball (B) constant.

The distance
between the two
balls increases.

74 Kinematics

The diagram below shows four cannons firing shells with different masses at
different angles of elevation. The
horizontal component of the shell's velocity is the same in all four cases. In
which case will the shell have the
greatest range if air resistance is neglected?

cannon A

cannon B only

cannon C only

cannon D

Both cannons B
and C have the
greatest range

75 Kinematics

Relief supplies are being dropped to flood victims from an airplane flying
horizontally at a speed v. If the
airplane is at an altitude of h above the ground, what distance d in front of the
landing site should the supplies
be dropped?
E

2v^1/2(h/g)

2vh/g

2^1/2(vh/h)

2vh^2/g^2

v^1/2(2h/g)

10hr

12hr

12.5hr

13.5hr

15hr

0m

14.2m

24.5m

70.8m

122.5m

half as far

twice as far

three times as far

four times as far

sixteen times as
far

150m

300m

450m

600m

800m

100 m/s up

20 m/s up

20 m/s

40 m/s

100 m/s

200 m/s

400 m/s

v=(vi+vf)/2=d/t
vf = 40 m/s (negative since it is
moving down when landing). Use vf = vi
+ (g)t
For a horizontal projectile (viy = 0 m/s)
to fall 0.05 m takes (using 0.05 m =
gt^2) 0.1 seconds.
To travel 20 m in this time requires a
speed of d/t = (20 m)/(0.1 s)

270 m/s

37 m/s

27 m/s

17 m/s

10 m/s

9.8 m/s^2 means the speed changes by


9.8 m/s each second

69 Kinematics

76 Kinematics

77 Kinematics

78 Kinematics

79 Kinematics

80 Kinematics

81 Kinematics

82 Kinematics

An airliner flies at a speed of 500 km/hr with respect to the air. The jet stream
blows from west to east with a
speed of 100 km/hr. What is the minimum time in which the airliner could fly
3000 km due west and then back
to its original starting position?
C
A punter in a football game kicks the ball with an initial speed of 28.3 m/s at
an angle of 60 with respect to the
ground. The ball is in the air for a total of 5.00 s before hitting the ground. If
we assume that air resistance is
negligible, what would be the ball's horizontal displacement?
D
Starting from rest, object 1 falls freely for 4.0 seconds, and object 2 falls freely
for 8.0 seconds. Compared to
object 1, object 2 falls:
D
A car starts from rest and uniformly accelerates to a final speed of 20.0 m/s in
a time of 15.0 s. How far does the
car travel during this time?
A
A ball is thrown off a high cliff with no horizontal velocity. It lands 6.0 s later
with a velocity of 40 m/s. What
was the initial velocity of the ball?
B
An arrow is aimed horizontally, directly at the center of a target 20 m away.
The arrow hits 0.050 m below the
center of the target. Neglecting air resistance, what was the initial speed of
the arrow?
D
A freely falling body is found to be moving downwards at 27 m/s at one
instant. If it continues to fall, one
second later the object would be moving with a downward velocity closest to: B

0 20 m/s down

100 m/s down

83 Kinematics

A rocket near the surface of the earth is accelerating vertically upward at 10


m/s2. The rocket releases an
instrument package. Immediately after release the acceleration of the
instrument package is:

84 Kinematics

20 m/s^2 up

10 m/s^2 up

A car starts from rest and accelerates at 0.80 m/s2 for 10 s. It then continues
at constant velocity. Twenty
seconds (20 s) after it began to move, the car has a
C

velocity of 8m/s
and has traveled
40m

velocity of 8m/s
and has traveled
80m

velocity of 8m/s
and has traveled
120m

velocity of 16m/s
and has traveled
160m

velocityof 16m/s
and has traveled
320m

A
A

B
B

C
C

D
D

E
E

87 Kinematics

A solid metallic sphere of radius R has charge Q uniformly distributed on its


outer surface. A graph of electric potential V as a function of position r is
shown above. Which of the following graphs best represents the magnitude of
the electric field E as a function of position r for this sphere?
C
What is the magnitude of the resultant electric field at the center of the circle? A
With the six particles held fixed, how much work would be required to bring a
seventh particle of charge + Q from very far away and place it at the center of
the circle?
D

88 Kinematics

Which vector best describes the direction of the electric field at point A ?

89 Kinematics

At which point does the electric field have the greatest magnitude?

-20 micro joules

-10 micro joules

10 micro joules

20 micro joules

30 micro joules

Once released, the package is in freefall (subject to gravity only)


For the first part v = at = 8.0 m/s and d
= at2 = 40 m. In the second part of
the trip, the speed
remains at 8 m/s, and travels an
additional d = vt = 80 m
Since E = V/d, E represents the
slope of the line on the graph
which could be choice C or D.
since V 1/r the slope is
proportional to V/r = (1/r)/r =
1/r2C which is choice C
by symmetry, all the vectors cancel
W = qV = +Q(Vcenter V) =
+QVcenter where VcenterD = V =
kQ/r = 6kQ/R
E points from high potential to low
potential, perpendicular to equipotential
lines (the direction of the force on a
positive charge)
E is greatest in magnitude where V
changes most rapidly with position (the
largest gradient) which is where the
lines are closest together.
VCE = VE VCB = 10 V. The
amount of work, W = qV = 1 C 10
V = 10 C. Since the external force
must push against the negative charge
to keep it from accelerating and bring it
to rest at point E, the work done by the
external force must be negative

C = K0D A/d. Only changes in the


geometry of the capacitor will change
the capacitance, not changes to the
battery or resistor

85 Kinematics
86 Kinematics

0 10 m/s^2 down

91 Kinematics

How much net work must be done by an external force to move a 1 C point
charge from rest at point C to rest at point E ?
B
The plates of a parallelplate capacitor of cross sectional area A are
separated by a distance d, as shown above. Between the plates is a dielectric
material of constant K. The plates are connected in series with a variable
resistance R and a power supply of potential difference V. The capacitance C
of this capacitor will increase if which of the following is decreased?
D

92 Kinematics

A physics problem starts: "A solid sphere has charge distributed uniformly
throughout. . . " It may be correctly concluded that the

93 Kinematics

A uniform spherical charge distribution has radius R.. Which of the following
is true of the electric field strength due to this charge distribution at a distance
r from the center of the charge?
D

It is directly
It is greatest when It is greatest when proportional to r
r = 0.
r = R/2.
when r > R.

94 Kinematics

When a negatively charged rod is brought near, but does not touch, the
initially uncharged electroscope shown above, the leaves spring apart (I).
When the electroscope is then touched with a finger, the leaves collapse (II).
When next the finger and finally the rod are removed, the leaves spring apart
a second time (III). The charge on the leaves is
D

positive in both I
and III

negative in both I
and III

positive in I,
negative in III

95 Kinematics

An ideal elastic rubber ball is dropped from a


height of about 2 meters, hits the floor and rebounds to its original height.
Which of the following graphs would best represent the distance above the
floor versus time for the above
bouncing ball?

(A)

(B)

(C)

96 Kinematics

An ideal elastic rubber ball is dropped from a


height of about 2 meters, hits the floor and rebounds to its original height.
Which of the following graphs would best represent acceleration versus time
for the bouncing ball?

(A)

(B)

(C)

97 Kinematics

During which time interval would cars #2 and #3 be moving at the same
average speed?

t0 to t1

t1 to t2

t2 to t3

98 Kinematics

About what position after t0 would car #1 and car #2 have been side by side? D

0m

99 Kinematics

car #1

15 m
car #2 and car #3
had the same
average speed

26 m

Which of the three cars had the greatest average speed during these 5
seconds?

90 Kinematics

R
electric field inside
the sphere is the
electric field is zero same as the
everywhere inside electric field
the sphere
outside

K
electric potential
on the surface of
the sphere is not
constant

car #2

20 m/s^2 down

electric potential in
the center of the
sphere is not made For charge to be distributed throughout
sphere is zero
of metal
a material, it must be non-conducting
Advanced question (not exactly in the B
curriculum, but interesting). Like gravity
inside a uniform sphere of mass, the
It is directly
field is directly proportional to r when
proportional to r
It is directly
inside the sphere (and proportional to
when r < R.
proportional to r2 1/r2D when outside)
In I, charge separation occurs (negative
charges repel to the leaves). The
whole process describes charging by
induction, where the electrons leave the
electroscope to ground (the finger) and
impossible to
once contact with ground is broken, the
negative in I,
determine in either electroscope is left with a positive
positive in III
I or III
charge (III)
One could analyze the graphs based on
slope, but more simply, the graph of
position versus time
should represent the actual path
followed by the ball as seen on a
platform moving past you at
(D)
(E)
constant speed.
Other than the falling portions (a = 9.8
m/s2
) the ball should have a spike in the
acceleration B
when it bounces due to the rapid
change of velocity from downward to
(D)
(E)
upward.
The same average speed would be
indicated by the same distance
t3 to t4
t4 to t5
travelled in the time interval
At t3, car #1 is ahead of car #2 and at
t4, car #1 is behind car #2. They were
in the same position somewhere in
37 m
39 m
between
all three cars had
Average speed = (total distance)/(total
the same average
time). Cars #2 and #3 travelled the
speed
car #3
same distance.

85.kinematics.png
86.kinematics.png

87.kinematics.png

88.kinematics.png

89.kinematics.png

90.kinematics.png

91.kinematics.png

94.kinematics.png

95.kinematics.png

96.kinematics.png

97.kinematics.png

98.kinematics.png

99.kinematics.png

101 Kinematics

If car #3 continues to constantly accelerate at the same rate what will be its
position at the end of 6 seconds?
The graph represents the relationship between distance and time for an
object that is moving along a straight
line. What is the instantaneous speed of the object at t = 5.0 seconds?

102 Kinematics

103 Kinematics

100 Kinematics

104 Kinematics
105 Kinematics
106 Kinematics

0.0 m/s

Between what times did the object have a non-zero acceleration?

0 s only

An airplane takes off and flies 300 miles at an angle of 30 north of east. It
then changes direction and flies 600
miles due west before landing. In what direction is the planes landing point
from its starting point?

If a ball is thrown directly upwards with twice the initial speed of another, how
much higher will it be at its
apex?
C
What was the average speed of the cart between 0.1 seconds and 0.3
seconds?
E
What was the acceleration of the cart during the first 0.4 seconds?
What was the instantaneous velocity of the cart at 96 centimeters from the
start?

107 Kinematics
108 Kinematics

What would be the acceleration of the clown at 5s?

109 Kinematics

After 12 seconds, how far is the clown from her original starting point?
E
A box falls to the ground from a delivery truck traveling at 30 m/s. After hitting
the road, it slides 45 meters to rest. How long does it take the box to come to
rest?
D

110 Kinematics

111 Kinematics

22 m

When an object falls freely in a vacuum near the surface of the earth

68 m

0.8 m/s

72 m

2.5 m/s

78 m

94 m

4.0 m/s
6.8 m/s
the object was not
accelerating at any
time
5 s to 8 s

100.kinematics.png

Instantaneous speed is the slope of the


line at that point.
101.kinematics.png

A non-zero accleeration is inidcated by


a curve in the line
102.kinematics.png
Net displacement north = 300 miles sin
30 = 150 miles
Net displacement east = (300 miles cos
30 600 miles) = 340 miles, or 340
miles west.
14.2 north of west 66.2 north of west 23.8 north of west 75.9 north of west 37.4 north of west Angle north of west
Maximum height of a projectile is found
from vy= 0 m/s at max height and (0
m/s)2= v2+ 2gh and gives h = v2/2g. At
twice the initial speed, the height will be
8 times
2 times
4 times
2 times
22 times
4 times as much
Average speed = total distance divided 105-107.kinematics.
0.6 m/s
4.8 m/s
1.9 m/s
60 m/s
2.4 m/s
by total time = (0.48 m)/(0.2 s)
png
105-107.kinematics.
6.0 m/s^2
25 m/s^2
9.8 m/s^2
50 m/s^2
12 m/s^2
d = at^2 (use any point)
png
105-107.kinematics.
0.6 m/s
4.8 m/s
1.9 m/s
60 m/s
2.4 m/s
v = viB + at
png
Acceleration is the slope of the line
108-109.kinematics.
1.6 m/s^2
8.0 m/s^2
2.0 m/s^2
none of the above 3.4 m/s^2
segment
png
108-109.kinematics.
0m
10 m
34 m
47 m
74 m
Displacement is the area under the line png
0 s to 8 s

0.67 s

0 s to 5 s

1.5 s
the terminal
velocity will be
the velocity cannot greater than when
exceed 10 m/s
dropped in air

2.0 s
the velocity will
increase but the
acceleration will be
zero

3.0 s

6.0 s

vi=30m, vf=0, d=45m; v=(vi+vf)/2=d/t

the acceleration
will remain
constant

Ina vaccum, there is no air resistance


and henve no terminal velocity. It will
continue to accelerate.

they both arrive


together

the acceleration
will constantly
increase
it depends on the
elecation where
the arrows are
launched

arrow A arrives
first

arrow B arrives
first

A projectile launched at a smaller angel


does not go as high and will fall to the
ground first.

the cosine of the


angle

the tangent of the


angle

it depends on the
elevation where
the arrows land
the value of the
gravitational
acceleration
from t=3.2s to t=3.
6s, from t=4.8 to
t=5s, and from t=6.
8s to t=7.2s only
from t = 3.3 s to t =
3.7 s, from t = 4.8
s to t = 5.0 s, and
from t = 6.8 s to t =
7.2 s only
0.8 m/s2

the angle

the sine of the


angle

113 Kinematics

Two arrows are launched at the same time with the same speed. Arrow A at
an angle greater than 45 degrees, and arrow B at an angle less than 45
degrees. Both land at the same spot on the ground. Which arrow arrives first? B
A ball is thrown into the air at an angle as measured from the horizontal
with a velocity v. The horizontal velocity of the ball will be directly proportional
to which of the following
C

114 Kinematics

For which time interval(s) did the marble have a negative velocity?

115 Kinematics
116 Kinematics

D
E

117 Kinematics

For which time interval(s) did the marble have a positive acceleration?
What is the marbles average acceleration between t = 3.1 s and t = 3.8 s?
A car accelerates uniformly from rest for a time of 2.00 s through a distance
of 4.00 m. What was the acceleration of the car?

from t=4.8s to t=6.


2s and from t=6.9s
to t=10s only
from t = 2.0 s to t =
2.5 s, from t = 5.8
from t = 3.8 s to t = s to t = 6.2 s, and
from t = 0.0 s to t = from t = 0.0 s to t = 4.8 s and t = 6.2 s from t = 8.4 s to t =
8.0 s only
3.6 only
to t = 6.8 s only
8.8 s only
2.0 m/s2
2.0 m/s2
0.0 m/s2
3.0 m/s2

0.50 m/s2

0.71 m/s2

1.00 m/s2

1.41 m/s2

2.00 m/s2

118 Kinematics

What is the acceleration of the toy car at t = 4 s?

1 m/s2

0 m/s2

1 m/s2

2 m/s2

4 m/s2

119 Kinematics

What was the total displacement of the toy car for the entire 10 second
interval shown?

0 meters

6.5 meters

9 meters

10 meters

120 Kinematics

An object is thrown upwards with a velocity of 30 m/s near the surface of the
earth. After two seconds what would be the direction of the displacement,
velocity and acceleration?

up; up; up

up; up; down

up; down; down

up; down; up

121 Kinematics

Which of the following graphs could correctly represent the motion of an


object moving with a constant speed in a straight line?

Graph I only

Graphs II and V
only

Graph II only

Graphs I and IV
only

122 Kinematics
123 Kinematics

What is the average speed of the ball between 3 and 4 seconds?


Which of the following is closest to the acceleration of the ball?

B
C

3.0 cm/s
1 cm/s2

7.0 cm/s
4 cm/s2

3.5 cm/s
2 cm/s2

12.5 cm/s
5 cm/s2

124 Kinematics

Three stones of different mass (1 m, 2m & 3m) are thrown vertically upward
with different velocities (l v, 2v &
3v respectively). The diagram indicates the mass and velocity of each stone.
Rank from high to low the
maximum height of each stone. Assume air resistance is negligible.

I, II, III

II, I, III

III, II, I

I, III, II

112 Kinematics

If you look at the distance covered in


each time interval you should nitce a
pattern: 2 m, 6 m, 10
m, 14 m, 18 m; making the distance in
the next second 22 m.

from t=8.0s to
t=10.0s only

from t=6.9s to 10.0 from t=3.8s to


s only
t=10s only

vx=vicos

velocity is the slope of the line

114.kinematics.png

Positive acceleration is an upward


curvature
Average acceleration = v/t

114.kinematics.png
114.kinematics.png

d = at2
Acceleration is the slope of the line
segment
Displacement is the area between the
line and the t-axis. Area is negative
11.5 meters
when the line is below the t-axis.
After two seconds, the object would be
above its original position, still moving
upward, but the acceleration due to
down; down; down gravity is always pointing down.
Constant speed is a constant slope on
a position-time graph, a horizontal line
All of the above
on a velocity time
graphs represent graph or a zero value on an
constant velocity
acceleration-time graph
Average speed = total distance divided
4.0 cm/s
by total time = (7 cm)/(1 s)
3 cm/s2
d = at2 (use any point)
Maximum height of a projectile is found
from vy = 0 m/s at max height and (0
m/s)2 = v2 + 2gh
and gives h = v2
C
all reach the same /2g. Mass is irrelevant. Largest initial
height
speed = highest.

118.kinematics. png

118.kinematics.png

118.kinematics.png

121.kinematics.png
122.kinematics.png
122.kinematics.png

125 Kinematics

126 Kinematics

127 Kinematics

128 Kinematics

A rubber ball bounces on the ground as shown. After each bounce, the ball
reaches one-half the height of the
bounce before it. If the time the ball was in the air between the first and
second bounce was 1 second. What
would be the time between the second and third bounce?

B.

The driver of a car makes an emergency stop by slamming on the car's


brakes and skidding to a stop. How far
would the car have skidded if it had been traveling twice as fast?
A
A pebble is dropped from a high vertical cliff. The collision of the pebble with
the ground below is seen 1.50
seconds after the pebble is dropped. With what speed did the pebble hit the
ground? Ignore air resistance.
B.
A snail is moving along a straight line. Its initial position is x0= 5 meters and
it is moving away from the
origin and slowing down. In this coordinate system, the signs of the initial
position, initial velocity and
acceleration, respectively, are
B.

131 Kinematics

An arrow is shot horizontally toward a target 20 m away. In traveling the first 5


m horizontally, the arrow falls
0.2 m. In traveling the next 5 m horizontally, it will fall an additional
A
How tall is a tree if the sun is at a 53 angle above the horizon and the
shadow is 8.0 meters long?
B.
Three students were arguing about the height of a parking garage. One
student suggested that to determine the
height of the garage, they simply had to drop tennis balls from the top and
time the fall of the tennis balls. If the
time for the ball to fall was 1.4 seconds, approximately how tall is the parking
garage?
C

132 Kinematics

An arrow is shot from a bow at an angle of 40 from the horizontal at a speed


of 24.0 m/s. Ignoring air
resistance, what is the arrows maximum height above its launch point?
B.

133 Kinematics

A car has the velocity versus time curve shown above. Which of the following
statements regarding its motion
is INCORRECT?
D

134 Kinematics

A rock is dropped from the top of a tall tower. Half a second later another
rock, twice as massive as the first, is
dropped. Ignoring air resistance,

135 Kinematics

A cart is initially moving at 0.5 m/s along a track. The cart comes to rest after
traveling 1 m. The experiment is repeated on the same track, but now the cart
is initially moving at 1 m/s. How far does the cart travel before coming to rest? B

129 Kinematics
130 Kinematics

136 Kinematics

137 Kinematics

During an interval of time, a tennis ball is moved so that the angle between
the velocity and the acceleration of the ball is kept at a constant 120o. Which
statement is true about the tennis ball during this interval of time?
B
A dog starts from rest and runs in a straight line with a constant acceleration
of 2.5 m/s2. How much time does it take for the dog to run a distance of 10.0
m?
C

0.50 sec

0.71 sec

4 times as far

the same distance 2 times as far

square root of 2
times as far

Using d=1/2at^2 shows the height is


proportional to the time squared. 1/2
the maximum height is 1/sq root of 2
2.0 sec
times the time.
125.kinematics.png
Stopping distance is found using
Vt=0=vi^2+2ad which gives d=vi/2a
the mass of the car where stopping distance is proportional
must be known
to initial speed squared.

10 m/s

15 m/s

48.6 m/s

100.4 m/s

343 m/s

0.6 m

0.4 m

0.3 m

0.2 m

0.1 m

Vt=Vi+gt
Moving away from the origin will
maintain a negative position and
velocity. Slowing down
indicates the acceleration is opposite in
direction to the velocity.
128.kinematics.png
The arrow travels equal horizontal
distances in equal amounts of time. The
distance fallen is
proportional to time squared. The arrow
will have fallen a total of 0.8 m in the
next 5 m
horizontally, or an additional 0.6 m.

4.8 m

10.6 m

6.0 m

13.3 m

8.0 m

Tan 53 degrees=h/(8m)

4.9 m

7.0 m

9.8 m

13.8 m

19.6 m

5.9 m

11.9 m

16.9 m

d=1/2at^2
Maximum height of a projectile is found
from vy = 0 at max height and vy
2 = viy 2 + 2gh and gives hmax = viy
2/2g = (vi sin )2 B/2g

1.0 sec

1.4 sec

The car is at rest


The car is moving at approximately
fastest at 2.0 s
5.2 s.
the distance
the acceleration is
between the rocks greater for the
increases while
more massive
both are falling.
rock.

23.8 m
The car has
The car is
negative
speeding up from t acceleration at t =
= 0 to t = 2.0 s
4.5 s.
they strike the
the speed of both ground more than
rocks is constant
half a second
while they fall.
apart.

28.8 m
The car has no
acceleration at the
instant t = 2 s.
24
they strike the
ground with the
same kinetic
energy.

1m

2m

3m

4m

Its speed
increases and it is
changing its
direction of travel.

Its speed
decreases and it is
changing its
direction of travel.

Its speed remains


constant, but it is
changing its
direction of travel.

Its speed remains


constant and it is
not changing its
direction of travel.

Acceleration is the slope of the line


Since the first rock is always traveling
faster, the relative distance between
them is always
increasing.
Stopping distance is found using vf
= 0 = vi^2+ 2ad which gives d = vi^2/2a
where stopping B
distance is proportional to initial speed
8m
squared.
At an angle of 120, there is a
component of the acceleration
perpendicular to the velocity
Its speed
causing the direction to change and a
decreases and it is component in the opposite direction of
not changing its
the velocity, causing
direction of travel. it to slow down.

8.0 s

4.0 s

2.8s

2.0 s

1.4 s

138 Kinematics

What is the direction of the average velocity during this time interval?

The average
velocity is zero.

139 Kinematics

What is the direction of the average acceleration during this time interval?

The average
acceleration is
zero.

140 Kinematics

At which time is the car the greatest distance from the origin?

t = 10 s

t=6s

t=5s

t=3s

t=0s

141 Kinematics

What is the average speed of the car for the 10 second interval?
Consider the motion of an object given by the position vs. time graph shown.
For what time(s) is the speed of the object greatest?
The free fall trajectory of an object thrown horizontally from the top of a
building is shown as the dashed line in the figure. Which sets of arrows best
correspond to the directions of the velocity and of the acceleration for the
object at the point labeled P on the trajectory?

1.20 m/s
At all times from
=0.0st=2.0s

1.40 m/s

3.30 m/s

At time t=3.0s

At time t=4.0s

5.00 m/s
5.40 m/s
At all times from t
= 5.0 s t = 7.0 s At time t = 8.5 s

142 Kinematics

143 Kinematics

d = a^2
The displacement is directly to the left.
The average velocity is proportional to
the displacement
The velocity is initially pointing up, the
final velocity points down. The
acceleration is in the
same direction as v = vf
+ (vi)
The car is the greatest distance just
before it reverses direction at 5
seconds
Average speed = (total distance)/(total
time), the total distance is the
magnitude of the area under
the line (the area below the t-axis is
considered positive)

138.kinematics.png

138.kinematics.png

140.kinematics.png

140.kinematics.png

Speed is the slope of the line.

142.kinematics.png

velocity is pointing tangent to the path,


acceleration (gravity) is downward.

143.kinematics.png

144 Kinematics

A toy car moves 3.0 m to the North in one second. The car then moves at 9.0
m/s due South for two seconds. What is the average speed of the car for this
three second trip?
D

145 Kinematics

A projectile launched from the ground landed a horizontal distance of 120.0 m


from its launch point. The
projectile was in the air for a time of 4.00 seconds. If the projectile landed at
the same vertical position from
which it was launched, what was the launch speed of the projectile? Ignore
air resistance.
c

146 Kinematics

Two automobiles are 150 kilometers apart and traveling toward each other.
One automobile is moving at 60
km/h and the other is moving at 40 km/h. In how many hours will they meet?

147 Kinematics

A particle moves on the x-axis. When the particles acceleration is positive


and increasing

148 Kinematics

What does one obtain by dividing the distance of 12 Mm by the time of 4 Ts? B

149 Kinematics

150 Kinematics

151 Kinematics

152 Kinematics

. Is it possible for an objects velocity to increase while its acceleration


decreases?
E
During a recent winter storm, bales of hay had to be dropped from an airplane
to a herd of cattle below. Assume
the airplane flew horizontally at an altitude of 180 m with a constant velocity
of 50 m/s and dropped one bale of
hay every two seconds. It is reasonable to assume that air resistance will be
negligible for this situation. As the bales are falling through the air, what will
happen to their distance of separation?
A
During a recent winter storm, bales of hay had to be dropped from an airplane
to a herd of cattle below. Assume
the airplane flew horizontally at an altitude of 180 m with a constant velocity
of 50 m/s and dropped one bale of
hay every two seconds. It is reasonable to assume that air resistance will be
negligible for this situation. About how far apart from each other will the bales
land on the ground?
D

If a boat can travel with a speed of v in still water, which of the following trips
will take the least amount of
time.

Average speed = (total distance)/(total


time)
To travel 120 m horizontally in 4 s gives
vx
= 30 m/s. The time to reach maximum
height was 2
seconds and vy
= 0 at the maximum height which gives
viy = 20 m/s. vi
SQRT(Vx^2+Viy^2)
22.4m/s
30.0m/s
36.1m/s
42.4m/s
50.0m/s
=v
The relative speed between the two
cars is v1 v2
= (60 km/h) (40 km/h) = 100 km/h.
They
will meet in t = d/vrelative
A
1.5
1.75
2
2.5
3 = 150 km/100 km/h
Acceleration is independent of velocity
(you can accelerate in any direction
its velocity must be its velocity must be it must be slowing it must be
none of the above while traveling in a
positive
negative
down
speeding up
must be true.
ny direction).
12/4 =3, now the units: M = 106
, T = 1012: M/T = 106
3 nm/s
3 m/s
3mm/s
3km/s
3 Gm/s
= micro ()
No, this is
Acceleration is independent of velocity
impossible
No, because if
No, because
Yes, an example
Yes, an example
(you can accelerate in any direction
because of the
acceleration is
velocity and
would be a falling would be a falling while traveling in a
way in which
decreasing the
acceleration must object near the
object in the
ny direction). If the accleration is in the
acceleration is
object will be
always be in the
surface of the
presence of air
same direction as the velocity, the
defined
slowing down
same direction
moon
resistance
object is speeding up.
4.0 m/s

5.0 m/s

6.0 m/s

7.0 m/s

12 m/s

the distance of
separation will
increase

the distance of
separation will
decrease

the distance of
separation will
remain constant

the distance of
separation will
depend on the
mass of the bales

As the first bales dropped will always


be traveling faster than the later bales,
their relative
none of the above velocity will cause their separation to
are always true
always iincrease.

9000m

300m

180m

100m

traveling a
distance of 2d in
still water

traveling a
distance of 2d
across
(perpendicular to)
the current in a
stream

traveling a
distance d
downstream and
returning a
distance d
upstream

traveling a
distance d
upstream and
returning a
distance d
downstream

atleast 250m

atleast 83m

atleast 200m

atleast 67m

T1=T2

T1> T2

T1< T2

Tl+ T2= mg

1 Dynamics

Suppose two cars are racing on a circular track one kilometer in


circumference. The first car can circle the
track in 15 seconds at top speed while the second car can circle the track in
12 seconds at top speed. How much
lead does the first car need starting the last lap of the race not to lose?
A ball of mass m is suspended from two strings of unequal length as shown
above. The magnitudes of the
tensions T1 and T2 in the strings must satisfy which of the following
relations?

2 Dynamics
3 Dynamics

Which of the following diagrams best represents the gravitational force W. the
frictional force f, and the normal
force N that act on the block?
E
The magnitude of the frictional force along the plane is most nearly
C

A
2.5 N

B
5N

C
6N

D
10 N

4 Dynamics

When the frictionless system shown above is accelerated by an applied force


of magnitude the tension in the
string between the blocks is
E

2F

2/3 F

1/2 F

153 Kinematics

Horizontally, the bales will all travel at


the speed of the plane, as gravity will
not affect their
horizontal motion. D = vt = (50 m/s)(2
5m
seconds apart)
Traveling in still water will take a time t
= d/v = 2d/v. Traveling perpendicularly
across the
stream requires the boat to head at an
angle into the current, causing the
relative velocity of the
boat to the shore to be less than when
in still water and therefore take a longer
time. Since this
eliminates choice E and choices D and
all of the above will C are identical, that leaves A as the
take equal times
only single option.
When the first car starts the last lap, it
will finish the race in 15 seconds from
that point. In 15
seconds, the second car will travel (1
km/12 s) 15 s = 1250 m so the first
car must be at least
250 m ahead when starting the last lap
atleast 104m
to win the race.
As T2 is more vertical, it is supporting
more of the weight of the ball. The
horizontal
T1 T2= mg
components of T1 and T2 are equal
Normal force is perpendicular to the
incline, friction acts up, parallel to the
incline (opposite the
motion of the block), gravity acts
E
straight down.
16 N
F = ma; mgsin f = ma
The diluted force between objects is
the applied force times the ratio of the
mass behind the
rope to the total mass being pulled.
This can be derived from a = F/m(total)
and FT = m(behind the
1/3 F
rope)a

1.dynamics.png

2.dynamics.png
3.dynamics.png

4.dynamics.png

7 Dynamics

A ball falls straight down through the air under the influence of gravity. There
is a retarding force F on the ball
with magnitude given by F = bv, where v is the speed of the ball and b is a
positive constant. The magnitude of
B
the acceleration, a of the ball at any time is equal to which of the following?
A push broom of mass m is pushed across a rough horizontal floor by a force
of magnitude T directed at angle
as shown above. The coefficient of friction between the broom and the floor is
. The frictional force on the
broom has magnitude
A
A block of weight W is pulled along a horizontal surface at constant speed v
by a force F, which acts at an angle
of with the horizontal, as shown above. The normal force exerted on the
block by the surface has magnitude
B

8 Dynamics

A uniform rope of weight 50 newtons hangs from a hook as shown above. A


box of weight 100 newtons hangs
from the rope. What is the tension in the rope?

5 Dynamics

6 Dynamics

9 Dynamics

10 Dynamics

11 Dynamics

12 Dynamics

13 Dynamics

14 Dynamics

15 Dynamics

16 Dynamics

17 Dynamics

18 Dynamics

19 Dynamics

20 Dynamics

When an object of weight W is suspended from the center of a massless


string as shown above, the tension at
any point in the string is
An ideal spring obeys Hooke's law, F = kx. A mass of 0.50 kilogram hung
vertically from this spring
stretches the spring 0.075 meter. The value of the force constant for the
spring is most nearly
A block of mass 3m can move without friction on a horizontal table. This block
is attached to another block of
mass m by a cord that passes over a frictionless pulley, as shown above. If
the masses of the cord and the pulley
are negligible, what is the magnitude of the acceleration of the descending
block?
A plane 5 meters in length is inclined at an angle of 37, as shown above. A
block of weight 20 newtons is
placed at the top of the plane and allowed to slide down. The mass of the
block is most nearly:
A plane 5 meters in length is inclined at an angle of 37, as shown above. A
block of weight 20 newtons is
placed at the top of the plane and allowed to slide down. The magnitude of
the normal force exerted on the block by the plane is most nearly:
The forces act on an object. If the object is in translational equilibrium, which
of the following must be true? I. The vector sum of the three forces must
equal zero. II. The magnitudes of the three forces must be equal. III. All three
forces must be parallel.
Three objects can only move along a straight, level path. The graphs above
show the position d of each of the
objects plotted as a function of time t. The sum of the forces on the object is
zero in which of the cases?

g-b

g bv/m

g + bv/m

g/b

bv/m

F = ma; mg bv = ma

(mg + Tsin)

(mg Tsin)

(mg + Tcos)

(mg Tcos)

mg

Ff= F(N) where F(N) is found from


Fy= 0 = (F(N) mg Tsin)

W F cos

W F sin

W + F sin

50 N throughout
the rope

75 N throughout
the rope

100 N throughout
the rope

150 N throughout
the rope

W + F cos
It varies from 100
N at the bottom of
the rope to 150 N
at the top.

2Wcos

Wcos

Wcos

W/(2cos)

W/(cos)

0.33 N/m

0.66 N/m

6.6 N/m

33 N/m

66 N/m

Fy= 0 = Fsin + F(N) W


7.dynamics.png
The bottom of the rope supports the
box, while the top of the rope must
support the rope itself
and the box.
8.dynamics.png
The vertical components of the tension
in the rope are two equal upward
components of Tcos,
which support the weight. Fy= 0 =
2Tcos W
F = mg = kx (the negative sign merely
indicates the direction of the spring
force relative to the
displacement)

6.dynamics.png

Zero

g/4

g/3

2g/3

F(external) = m(total)a;
mg is the only force acting from outside
the system of masses so we have mg =
(4m)a
11.dynamics.png

1.0 kg

1.2 kg

1.6 kg

2.0 kg

2.5 kg

W = mg

10 N

12 N

16 N

20 N

33 N

I only

II only

I and III only

II and III only

I, II, and III

II only

III only

I and II only

I and III only

I, II, and III

None of these
motions
guarantees zero
accleration.

For which of the following motions of an object must the acceleration always
be zero? I. Any motion in a straight line. II. Simple harmonic motion. III. Any
motion in a circle.
E
A rope of negligible mass supports a block that weighs 30 N, as shown
above. The breaking strength of the
rope is 50 N. The largest acceleration that can be given to the block by pulling
up on it with the rope without
breaking the rope is most nearly:
B
A horizontal, uniform board of weight 125 N and length 4 m is supported by
vertical chains at each end. A
person weighing 500 N is sitting on the board. The tension in the right chain is
250 N. What is the tension in the left chain?
B

A horizontal, uniform board of weight 125 N and length 4 m is supported by


vertical chains at each end. A
person weighing 500 N is sitting on the board. The tension in the right chain is
250 N. How far from the left end of the board is the person sitting?
B
The cart of mass 10 kg shown above moves without frictional loss on a level
table. A 10 N force pulls on the
cart horizontally to the right. At the same time, a 30 N force at an angle of 60
above the horizontal pulls on the cart to the left. What is the magnitude of the
horizontal acceleration of the cart?
A

F(N) = mgcos, cos =


adjacent/hypotenuse = 4/5
Three vectors add to zero if they form
the sides of a triangle, there is no
requirement they be
equal or parallel, though it is possible.

12.dynamics.png

13.dynamics.png

Any curvature of the line in a d-t graph


indicates a non-zero acceleration
15.dynamics.png
Motion in a straight line does not mean
the speed is constant. Simple harmonic
motion is a constantly changing velocity
and can only occur with an
acceleration. Motion in a circle requires
centripetal acceleration.

I only

II only

III only

Either I or III, but


not II

6 m/s^2

6.7 m/s^2

10 m/s^2

15 m/s^2

16.7 m/s^2

250 N

375 N

500 N

625 N

875 N

0.4 m

1.5 m

2m

2.5 m

3m

F = ma; FT mg = ma; Let FT = 50 N


(the maximum possible tension) amd m
= W/g = 3 kg
17.dynamics.png
The sum of the tensions in the chains
(250 N + Tleft) must support the weight
of the board and B
the person (125 N + 500 N)
From symmetry, each chain supports
half of the weight of the board (62.5 N),
The weight of the
person is then split between the chains
with the left chain holding 375 N 62.5
N = 312.5 N
and the right chain supporting 250 N
62.5 N = 187.5 N or 3/5 of the tension
in the left chain.
This means if the person sits a distance
x from the left end, they sit a distance
(5/3)x from the
right end. This gives x + (5/3)x = 4 m

0.5 m/s^2

1.6 m/s^2

2.0 m/s^2

2.5 m/s^2

2.6 m/s^2

F = ma; 10 N (30 N cos60) = (10


kg)a

20.dynamics.png

21 Dynamics

22 Dynamics

23 Dynamics
24 Dynamics

25 Dynamics

26 Dynamics

27 Dynamics

28 Dynamics

29 Dynamics

30 Dynamics
31 Dynamics

An object of mass m is initially at rest and free to move without friction in any
direction in the xy-plane. A
constant net force of magnitude F directed in the +x direction acts on the
object for 1 s. Immediately thereafter
a constant net force of the same magnitude F directed in the +y direction acts
on the object for 1 s. After this, no
forces act on the object. Which of the following vectors could represent the
velocity of the object at the end of
3 s, assuming the scales on the x and y axes are equal?
C

Two people are pulling on the ends of a rope. Each person pulls with a force
of 100 N. The tension in the rope
is:
C
The parabola above is a graph of speed v as a function of time t for an object.
Which of the following graphs
best represents the magnitude F of the net force exerted on the object as a
function of time t?
E
A 100-newton weight is suspended by two cords as shown above. The
tension in the slanted cord is
D

Two blocks are pushed along a horizontal frictionless surface by a force of 20


newtons to the right, as shown
above. The force that the 2-kilogram block exerts on the 3-kilogram block is
A ball initially moves horizontally with velocity vi
, as shown above. It is then struck by a stick. After leaving
the stick, the ball moves vertically with a velocity vf
, which is smaller in magnitude than vi
. Which of the
following vectors best represents the direction of the average force that the
stick exerts on the ball?
Two 0.60-kilogram objects are connected by a thread that passes over a light,
frictionless pulley, as shown
above. The objects are initially held at rest. If a third object with a mass of
0.30 kilogram is added on top of one
of the 0.60-kilogram objects as shown and the objects are released, the
magnitude of the acceleration of the
0.30-kilogram object is most nearly
Two identical massless springs are hung from a horizontal support. A block of
mass 1.2 kilograms is suspended
from the pair of springs, as shown above. When the block is in equilibrium,
each spring is stretched an
additional 0.15 meter. The force constant of each spring is most nearly
A ball is thrown and follows a parabolic path, as shown above. Air friction is
negligible. Point Q is the highest
point on the path. Which of the following best indicates the direction of the net
force on the ball at point P ?
A block of mass 5 kilograms lies on an inclined plane, as shown above. The
horizontal and vertical supports
for the plane have lengths of 4 meters and 3 meters, respectively. The
coefficient of friction between the plane
and the block is 0.3. The magnitude of the force F necessary to pull the block
up the plane with constant speed
is most nearly
What is the acceleration of the block?

0N

50 N

100 N

141 N

200 N

Since the same force acts for the same


time in each direction, the velocity in
each direction is
the same. The vector should then point
at a 45 angle in the first quadrant.
21.dynamics.png
Consider that no part of the system is in
motion, this means at each end of the
rope, a person
pulling with 100 N offorce is reacted to
with a tension in the rope of 100 N.

As v is proportional to t2
and a is proportional to v/t, this means
a should be proportional to t
23.dynamics.png

50 N

100 N

150 N

200 N

250 N

8 newtons to the
left

8 newtons to the
right

10 newtons to the
left

12 newtons to the
right

20 newtons to the
left

Fy= 0 = Tsin 30 mg
F = ma gives 20 N = (5 kg)a or an
acceleration of 4 m/s2. The 2 kg block
is accelerating due to the contact force
from the 3 kg block Fcontact = ma = (2
kg)(4 m/s2) = 8 N. The 2 kg pushes
back on the 3 kg block with a force
equal in magnitude and opposite in
direction.

26.dynamics.png

27.dynamics.png

24.dynamics.png

25.dynamics.png

The direction of the force is the same


as the direction of the acceleration,
which is proportional to v = vf+ (vi)

10.0 m/s2

6.0 m/s2

3.0 m/s2

2.0 m/s2

1.0 m/s2

Fexternal = mtotala gives (0.90 kg


10m/s2) (0.60 kg 10 m/s2) = (1.5
kg)a

40 N/m

48 N/m

60 N/m

80 N/m

96 N/m

Each spring supports half of the weight,


or 6 N. F = kx
28.dynamics.png

gravity acts downward

29.dynamics.png

At constant speed F = 0; The forces


acting parallel to the incline are F (up),
Ff
(down) and mgsin (down), which gives
F Ff mgsin = 0, where Ff= FN =
mgcos and cos = 4/5
30.dynamics.png
F = ma = Fcos f
31.dynamics.png
f = F
N where FN = mg Fsin
32.dynamics.png

B
D

30 N
F/m

42 N
(Fcos)/m

49 N
(Ff)/m

50 N
(Fcosf)/m

58 N
(Fsinmg)/m

f/mg

mg/f

(mgFcos)/f

f/(mgFcos)

f/(mgFsin)

The string pulling all three masses (total


6m) must have the largest tension.
String A is only
pulling the block of mass 3m and string
B is pulling a total mass of 5m.
33.dynamics.png

34 Dynamics

What is the coefficient of friction between the block and the surface?
E
Three blocks of masses 3m, 2m, ands are connected to strings A, B, and Cas
shown above. The blocks are
pulled along a rough surface by a force of magnitude F exerted by string C.
The coefficient of friction between
each block and the surface is the same. Which string must be the strongest in
order not to break?
C
A block of mass 3 kg, initially at rest, is pulled along a frictionless, horizontal
surface with a force shown as a
function of time t by the graph above. The acceleration of the block at t = 2 s
is
B

35 Dynamics

An object weighing 300 N is suspended by means of two cords, as shown


above. The tension in the horizontal
cord is...

At t = 2 s the force is 4 N. F = ma
34.dynamics.png
The upward component of the slanted
cord is 300 N to balance the weight of
the object. Since
the slanted cord is at an angle of 45, it
has an equal horizontal component.
The horizontal
component of the slanted cord is equal
to the tension in the horizontal cord

32 Dynamics

33 Dynamics

It is impossible to
determine without
knowing the
They must all be
coefficient of
the same strength. friction.

3/4 m/s^2

4/3 m/s^2

2 m/s^2

8 m/s^2

12 m/s^2

0N

150 N

210 N

300 N

400 N

36 Dynamics

37 Dynamics

38 Dynamics

39 Dynamics

A small box is on a ramp tilted at an angle above the horizontal. The box
may be subject to the following
forces: frictional (f ) ,gravitational (mg), pulling or pushing (FP) and normal (I).
In the following free-body
diagrams for the box, the lengths of the vectors are proportional to the
magnitudes of the forces. Which figure best represents the free-body diagram
for the box if it is accelerating up the ramp?
A small box is on a ramp tilted at an angle above the horizontal. The box
may be subject to the following
forces: frictional (f ) ,gravitational (mg), pulling or pushing (FP) and normal (I).
In the following free-body
diagrams for the box, the lengths of the vectors are proportional to the
magnitudes of the forces. Which figure best represents the free-body diagram
for the box if it is at rest on the ramp?
A small box is on a ramp tilted at an angle above the horizontal. The box
may be subject to the following
forces: frictional (f ) ,gravitational (mg), pulling or pushing (FP) and normal (I).
In the following free-body
diagrams for the box, the lengths of the vectors are proportional to the
magnitudes of the forces.
Two blocks of masses M and m, with M > m, are connected by a light string.
The string passes over a
frictionless pulley of negligible mass so that the blocks hang vertically. The
blocks are then released from rest.
What is the acceleration of the block of mass M ?

Figure B

Figure C

Figure D

Figure A

Figure B

Figure C

Figure D

Figure E

Figure A

Figure B

Figure C

Figure D

Figure E

(M-m/M)g

(M+m/M)g

(M+m/M-m)g

(M-m/M+m)g

mg

mg

mg/

mg(1 - )

mg(1 + )

41 Dynamics

42 Dynamics

As shown in the accompanying figure, a force F is exerted at an angle of .


The block of weight mg is initially moving the right with speed v. The
coefficient of friction between the rough floor and the block is . The frictional
force acting on the block is:
E

43 Dynamics

The reaction force does not cancel the action force because:

44 Dynamics

A student pulls a wooden box along a rough horizontal floor at constant


speed by means of a force P as shown to the right. Which of the following
must be true?

2.0 kg

1.5 kg

1.0 kg

0.5 kg

1000 N

2000 N

3000 N

20000 N

30000 N

1.6 kN down

1.6 kN up

6.4 kN up

8.0 kN down

9.6 kN down

47 Dynamics

48 Dynamics

The 10.0 kg box shown in the figure to the right is sliding to the right along the
floor. A horizontal force of 10.0 N is being applied to the right. The coefficient
of kinetic friction between the box and the floor is 0.20. The box is moving
with:
A

acceleration to the centripetal


left.
acceleration.

constant speed
acceleration to the and constant
right.
velocity.

49 Dynamics

Two blocks X and Y are in contact on a horizontal frictionless surface. A 36 N


constant force is applied to X as shown to the right. The force exerted by X on
Y is:
D

1.5 N

6.0 N

29 N

30 N

50 Dynamics

Assume the objects in the following diagrams have equal mass and the
strings holding them in place are identical. In which case would the string be
most likely to break?

46 Dynamics

The normal force must point


perpendicular to the surface and the
weight must point down. If
the box is at rest on the ramp, friction
acts up the ramp, opposing the
tendency to slide down
The normal force must point
perpendicular to the surface and the
weight must point down. If
the box is sliding down at constant
speed, friction acts up the ramp,
opposing the motion

36.dynamics.png

36.dynamics.png

Fexternal = m totala gives (Mg) (mg)


= (M + m)a
. To keep the box from slipping, friction
up the wall must balance the weight of
the block, or Ff
= mg, where Ff
= F
N and FN
= the applied force F. This gives F =
mg
40.dynamics.png

Fexternal = mtotala gives (mg) (10


N) = (m + 1 kg)(5 m/s^2)
41.dynamics.png
Friction opposes the motion of the block
and therefore points to the left. The
normal force is found from Fy = 0 =
mg F sin to
(mg F cos ) to (mg + F sin ) to FN mg Fsin and the force of
mg to the left.
mg to the right.
the left.
the right.
the left.
friction Ff = FN
42.dynamics.png
The reaction exists When an object exerts a force on a
The action force is The action force is
only after the
second object, the second object exerts
greater than the
less than the
They act on
They are in the
action force is
an equal and opposite force back on
reaction force.
reaction force.
different bodies.
same direction.
removed.
the first object.
Since P is at an upward angle, the
normal force is decreased as P
supports some of the weight. Since a
component of P balances the frictional
P > f and N < W.
P > f and N = W.
P = f and N > W.
P = f and N = W.
P < f and N = W.
force, P itself must be larger than f.
44.dynamics.png
Newtons 2nd law applied to an object
sliding to rest gives F = Ff = FN =
ma. On a horizontal surface, FN = mg
and we have mg = ma, or a = g.
Use this acceleration with vf^2 = vi^2 +
0.8
0.4
0.2
0.1
0.05 2ad.
3.0 kg

A block with initial velocity 4.0 m/s slides 8.0 m across a rough horizontal floor
before coming to rest. The coefficient of friction is:
D
A car whose mass is 1500 kg is accelerated uniformly from rest to a speed of
20 m/s in 10 s. The magnitude of the net force accelerating the car is:
C
An 800-kg elevator accelerates downward at 2.0 m/s^2. The force exerted by
the cable on the elevator is:
C

45 Dynamics

Figure E

A horizontal force F pushes a block of mass m against a vertical wall. The


coefficient of friction between the
block and the wall is . What value of F is necessary to keep the block from
slipping down the wall?
C
One end of a massless rope is attached to a mass m; the other end is
attached to a mass of 1.00 kg. The rope is
hung over a massless frictionless pulley asshown in the accompanying figure.
Mass m accelerates downward at
5.0 m/s2. What is m?
A

40 Dynamics

Figure A

The normal force must point


perpendicular to the surface and the
weight must point down. In
order to accelerate up the ramp, there
must be an applied force up the ramp. If
the box is
accelerating up the ramp, friction acts
down the ramp, opposite the motion
36.dynamics.png

F = ma = mv/t
F = ma; Fcable mg = ma = m(2
m/s^2)
The force of friction = FN = 0.2 10
kg 9.8 m/s^2 = 19.6 N, which is
constant speed but greater than the applied force, which
not constant
means the object is accelerating to the
velocity.
left, or slowing down
48.dynamics.png
F = ma gives 36 N = (24 kg)a or an
acceleration of 1.5 m/s^2. The 20 kg
block is accelerating due to the contact
force from the 4 kg block Fcontact = ma
36 N
= (20 kg)(1.5 m/s^2) = 30 N.
49.dynamics.png
The upward component of the tension
is Tup = Tsin, where is the angle to
the horizontal. This gives T = Tup/sin.
Since the upward components are all
equal to one half the weight, the rope at
the smallest angle (and the smallest
value of sin) will have the greatest
E
tension, and most likely break
50.dynamics.png

52 Dynamics

A string with masses of 1.5 kg and 3.0 kg on its ends is hung over a
frictionless, massless pulley as shown to the
right. What is the approximate magnitude of the acceleration of the masses?
Two blocks of mass 1.0 kg and 3.0 kg are connected by a string which has a
tension of 2.0 N. A force F acts in
the direction shown to the right. Assuming friction is negligible, what is the
value of F?

1.0 N

2.0 N

4.0 N

6.0 N

8.0 N

53 Dynamics

An object in equilibrium has three forces, F1 of 30 N, F2 of 50 N, and F3 of 70


N, acting on it. The magnitude of the resultant of F1 and F2 is
D

10 N

20 N

40 N

70 N

80 N

350 N

450 N

500 N

550 N

650 N

From the 1 kg block: F = ma giving a =


2 m/s2. For the system: F = (4 kg)(2
m/s2)
52.dynamics
For three forces in equilibrium, any one
of the forces is equal and opposite to
the resultant of the other two forces.
Elevator physics: FN represents the
scale reading. F = ma; FN mg = ma,
or FN = m(g + a). The velocity of the
elevator is irrelevant.

1/4 as great.
4 times greater
than the force of
the tractor on the
trailer.

1/2 as great.
2 times greater
than the force of
the tractor on the
trailer.

2 times greater.

4 times greater.

unchanged

F = ma, if F is doubled, a is doubled. If


m is halved, a will be doubled

equal to the force


of the tractor on
the trailer.

the force of the


tractor on the
trailer

zero since the


tractor is pulling
the trailer forward. Newtons third law

51 Dynamics

54 Dynamics

55 Dynamics

56 Dynamics

57 Dynamics

58 Dynamics

A 50-kg student stands on a scale in an elevator. At the instant the elevator


has a downward acceleration of 1.0
m/s2 and an upward velocity of 3.0 m/s, the scale reads approximately
B
If the net force on an object were doubled while at the same time the mass of
the object was halved, then the
acceleration of the object is
D
A tractor-trailer truck is traveling down the road. The mass of the trailer is 4
times the mass of the tractor. If the
tractor accelerates forward, the force that the trailer applies on the tractor is
C
Two boxes are accelerated to the right on a frictionless horizontal surface as
shown. The larger box has a mass
of 9 kilograms and the smaller box has a mass of 3 kilograms. If a 24 newton
horizontal force pulls on the larger
box, with what force does the larger box pull on the smaller box?
B

62 Dynamics

63 Dynamics

64 Dynamics
65 Dynamics

Shown below is the velocity vs. time graph for a toy car moving along a
straight line. What is the maximum
displacement from start for the toy car?
A cannon fires projectiles on a flat range at a fixed speed but with variable
angle. The maximum range of the
cannon is L. What is the range of the cannon when it fires at an angle /6
above the horizontal? Ignore air
resistance.
A ball is launched upward from the ground at an initial vertical speed of v0
and begins bouncing vertically.
Every time it rebounds, it loses a proportion of the magnitude of its velocity
due to the inelastic nature of the
collision, such that if the speed just before hitting the ground on a bounce is v,
then the speed just after the
bounce is rv, where r < 1 is a constant. Calculate the total length of time that
the ball remains bouncing,
assuming that any time associated with the actual contact of the ball with the
ground is negligible.
The graph shows velocity as a function of time for a car. What was the
acceleration at time t = 90 seconds?

51.dynamics.png

56.dynamics

F = ma gives 24 N = (12 kg)a or an


acceleration of 2 m/s2. The 3 kg block
is accelerating due to the tension in the
rope FT = ma = (3 kg)(2 m/s2) = 6 N.
57.dynamics

250 N

400 N

2500 N

0.5 d

1.41 d

2d

4d

3m

5m

6.5 m

7m

7.5 m

F = ma, where m = W/g = 10 kg


Newtons 2nd law applied to an object
sliding to rest gives F = Ff = FN =
ma. On a
horizontal surface, FN = mg and we
have mg = ma, or a = g. Using this
acceleration with
vf2 = vi2 + 2ad gives d = vi2/2g. There
is no dependence on mass.
Displacement is the area under the
curve. Maximum displacement is just
before the car turns
around at 2.5 seconds.
62.dynamics.png

1/3 L

Range of a projectile R = (vi^2 sin 2)/g


and maximum range occurs at = 45,
which gives viRg. Using = 30 gives
Rnew= R sin 60

3/2 L

1/2 L

1/3 L

1/2 L

24 N

Fexternal = mtotala gives (3.0 kg


10 m/s^2) (1.5 kg 10 m/s^2) = (4.5
kg)a

40 N

the objects inertia


is unchanged
Inertia is mass
The normal force is mgcos. For a
horizontal surface, FN = mg. At any
more information is angle FN < mg and Ff is proportional to
required
FN

(2vo/g)*(1/1r)

(vo/g)*(r/1r)

(2vo/g)*(1r/r)

(2vo/g)*(1/1r^2)

(2vo/g)*(1/1+(1-r)
^2)

0.22 m/s^2

0.33 m/s^2

1.0 m/s^2

9.8 m/s^2

30 m/s^2

2h

3h

4h

h^2

45 m

60 m

90 m

105 m

120 m

2.3 s

3.1 s

5.4 s

5.9 s

8.2 s

67 Dynamics

An object is released from rest and falls a distance h during the first second of
time. How far will it fall during
the next second of time?
C
A stone is thrown straight downward with a speed of 20 m/s from the top of a
tall building. If the stone strikes
the ground 3.0 s later, about how tall is the building? Assume air resistance is
negligible.
D

68 Dynamics

A coyote can run at a speed of 20 m/s while a prairie dog can manage only
5.5 m/s. If a prairie dog is 45 m in
front of a coyote, what is the maximum time it has to reach its hole without
being caught?

66 Dynamics

10 m/s^2

25 N

61 Dynamics

8N
the objects inertia
becomes 4 times
greater

6.7 m/s^2

the inclined case


has less frictional
force

A car of mass m slides across a patch of ice at a speed v with its brakes
locked. It then hits dry pavement and skids to a stop in a distance d. The
coefficient of kinetic friction between the tires and the dry road is . If the car
had a mass of 2m, it would have skidded a distance of

6N
the objects inertia
becomes 2 times
greater

3.3 m/s^2

the frictional force the inclined case


is the same in both has a greater
cases
frictional force

60 Dynamics

3N
the objects inertia
becomes 2 times
greater

3.0 m/s^2

18 N
the objects inertia
becomes 8 times
greater
the frictional force
increases with
angle until the
angle is 90, then
drops to zero

What happens to the inertia of an object when its velocity is doubled?


E
A wooden box is first pulled across a horizontal steel plate as shown in the
diagram A. The box is then pulled
across the same steel plate while the plate is inclined as shown in diagram B.
How does the force required to
overcome friction in the inclined case (B) compare to the horizontal case (A)? C
An object near the surface of the earth with a weight of 100 newtons is
accelerated at 4 m/s2. What is the netforce on the object?
B

59 Dynamics

1.5 m/s^2

(advanced question!) The time for one


bounce is found from v = v + (g)t
which gives t = 2v/g.
We are summing the time for all
bounces, while the velocity (and hence
the time) converge in a
geometric series with the ratio
vn+1/vn = r<1 to (1/1 r)
The acceleration is the slope of the
curve at 90 seconds.
65.dynamics.png
From the equation d = at^2,
displacement is proportional to time
squared. Traveling from rest
for twice the time gives 4 times the
displacement (or 4 h). Since the object
already travelled h in
the first second, during the time interval
from 1 s to 2 s the object travelled the
remaining 3h

d = vit + gt^2
The relative speed between the coyote
and the prairie dog is 14.5 m/s. To
cover the 45 m
distance between them will take t = d/v
= (45 m)/(14.5 m/s)

69 Dynamics

70 Dynamics

71 Dynamics

72 Dynamics

73 Dynamics

74 Dynamics

75 Dynamics

76 Dynamics

77 Dynamics

78 Dynamics

79 Dynamics

80 Dynamics

A model rocket accelerates from rest upwards at 50 m/s2


for 2.0 s before its engine burns out. The rocket then
coasts upward. What is the maximum height that the rocket reaches? You
may assume air resistance is
negligible.
A hunter in a forest walks 800 m west. He then turns south and walks 400 m
before turning west again and
walking a final 300 m. At the end of the walk, what is the magnitude of the
hunter's displacement from the
beginning?
Robin Hood aims his longbow horizontally at a target's bull's eye 30 m away.
If the arrow strikes the target
exactly 1.0 m below the bull's eye, how fast did the arrow move as it was shot
from the bow? Assume air
resistance is negligible.
A 2 kg mass and a 4 kg mass on a horizontal frictionless surface are
connected by a massless string A. They are
pulled horizontally across the surface by a second string B with a constant
acceleration of 12 m/s2. What is the magnitude of the force of string B on the
2 kg mass?
A 2 kg mass and a 4 kg mass on a horizontal frictionless surface are
connected by a massless string A. They are
pulled horizontally across the surface by a second string B with a constant
acceleration of 12 m/s2. What is the magnitude of the force of string A on the
4 kg mass?
A 2 kg mass and a 4 kg mass on a horizontal frictionless surface are
connected by a massless string A. They are
pulled horizontally across the surface by a second string B with a constant
acceleration of 12 m/s2. What is the magnitude of the net force on the 2 kg
mass?
A mass is suspended from the roof of a lift (elevator) by means of a spring
balance. The lift (elevator) is moving
upwards and the readings of the spring balance are noted as follows:
Speeding up: RU
Constant speed: RC
Slowing down: RD
Which of the following is a correct relationship between the readings?
A small box of mass m is placed on top of a larger box of mass 2m as shown
in the diagram at right. When a
force F is applied to the large box, both boxes accelerate to the right with the
same acceleration. If the
coefficient of friction between all surfaces is , what would be the force
accelerating the smaller mass?
The S.I. unit of force is named the newton in honor of Sir Isaac Newton's
contributions to physics. Which of
the following combination of units is the equivalent of a newton?

100 m

510 m

610 m

1020 m

1220 m

For the first part of the trip (the thrust):


d1 = vit + at^2 = 0 m + (50 m/s^2)
(2 s)^2
For the second part, we first find the
velocity after the thrust v = at = 100 m/s
and at the
maximum height v
= 100 m
f = 0, so to find d2 we use vf^2 = vi^2 +
2ad2 which gives d2
= 510 m

640 m

890 m

1170 m

1390 m

1500 m

6.0 m/s

13 m/s

33 m/s

67 m/s

150 m/s

Total displacement west = 1100 m; total


displacement south = 400 m. Use the
Pythagorean
theorem.
For a horizontal projectile (viy = 0 m/s)
to fall 1 m takes (using 1 m = gt^2)
0.45 seconds. To
travel 30 m in this time requires a
speed of d/t = (30 m)/(0.45 s)

72 N

48 N

24 N

6N

3N

String B is pulling both masses so FB =


(6 kg)(12 m/s2)
72.dynamics.png

72 N

48 N

24 N

6N

3N

String A is only pulling the 4 kg mass so


FA = (4kg)(12 m/s2)
72.dynamics.png

72 N

48 N

24 N

6N

3N

RU > RC

RU = RD

RC = RD

RC < RD

RU < RD

Fnet = ma
Elevator physics: R represents the
scale reading. F = ma; R mg = ma,
or R = m(g + a). This
ranks the value of R from largest to
smallest as accelerating upward,
constant speed,
accelerating downward

F = ma for the whole system gives F


(3m)g = (3m)a and solving for a gives
a = (F 3mg)/3m. For the top block,
Fm
= ma = m[(F 3mg)/3m]
76.dynamics.png

Static friction acts


upward on the
block.

Kinetic friction acts


Kinetic friction acts Static friction acts
upward on the
No friction acts on downward on the downward on the
block
the block
block
block.

A 6.0 kg block initially at rest is pushed against a wall by a 100 N force as


shown. The
coefficient of kinetic friction is 0.30 while the coefficient of static friction is
0.50. What is
true of the friction acting on the block after a time of 1 second?
D
A homeowner pushes a lawn mower across a horizontal patch of grass with a
constant speed by applying a
force P. The arrows in the diagram correctly indicate the directions but not
necessarily the magnitudes of the
various forces on the lawn mower. Which of the following relations among the
various force magnitudes, W, f, N, P
is CORRECT?
A
A mass, M, is at rest on a frictionless surface, connected to an ideal
horizontal spring that is unstretched. A
person extends the spring 30 cm from equilibrium and holds it at this location
by applying a 10 N force. The
spring is brought back to equilibrium and the mass connected to it is now
doubled to 2M. If the spring is
extended back 30 cm from equilibrium, what is the necessary force applied by
the person to hold the mass
stationary there?
C

72.dynamics.png

m a = kg m/s^2
77.dynamics.png
The normal force comes from the
perpendicular component of the applied
force which is Fcos
= 50 N. The maximum value of static
friction is then F
N = 25 N. The upward component of
the applied force is Fsin = 87 N. Fy
= Fup mg = 87 N 60 N > 25 N. Since
the net force on
the block is great than static friction can
hold, the block will begin moving up the
wall. Since it
is in motion, kinetic friction is acting
opposite the direction of the blocks
motion
78.dynamics.png

P > f and N> W

P < f and N = W

P > f and N< W

P = f and N > W

Since P is at a downward angle, the


normal force is increased. Since a
component of P balances
the frictional force, P itself must be
none of the above larger than f.

20.0 N

14.1 N

10.0 N

7.07 N

5.00 N

Since the force is applied horizontally,


the mass has no effect.

79.dynamics.png

82 Dynamics

A book leans against a crate on a table. Neither is moving. Which of the


following statements concerning this situation is CORRECT?

83 Dynamics

A crate of toys remains at rest on a sleigh as the sleigh is pulled up a hill with
an increasing speed. The crate is not fastened down to the sleigh. What force
is responsible for the crates increase in speed up the hill?
B

smaller than the


force the bat
exerts on the ball.
The direction of
the frictional force
Although there is
acting on the book
no friction acting
is in the same
The force of the
on the crate, there
direction as the
book on the crate must be friction
The net force
frictional force
is less than that of acting on the book acting on the book acting
crate on the book. or else it will fall.
is zero.
on the crate.
the contact force
the contact force
(normal force) of
the force of static (normal force) of
the gravitational
the ground on the friction of the
the sleigh on the
force acting on the
sleigh
sleigh on the crate crate
sleigh

84 Dynamics

A student weighing 500 N stands on a bathroom scale in the schools


elevator. When the scale reads 520 N, the elevator must be

accelerating
upward.

accelerating
downward.

moving upward at
a constant speed.

85 Dynamics

In which one of the following situations is the net force constantly zero on the
object?
E

A mass attached
to a string and
swinging like a
pendulum.

A stone falling
freely in a
gravitational field.

An astronaut
floating in the
International
Space Station.

f<M<T

M<f<T

M<T<f

The only force in the direction of the


crates acceleration is the force of
no force is needed friction from the sleigh
Elevator physics: FN represents the
scale reading. F = ma; FN mg = ma,
moving downward
or FN = m(g + a). When FN > mg, the
at a constant
elevator is accelerating upward (a is
speed.
at rest.
positive)
Changing direction (choices A and C
(the astronaut is still orbiting the earth!))
A snowboarder
A skydiver who
cannot occur
riding down a
has reached
with a zero net force. Choices B and D
steep hill.
terminal velocity.
represent accelerated motion.
Given that the box accelerates toward
Ted, Teds force must be greater than
Marios force plus the force of friction.
Since Marios force is of Teds force,
It cannot be
the force of frction must be less than
f=M<T
determined.
half of Teds force.

ball on the hand

Earth on the ball

ball on the Earth

Earth on your hand hand on the ball

X-componentpositive 6 N Ycomponentnegative 8 N

X-componentpositive 8 N Ycomponentnegative 6 N

X-componentnegative 6 N Ycomponentpositive 8 N

X-componentnegative 8 N Ycomponentpositive 6 N

For a Newtons third law pair, just


switch the nouns
The component of gravity acting down
the incline (+x) is mgsin and the
component
X-component- 0 N perpendicularly intothe incline (y) is
Y-componentmgcos. 36.9 indicates a 3-4-5
positive 10 N
triangle.

4800 N

4000 N

3200 N

800 N

400 N

k = 0.013

k = 0.394

k= 0.509

k= 0.866

k= 1.055

F = ma; F mg = m(5g) or F = 6mg


Fy= Fsin + FN mg = 0, which gives
FN = 170 N. The force of friction is
equal to the horizontal component of
the force applied by the student which
is Fcos = 86.6N. Ff= FN

0N

21 N

30 N

51 N

76 N

constant speed means F(net) = 0 N

0.938 m/s2

As the initial and final velocities and the


displacement are given, as well as an
indication that the acceleration is
constant, this is merely a kinematics
problem. vf2 = vi2 + 2ad
92.dynamics.png

81 Dynamics

86 Dynamics

87 Dynamics

A baseball is thrown by a pitcher with a speed of 35 m/s. The batter swings


and hits the ball. The magnitude of
the force that the ball exerts on the bat is always

A box slides to the right across a horizontal floor. A person called Ted exerts
a force T to the right on the box. A person called Mario exerts a force M to the
left, which is half as large as the force T. Given that there is friction f and the
box accelerates to the right, rank the sizes of these three forces exerted on
the box
A
You hold a rubber ball in your hand. The Newton's third law companion force
to the force of gravity on the ball is the force exerted by which object onto
what other object?
C

93 Dynamics

An object on an inclined plane has a gravitational force of magnitude 10 N


acting on it from the Earth. Which of the following gives the correct
components of this gravitational force for the coordinate axes shown in the
figure? The y-axis is perpendicular to the inclines surface while the x-axis is
parallel to the inclined surface.
A spaceman of mass 80 kg is sitting in a spacecraft near the surface of the
Earth. The spacecraft is accelerating upward at five times the acceleration
due to gravity. What is the force of the spaceman on the spacecraft?
. A 22.0 kg suitcase is dragged in a straight line at a constant speed of 1.10
m/s across a level airport floor by a student on the way to Mexico. The
individual pulls with a 1.0 102 N force along a handle which makes an
upward angle of 30.0 degrees with respect to the horizontal. What is the
coefficient of kinetic friction between the suitcase and the floor?
A person pushes a block of mass M = 6.0kg with a constant speed of 5.0 m/s
straight up a flat surface inclined
30.0 above the horizontal. The coefficient of kinetic friction between the
block and the surface is = 0.40.
What is the net force acting on the block?
In the figure above, a box moves with speed 5.0 m/s at the bottom of a rough,
fixed inclined plane. The box
slides with constant acceleration to the top of the incline as it is being pushed
directly to the left with a constant
force of F = 240 N. The box, of mass m = 20.0 kg, has a speed of 2.50 m/s
when it reaches the top of the
incline. What is the magnitude of the acceleration of the box as it slides up
the incline?
A 20.0 kg box remains at rest on a horizontal surface while a person pushes
directly to the right on the box with
a force of 60 N. The coefficient of kinetic friction between the box and the
surface is k = 0.20. The coefficient
of static friction between the box and the surface is s= 0.60. What is the
magnitude of the force of friction
acting on the box during the push?

94 Dynamics

Two identical blocks of weight W are placed one on top of the other as shown
in the diagram above. The upper
block is tied to the wall. The lower block is pulled to the right with a force F.
The coefficient of static friction
between all surfaces in contact is . What is the largest force F that can be
exerted before the lower block starts
to slip?
E

88 Dynamics

89 Dynamics

90 Dynamics

91 Dynamics

92 Dynamics

zero as it is only
equal to the
the bat that exerts gravitational force
a force on the ball. acting on the ball.

12.0 m/s^2

10.0 m/s^2

larger than the


force the bat
exerts on the ball.

5.88 m/s^2

1.88 m/s^2

equal to the force


that the bat exerts
on the ball.
Newtons third law

The Newtons
Third Law reaction
force to the weight
of the book is the
normal force from
the table

200 N

120 N

60 N

40 N

0N

3W/2

2W

5W/2

3W

If they are not moving, the net force


must be zero. While the book and crate
are pushing each other apart, there is
friction from the table pointing inward
against each object on the table to keep
them at rest.
82.dynamics.png

88.dynamics.png

91.dynamics.png

The maximum value of static friction in


this case is sFN = 120 N. Since the
person is pushing with only 60 N of
force, the box remains at rest.
Between the lower block and the
tabletop, there is a force of friction to
the left of maximum
magnitude (2W) as both blocks are
pushing down on the tabletop. There is
also a force of
friction acting to the left on the upper
surface of the lower block due to the
upper block of
maximum magnitude W. The total
maximum static frictional force holding
the lower block in
place is therefore (2W) + W
94.dynamics.png

mg

mgcos

96 Dynamics

A mass m is resting at equilibrium suspended from a vertical spring of natural


length L and spring constant k
inside a box as shown. The box begins accelerating upward with acceleration
a. How much closer does the equilibrium position of the
mass move to the bottom of the box?
C

(aL)/g

(gL)/a

97 Dynamics

When the speed of a rear-drive car is increasing on a horizontal road, what is


the direction of the frictional force
on the tires?
A

backward on the
front tires and
forward on the rear
tires

forward on the
front tires and
backward on the
rear tires.

98 Dynamics

A ball of mass m is launched into the air. Ignore air resistance, but assume
that there is a wind that exerts a
constant force Fo in the x direction. In terms of Fo and the acceleration due
to gravity g, at what angle above
the positive x-axis must the ball be launched in order to come back to the
point from which it was launched?

tan-1(Fo/mg)

tan-1(mg/Fo)

m1g/(m1 + m2 +
m3)

g(m1 + m2)/(m1
+ m2 + m3)

5N

7N

The normal force on the block can be


found from Fy = 0 = FN mgcos F.
The force of
friction necessary to hold the block in
place is mgsin. Setting the force of
friction equal to
mgsin gives FN = mgsin = (F +
mgsin
mgsin/
mg(sin cos)/ mgcos)
In equilibrium, mg = kx and the
equilibrium position x = mg/k. In an
accelerating elevator, we
can just adjust gravity to its effective
value g(eff) = g + a, thus making the
new equilibrium
(m(g+a))/k
(m(g-a))/k
(ma)/k
position mg(eff)/k
This is a tricky one. In order to move
the car forward, the rear tires roll back
against the
ground, the force of friction pushing
forward on the rear tires. The front tires,
however, are not
trying to roll on their own, rather they
begin rolling due to the friction acting
backward on all
backward,
forward on all tires tires
zero
increasing their rate of rotation
Gravity is still the only force acting
vertically so we can find the total time in
the air from
kinematics: vy = 0 at the top = vosin
gt giving t (to the top) = v0sin/g and
the total time is
twice the time to the top, or 2vosin/g.
In this time, the ball is also accelerating
horizontally
(think of it as a sideways gravity) and
in this time, should return to its starting
location.
the angle depends
Using x = 0 = (vocos)t + at2, where
on the launch
no such angle is
a = Fo/m and t is found above, we can
sin-1(Fo/mg)
speed
possible
solve for
The external forces acting on the
system of masses are the weights of
block 1 (pulling the
system to the left), the weight of block 3
(pulling the system to the right) and the
force of
friction on block 2 (pulling the system to
the left with a magnitude FN = m2g)
g(m1 + m2 + m3)/ g(m1 m2 m3)/ g(m1 m2 m3)/ F(external) = m(total)a gives (m1g
(m1 m2 m3)
(m1 + m2 + m3)
(m1 + m2 + m3)
m2g m3g) = (m1 + m2 + m3)a
F = ma gives 30 N = (12 kg)a or an
acceleration of 2.5 m/s2. The 5 kg block
is accelerating due
to the tension in the rope FT = ma = (5
12.5 N
17.5 N
30 N
kg)(2.5 m/s2) = 12.5 N.

9.8 m/s^2

8.4 m/s^2

6.3 m/s^2

3.8 m/s^2

2.5 m/s^2

all three average


velocities are
equal

not enough
information is
provided

95 Dynamics

A force F is used to hold a block of mass m on an incline as shown in the


diagram (see above). The plane makes
an angle of with the horizontal and F is perpendicular to the plane. The
coefficient of friction between the
plane and the block is . What is the minimum force, F, necessary to keep
the block at rest?

99 Dynamics

100 Dynamics

101 Dynamics

102 Dynamics

103 Dynamics

104 Dynamics

105 Dynamics

Given the three masses as shown in the diagram above, if the coefficient of
kinetic friction between the large
mass (m2) and the table is , what would be the upward acceleration of the
small mass (m3)? The mass and
friction of the cords and pulleys are small enough to produce a negligible
effect on the system.
Two masses 5.0 and 7.0 kg are originally at rest on a frictionless surface. The
masses are connected by a light
cord. A second cord is attached to the 7.0 kg mass and pulled with a
horizontal force of 30 N. What is the
tension in the cord that connects the two masses?
Two masses are connected by a light cord which is looped over a light
frictionless pulley. If one mass is 3.0 kg
and the second mass is 5.0 kg, what is the downward acceleration of the
heavier mass? Assume air resistance is
negligible.
Three identical laboratory carts A, B, and C are each subject to a constant
force FA, FB, and FC, respectively.
One or more of these forces may be zero. The diagram below shows the
position of each cart at each second of
an 8.0 second interval.
Which car has the greatest
average velocity during the interval?
Three identical laboratory carts A, B, and C are each subject to a constant
force FA, FB, and FC, respectively.
One or more of these forces may be zero. The diagram below shows the
position of each cart at each second of
an 8.0 second interval.
How does the magnitude of the
force acting on each car compare?

FA > FB > FC

FA = FC > FB

FA > FC = FB

The ratio of the


forces is equal to
the ratio of the
velocities

The ratio of the


forces is equal to
the ratio of the
velocities

A skydiver is falling at terminal velocity before opening her parachute. After


opening her parachute, she falls at
a much smaller terminal velocity. How does the total upward force before she
opens her parachute compare to
the total upward force after she opens her parachute?
E
Each of the diagrams below represents two weights connected by a massless
string which passes over a
massless, frictionless pulley. In which diagram will the magnitude of the
acceleration be the largest?
A

FA = FB > FC
The upward force
the upward force
before the
with the parachute parachute will be
will depend on the greater because of
size of the
the greater
parachute
velocity.

not enough
information

95.dynamics.png

96.dynamics.png

99.dynamics.png

100.dynamics.png

F(external) = m(total)a gives (5.0 kg


10 m/s2) (3 kg 10 m/s2) = (8 kg)a
101.dynamics.png

As they are all at the same position


after 8 seconds, they all have the same
average velocity
104.dynamics.png
Car A decelerates with the same
magnitude that C accelerates. Car B is
moving at constant
speed, which means FB
B
= 0.
104.dynamics.png

When falling with terminal velocity, the


The upward force force of air resistance equals your
in both cases must weight, regardless of
be the same.
the speed.
For each case, Fexternal = mtotal*a
gives Mg mg = (M + m)a or a=
[(M-m)/(M+m)]g

105.dynamics.png

111 Dynamics

A simple Atwood's machine is shown in the diagram above. It is composed of


a frictionless lightweight pulley
with two cubes connected by a light string. If cube A has a mass of 4.0 kg and
cube B has a mass of 6.0 kg, the
system will move such that cube B accelerates downwards. What would be
the tension in the two parts of the
string between the pulley and the cubes?
If a net force F applied to an object of mass m will produce an acceleration of
a, what is the mass of a second
object which accelerates at 5a when acted upon by a net force of 2F?
A simple Atwood's machine remains motionless when equal masses M are
placed on each end of the chord.
When a small mass m is added to one side, the masses have an acceleration
a. What is M? You may neglect
friction and the mass of the cord and pulley.
Block 1 is stacked on top of block 2. Block 2 is connected by a light cord to
block 3, which is pulled along a
frictionless surface with a force F as shown in the diagram. Block 1 is
accelerated at the same rate as block 2
because of the frictional forces between the two blocks. If all three blocks
have the same mass m, what is the
minimum coefficient of static friction between block 1 and block 2?
An object originally traveling at a velocity, v0, is accelerated to a velocity, v, in
a time, t, by a constant force, F.
What would be the mass of the object?
A frictionless air puck of mass m is placed on a plane surface inclined at an
angle of 60 with respect to the
horizontal. A string of length l is attached to the puck at one end and the
upper edge of the inclined plane at the
other to constrain the movement of the puck. What would be the magnitude of
the normal force from the plane
acting on the puck?

mg(sin 60)

mg(cos 30)

mg(tan 30)

mg/(tan 60)

112 Dynamics

Three blocks (m1, m2, and m3) are sliding at a constant velocity across a
rough surface as shown in the
diagram above. The coefficient of kinetic friction between each block and the
surface is . What would be the
force of m1 on m2?

(m2 + m3)g

F (m2 m3)g

(m1+ m2+ m3)g

113 Dynamics

Two 5 kg masses are attached to opposite ends of a long massless cord


which passes tautly over a massless
frictionless pulley. The upper mass is initially held at rest on a table 50 cm
from the pulley. The coefficient of
kinetic friction between this mass and the table is 0.2. When the system is
released, its resulting acceleration is
closest to which of the following?

9.8 m/s^2
greater in
magnitude than
the frictional force
exerted on the
person by the
merry-go-round

7.8 m/s^2
opposite in
direction to the
frictional force
exerted on the
merry-go-round by
the person

4.9 m/s^2

3.9 m/s^2

One-quarter the
original speed

One-half the
original speed

The same as the


original speed

Twice the original


speed

Four times the


original speed

clockwise at 30
m/s

clockwise at 10
m/s

counterclockwise
at 30 m/s

counterclockwise
at 10 m/s

with constant
velocity

F = mv^2/r ; v=sqrt((Fr)/m) all other


variables being constant, if r is
quadrupled, v is doubled
With acceleration south the car is at the
top (north side) of the track as the
acceleration points toward the center of
the circular track. Moving east indicates
the car is travelling clockwise. The
magnitude of the acceleration is found
from a = v2/r
3.circular.png

The frictional force acts as the


centripetal force (toward the center)

I only

III only

I and II only

II and III only

I, II, and III.

Acceleration occurs when an object is


changing speed and/or direction

Velocity is tangential, acceleration


points toward the center of the circular
path

106 Dynamics

107 Dynamics

108 Dynamics

109 Dynamics

110 Dynamics

1 Circular

2 Circular

3 Circular

4 Circular

5 Circular

6 Circular

TA = 47 N ; TB =
71 N

TA = 47 N ; TB =
47 N

TA = 47 N ; TB =
42 N

TA = 39 N ; TB =
59 N

TA = 39 N ; TB =
39 N

The two ends of the light string must


have the same tension, eliminating
choices A, C and D. If
choice E was correct, both masses
would be accelerating downward and
TA must be greater than the weight of
block A.

(2/5)m

2m

(5/2)m

5m

10m

If F = ma, then m = F/a. For the second


object m = 2F/5a = 2/5(F/a) = (2/5)m
107.dynamics.png

Fexternal = mtotala gives (M + m)g


Mg = (2M + m)a

As the entire system moves as one, F =


(3m)a, or a = F/(3m). The force of
friction acting on
block 1 is the force moving block 1 and
we have mg = m(F/(3m))

2F/3mg

F/mg

3F/2mg

F/3mg

When a person stands on a rotating merry-go-round, the frictional force


exerted on the person by the
merry-go-round is
B
A ball attached to a string is whirled around in a horizontal circle having a
radius r. If the radius of the circle is
changed to 4r and the same centripetal force is applied by the string, the new
speed of the ball is which of the
following?
D

A racing car is moving around the circular track of radius 300 meters shown
above. At the instant when the car's
velocity is directed due east, its acceleration is directed due south and has a
magnitude of 3 meters per second
squared. When viewed from above, the car is moving
The horizontal turntable shown above rotates at a constant rate. As viewed
from above, a coin on the turntable
moves counterclockwise in a circle as shown. Which of the following vectors
best represents the direction of the
frictional force exerted on the coin by the turntable when the coin is in the
position shown?
In which of the following situations would an object be accelerated?
I. It moves in a straight line at constant speed.
II. It moves with uniform circular motion.
III. It travels as a projectile in a gravitational field with negligible air resistance.
An automobile moves at constant speed down one hill and up another hill
along the smoothly curved surface
shown above. Which of the following diagrams best represents the directions
ofthe velocity and the
acceleration of the automobile at the instant that it is at the lowest position. as
shown?

2F/mg

F = ma = mv/t

106.dynamics.png

108.dynamics.png

110.dynamics.png

This is really no different than any other


incline problem. The normal force on an
incline with
no other forces acting into the incline is
None of these
mgcos
11.dynamics.png
Since the system is moving at constant
velocity, m1 is pushing m2 and m3
with a force equal tothe force of friction
acting on those two blocks, which is
(FN2 + FN3
m1g (m2+ 3)g )
112.dynamics.png
Fexternal = mtotala gives (5 kg 9.8
m/s2
) Ff
= (10 kg)a, where Ff
is the force of friction acting
on the 5 kg block on the table: mg =
0.2 5 kg 9.8 m/s2
D
1.9 m/s^2
= 9.8 N
113.dynamics.png

directed away from


the center of the
zero if the rate of independent of the
merry-go-round
rotation is constant person's mass
Newtons third law

4.circular.png

6.circular.png

7 Circular

8
9
10
11
12
13
14
15
16
17
18

Circular
Circular
Circular
Circular
Circular
Circular
Circular
Circular
Circular
Circular
Circular

19 Circular

20 Circular

21 Circular

22 Circular

23 Circular

24 Circular

25 Circular

26 Circular

27 Circular

28
29
30
31
1
2
3
4
5
6
7

Circular
Circular
Circular
Circular
Torque
Torque
Torque
Torque
Torque
Torque
Torque

A car initially travels north and then turns to the left along a circular curve.
This causes a package on the seat of
the car to slide toward the right side of the car. Which of the following is true
of the net force on the package
while it is sliding?
A child has a toy tied to the end of a string and whirls the toy at constant
speed in a horizontal circular path of
radius R. The toy completes each revolution of its motion in a time period T.
What is the magnitude of the
acceleration of the toy?

The force is
directed away from
the center of the
The force is
circle
directed north.

There is not
enough force
directed north to
keep the package
from sliding.

There is not
enough force
tangential to the
car's path to keep
the package from
sliding.

There is not
enough force
directed toward the
center of the circle
to keep the
package from
sliding.

To move in a circle, a force directed


toward the center of the circle is
required. While the
package slides to the right in the car, it
is actually moving in its original straight
line path while
the car turns from under it.

Zero

(4 pi^2 R)/T^2

(pi R)/T^2

2*pi*g

a = v^2/r and v = 2(pi r)/T giving a = (6


pi^2 r)/T^2

The speed of the


ball is constant

The velocity of the The radius is


ball is consant
constant

The magnitude of
the ball's
acceleration is
constant

The acceleration of
the ball is directed
radially inwards
towards the center

While speed may be constant, the


changing direction means velocity
cannot be constant as
velocity is a vector

F/2

2F

4F

8F

Upward

Downward

Forward

Backward

F = mv2/r. Fnew = (2m)(2v)2/(2r) = 4


(mv2/r) = 4F
Assuming the track is circular at the
bottom, the acceleration points toward
the center of the
circular path

A child whirls a ball at the end of a rope, in a uniform circular motion. Which
of the following statements is
NOT true?
B
An astronaut in an orbiting space craft attaches a mass m to a string and
whirls it around in uniform circular
motion. The radius of the circle is r, the speed of the mass is v, and the
tension in the string is F. If the mass,
radius, and speed were all to double the tension required to maintain uniform
circular motion would be
D
Assume the roller coaster cart rolls frictionlessly along
the curved track from point A to point C under the
influence of gravity. What would be the direction of the
cart's acceleration at point B?
A

Which car has had the lowest average speed during the race so far?

Which car at the moment of the snapshot MUST have a net force acting on
it?
A centripetal force of 5.0 newtons is applied to a rubber stopper moving at a
constant speed in a horizontal
circle. If the same force is applied, but the radius is made smaller, what
happens to the speed, v, and the
frequency, f, of the stopper?
What is the centripetal acceleration of an object (mass = 50 g) on the end of
an 80-cm string rotating at a
constant rate of 4 times a second?
What net force is necessary to keep a 1.0 kg puck moving in a circle of radius
0.5 m on a horizontal frictionless
surface with a speed of 2.0 m/s?

Car A

Car B

Car C

Car A

Car B

Car C

No acceleration
Cannot be
All three cars have determined with
had the same
information
average speed
provided
Cannot be
All three cars have determined with
net forces acting
information
on them
provided

v increases and f
increases

v decreases and f
decreases

v increases and f
decreases

v decreases and f
increases

neither changes

Average speed = (total distance)/(total


time). Lowest average speed is the car
that covered the least distance
22.circular.png
As all the cars are changing direction,
there must be a net force to change the
direction of their
velocity vectors
22.circular.png
F = mv2/r; v2 = rF/m, if r decreases, v
will decrease with the same applied
force. Also, v = 2rf
so 42r2f = rF/m, or f = F/(42rm) and
as r decreases, f increases.

25 m/s^2

32 m/s^2

100 m/s^2

500 m/s^2

2500 m/s^2

f = 4 rev/sec. a = v2/r and v = 2rf

ON

2.0 N

4.0 N

8.0 N

16 N

Astronauts on the Moon perform an experiment with a simple pendulum that


is released from the horizontal
position at rest. At the moment shown in the diagram with 0 < < 90, the
total acceleration of the mass may
be directed in which of the following ways?
A
A 4.0 kg mass is attached to one end of a rope 2 m long. If the mass is swung
in a vertical circle from the free
end of the rope, what is the tension in the rope when the mass is at its
highest point if it is moving with a speed
of 5 m/s?
B

Straight to the right Straight to the left

Straight upward

F = mv2/r
There is a force acting downward
(gravity) and a centripetal force acting
toward the center of
Straight along the the circle (up and to the right). Adding
connecting string these vectors cannot produce
toward point P (the resultants in the directions
Straight downward pivot)
of B, C, D or E.

5.4 N

21.6 N

50 N

10.8 N

65.4 N

F = ma; mg + FT = mv2/r giving FT =


mv2/r mg

21.circular.png

27.circular.png

8 Torque

9 Torque

10 Torque

11 Torque

12 Torque

To weigh a fish, a person hangs a tackle box of mass 3.5 kilograms and a
cooler of mass 5 kilograms from the ends of a uniform rigid pole that is
suspended by a rope attached to its center. The system balances when the
fish hangs at a point 1/4 of the rods length from the tackle box. What is the
mass of the fish?

Two objects, of masses 6 and 8 kilograms, are hung from the ends of a stick
that is 70 cm long and has marks every 10 cm, as shown. If the mass of the
stick is negligible, at which of the points indicated should a cord be attached if
the stick is to remain horizontal when suspended from the cord?
D
A wheel of radius R and negligible mass is mounted on a horizontal
frictionless axle so that the wheel is in a vertical plane. Three small objects
having masses m, M, and 2M, respectively, are mounted on the rim of the
wheel, as shown. If the system is in static equilibrium, what is the value of m
in terms of M?
C
A rod on a horizontal tabletop is pivoted at one end and is free to rotate
without friction about a vertical axis, as shown. A force F is applied at the
other end, at an angle to the rod. If F were to be applied perpendicular to
the rod, at what distance from the axis should it be applied in order to
produce the same torque?
A horizontal, uniform board of weight 125 N and length 4 m is supported by
vertical chains at each end. A person weighing 500 N is sitting on the board.
The tension in the right chain is 250 N.What is the tension in the left chain?

15 Torque

A horizontal, uniform board of weight 125 N and length 4 m is supported by


vertical chains at each end. A person weighing 500 N is sitting on the board.
The tension in the right chain is 250 N.How far from the left end of the board
is the person sitting?
Torque is the rotational analogue of
A square piece of plywood on a horizontal tabletop is subjected to the two
horizontal forces shown. Where should a third force of magnitude 5 newtons
be applied to put the piece of plywood into equilibrium?

16 Torque

A uniform rigid bar of weight W is supported in a horizontal orientation as


shown by a rope that makes a 30 angle with the horizontal. The force
exerted on the bar at point O, where it is pivoted, is best represented by a
vector whose direction is which of the following?

13 Torque
14 Torque

17 Torque

18 Torque

19 Torque

20 Torque

In which of the following diagrams is the torque about point O equal in


magnitude to the torque about point X in the diagram? (All forces lie in the
plane of the paper.)
A rod of length L and of negligible mass is pivoted at a point that is off-center
with lengths shown in the figure
below. The figures show two cases in which masses are suspended from the
ends of the rod. In each case the
unknown mass m is balanced by a known mass, M1 or M2, so that the rod
remains horizontal. What is the value
of m in terms of the known masses?
A system of two wheels fixed to each other is free to rotate about a
frictionless axis through the common center of the wheels and
perpendicular to the page. Four forces are exerted tangentially to the
rims of the wheels, as shown. The magnitude of the net torque on
the system about the axis is
For the wheel-and-axle system shown, which of the following
expresses the condition required for the system to be in static
equilibrium?

1.5 kg

2 kg

3 kg

6 kg

6.5 kg

M/2

3M/2

2M

5M/2

Lsin

Lcos

Ltan

2^(1/2) L

250 N

375 N

500 N

625 N

875 N

B
D

.4 m
kinetic energy

1.5 m
linear momentum

2m
acceleration

2.5 m
force

3m
mass

Ml + M2

(Ml + M2)

Ml M2

M1M2

M1M2

Apply rotational equilibrium using the


rope as the pivot point. (3.5)(9.8)(L/2)
+ m(9.8)(L/4) (5)(9.8)(L/2) = 0
therefore m=3 kg
To balance the torques on each side,
we obviously need to be closer to the
heavier mass. Trying point D as a pivot
point we have:
(m1g) r1 ?=? (m2g) r2
(6kg) (40 cm) ?=? (8kg) (30 cm) and we
see it works
Applying rotational equilibrium at the
center pivot we get: +mg(R) + Mg
(Rcos60) 2Mg(R) = 0.
Using cos60 = 12 we arrive at the
answer 3M/2
Finding the torque in the current
configuration we have: (Fsin)(L) = FL
sin .
To get the same torque with F applied
perpendicular we would have to change
the L to get F (Lsin)
Simple Fnet(y) = 0
T 500 + 250 125 = 0
Apply rotational equilibrium using left
end as pivot:
Same Diagram
+ (250)(4) (125)(2) (500)(r) = 0
therefore r =1.5m
Definition of torque
To balance the forces (Fnet=0) the
answer must be A or D, to prevent
rotation, obviously A would be needed
Since the rope is a tan angle it has x
and y B components of force.
Therefore, H would have to exist to
counteract Tx. Based on net = 0
requirement, V also would have to exist
to balance W if we were to chose a
pivot
point at the right end of the bar
In the given diagram the torque is = FL.
Finding the torque of all the choices
reveals C as correct. (2Fsin60) (L) =
2F 12 L = FL
Applying rotational equilibrium to each
diagram gives
DIAGRAM 1: (mg)(L1) = (M1g)(L2), L1
= M1(L2) /m. (sub this L1) into the
Diagram 2 eqn, and solve. DIAGRAM
2: (M2g)(L1) = mg(L2), M2 (L1) = m
(L2)

8.torque.png

9.torque.png

10.torque.png

11.torque.png

15.torque.png

16.torque.png

17.torque.png

18.torque.png

zero

FR

2FR

5FR

14FR

m1 = m2

am1 = bm2

am2 = bm1

a^2m1 = b^2m^2

b^2m1 = a^2m2

2.0 Nm

8.0 Nm

10 Nm

14 Nm

16 Nm

Find the torques of each using proper


signs and add up. + (1) (2) + (3) + (4)
+F(3R) (2F)(3R) + F(2R) +F(3R) =
2FR
19.torque.png
Simply apply rotational equilibrium
(m1g) r1 = (m2g) r2
m1a = m2b
20.torque.png
Question says meterstick has no mass,
so ignore that force.
Pivot placed at 0.60 m. Based on the
applied masses, this
meterstick would have a net torque and
rotate. Find the net
Torque as follows
net = + (m1g) r1 (m2g) r2
+ (2)(10 m/s2)(0.6 m) (1)(10 m/s2)
(0.4 m)

Am/k

(1/A)k/m

(1/A)m/k

Conservation of Energy, Usp = K,


kA2 = mv2 solve for v

Fd cos

-mgd

- mgd cos

Constant velocity ---> Fnet=0, fk = Fx =


Fcos Wfk = fkd = Fcos d

A meterstick of negligible mass is placed on a fulcrum at the 0.60 m mark,


with a 2.0 kg mass hung at the 0 m
mark and a 1.0 kg mass hung at the 1.0 m mark. The meterstick is released
from rest in a horizontal position.
Immediately after release, the magnitude of the net torque on the meterstick
21 Torque
about the fulcrum is most nearly
B
A mass m attached to a horizontal massless spring with spring constant k, is
set
into simple harmonic motion. Its maximum displacement from its equilibrium
position is A. What is the masses speed as it passes through its equilibrium
1 WorkPowerEnergy
position?
B
A force F at an angle above the horizontal is used to pull a heavy suitcase
of weight mg a distance d along a
level floor at constant velocity. The coefficient of friction between the floor and
the suitcase is . The work
2 WorkPowerEnergy
done by the frictional force is:
A

0 Ak/m

Fd cos

mgh Fd cos

1.workpowerenergy.
png

If the unit for force is F, the unit for velocity V, and the unit for time T, then the
3 WorkPowerEnergy
unit for energy is:
A
A force of 10 N stretches a spring that has a spring constant of 20 N/m. The
potential energy stored in the
4 WorkPowerEnergy
spring is:
A

A 2 kg ball is attached to a 0.80 m string and whirled in a horizontal circle at a


constant speed of 6 m/s. The
5 WorkPowerEnergy
work done on the ball during each revolution is:
E
A pendulum bob of mass m on a cord of length L is pulled sideways until the
cord makes an angle with the vertical as shown in the figure to the right.
The
6 WorkPowerEnergy
change in potential energy of the bob during the displacement is:
A

FVT

F/T

FV/T

F/T^2

FV^2/T^2

2.5 J

5.0 J

10 J

40 J

200 J

450 J

90 J

72 J

16 J

zero

mgL (1cos )

mgL (1sin )

mgL sin

mgL cos

2mgL (1sin )

Try out the choices with the proper


units for each quantity.
Choice A FVT = (N) (m/s) (s) = Nm
which is work in joules same as energy.
Two step problem. Do F = kx, solve
for x then sub in the Usp = kx2
In a circle moving at a constant speed,
the work done is zero since the Force is
always
perpendicular to the distance moved as
you move incrementally around the
circle
The potential energy at the first position
will be the amount lost as the ball falls
and this will be the change in potential. 6.workpowerenergy.
U=mgh = mg(LLcos )
png
A force directed above the horizontal
looks like this To find the work done
by this
force we use the parallel component of
the force (Fx) x distance. = (Fcos ) d
The maximum speed would occur if all
of the potential energy was converted
to kinetic
U = K 16 = mv2
16 = (2) v2
The work done by the stopping force
equals the loss of kinetic energy.
W=K
Fd = mvf
2
mvi
2
vf = 0 F = mv2
/2d
The work done by friction equals the
loss of kinetic energy
fk d = mvf
2
mvi
2
vf = 0, plug in values to get answer

A force F directed at an angle above the horizontal is used to pull a crate a


distance D across a level floor.
7 WorkPowerEnergy
The work done by the force F is

FD

FD cos

FD sin

mg sin

mgD cos

A compressed spring has 16 J of potential energy. What is the maximum


8 WorkPowerEnergy
speed it can impart to a 2 kg object?

mgD cos

mgD cos

mgD cos

mgD cos

mgD cos

A softball player catches a ball of mass m, which is moving towards her with
horizontal speed V. While bringing the ball to rest, her hand moved back a
distance d. Assuming constant deceleration, the horizontal force exerted on
9 WorkPowerEnergy
the ball by the hand is

mgD cos

mgD cos

mgD cos

mgD cos

mgD cos

mgD cos

mgD cos

mgD cos

mgD cos

mgD cos

mgD cos

mgD cos

mgD cos

mgD cos

mgD cos

A pendulum is pulled to one side and released. It swings freely to the


opposite side and stops. Which of thefollowing might best represent graphs of
12 WorkPowerEnergy
kinetic energy (Ek), potential energy (Ep) and total mechanical energy (ET)
C

mgD cos

mgD cos

mgD cos

mgD cos

mgD cos

mgD cos

mgD cos

mgD cos

mgD cos

mgD cos

mgD cos

mgD cos

mgD cos

mgD cos

mgD cos

mgD cos

mgD cos

mgD cos

mgD cos

Same relationship as above double


the v gives 4x the distance
Half way up you have gained half of the
height so you gained of potential
energy. Therefore
you must have lost of the initial
kinetic energy so E2
= (Ek/2).
Subbing into this relationship

mgD cos

mgD cos

mgD cos

mgD cos

mgD cos

At the top, the ball is still moving (vx) so


would still possess some kinetic energy

v=(kd/m)^1/2

v2=kd/m

v=kd/mg

v2=mgd/k

v=d*(k/m)^1/2

Conservation of energy

A 3 kg block with initial speed 4 m/s slides across a rough horizontal floor
before coming to rest. The frictional force acting on the block is 3 N. How far
10 WorkPowerEnergy
does the block slide before coming to rest?
A construction laborer holds a 20 kg sheet of wallboard 3 m above the floor
for 4 seconds. During these 4 seconds how much power was expended on
11 WorkPowerEnergy
the wallboard?

Problems 13 and 14 refer to the following situation: A car of mass m slides


across a patch of ice at a speed v with its brakes locked. It the hits dry
pavement and skids to a stop in a distance d. The coefficient of kinetic friction
13 WorkPowerEnergy
between the tires and the dry road is.
B
Problems 13 and 14 refer to the following situation: A car of mass m slides
across a patch of ice at a speed v with
its brakes locked. It the hits dry pavement and skids to a stop in a distance d.
The coefficient of kinetic friction
14 WorkPowerEnergy
between the tires and the dry road is.
E

A ball is thrown vertically upwards with a velocity v and an initial kinetic


energy Ek. When half way to the top of its flight, it has a velocity and kinetic
15 WorkPowerEnergy
energy respectively of
A football is kicked off the ground a distance of 50 yards downfield.
Neglecting air resistance, which of the
following statements would be INCORRECT when the football reaches the
16 WorkPowerEnergy
highest point?
A mass m is attached to a spring with a spring constant k. If the mass is set
into motion by a displacement d from its equilibrium position, what would be
17 WorkPowerEnergy
the speed, v, of the mass when it returns to equilibrium position?

P = Fd / t. Since there is no distance


moved, the power is zero
This is a conservative situation so the
total energy should stay same the
whole time. It should also start with max
potential and min kinetic, which only
occurs in choice C
Stopping distance is a work-energy
relationship. Work done by friction to
stop = loss of kinetic
fk d = mvi
2
k
mg = mvi
2
The mass cancelsin the relationship
above so changing mass doesnt
change the distance

If M represents units of mass, L represents units of length, and T represents


18 WorkPowerEnergy
units of time, the dimensions of power would be:
C
An automobile engine delivers 24000 watts of power to a cars driving wheels.
If the car maintains a constant speed of 30 m/s, what is the magnitude of the
19 WorkPowerEnergy
retarding force acting on the car?
A

P = F d / t = (ma)d / t = (kg)(m/s2
)(m) / (s) = kg m2
/ s3
P = Fv, plug in to get the pushing force
F and since its constant speed, Fpush
= fk
Total energy is always conserved so as
the air molecules slow and lose their
kinetic energy, there
is a heat flow which increases internal
(or thermal) energy

ML/T2

ML2/T2

ML2/T3

ML/T

ML2/T

800 N

960 N

1950 N

720000 N

1560000 N

it disapears

it turns into
potential energy

it turns into thermal it turns into sound


enegy
energy

it turns into
electrical energy

13 J

26 J

52 J

130 J

260 J

20 N

14.1 N

10 N

7.07 N

5N

2500 J

3750 J

10000 J

25000 J

37500 J

the distance
between the rocks
increases while
both are falling.

the acceleration is
greater for the
the speed of both
more massive
rocks is constant
rock.
while they fall.

they strike the


ground more than
half a second
apart.

they strike the


ground with the
same kinetic
energy

0J

45 J

280 J

635 J

The kinetic and


potential energies
are equal to each
other at all times.

The kinetic and


potential energies
are both constant

The maximum
potential energy is
achieved when the
mass passes
through its
equilibrium
position.

2700 J
The maximum
kinetic energy
The maximum
occurs at
kinetic energy and maximum
maximum potential displacement of
energy are equal, the mass from its
but occur at
equilibrium
different times
position

The work done must equal the total


gain in potential energy
10 boxes * mgh (25)(10)(1.5) of each
Eliminating obviously wrong choices
only leaves A as an option. The answer
is A because since
the first ball has a head start on the
second ball it is moving at a faster rate
of speed at all
times. When both are moving in the air
together for equal time periods the first
faster rock
will gain more distance than the slower
one which will widen the gap between
them.
All of the K = m v2
is converted to U. Simply plug in the
values
For a mass on a spring, the max U
occurs when the mass stops and has
no K while the max K
occurs when the mass is moving fast
and has no U. Since energy is
conserved it is
transferred from one to the other so
both maximums are equal

Vo^2/2g

mg/k Vo

m/k Vo

m/k Vo ^(1/2)

k/m Vo ^(1/2)

Simple energy conservation K=Usp


mvo^2
= k x2 solve for x

1.0 kg

1.2 kg

1.6 kg

2.0 kg

2.5 kg

10 N

12 N

16 N

20 N

33 N

The work done on the block by the gravitational force during the 5-meter slide
40 WorkPowerEnergy
down the plane is most nearly
B

20 J

60 J

80 J

100 J

130 J

A fan blows the air and gives it kinetic energy. An hour after the fan has been
20 WorkPowerEnergy
turned off, what has happened to the kinetic energy of the air?
A box of old textbooks is on the middle shelf in the bookroom 1.3 m from the
floor. If the janitor relocates the
box to a shelf that is 2.6 m from the floor, how much work does he do on the
21 WorkPowerEnergy
box? The box has a mass of 10 kg.
A mass, M, is at rest on a frictionless surface, connected to an ideal
horizontal spring that is unstretched. A person extends the spring 30 cm from
equilibrium and holds it by applying a 10 N force. The spring is brought back
to equilibrium and the mass connected to it is now doubled to 2M. If the
spring is extended back 30 cm from equilibrium, what is the necessary force
22 WorkPowerEnergy
applied by the person to hold the mass stationary there?
A deliveryman moves 10 cartons from the sidewalk, along a 10-meter ramp to
a loading dock, which is 1.5 meters above the sidewalk. If each carton has a
mass of 25 kg, what is the total work done by the deliveryman on the cartons
23 WorkPowerEnergy
to move them to the loading dock?

A rock is dropped from the top of a tall tower. Half a second later another
24 WorkPowerEnergy
rock, twice as massive as the first, is dropped. Ignoring air resistance,
A
A 60.0-kg ball of clay is tossed vertically in the air with an initial speed of 4.60
m/s. Ignoring air resistance, what is the change in its potential energy when it
25 WorkPowerEnergy
reaches its highest point?
D

26
27
28
29
30
31
32
33
34
35
36

37

38

Which of the following is true for a system consisting of a mass oscillating on


WorkPowerEnergy
the end of an ideal spring?
D
WorkPowerEnergy
WorkPowerEnergy
WorkPowerEnergy
WorkPowerEnergy
WorkPowerEnergy
WorkPowerEnergy
WorkPowerEnergy
WorkPowerEnergy
WorkPowerEnergy
WorkPowerEnergy
A block of mass m slides on a horizontal frictionless table with an initial speed
vo. It then compresses a spring
of force constant k and is brought to rest. How much is the spring
WorkPowerEnergy
compressed from its natural length?
D
A plane 5 meters in length is inclined at an angle of 37, as shown. A block
of weight 20 newtons is placed at the top of the plane and allowed to slide
WorkPowerEnergy
down. The mass of the block is most nearly
D

The magnitude of the normal force exerted on the block by the plane is
39 WorkPowerEnergy
most nearly

The work done must equal the increase


in the potential energy mgh = (10)(10)
(1.3)
Based on F = k x. The mass attached
to the spring does not change the
spring constant so the
same resistive spring force will exist, so
the same stretching force would be
required

Simple application of Fg=mg


38.powerenergy.png
Fn= Fgy = mg cos . Since you are
given the incline with sides listed, cos
can be found by
using the dimensions of the incline
cos = adj / hyp = 4/5 to make math
simple. This is
a good trick to learn for physics
problems
38.powerenergy.png
As the box slides down the incline, the
gravity force is parallel to the height of
the incline the
whole time so when finding the work or
gravity you use the gravity force for F
and the
height of the incline as the parallel
distance. Work = (Fg)(d) = (20)(3)
38.powerenergy.png

The student must exert an average


force equal to their weight (Fg) in order
to lift themselves so
the lifting force F=mg. The power is
then found with P = Fd / t = (Fg)d / t
As the object oscillates its total
mechanical energy is conserved and
transfers from U to K back
and forth. The only graph that makes
sense to have an equal switch
throughout is D
To push the box at a constant speed,
the child would need to use a force
equal to friction so
F=fk
=mg. The rate of work (W/t) is the
power. Power is given by P=Fv
mgv
Two steps. I) use hookes law in the first
situation with the 3 kg mass to find the
spring constant
(k). Fsp=kx, mg=kx, k = 30/.12 =
250. II) Now do energy conservation
with the second
scenario (note that the initial height of
drop will be the same as the stretch
x).
U top = Usp bottom, mgh = k x^2,
(4)(10)(x) = (250) (x^2)
In a circular orbit, the velocity of a
satellite is given by
V= (GMe/r)^(1/2) with Me = M. Kinetic
energy of the satellite is given by K =
m v^2
. Plug in v from above to get answer

A student weighing 700 N climbs at constant speed to the top of an 8 m


vertical rope in 10 s. The average
41 WorkPowerEnergy
power expended by the student to overcome gravity is most nearly

1.1 W

87.5 W

560 W

875 W

5600 W

The graph shown represents the potential energy U as a function of


displacement x for an object on the end of a spring moving back and
forth with amplitude x0. Which of the following graphs represents
42 WorkPowerEnergy
the kinetic energy Kof the object as a function of displacement x ?

A child pushes horizontally on a box of mass m which moves with constant


speed v across a horizontal floor.
The coefficient of friction between the box and the floor is . At what rate
43 WorkPowerEnergy
does the child do work on the box?

mgv

mgv

mg/v

m^2g/v^2

mv^2

A block of mass 3.0 kg is hung from a spring, causing it to


stretch 12 cm at equilibrium, as shown. The 3.0 kg block is then
replaced by a 4.0 kg block, and the new block is released from
the position shown, at which the spring is unstretched. How far
44 WorkPowerEnergy
will the 4.0 kg block fall before its direction is reversed?

9 cm

18 cm

24 cm

32 cm

48 cm

Zero

1/2 GMm/R

1/4 GMm/R

1/2 GMm/R^2

GMm/R^2

hvo^(1/2)

h/v0

hVo/g

2h/g

2h/g^(1/2)

mgh

mv^2

mv^2+mgh

mgh- mv^2

The maximum
kinetic energy is
attained as the
sphere passes
through its
equilibrium
position.

mv^2 - mgh
The minimum
gravitational
potential energy is
The maximum
attained as the
kinetic energy is
sphere passes
attained as the
through its
sphere reaches its equilibrium
point of release.
position.

The maximum
gravitational
potential energy is
attained when the
sphere reaches its
point of release.

The maximum total


energy is attained
only as the sphere
passes through its
equilibrium
A is true; both will be moving the fastest
position.
when they move through equilibrium.
X and Y directions are independent and
both start with the same velocity of zero
in each
direction. The same force is applied in
each direction for the same amount of
time so each
should gain the same velocity in each
E
respective direction.
Kinetic energy is not a vector and the
total resultant velocity should be used
to determine the KE.
For the 1st second the object gains
speed at a uniform rate in the x
direction and since KE is
proportional to v2 we should get a
parabola. However, when the 2nd
second starts the new
gains in velocity occur only in the y
direction and are at smaller values so
the gains
essentially start over their parabolic
E
trend as shown in graph B

400W

900W

800W

1000W

3600W

mgHT

mgH/T

mg/HT

mgT/H

Zero

What is the kinetic energy of a satellite of mass m that orbits the Earth, of
45 WorkPowerEnergy
mass M, in a circular orbit of radius R?
B
A rock of mass m is thrown horizontally off a building from a height h, as
shown above. The speed of the rock as it
leaves the throwers hand at the edge of the building is v
0. How much time does it take the rock to travel from the edge of the building
46 WorkPowerEnergy
to the ground?
E

47 WorkPowerEnergy
What is the kinetic energy of the rock just before it hits the ground?

48 WorkPowerEnergy
Which of the following is true for both spheres?
An object of mass m is initially at rest and free to move without friction in any
direction in the xy-plane. Aconstant net force of magnitude F directed in the
+x direction acts on the object for 1 s. Immediately there after a constant net
force of the same magnitude F directed in the +y direction acts on the object
for 1 s. After this, no forces act on the objectWhich of the following vectors
could represent the velocity of the object at the end of 3 s, assuming the
scales
49 WorkPowerEnergy
on the x and y axes are equal?

An object of mass m is initially at rest and free to move without friction in any
direction in the xy-plane. Aconstant net force of magnitude F directed in the
+x direction acts on the object for 1 s. Immediately there after a constant net
force of the same magnitude F directed in the +y direction acts on the object
for 1 s. After this, noforces act on the objectWhich of the following graphs
50 WorkPowerEnergy
best represents the kinetic energy K of the object as a function of time?
A constant force of 900 N pushes a
100 kg mass up the inclined plane
shown at a uniform speed of 4 m/s.
The power developed by the 900 N
51 WorkPowerEnergy
force is most nearly

An object of mass m is lifted at constant velocity a vertical distance H in time


52 WorkPowerEnergy
T. The power supplied by the lifting force is

Projectile. Vx doesnt matter Viy = 0.


Using d = viyt + at^2
we get the answer
Energy conservation Etop = Ebot,
Kt+Ut = Kb. Plug in for K top and U top
to get answer

42.workpowerenergy.
png

44.workpowerenergy.
png

46.workpowerenergy.
png
47.workpowerenergy.
png

48.workpowerenergy.
png

49.workpowerenergy.
png

50.workpowerenergy.
png

51.workpowerenergy.
Simple P = Fv
png
The force needed to lift something at a
constant speed is equal to the object
weight F=mg. The
power is then found by P = Fd / t = mgh
/t

A system consists of two objects having masses ml and m2 (ml < m2). The
objects are
connected by a massless string, hung over a pulley as shown, and then
released.
When the object of mass m2
has descended a distance h, the potential energy of the
53 WorkPowerEnergy
system has decreased by
The following graphs, all drawn to the same scale, represent the net force F
as a function of displacement x for
an object that moves along a straight line. Which graph represents the force
that will cause the greatest change
in the kinetic energy of the object from x = 0 to x = x1?
54 WorkPowerEnergy
151
From the top of a 70-meter-high building, a l-kilogram ball is thrown directly
downward with an initial speed of
10 meters per second. If the ball reaches the ground with a speed of 30
meters per second, the energy lost to
55 WorkPowerEnergy
friction is most nearly

(m2-m1)gh

M2gh

(M1+M2)gh

(ml + m2)gh

sqrt(gl)

sqrt(2gl)

.5gl

gl

2gl

As the system moves, m2 loses energy


over distance h and m1 gains energy
over the same
distance h but some of this energy is
converted to KE so there is a net loss of
U. Simply
0 subtract the U2 U1 to find this loss
In a force vs. displacement graph, the
area under the line gives the work done
by the force and
the work done will be the change in the
K so the largest area is the most K
change

53.workpowerenergy.
png

54.workpowerenergy.
png

A rock is lifted for a certain time by a force F that is greater in magnitude than
the rock's weight W. The change
57 WorkPowerEnergy
in kinetic energy of the rock during this time is equal to the
A

work done by the


net force (F - W)

work done by F
alone

work done by W
alone

difference in
the momentum of
the rock before
and after this time.

A ball is thrown upward. At a height of 10 meters above the ground, the ball
has a potential energy of 50 joules
(with the potential energy equal to zero at ground level) and is moving upward
with a kinetic energy of 50
joules. Air friction is negligible. The maximum height reached by the ball is
58 WorkPowerEnergy
most nearly
B

10 m

20 m

30 m

40 m

The potential
energy of the
spring is at a
minimum at x = 0.

The potential
energy of the
The kinetic energy The kinetic energy
spring is at a
of the block is at a of the block is at a
minimum at x = A. minimum at x =0. maximum at x = A.

Compare the U+K ( mgh + mv2 ) at


the top, to the K ( mv2 ) at the bottom
and subtract them
700J
to get the loss.
Use energy conservation, U top = K
bottom. As in problem #6 (in this
document), the initial
height is given by L Lcos , with cos 56.workpowerenergy.
2gl
60 = .5 so the initial height is L.
png
Use application of the net work energy
difference in the
theorem which says Wnet = K. The
potential energy of net work is the
the rock before
work done by the net force which gives
and after this time. you the answer
First use the given location (h=10m)
and the U there (50J) to find the mass.
U=mgh, 50=m(10)(10), so m = 0.5 kg.
The total mechanical energy is given in
the problem
as U+K = 100 J. The max height is
achieved when all of this energy is
potential. So set
50 m
100J = mgh and solve for h
The kinetic energy
of the block is
always equal to
the potential
There is no Usp at position x=0 since
energy of the
there is no x here so this is the
59.workpowerenergy.
spring.
minimum U location
png

16 N

8N

6N

4N

2N

h/4

h/2(2^1/2)

h/2

h/(2^1/2)

P = Fv
Using energy conservation in the first
situation presented K=U gives the initial
velocity as
v = (2gh)^1/2 . The gun will fire at this
velocity regardless of the angle. In the
second
scenario, the ball starts with the same
initial energy but at the top will have
both KE and PE
so will be at a lower height. The velocity
at the top will be equal to the vx
vx = vcos = ((2gh)^1/2) cos45 = (2gh)
^1/2 ((2^1/2)/2) = (gh^1/2)
at the beginning
. Now sub into the full energy
conservation problem for situation 2
and solve for h2. Kbottom = Utop +
Ktop. 1/2m ((2gh^1/2)^2)= mgh + 1/2m
((gh^1/2)^2)

Fd sin

Fd cos

Fm cos

Fm tan

Fmd sin

To find work we use the parallel


component of the force to the distance, 62.workpowerenergy.
this gives Fcos d
png

1000 N

The centripetal force is the force


allowing the circular motion which in
this case is the spring
force Fsp=kx=(100)(.03)

A pendulum consists of a ball of mass m suspended at the end of a


massless cord of length L as shown. The pendulum is drawn aside
through an angle of 60 with the vertical and released. At the low point
56 WorkPowerEnergy
of its swing, the speed of the pendulum ball is

0J

100J

300J

400J

sqrt(gl)

sqrt(2gl)

.5gl

gl

A block on a horizontal frictionless plane is


attached to a spring, as shown. The block
oscillates along the x-axis with amplitude A.
Which of the following statements about
59 WorkPowerEnergy
energy is correct?
A
During a certain time interval, a constant force delivers an average power of 4
watts to an object. If the object
has an average speed of 2 meters per second and the force acts in the
direction of motion of the object, the
60 WorkPowerEnergy
magnitude of the force is
E

A spring-loaded gun can fire a projectile to a height h if it is fired straight up. If


the same gun is pointed at an
angle of 45 from the vertical, what maximum height can now be reached by
61 WorkPowerEnergy
the projectile?
C
A force F is exerted by a broom handle on the head of the broom, which has
a
mass m. The handle is at an angle to the horizontal, as shown. The work
done by the force on the head of the broom as it moves a distance d across a
62 WorkPowerEnergy
horizontal floor is
B
A spring has a force constant of 100 N/m and an unstretched length of 0.07
m. One
end is attached to a post that is free to rotate in the center of a smooth table,
as
shown in the top view. The other end is attached to a 1 kg disc moving in
uniform circular motion on the table, which stretches the spring by 0.03 m.
63 WorkPowerEnergy
Friction is negligible. What is the centripetal force on the disc?
B

.3 N

3N

10 N

300 N

63-64.
workpowerenergy.
png

A spring has a force constant of 100 N/m and an unstretched length of 0.07
m. One
end is attached to a post that is free to rotate in the center of a smooth table,
as
shown in the top view. The other end is attached to a 1 kg disc moving in
uniform circular motion on the table, which stretches the spring by 0.03 m.
Friction is negligible. What is the work done on the disc by the spring during
64 WorkPowerEnergy
one full circle?

0J

94 J

186 J

314 J

628 J

A frictionless pendulum of length 3 m swings with an amplitude of 10. At its


maximum displacement, the
potential energy of the pendulum is 10 J. What is the kinetic energy of the
pendulum when its potential energy
65 WorkPowerEnergy
is 5 J ?

3.3 J

5J

6.7 J

10 J

15 J

4 m/s

10 m/s

13 m/s

16 m/s

21 m/s

t/2

2^1/2 t

2t

4t

v(m/k)^(1/2)

v(k/m)^(1/2)

v((m+M)/k)^(1//2)

(m+M)v/(mk)^1/2

mv/((m+M)k)^1/2

.25s

.5s

1.0s

2.0s

4.0s

66
67
68
69
70
71
72
73
74
75
76

A descending elevator of mass 1,000 kg is uniformly decelerated to rest


over a distance of 8 m by a cable in which the tension is 11,000 N. The
speed vi of the elevator at the beginning of the 8 m descent is most
WorkPowerEnergy
nearly
A
WorkPowerEnergy
WorkPowerEnergy
WorkPowerEnergy
WorkPowerEnergy
WorkPowerEnergy
WorkPowerEnergy
WorkPowerEnergy
WorkPowerEnergy
WorkPowerEnergy
WorkPowerEnergy
A car of mass m, traveling at speed v, stops in time t when maximum braking
force is applied. Assuming the
braking force is independent of mass, what time would be required to stop a
momentum
car of mass 2m traveling at speed v?
D

3 momentum

A block of mass M is initially at rest on a frictionless floor. The block, attached


to a massless spring with spring
constant k, is initially at its equilibrium position. An arrow with mass m and
velocity v is shot into the block.
The arrow sticks in the block. What is the maximum compression of the
spring?
E
How long must a 100 N net force act to produce a change in momentum of
200 kgm/s?
D

4 momentum

Which is a vector quantity

energy

mass

impulse

power

work

5 momentum

When the velocity of a moving object is doubled, its _____ is also doubled

acceleration

kinetic energy

mass

momentum

potential energy

2 momentum

7 momentum

Two objects, P and Q, have the same momentum. Q can have more kinetic
energy than P if it has:
C
A spring is compressed between two objects with unequal masses, m and M,
and held together. The objects are
initially at rest on a horizontal frictionless surface. When released, which of
the following is true?
E

More mass than P


Kinetic energy is
the same as
before begin
released.

8 momentum

Net Impulse is best related to

momentum

6 momentum

The same mass as


The same speed
P
More speed thanP at P
The speed of one
The total final
The two objects
object is equal to
kinetic energy is
have equal kinetic the speed of the
zero.
energy.
other.
change in
change in kinetic
momentum
kinetic energy
energy

Q can not have


more kinetic
energy than P

In a circle at constant speed, the work


done is zero since the Force is always
perpendicular to the
distance moved as you move
incrementally around the circle
At the maximum displacement the K=0
so the 10J of potential energy at this
spot is equal to the
total amount of mechanical energy for
the problem. Since energy is conserved
in this
situation, the situation listed must have
U+K add up to 10J.
Using the work-energy theorem. Wnc =
ME,
WFt = U+K,
Fd = (mghf mghi) + ( mvf
^2 mvi
^2),
(11000)(8) = (0 (1000)(10)(8)) + (0
(1000)(vi
^2) solve for vi

Based on Ft = mv, doubling the mass


would require twice the time for same
momentum change
Two step problem.
I) find velocity after collision with arrow.
mvi = (ma+mb) vf vf = mv / (m+M)
II)now use energy conservation Ki=U
(m+M)vf
2 = k X2, sub in vf
from I
Use J=p Ft=p (100)t=200 t=2
Definition. Impulse, just like momentum,
needs a direction and is a vector
Since p=mv, by doubling v you also
double p
Since the momentum is the same, that
means the quantity m1v1 = m2v2. This
means that the
mass and velocity change
proportionally to each other so if you
double m1 you would have
to double m2 or v2 on the other side as
well to maintain the same momentum.
Now we
consider the energy formula KE=
mv2 since the v is squared, it is the
more important term
to increase in order to make more
energy. So if you double the mass of 1,
then double the
velocity of 2, you have the same
momentum but the velocity of 2 when
squared will make a
greater energy, hence we want more
velocity in object 2 to have more
energy.

The total final


Due to momentum conservation, the
momentum of the total before is zero therefore the total
two objects is zero. after must also be zero
none of the above Definition. Jnet = p

63-64.
workpowerenergy.
png

66.workpowerenergy.
png

9 momentum

10 momentum

11 momentum

12 momentum

13 momentum
14 momentum

15 momentum

16 momentum

17 momentum

Two football players with mass 75 kg and 100 kg run directly toward each
other with speeds of 6 m/s and 8 m/s
respectively. If they grab each other as they collide, the combined speed of
the two players just after the
collision would be:
A 30 kg child who is running at 4 m/s jumps onto a stationary 10 kg
skateboard. The speed of the child and the
skateboard is approximately:
A 5000 kg freight car moving at 4 km/hr collides and couples with an 8000 kg
freight car which is initially at
rest. The approximate common final speed of these two cars is
A rubber ball is held motionless a height ho above a hard floor and released.
Assuming that the collision with
the floor is elastic, which one of the following graphs best shows the
relationship between the total energy E of
the ball and its height h above the surface.

Two carts are held together. Cart 1 is more massive than Cart 2. As they are
forced apart by a compressed
spring between them, which of the following will have the same magnitude for
both carts.
If the unit for force is F, the unit for velocity is v and the unit for time t, then
the unit for momentum is
A ball with a mass of 0.50 kg and a speed of 6 m/s collides perpendicularly
with a wall and bounces off with a
speed of 4 m/s in the opposite direction. What is the magnitude of the impulse
acting on the ball?
A cart with mass 2m has a velocity v before it strikes another cart of mass 3m
at rest. The two carts couple and
move off together with a velocity of

20 momentum

Which of the above statements would be correct?


A mass m has speed v. It then collides with a stationary object of mass 2m. If
both objects stick together in a
perfectly inelastic collision, what is the final speed of the newly formed
object?
A Freight car is moving freely along a railroad track at 7 m/s and collides with
a tanker car that is at rest. After the collision, the two cars stick together and
continue to move down the track. What is the magnitude of the final velocity
of the cars if the freight car has a mass of 1200 kg and the tanker car has a
mass of 1600 kg?
A 50 kg skater at rest on a frictionless rink throws a 2 kg ball, giving the ball a
velocity of 10 m/s. Which statement describes the skaters subsequent
motion?

21 momentum

A certain particle undergoes erratic motion. At every point in its motion, the
direction of the particles momentum is ALWAYS

22 momentum

A student initially at rest on a frictionless frozen pond throws a 1 kg hammer


in one direction. After the throw,
the hammer moves off in one direction while the student moves off in the
other direction. Which of the
following correctly describes the above situation?

18 momentum

19 momentum

2 m/s

3.4 m/s

4.6 m/s

7.1 m/s

8 m/s

Perfect inelastic collision. m1v1i +


m2v2i = mtot(vf) (75)(6) + (100)(8)
= (175) vf

3m/s

4m/s

5m/s

6m/s

7m/s

Perfect inelastic collision. m1v1i = mtot


(vf) (30)(4) = (40)vf

1 km/h

1.3 km/h

1.5 km/h

2.5 km/h

4 km/h

acceleration

change of velocity force

speed

velocity

Ft

Ftv

Ft2v

Ft / v

Fv / t

Perfect inelastic collision. m1v1i = mtot


(vf) ... (5000)(4) = (13000)vf
Energy is conserved during fall and
since the collision is elastic, energy is
also conserved during the collision and
always has the same total value
throughout.
12.momentum.png
To conserve momentum, the change in
momentum of each mass must be the
same so each must receive the same
impulse. Since the spring is in contact
with each mass for the same
expansion time, the applied force must
be the same to produce the same
impulse.
Momentum is equivalent to impulse
which is Ft

13 J

1 Ns

5 Ns

2 m/s

10 m/s

Use J=p J = mvf mvi J = (0.5)( 4)


(0.5)(6)

v/5

2v/5

3v/5

2v/3

(2/5)1/2 v

I only

II only

III only

I and II only

I and III only

Perfect inelastic collision. m1v1i = mtot


(vf) ... (2m)(v) = (5m) vf
First of all, if the kinetic energies are the
same, then when brought to rest, the
non conservative work done on each
would have to be the same based on
work-energy principle. Also, since
both have the same kinetic energies we
have 12 m1v12 = 12 m2v22 ... since
the velocity is
squared an increase in mass would
need a proportionally smaller decrease
in velocity to keep
the terms the same and thus make the
quantity mv be higher for the larger
mass. This can be seen through
example: If mass m1 was double mass
m2 its velocity would be v / 2 times in
comparison to mass m2s velocity. So
you get double the mass but less than
half of the velocity which makes a
larger mv term.
17.momentum.png

v/3

v/2

2v / 3

3v / 2

Perfect inelastic collision. m1v1i = mtot


(vf) ... (m)(v) = (3m) vf

0 m/s
0.4 m/s in the
same direction as
the ball.

1 m/s
0.4 m/s in the
opposite direction
of the ball

3 m/s
2 m/s in the same
direction as the
ball

6 m/s
4 m/s in the
opposite direction
of the ball

the same as the


direction of its
velocity

the same as the


direction of its
acceleration

4 m/s
4 m/s in the same
direction as the
ball
the same as the
the same as the
direction of the
direction of the net kinetic energy
force
vector

The hammer will


have the
momentum with
the greater
magnitude

The student will


have the
momentum with
the greater
magnitude

The hammer will


have the greater
kinetic energy

Since p=mv and both p and v are


vectors, they must share the same
none of the above direction
Explosion, momentum before is zero
and after must also be zero. To have
equal momentum the
heavier student must have a much
The student and
smaller velocity and since that smaller
the hammer will
velocity is squared
have equal but
it has the effect of making the heavier
opposite amounts object have less energy than the
of kinetic energy
smaller one

The student will


have the greater
kinetic energy

Perfect inelastic collision. m1v1i = mtot


(vf) ... (1200)(7) = (2800)vf
Explosion. pbefore = 0 = pafter ... 0 =
m1v1f + m2v2f ... 0 = (50)(v1f) + (2)(10)

both toy cars will


both toy cars will
acquire equal but acquire equal
opposite momenta kinetic energies

Use J=p Ft = mvf mvi Ft = m(vf vi)


note: since m is not given we will
plug in
Fg / g with g as 10 to be used in the
impulse equation.
1.5 s
3.8 s
15 s
(24000)(t) = (15000 / 10m/s2 ) (3612)
This is a rather involved question. First
find speed of impact using free fall or
energy. Define up
as positive and Let v1 = trampoline
impact velocity and v2 be trampoline
rebound velocity.
With that v1 = 80 and v2 = 80. Now
analyze the impact with the pad using
Jnet =p
Fnet t = mv2 mv1 At this point we
realize we need the time in order to find
the Fnet and
therefore cannot continue. If the time
was given, you could find the Fnet and
More information is then use
500 N
2000 N
required
Fnet = Fpad mg to find Fpad.
Based on momentum conservation both
carts have the same magnitude of
momentum mv but
based on K = m v2 the one with the
larger mass would have a directly
proportional smaller
velocity that then gets squared. So by
squaring the smaller velocity term it has
the effect of
making the bigger mass have less
energy. This can be shown with an
example of one object
the smaller toy car
of mass m and speed v compared to a
will experience an
second object of mass 2m and speed
the more massive acceleration of the
v/2. The larger
toy car will acquire greatest
mass ends up with less energy even
the least speed
magnitude
through the momenta are the same.

8.33 N

18.75 N

27.08 N

45.83 N

458 N

Use J=p Ft = mvf mvi Ft = m(vf vi)


F (0.03) = (0.125)( 6.5 4.5)

0.2 kg

0.5 kg

0.8 kg

1 kg

2 kg

Momentum conservation. pbefore =


pafter m1v1i = m1v1f + m2v2f (0.1)(30)
= (0.1)(20)+(ma)(2)

23 momentum

The net force on a rocket with a weight of 1.5 x 104 N is 2.4 x 104 N. How
much time is needed to increase the
rockets speed from 12 m/s to 36 m/s near the surface of the Earth at takeoff? C

0.62 s

0.78 s

24 momentum

A 50 kg gymnast falls freely from a height of 4 meters on to a trampoline. The


trampoline then bounces her
back upward with a speed equal to the speed at which she first struck the
trampoline. What is the average force
the trampoline applies to the gymnast.
E

50 N

200 N

28 momentum

Two toy cars with different masses originally at rest are pushed apart by a
spring between them. Which of the
following statements would NOT be true?
A bat striking a 0.125 kg baseball is in contact with the ball for a time of 0.03
seconds. The ball travels in a
straight line as it approaches and then leaves the bat. If the ball arrives at the
bat with a speed of 4.5 m/s and
leaves with a speed of 6.5 m/s in the opposite direction, what is the
magnitude of the average force acting on the
ball?
An arrow is shot through an apple. If the 0.1 kg arrow changes speed by 10
m/s during the collision (from 30
m/s to 20 m/s) and the apple goes from rest to a speed of 2 m/s during the
collisions, then the mass of the apple
must be
A railroad flatcar of mass 2,000 kilograms rolls to the right at 10 meters per
second and collides with a flatcar of
mass 3,000 kilograms that is rolling to the left at 5 meters per second. The
flatcars couple together. Their speed
after the collision is

29 momentum

Which of the following quantities is a scalar that is always positive or zero?

25 momentum

26 momentum

27 momentum

1 m/s

2.5 m/ s

5 m/ s

Power

Work

Kinetic energy

7 m/ s

7.5 m/ s
Angular
Linear momentum momentum

2mv

2mv sin

2mv cos

31 momentum

A tennis ball of mass m rebounds from a racquet with the same speed v as it
had
initially as shown. The magnitude of the momentum change of the ball is
E
Two bodies of masses 5 and 7 kilograms are initially at rest on a horizontal
frictionless surface. A light spring is compressed between the bodies, which
are held together by a thin thread. After the spring is released by burning
through the thread, the 5 kilogram body has a speed of 0.2 m/s. The speed of
the 7 kilogram body is (in m/s)
B

1/12.

1/7.

1/sqrt(35)

1/5.

7/25.

32 momentum

A satellite of mass M moves in a circular orbit of radius R at a constant speed


v. Which of the following must be true?
I. The net force on the satellite is equal to MR and is directed toward the
center of the orbit.
II. The net work done on the satellite by gravity in one revolution is zero.
III. The angular momentum of the satellite is a constant.
D

I only

III only

I and II only

II and III only

I, II, and III

30 momentum

0 mv

Perfect inelastic collision. m1v1i +


m2v2i = mtot(vf) (2000)(10) + (3000)
(5) = (5000) vf
Kinetic energy has no direction and
based on K = m v2 must always be +
A 2d collision must be looked at in both
x-y directions always. Since the angle is
the same and
the v is the same, vy is the same both
before and after therefore there is no
momentum
change in the y direction. All of the
momentum change comes from the x
direction.
vix = v cos and vfx = v cos . p =
mvfx mvix mv cos mv cos

Explosion. pbefore = 0 = pafter ... 0 =


m1v1f + m2v2f ... 0 = (7)(v1f) + (5)(0.2)
In a circle at constant speed, work is
zero since the force is parallel to the
incremental distance moved during
revolution. Angular momentum is given
by mvr and since none of those
quantities are changing it is constant.
However the net force is NOT = MR, its
Mv2/R

33 momentum

Two pucks are firmly attached by a stretched spring and are initially held at
rest on a frictionless surface, as shown above. The pucks are then released
simultaneously. If puck I has three times the mass of puck II, which of the
following quantities is the same for both pucks as the spring pulls the two
pucks toward each other?

37 momentum

Momentum was
not conserved
therefore the
report is false.

If the objects had


different masses
If the surface was
the report could be inclined the report
true.
could be true.

Inertia

Speed

Momentum

Kinetic energy

v/2

mv/M

Mv/m

(m+M)v/m

40 momentum

An open cart on a level surface is rolling without frictional loss through a


vertical downpour of rain, as shown above. As the cart rolls, an appreciable
amount of rainwater accumulates in the cart. The speed of the cart will
C
A2kgobjectmovesinacircleofradius4mataconstantspeedof3m/s. Anetforceof4.
5Nactson the object. What is the angular momentum of the object with
respect to an axis perpendicular to the circle and through its center?
E
Two objects of mass 0.2 kg and 0.1 kg, respectively, move parallel to the xaxis, as shown above. The 0.2 kg object overtakes and collides with the 0.1
kg object. Immediately after the collision, the y-component of the velocity of
the 0.2 kg object is 1 m/s upward. What is the y-component of the velocity of
the 0.1 kg object immediately after the collision'?
A

41 momentum

The magnitude of the momentum of the object is increasing in which of the


cases?

42 momentum

The sum of the forces on the object is zero in which of the cases?

43 momentum

A ball of mass 0.4 kg is initially at rest on the ground. It is kicked and leaves
the kicker's foot with a speed of 5.0 m/s in a direction 60 above the
horizontal. The magnitude of the impulse imparted by the ball to the foot is
most nearly

38 momentum

39 momentum

The direction of
motion of the
objects that are
stuck together
depends on
whether the hit is a
head-on collision.

If potential energy
was released to
the objects during
the collision the
report could be
true.

35 momentum

A solid metal ball and a hollow plastic ball of the same external radius are
released from rest in a large vacuum chamber. When each has fallen 1m,
they both have the same
A railroad car of mass m is moving at speed v when it collides with a second
railroad car of mass M which is at rest. The two cars lock together
instantaneously and move along the track. What is the speed of the cars
immediately after the collision?

The speed of the


objects that are
stuck together will
be less than the
initial speed of the
less massive
object.

The collision is
elastic.

Two objects having the same mass travel toward each other on a flat surface
each with a speed of 1.0 meter per second relative to the surface. The
objects collide head-on and are reported to rebound after the collision, each
with a speed of 2.0 meters per second relative to the surface. Which of the
following assessments of this report is most accurate?
B

36 momentum

Magnitude of
momentum

All of the initial


kinetic energy of
the less massive
object is lost.

34 momentum

increase because
of conservation of
momentum

9Nm/kg

Velocity

Kinetic energy

Acceleration
The momentum of
the objects that are
stuck together has
a smaller
magnitude than
the initial
momentum of the
less-massive
object.

Which of the following is true when an object of mass m moving on a


horizontal frictionless surface hits and sticks to an object of mass M > m,
which is initially at rest on the surface?

Speed

In a perfect inelastic collision with one


of the objects at rest, the speed after
will always be less no matter what the
masses. The increase of mass in mv
is offset by a decrease in velocity
Since the total momentum before and
after is zero, momentum conservation
is not violated, however the objects
If there was no
gain energy in the collision which is not
friction between
possible unless there was some
the objects and the energy input which could come in the
surface the report form of inputting stored potential energy
could be true.
in some way.
The plastic ball is clearly lighter so
anything involving mass is out, this
Change in
leaves speed which makes sense
potential energy
based on free-fall

mv/(m+M)
remain the same
because the
raindrops are
falling
increase because decrease because decrease because perpendicular to
of conservation of of conservation of of conservation of the direction of the
mechanical energy momentum
mechanical energy cart's motion

12m^2/s

13.5kgm^2/s^2

18Nm/kg

24kgm^2/s.

2 m/s downward

0.5 m/s downward 0 m/s

0.5 m/s upward

2 m/s upward

IIonly

IIIonly

IandIIonly

IandIIIonly

I,II,andIII

IIonly

IIIonly

IandIIonly

IandIIIonly

I,II,andIII

1Ns

(3)^1/2 N s

2Ns

44 momentum

Two people of unequal mass are initially standing on ice with neglibible
friction. They then simultaneously push each other horizontally. Afterward,
which of the following is true?

45 momentum

A stationary object exploses, breaking into three pieces of masses m, m, and


3m. The two pieces of mass m move off at right angles to each other with the
same magnitude of momentum mV, as shown in the diagram above. What
are the magnitude and direction of the velocity of the piece having mass 3m? D

4Ns

The kinetic
the speeds of the
energies of the two two people are
people are equal. equal.

2/(3)^1/2 N s
The center of
mass of the twoperson system
moves in the
The momenta of
direction of the
the two people are less massive
equal.
person.

Since the momentum before is zero,


the momentum after must also be zero.
Each mass must have equal and
opposite momentum to maintain zero
total momentum.
33.momentum.png

The less massive


person has a
smaller initial
acceleration than
the more massive
person.

Perfect inelastic collision. m1v1i = mtot


(vf) ... (m)(v) = (m+M) vf
As the cart moves forward it gains
mass due to the rain but in the x
direction the rain does not provide any
impulse to speed up the car so its
speed must decrease to conserve
momentum

38.momentum.png

Angular momentum is given by the


formula L = mvr = (2)(3)(4)
2D collision. Analyze the y direction.
Before py = 0 so after py must equal 0.
0 = m1v1fy + m2v2fy 0 = (0.2)(1) +
(0.1)(V2fy)
40.momentum.png
Momentum increases if velocity
increases. In a d-t graph, III shows
increasing slope (velocity)
41.momentum.png
The net force is zero if velocity (slope)
does not change, this is graphs I and II 42.momentum.png
Since the impulse force is applied in the
same direction (60) as the velocity, we
do not need to C use components but
use the 60 inclined axis for the impulse
momentum problem. In that
direction. J=p J=mvf mvi =m(vf vi)=
(0.4)(05)

Initially, before the push, the two people


are at rest and the total momentum is
zero. After, the C total momentum must
also be zero so each man must have
equal and opposite momenta.
Since the initial object was stationary
and the total momentum was zero it
must also have zero D total momentum
after. To cancel the momentum shown
of the other two pieces, the 3m
piece would need an x component of
momentum px = mV and a y
component of momentum
py = mV giving it a total momentum of
2 mV using Pythagorean theorem.
Then set this
total momentum equal to the mass *
velocity of the 3rd particle. 2 mV =
(3m) Vm3 and solve for Vm3
45.momentum.png

46 momentum

47 momentum
48 momentum
49 momentum

A ball is thrown stright up in the air. When the ball reaches its highest point,
which of the following is true?

An empty sled of mass M moves without friction across a frozen pond at


speed vo. Two objects are dropped vertically into the sled one at a time: first
an object of mass m and then an object of mass 2m. Afterward the sled
moves with speed vf. What would be the final speed of the sled if the objects
were dropped into it in reverse order?
C
The slope of the best fit straight line is most nearly
A
The increase in the momentum of the object between t=0 s and t=4 s is most
nearly
C

51 momentum

How does an air mattress protect a stunt person landing on the ground after a
stunt?
E
Two objects, A and B, initially at rest, are "exploded" apart by the release of a
coiled spring that was compressed between them. As they move apart, the
velocity of object A is 5 m/s and the velocity of object B is 2 m/s. The ratio
of the mass of object A to the mass object B, ma/mb is
B

52 momentum

The two blocks of masses M and 2M shown above initially travel at the same
speed v but in opposite directions. They collide and stick together. How much
mechanical energy is lost to other forms of energy during the collision?
D

50 momentum

It has zero
It is in equilibrium. acceleration.

It has maximum
momentum.

It has maximum
kinetic energy.

None of these are


true.

vf /3
5 N/s

vf /2
6 N/s

vf
7 N/s

2vf
8 N/s

3vf
10 N/s

40 Ns

50 Ns

60 Ns

80 Ns
It shortens the
stopping time of
It reduces the
It reduces the
It increases the
the stunt person
kinetic energy loss momentum
momentum
and increases the
of the stunt
change of the stunt change of the stunt force applied
person.
person.
person.
during the landing.

0.16

0.4

Zero

.5mv

.75mv

1.3mv

1.5mv

v0/4

v0/2

(v02)/2

(v03)/2

v0

zero

mva

mvx0

mv(x+a)

mva/(x+a)

54 momentum
55 momentum

The final speed of the first 4-kilogram mass is

0 m/s

2 m/s

3 m/s

4 m/s

56 momentum

The final speed of the two 4-kilogram masses that stick together is

0 m/s

2 m/s

3 m/s

4 m/s

57 momentum

A projectile of mass M1 is fired horizontally from a spring gun that is initially at


rest on a frictionless surface. The combined mass of the gun and projectile is
M2. If the kinetic energy of the projectile after firing is K, the gun will recoil
with a kinetic energy equal to
D

M2*K/M1

[(M1/M2)^2]*K

(M1*K)/(M2-M1)

58 momentum

Two balls are on a frictionless horizontal tabletop. Ball X initially moves at 10


meters per second, as shown in Figure I above. It then collides elastically with
identical ball Y which is initially at rest. After the collision, ball X moves at 6
meters per second along a path at 53 to its original direction, as shown in
Figure II above. Which of the following diagrams best represents the motion
of ball Y after the collision?
D

If one knows only the constant resultant force acting on an object and the
time during which this force acts, one can determine the

C
change in kinetic
energy of the
object

59 momentum

A
change in
momentum of the
object

100 Ns
It lengthens the
stopping time of
the stunt person
and reduces the
force applied
Basic principle of impulse. Increased
during the landing. time lessens the force of impact.

2.5

Two particles of equal mass mo, moving with equal speeds vo along paths
inclined at 60 to the x-axis as shown, collide and stick together. Their
velocity after the collision has magnitude
B
A particle of mass m moves with a constant speed v along the dashed line y
= a. When the x-coordinate of the particle is xo, the magnitude of the angular
momentum of the particle with respect to the origin of the system is
B

53 momentum

change in velocity
of the object

None of the statements are true. I) it is


accelerating so is not in equilibrium, II)
Its acceleration is E 9.8 at all times, III)
Its momentum is zero because its
velocity is momentarily zero, IV) Its
kinetic energy is also zero since its
velocity is momentarily zero.
Its does not matter what order to
masses are dropped in. Adding mass
reduces momentum C proportionally.
All that matters is the total mass that
was added. This can be provided by
finding the velocity after the first drop,
then continuing to find the velocity after
the second
drop. Then repeating the problem in
reverse to find the final velocity which
will come out
the same
Stupid easy. Find slope of line
48.momentum.png
Increase in momentum is momentum
change which
49.momentum.png

Explosion. pbefore = 0 = pafter 0 =


6.25 m1v1f + m2v2f 0 = m1(5) + m2(2)
Perfect inelastic collision. m1v1i +
m2v2i = mtot*(vf) Mv + ( 2Mv) =
(3M)*vf gives vf = v/3.
Then to find the energy loss subtract
the total energy before the total
energy after
[ Mv+ (2M)v] (3M) (v/3) =
3/6 Mv+ 6/6 Mv 1/6 Mv
52.momentum.png
2D collision. The y momentums are
equal and opposite and will cancel out
leaving only the x momentums which
are also equal and will add together to
give a total momentum equal to twice
the x component momentum before
hand. pbefore = pafter 2m0v0cos60
= (2m0)*vf
53.momentum.png

Angular momentum is given by L = mvr


= mva
Perfect inelastic collision. m1v1i = mtot
6 m/s
(vf) (4)(6) = (8)vf
Perfect inelastic collision. m1v1i = mtot
6 m/s
(vf) (8)(3) = (12)vf
First use the given kinetic energy of
mass M1 to determine the projectile
speed after.
K= M1v1f v1f = (2K/M1) . Now
solve the explosion problem with
pbefore=0 = pafter. Note that the mass
of the gun is M2M1 since M2 was
given as the total mass. 0 = M1v1f +
(M2M1)v2f now sub in from above
for v1f. M1((2K/M1)) = (M2M1) v2f
and find v2f v2f = M1((2K/M1)) /
(M2M1) .
Now sub this into K2 = (M2M1) v2f
K*[M1/(M2-M1)]^.5 and simplify.
Since there is no y momentum before,
there cannot be any net y momentum
after. The balls have equal masses so
you need the y velocities of each ball to
be equal after to cancel out the
momenta. By inspection, looking at the
given velocities and angles and without
doing any math, the only one that could
possibly make equal y velocities is
E
choice D

acceleration of the
mass of the object object
Definition. Jnet = p. Fnet*t = p

54.momentum.png
55.momentum.png
55.momentum.png

58.momentum.png &
58.momentumm.png

60
61
62
63
64
65
66
67
68
69
70

momentum
momentum
momentum
momentum
momentum
momentum
momentum
momentum
momentum
momentum
momentum

71 momentum

72 momentum

73 momentum

An object of mass m is moving with speed vo to the right on a horizontal


frictionless surface, as shown above,
when it explodes into two pieces. Subsequently, one piece of mass 2/5 m
moves with a speed vo/2 to the left.
The speed of the other piece of the object is

A 2 kg object initially moving with a constant velocity is subjected


to a force of magnitude F in the direction of motion. A graph of F
as a function of time t is shown. What is the increase, if any, in the
velocity of the object during the time the force is applied?

A disk slides to the right on a horizontal, frictionless air table and collides with
another disk that was initially
stationary. The figures below show a top view of the initial path I of the sliding
disk and a hypothetical path H
for each disk after the collision. Which figure shows an impossible situation? B
A ball of mass m with speed v strikes a wall at an angle with the normal, as
shown. It then rebounds with the same speed and at the same angle. The
impulse delivered by the ball to the wall is
E

vo / 2

vo / 3

7vo / 5

3vo / 2

2vo

0 m/s

2.0 m/s

3.0 m/s

4.0 m/s

6.0 m/s

zero

mv sin

mv cos

2mv sin

2mv cos

1 gravitation

Each of five satellites makes a circular orbit about an object that is much
more massive than any of the
satellites. The mass and orbital radius of each satellite are given below.
Which satellite has the greatest speed?

m R

m R

mR

m 2R

2m R

2 gravitation

An asteroid moves in an elliptic orbit with the Sun at one focus as shown
above. Which of the following quantities increases as the asteroid moves
from point P in its orbit to point Q?

Speed

Angular
momentum

Total energy

Kinetic energy

Potential energy

3 gravitation

Two planets have the same size, but different masses, and no atmospheres.
Which of the following would be the same for objects with equal mass on the
surfaces of the two planets?

4 gravitation

A person weighing 800 newtons on Earth travels to another planet with twice
the mass and twice the radius of
Earth. The person's weight on this other planet is most nearly

400 N

800/2 N

800 N

8002

1,600 N

5 gravitation

Mars has a mass 1/10 that of Earth and a diameter 1/2 that of Earth. The
acceleration of a falling body near the
surface of Mars is most nearly

0.25 m/s2

0.5 m/s2

2 m/s2

4 m/s2

25 m/s2

6 gravitation

A satellite of mass M moves in a circular orbit of radius R at a constant speed


v. Which of the following must
be true?
E

Explosion with initial momentum.


pbefore = pafter mvo = mavaf + mbvbf
mvo = (2/5 m)( vo / 2) + (3/5 m)(vbf)
solve for vbf
60.momentum.png

The area under the F-t graph will give


the impulse which is equal to the
momentum change.
With the momentum change we can
find the velocity change.
J = area = 6 Then J = p = mv 6 = (2)
v v = 3 m/s
This is a 2D collision. Before the
collision, there is no y momentum, so in
the after condition the
y momenta of each disk must cancel
out. In choice B, both particles would
have Y
momentum downwards making a net Y
momentum after which is impossible.
This is the same as question 30 except
oriented vertically instead of
horizontally.
Orbital speed is found from setting
(GMm)/(r2) which gives v=(GM/r)1/2
where M is the
object being orbited. Notice that
satellite mass does not affect orbital
speed. The smallest radius
of orbit will be the fastest satellite.
As a satellite moves farther away, it
slows down, also decreasing its angular
momentum and
kinetic energy. The total energy
remains the same in the absence of
resistive or thrust forces.
The potential energy becomes less
negative, which is an increase.
With different masses, g would have a
different value, but the physical
characteristics of the
objects would not be affected.
g = GM / r2 so the acceleration due to
gravity (and the weight of an object) is
proportional to the
mass of the planet and inversely
proportional to the distance from the
center of the planet
squared. M 2 = g 2 and r 2 = g
4, so the net effect is the persons
weight is divided by 2
g = GM / r2 so the acceleration due to
gravity (and the weight of an object) is
proportional to the
mass of the planet and inversely
proportional to the distance from the
center of the planet
squared. M 10 = g 10 and r 2 = g
4, so the net effect is g 4/10
Circular orbit = constant r, combined
with constant speed gives constant
angular momentum
(mvr). As it is a circular orbit, the force
is centripetal, points toward the center
and is always
perpendicular to the displacement of
the satellite therefore does no work.

71.momentum.png

72.momentum.png

73.momentum.png

2.gravitation.png

3.gravitation.png

6.gravitation.png

(B) Velocity

D
(D) Displacement
from the center of
(C) Kinetic energy mass

The gravitational force on an object is


the weight, and is proportional to the
mass. In the same
circular orbit, it is only the mass of the
body being orbited and the radius of the
orbit that
contributes to the orbital speed and
acceleration.
7.gravitation.png

(E) Gravitational
force

Newtons third law

(A) zero

(B) 1.0 m/s2

(C) 1.9 m/s2

(D) 3.7 m/s2

(E) 9.8 m/s2

(A) 16W

(B)4W

(C)W

(D)4

10 gravitation

If Spacecraft X has twice the mass of Spacecraft Y, then true statements


about X and Y include which of the
following?
. The two spheres pictured above have equal densities and are subject only
to their mutual gravitational attraction. Which of the following quantities must
have the same magnitude for both spheres?
The planet Mars has mass 6.4 1023 kilograms and radius 3.4 106 meters.
The acceleration of an object in free-fall near the surface of Mars is most
nearly
An object has a weight W when it is on the surface of a planet of radius R.
What will be the gravitational force on the object after it has been moved to a
distance of 4R from the center of the planet?

11 gravitation

A new planet is discovered that has twice the Earth's mass and twice the
Earth's radius. On the surface of this new planet, a person who weighs 500 N
on Earth would experience a gravitational force of
B

(A) 125N

(B) 250N

(C) 500N

12 gravitation

A simple pendulum and a mass hanging on a spring both have a period of 1 s


when set into small oscillatory motion on Earth. They are taken to Planet X,
which has the same diameter as Earth but twice the mass. Which of the
following statements is true about the periods of the two objects on Planet X
compared to their periods on
E

(A) Both are


shorter.

(B) Both are the


same.

(C) Both are


longer.

(D) 1000N
(D) The period of
the mass on the
spring is shorter,
that of the
pendulum is the
same.

13 gravitation

A satellite of mass m and speed v moves in a stable, circular orbit around a


planet of mass M. What is the radius of the satellite's orbit?

(A) GM/mv

(B)Gv/mM

(C)GM/v^2

(D)GmM/v

14 gravitation

The mass of Planet X is one-tenth that of the Earth, and its diameter is onehalf that of the Earth. The acceleration due to gravity at the surface of Planet
X is most nearly

(A) 2 m/s2

(B) 4 m/s2

(C) 5 m/s2

(D) 7 m/s2

15 gravitation

.A satellite travels around the Sun in an elliptical orbit as shown above. As the
satellite travels from point X to point Y. which of the following is true about its
speed and angular momentum?
D

(A)

(B)

(C)

(D)

16 gravitation

16. A newly discovered planet, "Cosmo," has a mass that is 4 times the mass
of the Earth. The radius of the Earth is
R . The gravitational field strength at the surface of Cosmo is equal to that at
the surface of the Earth if the e
radius of Cosmo is equal to
C

(A) .5Re

(B)Re

(C)2Re

(D) Sqrt Re

(A) 0 m/s2

(B) 0.05 m/s2

(C) 5 m/s2

(D) 9 m/s2

g = GM/r^2
Force is inversely proportional to
distance between the centers squared.
(E)1/16W
R 4 = F 16
g = GM/r^2 so the acceleration due to
gravity (and the weight of an object) is
proportional to the mass of the planet
and inversely proportional to the
distance from the center of the planet
squared. M 2 = g 2 and r 2 = g
4, so the net effect is the persons
(E) 2000N
weight is divided by 2
A planet of the same size and twice the
(E) The period of mass of Earth will have twice the
the pendulum is
acceleration due to gravity. The period
shorter; that of the of a mass on a spring has no
mass on the spring dependence on g, while the period of a
is the same.
pendulum is inversely proportional to g.
Orbital speed is found from setting
r
mv
r
GMm 2
2 = which gives
r
(E)GmM/v^2
v = GM
g = GM/r^2 so the acceleration due to
gravity (and the weight of an object) is
proportional to the
mass of the planet and inversely
proportional to the distance from the
center of the planet
squared. M 10 = g 10 and r 2 = g
(E) 10 m/s2
4, so the net effect is g 4/10
14.gravitation.png
Keplers second law (Law of areas) is
based on conservation of angular
momentum, which
remains constant. In order for angular
momentum to remain constant, as the
satellite approaches
(E)
the sun, its speed increases.
g = GM/r^2 so the acceleration due to
gravity (and the weight of an object) is
proportional to the
mass of the planet and inversely
proportional to the distance from the
center of the planet
squared. M 4 = g 4 and if the net
effect is g = gEarth then r must be twice
(E) Re^2
that of Earth.
g = GM/r^2 . 300 km above the surface
of the Earth is only a 5% increase in the
distance (1.05
times the distance). This will produce
(E) 11 m/s2
only a small effect on g (1.052)

a2 =4a1

a2 =2a1

a2 =a1

a2 =a1/2

a2 =a1/4

v/2

v/(2^1/2)

vo

(2v)^1/2

2vo

F1 is much greater F1 is slightly


than F2.
greater than F2.

F1 is equal to F2.

F2 is slightly
greater than F1

F2 is much greater
than F1
Newtons third law

7 gravitation

8 gravitation

9 gravitation

17 gravitation

18 gravitation

19 gravitation

20 gravitation

(A) Acceleration

The radius of the Earth is approximately 6,000 kilometers. The acceleration of


an astronaut in a perfectly circular orbit 300 kilometers above the Earth would
be most nearly
D
Two artificial satellites, 1 and 2, orbit the Earth in circular orbits having radii
R1 and R2, respectively, as shown above. If R2 = 2R1, the accelerations a2
and a1 of the two satellites are related by which of the following?
E

A satellite moves in a stable circular orbit with speed vo at a distance R from


the center of a planet. For this satellite to move in a stable circular orbit a
distance 2R from the center of the planet, the speed of the satellite must be
If F1 is the magnitude of the force exerted by the Earth on a satellite in orbit
about the Earth and F2 is the magnitude of the force exerted by the satellite
on the Earth, then which of the following is true?

8.gravitation.png

a=g=GM ,if R =2R then a =14a1


18.Gravitation.png
Orbital speed is found from setting
GMm = mv 2 which gives v = GM
where M is the B
r2rr object being orbited. If r is doubled,
v decreases by 2

21 gravitation

22 gravitation

A newly discovered planet has twice the mass of the Earth, but the
acceleration due to gravity on the new planet's surface is exactly the same as
the acceleration due to gravity on the Earth's surface. The radius of the new
planet in terms of the radius R of Earth is
C
A satellite S is in an elliptical orbit around a planet P, as shown above, with r1
and r2 being its closest and farthest distances, respectively, from the center
of the planet. If the satellite has a speed v1 at its closest distance, what is its
speed at its farthest distance?
A

12R

((2^1/2)*R)/2

(2^1/2)*R

(r1*v1)/R2

(r2*v1)/R1

(r2r2)v1

g = GM/(r^2) so the acceleration due to


gravity (and the weight of an object) is
proportional to the
mass of the planet and inversely
proportional to the distance from the
center of the planet
squared. M 2 = g 2 and if the net
effect is g = gEarth then r must be 2^
(1/2) times that of Earth

2R

((r1+r2)*v)/2
Both a constant
gravitational force
Only a decreasing Only an increasing Only a constant
that acts
gravitational force gravitational force gravitational force downward and a
that acts
that acts
that acts
decreasing force
downward
downward
downward
that acts upward
equal to the
equal to one-half
equal to one-fourth
at its maximum
acceleration at the the acceleration at the acceleration at
value for the ball's surface of the
the surface of the the surface of the
flight
asteroid
asteroid
asteroid

From conservation of angular


((r2r1)*v)/ (r2+r1) momentum v1r1= v2r2

25 gravitation

A ball is tossed straight up from the surface of a small, spherical asteroid with
no atmosphere. The ball rises to a height equal to the asteroid's radius and
then falls straight down toward the surface of the asteroid. What forces, if
any, act on the ball while it is on the way up?
A
A ball is tossed straight up from the surface of a small, spherical asteroid with
no atmosphere. The ball rises to a height equal to the asteroid's radius and
then falls straight down toward the surface of the asteroid. The acceleration of
the ball at the top of its path is:
D
A satellite of mass M moves in a circular orbit of radius R with constant speed
v. True statements about this satellite include which of the following?
I. Its angular speed is v/R.
II. Its tangential acceleration is zero.
III. The magnitude of its centripetal acceleration is constant.
E

26 gravitation

Two identical stars, a fixed distance D apart, revolve in a circle about their
mutual center of mass, as shown above. Each star has mass M and speed v.
G is the universal gravitational constant. Which of the following is a correct
relationship among these quantities?
B

27 gravitation

A spacecraft orbits Earth in a circular orbit of radius R, as shown above.


When the spacecraft is at position P shown, a short burst of the ship's
engines results in a small increase in its speed. The new orbit is best shown
by the solid curve in which of the following diagrams?

A)

B)

C)

D)

E)

28 gravitation

The escape speed for a rocket at Earth's surface is ve. What would be the
rocket's escape speed from the surface of a planet with twice Earth's mass
and the same radius as Earth?

2ve

(2)^ ve

ve

ve/(2)^

ve

29 gravitation

A hypothetical planet orbits a star with mass one-half the mass of our sun.
The planets orbital radius is the
same as the Earths. Approximately how many Earth years does it take for
the planet to complete one orbit?

1/(2)^

23 gravitation

24 gravitation

No forces act on
the ball.

Zero

I only

II only

I and III only

II and III only

I, II, and III

v^2 =GM/D

v^2 =GM/2D

v^2 =GM/D^2

v^2 =MGD

v^2 =2GM^2/D

1 2^

As the ball moves away, the force of


gravity decreases due to the increasing
distance
g = GM/(r^2) At the top of its path, it
has doubled its original distance from
the center of the
asteroid
Angular speed (in radians per second)
is v/R. Since the satellite is not
changing speed, there is
no tangential acceleration and v2/r is
constant.
The radius of each orbit is D, while
the distance between them is D. This
gives
2/
2
2
D
Mv
D
GMM
An burst of the ships engine produces
an increase in the satellites energy.
Now the satellite is
moving at too large a speed for a
circular orbit. The point at which the
burst occurs must remain
part of the ships orbit, eliminating
choices A and B. The Earth is no longer
at the focus of the
ellipse in choice E.
Escape speed with the speed at which
the kinetic energy of the satellite is
exactly equal to the
negative amount of potential energy
within the satellite/mass system. That is
r
GMm
mv =
2
2
1
which gives the escape speed
r
GM
Orbital speed is found from setting
r
mv
r
GMm 2
2
= which gives
r
GM
v = where M is the
object being orbited. Also,
v
r
T
2
= . Since the mass is divided by 2, v is
2 divided by 2

22.Gravitation.png

26.Gravitation.png

27.Gravitation.png
and 27. Gravitation.
png

30 gravitation

A hypothetical planet has seven times the mass of Earth and twice the radius
of Earth. The magnitude of the
gravitational acceleration at the surface of this planet is most nearly
C

2.9 m/s^2

5.7 m/s^2

17.5 m/s^2

35 m/s^2

122 m/s^2

31 gravitation

Two artificial satellites, 1 and 2, are put into circular orbit at the same altitude
above Earthssurface. The mass
of satellite 2 is twice the mass of satellite 1. If the period of satellite 1 is T,
what is the period of satellite 2?
C

T/4

T/2

2T

4T

32 gravitation

A planet has a radius one-half that of Earth and a mass one-fifth the Earths
mass. The gravitational acceleration
at the surface of the planet is most nearly

4.0 m/s^2

8.0 m/s^2

12.5 m/s^2

25 m/s2

62.5 m/s2

33 gravitation

In the following problem, the word weight refers to the force a scale
registers. If the Earth were to stop
rotating, but not change shape,

the weight of an
object at the
equator would
increase.

the weight of an
object at the
equator would
decrease.

the weight of an
object at the north
pole would
increase.

the weight of an
object at the north all objects on Earth
pole would
would become
decrease.
weightless.

34 gravitation

Assume that the Earth attracts John Glenn with a gravitational force F at the
surface of the Earth. When he
made his famous second flight in orbit, the gravitational force on John Glenn
while he was in orbit was closest
to which of the following?

0.95F

0.50F

0.25F

0.10F

zero

35 gravitation

What happens to the force of gravitational attraction between two small


objects if the mass of each object is
doubled and the distance between their centers is doubled?

It is doubled

It is quadrupled

It is halved

It is reduced
fourfold

It remains the
same

36 gravitation

One object at the surface of the Moon experiences the same gravitational
force as a second object at the surface
of the Earth. Which of the following would be a reasonable conclusion?

both objects would the object on the


fall at the same
Moon has the
acceleration
greater mass

the object on the


Earth has the
greater mass

both objects have


identical masses

the object on Earth


has a greater
mass but the Earth
has a greater rate
of rotation

37 gravitation

Astronauts in an orbiting space shuttle are weightless because

of their extreme
distance from the
earth

the net
gravitational force
on them is zero

there is no
atmosphere in
space

the space shuttle


does not rotate

they are in a state


of free fall

38 gravitation

Consider an object that has a mass, m, and a weight, W, at the surface of the
moon. If we assume the moon has a
nearly uniform density, which of the following would be closest to the objects
mass and weight at a distance
halfway between Moons center and its surface?
D

m&W

m&W

1m&1W

1m&W

1m&W

39 gravitation

The mass of a planet can be calculated if it is orbited by a small satellite by


setting the gravitational force on the
satellite equal to the centripetal force on the satellite. Which of the following
would NOT be required in this
calculation?

the mass of the


satellite

the radius of the


satellite's orbit

the period of the


satellite's orbit

Newton's universal all of the above are


gravitational
required for this
constant
calculation

g = so the acceleration due to gravity


(and the weight of an object) is
proportional to the
mass of the planet and inversely
proportional to the distance from the
center of the planet
squared. M 7 = g 7 and r 2 = g
4, so the net effect is g 7/4
Orbital speed is found from setting
r
mv
r
GMm 2
2
= which gives
r
GM
v = where M is the
object being orbited. Notice that
satellite mass does not affect orbital
speed or period.
g = so the acceleration due to gravity
(and the weight of an object) is
proportional to the
mass of the planet and inversely
proportional to the distance from the
center of the planet
squared. M 5 = g 5 and r 2 = g
4, so the net effect is g 4/5
Part of the gravitational force acting on
an object at the equator is providing the
necessary
centripetal force to move the object in a
circle. If the rotation of the earth were to
stop, this part
of the gravitational force is no longer
required and the full value of this force
will hold the
object to the Earth.
Standard orbital altitudes are not a
large percentage of the radius of the
Earth. The acceleration
due to gravity is only slightly smaller in
orbit compared to the surface of the
Earth.
F = . F is proportional to each mass and
inversely proportional to the distance
between
their centers squared. If each mass is
doubled, F is quadrupled. If r is doubled
F is quartered.
Since the acceleration due to gravity is
less on the surface of the moon, to
have the same
gravitational force as a second object
on the Earth requires the object on the
Moon to have a
larger mass.
Satellites in orbit are freely falling
objects with enough horizontal speed to
keep from felling
closer to the planet
The mass of an object will not change
based on its location. As one digs into a
sphere of uniform
density, the acceleration due to gravity
(and the weight of the object) varies
directly with distance
from the center of the sphere.
Combining
r
mv
r
GMm 2
2 = with
v
Tr
2 = gives the equation corresponding
to Keplers second
law. The mass of the satellite cancels in
these equations.

40 gravitation

41 gravitation

42 gravitation

As a rocket blasts away from the earth with a cargo for the international
space station, which of the following
graphs would best represent the gravitational force on the cargo versus
distance from the surface of the Earth?
E
In chronological order (earliest to latest), place the following events:
(1) Henry Cavendish's experiment
(2) Newton's work leading towards the Law of Universal Gravitation
(3) Tycho Brahe takes astronomical data
(4) Nicolaus Copernicus proposes the heliocentric theory
(5) Johannes Kepler's work on the orbit of Mars.
A
A 20 kg boulder rests on the surface of the Earth. Assume the Earth has
mass 5.98 1024 kg and g = 10 m/s2
What is the magnitude of the gravitational force that the boulder exerts on the
Earth?
E

43521

5.98x10^25 N

43 gravitation

An astronaut on the Moon simultaneously drops a feather and a hammer. The


fact that they reach the surface at
the same instant shows that
E

no gravity forces
act on a body in a
vacuum.

44 gravitation

A scientist in the International Space Station experiences "weightlessness"


because

there is no
gravitational force
from the Earth
acting on her.

45 gravitation

A rocket is in a circular orbit with speed v and orbital radius R around a heavy
stationary mass. An external
impulse is quickly applied to the rocket directly opposite to the velocity and
the rockets speed is slowed to v/2,
putting the rocket into an elliptical orbit. In terms of R, the size of the semimajor axis a of this new elliptical
orbit is
E

46 gravitation

Keplers Second Law about sweeping out equal areas in equal time can be
derived most directly from which
conservation law?

47 gravitation

48 gravitation

= 1
4

42135

43251

5.98x10^24N

20N
the gravitational
force from the
the acceleration
Moon on heavier
due to gravity on
objects (the
the Moon is less
hammer) is equal
than the
to the gravitational
acceleration due to force on
gravity on the
lighter objects (the
Earth
feather).

= 7
11

35421

= 8
3

energy

angular
mechanical energy momentum

mass

Three equal mass satellites A, B, and C are in coplanar orbits around a planet
as shown in the figure. The
magnitudes of the angular momenta of the satellites as measured about the
planet are LA, LB, and LC. Which of
the following statements is correct?
A

LA > LB > LC

LC > LB > LA

LB > LA > LC

What is the value of the gravitational potential energy of the two star system? D

3 2

LB > LC > LA

2
2

3 2

40.gravitation.png

NO, this is not part of the curriculum,


but interesting to know (and a bit of
common sense if you
42351 follow the changing of the theories)

The boulder exerts


no force on the
Earth
200 N

a hammer and
feather have less
mass on the Moon
than on Earth.
at an orbit of 200
miles above the
Earth, the
the gravitational
gravitational force
pull of the Moon
she is in free fall
of the Earth on her
has canceled the along with the
is 2% less than on
pull of the Earth on Space Station and its
her.
its contents.
surface.

= 1
2

r2
GMm
F = so F is proportional to 1/r2
. Standard orbital altitudes are not a
large percentage of
the radius of the Earth. The
acceleration due to gravity is only
slightly smaller in orbit compared
to the surface of the Earth.

Gravitational force is also the weight.


mg.

in the absence of
air resistance all
bodies at a given
location fall with
the same
acceleration.

g is the same for all bodies in the


absence of air resistance

in space she has


no mass.

Satellites in orbit are freely falling


objects with enough horizontal speed to
keep from felling
closer to the planet.

= 4
7

The energy of a circular orbit is K+U=


(1/2)mv^2 + (-Gmm/r)= (1/2)m((GMm/r)
^1/2)^2 + (-GMm/r)= -GMm/2r. The
energy of
an elliptical orbit is (GMm/2a) where a
is the semimajor axis. If the speed is
cut in half we have K+U= (1/2)m(v/2)^2
+ (-Gmm/r)=(1/2)m((1/2)(GM/r)^1/2))^2
+
(-Gmm/r) = -(7GMm/8r). Setting (7
GMm/8r)= (-GMm/2a) gives a= (4/7)r

Sweeping out equal areas in based on


the satellite moving faster as it moves
closer to the body it
is orbiting. This is a result of
linear momentum conservation of angular momentum.
The angular momentum of each
satellite is conserved independently so
we can compare the
orbits at any location. Looking at the
common point between orbit A and B
shows that satellite
The relationship
A is moving faster at that point than
between the
satellite B, showing LA > LB. A similar
magnitudes is
analysis at the
different at various common point between B and C shows
instants in time.
LB > LC
47.gravitation.png

.
2
r
GMm
2
U =
48.gravitation.png

49 gravitation

Determine the period of orbit for the star of mass 3M

3
3

Since they are orbiting their center of


mass, the larger mass has a radius of
orbit of d
4
1
. The
speed can be found from
4
2
)3(
2
)3(
d
vM
d
MMG
= which gives
T
d
d
GM
v
)
4

49.gravitation.png

The energy of a circular orbit is


r
GMm
r
GMm
r
GM
m
r
Gmm
mvUK
2
)(
2
2
1
)(
2
2
1
=+=+ =+

The energy of an elliptical orbit is


a
GMm
2
where a is the semimajor axis. If the
speed is cut in
half we have
r
GMm
r
GMm
r
GM
m
r
v
Gmm
mUK
8
7
)(
2
2
1
2
1
)(
2
22
1
=+ =+ =+

50 gravitation

Two satellites are launched at a distance R from a planet of negligible radius.


Both satellites are launched in the
tangential direction. The first satellite launches correctly at a speed v
0 and enters a circular orbit. The second
satellite, however, is launched at a speed v
0.What is the minimum distance between the second satellite and
the planet over the course of its orbit?
E

1
2

1
2

1
3

1
4

1
7

Setting
a
GMm
r
GMm
82
7
= gives a = (4/7)r
The distance to the planet from this
point is r (the radius of the circular orbit
and aphelion for the

51 gravitation

Two satellites are launched at a distance R from a planet of negligible radius.


Both satellites are launched in the
tangential direction. The first satellite launches correctly at a speed v
0 and enters a circular orbit. The second
satellite, however, is launched at a speed v
0.What is the minimum distance between the second satellite and
the planet over the course of its orbit?
E
When two stars are very far apart, their gravitational potential energy is zero;
when they are separated by a
distance d, the gravitational potential energy of the system is U. What is the
gravitational potential energy of the
system if the stars are separated by a distance 2d?
B

52 gravitation

The gravitational force on a textbook at the top of Pikes Peak (elevation


14,100 ft) is 40 newtons. What would
be the approximate gravitational force on the same textbook if it were taken to
twice the elevation?
D

53 gravitation

Assume the International Space Station has a mass m and is in a circular


orbit of radius r about the center of the
Earth. If the Earth has a mass of M, what would be the speed of the Space
Station around the Earth?

54 gravitation

What would be the gravitational force of attraction between the proton in the
nucleus and the electron in an
orbit of radius 5.3 1011 m in a simple hydrogen atom?

2.0 1057 N

3.7 1047 N

6.1 1028 N

8.2 108 N

2F

4F

6F

8F

50 gravitation

55 gravitation

56 gravitation

57 gravitation

58 gravitation

59 gravitation

1 oscillations

2 oscillations

3 oscillations

4 oscillations

1
2

1
3

1
4

U/4

U/2

2U

5N

10 N

20 N

40 N

Two iron spheres separated by some distance have a minute gravitational


attraction, F. If the spheres are moved
to one half their original separation and allowed to rust so that the mass of
each sphere increases 41%, what would be the resulting gravitational force? D
A ball which is thrown upward near the surface of the Earth with a velocity of
50 m/s will come to rest about 5
seconds later. If the ball were thrown up with the same velocity on Planet X,
after 5 seconds it would be still
moving upwards at nearly 31 m/s. The magnitude of the gravitational field
near the surface of Planet X is what
fraction of the gravitational field near the surface of the Earth?
B
An object placed on an equal arm balance requires 12 kg to balance it. When
placed on a spring scale, the scale
reads 120 N. Everything (balance, scale, set of masses, and the object) is
now transported to the moon where the
gravitational force is one-sixth that on Earth. What are the new readings of
the balance and the spring scale
(respectively)?
A
Two artificial satellites I and II have circular orbits of radii R and 2R,
respectively, about the same planet. The
orbital velocity of satellite I is v. What is the orbital velocity of satellite II?
The gravitational acceleration on the surface of the moon is 1.6 m/s^2. The
radius of the moon is 1.7 10^6 m. What is the period of a satellite placed in
a low circular orbit about the moon?
A mass m, attached to a horizontal massless spring with spring constant k, is
set into simple harmonic motion.
Its maximum displacement from its equilibrium position is A. What is the
masss speed as it passes through its
equilibrium position?
The period of a spring-mass system undergoing simple harmonic motion is T.
If the amplitude of the springmass
systems motion is doubled, the period will be:
A simple pendulum of mass m and length L has a period of oscillation T at
angular amplitude = 5 measured
from its equilibrium position. If the amplitude is changed to 10 and everything
else remains constant, the new
period of the pendulum would be approximately.
A mass m is attached to a spring with a spring constant k. If the mass is set
into simple harmonic motion by a
displacement d from its equilibrium position, what would be the speed, v, of
the mass when it returns to the
equilibrium position?

1
2

1
7

.
r
GMm
4U
U =
The top of Pikes Peak is a very small
fraction of the radius of the Earth.
Moving to twice this
elevation will barely change the value of
80 N
g
Orbital speed is found from setting
r
mv
r
GMm 2
2
= which gives
r
GM
v = where M is the

object being orbited.
.
2
r
GMm
F = . The masses of the proton and
the only force of
electron can be found in the table of
attraction would be constants (these
electrical
masses do not need to be memorized)
2
r
GMm
F = ; If r 2, F 4. If each mass is
multiplied by 1.41, F is doubled (1.41
10 F
1.41)

0.16

0.39

0.53

Setting
a
GMm
r
GMm
82
7
= gives a = (4/7)r
The distance to the planet from this
point is r (the radius of the circular orbit
and aphelion for the
elliptical orbit). The opposite side of the
ellipse is 2a away, or 8r/7, making the
distance to the
planet at perihelion 8r/7 r = r/7

g = v/t = (31 m/s 50 m/s)/(5 s) = 3.8


m/s B
1.59 2

0.63

12 kg, 20 N

1 kg, 120 N

12 kg, 720 N

2 kg, 20 N

2 kg, 120 N

.5v

v/(sqrt(2))

(sqrt(2))v

2v

mass is unchanged, weight is changed


due to a change in the acceleration due
to gravity
Orbital speed is found from setting
GMm/(r^2)=mv^2/r which gives v =
(GM/r)^(1/2) where M is the
object being orbited.

1.0x10^3 s

6.5x10^3 s

1.1x10^6 s

5.0x10^6 s

7.1x10^12 s

g = v^2/r and v = 2r/T

.25T

.5T

2T

4T

Energy conservation. Usp = K, so k


A^2 = mv^2
1.oscillations.png
The period of a mass-spring is only
affected by mass and k so its stays the
same

2T

(sqrt(2))T

T/(sqrt(2))

.5T

The period of a pendulum is only


affected by length and g so it stays
the same

Energy conservation. Usp = K, so k


d^2 = mv^2

4.oscillations.png

5 oscillations
6 oscillations
7 oscillations

8 oscillations
9 oscillations

A mass on the end of a spring oscillates with the displacement vs.


time graph shown. Which of the following statements about its
motion is INCORRECT?
What is the period of a simple pendulum if the cord length is 67 cm and the
pendulum bob has a mass of 2.4 kg.
If the mass of a simple pendulum is doubled but its length remains constant,
its period is multiplied by a factor of

Which of the following is true for a system consisting of a mass oscillating on


the end of an ideal spring?
The length of a simple pendulum with a period on Earth of one second is
most nearly

The mass
experiences a
maximum in the
The amplitude of
The frequency of
size of the
the oscillation is
oscillation is 0.5
acceleration at t=1. The amplitude from the graph is 0.04
0.08 m.
Hz.
5 sec
not 0.08, the rest are true
5.oscillations.png
The mass is irrelevant, only the length
.259s
1.63s
3.86s
16.3s
24.3s
matters. T is found with 2 (L/g)
Mass does not affect the period, only
0.5 1/(sqrt(2))
1 (sqrt(2))
2 the length matters
The maximum
The maximum
kinetic energy
potential energy is The maximum
occurs at
achieved when the kinetic energy and maximum
The kinetic and
mass passes
maximum potential displacement of
Energy is conserved here and switches
potential energies The kinetic and
through its
energy are equal, the mass from its between kinetic and potential which
are equal to each potential energies equilibrium
but occur at
equilibrium
have maximums at
other at all times. are both constant. position.
different times.
position
different locations

0.12m

0.25m

0.50m

1.0m

A
B
C

The mass
achieves a
maximum in speed The period of
at 1 sec.
oscillation is 2 sec.

The maximum
kinetic energy is
attained as the
sphere passes
through its
equilibrium
position

C
The minimum
gravitational
potential energy is
The maximum
attained as the
kinetic energy is
sphere passes
attained as the
through its
sphere reaches its equilibrium
point of release.
position.

Sub into T = 2 (L/g) and solve for L


Only conservative forces are acting
which means mechanical energy must
be conserved so it stays constant as
E
the mass oscillates
The box momentarily stops at x(min)
and x(max) so must have zero K at
these points. The box accelerates the
most at the ends of the oscillation since
the force is the greatest there. This
changing acceleration means that the
box gains speed quickly at first but not
as quickly as it approaches equilibrium.
This means that the KE gain starts of
rapidly from the endpoints and
gets less rapid as you approach
equilibrium where there would be a
maximum speed and maximum K, but
zero force so less gain in speed. This
E
results in the curved graph.
Pendulum is unaffected by mass.
Mass-spring system has mass causing
the T to change proportional to m so
since the mass is doubled the period is
E.
changed by 2
At T/4 the mass reaches maximum +
displacement where the restoring force
is at a maximum and pulling in the
opposite direction and hence creating a
negative acceleration. At
maximum displacement the mass stops
v = 0, a = 0
momentarily and has zero velocity
At T/4 the mass reaches maximum +
displacement where the restoring force
is at a maximum and pulling in the
opposite direction and hence creating a
negative acceleration. At
maximum displacement the mass stops
E
momentarily and has zero velocity
Positive Acceleration occurs when the
mass is at negative displacements
since the force will be acting A in the
opposite direction of the displacement
to restore equilibrium. Based on F=kx
None of the above, the
because the
most force, and therefore the most
acceleration is
acceleration occurs where the most
constant
displacement is
As the object oscillates, its total
mechanical energy is conserved and
transfers from U to K back and forth.
The only graph that makes sense to
E
have an equal switch throughout is D

The maximum
gravitational
potential energy is
attained when the
sphere reaches its
point of release.

The maximum total


energy is attained
only as the sphere
passes through its
equilibrium
position.

For the spring, equilibrium is shown


where the maximum transfer of kinetic
energy has occurred and likewise for
the pendulum the bottom equilibrium
position has the maximum transfer of
potential energy into spring energy.

L / m1g

g/m2L

m2L/g

m1g/L

Set period formulas equal to each other


and rearrange for k
18.oscillations.png

10 oscillations

Which graph can represent the total mechanical energy of the block-spring
system as a function of x ?

11 oscillations

Which graph can represent the kinetic energy of the block as a function of x ? D

12 oscillations

An object swings on the end of a cord as a simple pendulum with period T.


Another object oscillates up and down on the end of a vertical spring also with
period T. If the masses of both objects are doubled, what are the
new values for the Periods?
B

A.

B.

C.

D.

13 oscillations

An object is attached to a spring and oscillates with amplitude A and period T,


as represented on the graph. The nature of the velocity v and acceleration a
of the object at time T/4 is best represented by which of the following?
D

v > 0, a > 0

v > 0, a < 0

v > 0, a = 0

v = 0, a < 0

14 oscillations

When an object oscillating in simple harmonic motion is at its maximum


displacement from the equilibrium position. Which of the following is true of
the values of its speed and the magnitude of the restoring force?

15 oscillations

A particle oscillates up and down in simple harmonic motion. Its height y as a


function of time t is shown in the diagram. At what time t does the particle
achieve its maximum positive acceleration?
A

1 second

2 seconds

3 seconds

4 seconds

16 oscillations

The graph shown represents the potential energy U as a function of


displacement x for an object on the end of a spring oscillating in simple
harmonic motion with amplitude x. Which of the following graphs represents
the kinetic energy K of the object as a function of displacement x ?
D

17 oscillations

18 oscillations

Which of the following is true for both spheres?


If both spheres have the same period of oscillation, which of the following is
an expression for the spring
constant

m1L/g

10.0m

10.oscillations.png

11.oscillations.png

12.oscillations.png

13.oscillations.png

14.oscillations.png

15.oscillations.png

16.oscillations.png

17.oscillations.png

19 oscillations

A block attached to the lower end of a vertical spring oscillates up and down.
If the spring obeys
Hooke's law, the period of oscillation depends on which of the following?
I. Mass of the block
II. Amplitude of the oscillation
III. Force constant of the spring

I only

II only

Both are shorter.

Both are the same. Both are longer.

In a mass-spring system, both mass


and spring constant (force constant)
I and II
I and III
affect the period.
The mass spring system in unaffected
because the attached mass and spring
constant are the
The period of the The period of the same. Based on g = GMp / R^2 and the
mass on the spring pendulum is
given values, g on planet X would be
is shorter; that of
shorter; that of the greater. Using
the pendulum is
mass on the spring the pendulum period formula, larger g
the same.
is the same
means smaller period.

T/4

T/2

2T

4T

III only

21 oscillations

A simple pendulum and a mass hanging on a spring both have a period of 1 s


when set into small oscillatory
motion on Earth. They are taken to Planet X, which has the same diameter as
Earth but twice the mass. Which of the
following statements is true about the periods of the two objects on Planet X
compared to their periods on Earth?
E
A simple pendulum of length l, whose bob has mass m, oscillates with a
period T. If the bob is replaced by one
of mass 4m, the period of oscillation is
C

22 oscillations

What is the amplitude, in meters, of the resulting simple harmonic motion of


the block?

m/40

m/20

m/4

m/2

23 oscillations

What will the resulting period of oscillation be?

/40s

/20s

/10s

/5s

/4ss

24 oscillations

A ball is dropped from a height of 10 meters onto a hard surface so that the
collision at the surface may be
assumed elastic. Under such conditions the motion of the ball is

simple harmonic
with a period of
about 1.4 s

simple harmonic
with a period of
about 2.8 s

simple harmonic
with an amplitude
of 5 m

periodic with a
period of about 2.8 motion with
s but not simple
constant
harmonic
momentum

20 oscillations

26 oscillations

Which of the following graphs shows the kinetic energy K of the particle as a
function of time t for one cycle of
motion?
Which of the following graphs shows the kinetic energy K of the particle as a
function of its displacement x ?

27 oscillations

A mass m is attached to a vertical spring stretching it distance d. Then, the


mass is set oscillating on a spring
with an amplitude of A, the period of oscillation is proportional to

25 oscillations

Mass does not affect the period of a


pendulum
At the current location all of the energy
is gravitational potential. As the spring
stretches to its
max location all of that gravitational
potential will become spring potential
when it reaches
its lowest position. When the box
oscillates back up it will return to its
original location
converting all of its energy back to
gravitational potential and will oscillate
back and forth
between these two positions. As such
the maximum stretch bottom location
represents twice
the amplitude so simply halving that
max x will give the amplitude. Finding
the max
stretch: The initial height of the box h
and the stretch x have the same value
(h=x)
U = Usp mg(x1) = kx1
2 mg = k x1 x1 = .05 m.
This is 2A, so the amplitude is 0.025 m
or 1/40 m.
Alternatively, we could simply find the
equilibrium position measured from the
initial top
position based on the forces at
equilibrium, and this equilibrium stretch
measured from the
top will be the amplitude directly. To do
this:
Fnet = 0 Fsp = mg kx2 = mg x2 =
0.025 m, which is the amplitude
22.oscillations.png
Plug into period for mass-spring system
T = 2 (m/k)
22.oscillations.png
Based on free fall, the time to fall down
would be 1.4 seconds. Since the
collision with the
ground is elastic, all of the energy will
be returned to the ball and it will rise
back up to its
initial height completing 1 cycle in a
total time of 2.8 seconds. It will continue
doing this
oscillating up and down. However, this
is not simple harmonic because to be
simple
harmonic the force should vary directly
proportional to the displacement but
that is not the
case in this situation
Energy will never be negative. The max
kinetic occurs at zero displacement and
the kinetic
energy become zero when at the
maximum displacement
25.oscillations.png
Same reasoning as above, it must be C 26.oscillations.png
First use the initial stretch to find the
spring constant. Fsp = mg = kx k = mg
/d
Then plug that into T = 2 (m/k)
27.oscillations.png

28
29
30
31
32
33
34
35
36
37
38

oscillations
oscillations
oscillations
oscillations
oscillations
oscillations
oscillations
oscillations
oscillations
oscillations
oscillations

Two objects of equal mass hang from independent springs of unequal spring
constant and oscillate up and down.
The spring of greater spring constant must have the

smaller amplitude
of oscillation

larger amplitude of shorter period of


oscillation
oscillation

longer period of
oscillation

lower frequency of Based on T = 2 (m/k) the larger


oscillation
spring constant makes a smaller period

39 oscillations

An object of mass m hanging from a spring of spring constant k oscillates with


a certain frequency. What is the
length of a simple pendulum that has the same frequency of oscillation?
B

mk/g

mg/k

kg/m

k/mg

g/mk

40 oscillations

A platform of mass 2 kg is supported by a spring of negligible mass as


shown.
The platform oscillates with a period of 3 s when the platform is pushed down
and released. What must be the mass of a block that when placed on the
platform doubles the period of oscillation to 6 s?
D

1kg

2kg

4kg

6kg

8kg

0 mg
a numerical
arrangement
pressure on a
where each
confined liquid is
number is the sum
transmitted equally of the two numbers
in all directions
above

2mg

3mg

4mg

two electrons
cannot occupy the
the same quantum
state at the same
time

the volume of a
gas is directly
reated to its
temperature

16300kg

17300kg

18300kg

19300kg

1/4.

1/2.

1 Fluids

2 Fluids

3 Fluids

4 Fluids

A cork has weight mg and density 25% of water density. A string is tied
around the cork and attached to the bottom of a water-filled container. The
cork is totally immersed. Express in terms of the cork weight mg, the tension
in the string

Which of the following is the best statement of Pascals Law?

When submerged under water, the apparent mass of one cubic meter of pure
gold is 18300 kg. What would be its mass in air?
D
An ideal fluid flows through a long horizontal circular pipe. In one region of the
pipe, it has radius R. The pipe
then widens to radius 2R. What is the ratio of the fluids speed in the region of
radius R to the speed of the fluid
in region with radius 2R
E

Since the frequencies are the same, the


periods are also the same. Set the
period for the
mass-spring system equal to the period
for the string pendulum and rearrange
for L.
Based on T = 2 (m/k), in order to
double the period, the mass would have
to be increased by
4x the original amount. Here is the
tricky part you are to increase the
mass to 4 x its
original value by adding mass the to
2kg tray. So to make the total mass
have a value of
8 kg, only 6 kg of extra mass would
need to be added to the tray.
40.oscillations.PNG
FBD has Ft pointing down Fb pointing
up and weight (mg) down. Fnet = 0 Fb
Ft
The buoyant force is given by the
weight of the displaced water. Since the
waters displaced
volume is equal to the corks displaced
volume and the water weight for the
same volume
would be 4 times heavier (based on the
given cork weight = 25% water weight)
compared to
the cork, the buoyant force is equal to 4
x the cork weight = 4mg. Using the
force equation
created initially. F
mg = 0
t = Fb mg = 4mg mg = 3mg
1.fluids.PNG

the farther away a


galaxy is the faster
it is receding
Definition of Pascal's principle
A 1 m3 volume cube under water
displaces 1m3 of water. This weight of
water = pVg =
1000(1)(10) = 10000 N which is
equivalent to the buoyant force. The
apparent weight in
water is mappg = 18300(10) = 183000
N. This apparent weight is lessened by
the buoyant
force pulling up with 10000 N of force.
So outside of the water, this upwards
force would
not exist and the actual weight would
be 193000 N which equal 19300 kg of
20300kg
mass.

Using fluid continuity. A1v1 = A2v2


4 R2v1 = (2R)2v2 v1 = 4 v2

5 Fluids

A fluid is forced through a pipe of changing cross section as shown. In which


section would the pressure of the
fluid be a minimum

II

III

IV

all sections have


same pressure

6 Fluids

Three fishing bobbers all float on top of water. They have the following
relationships:
-A,B: same mass, same density, different shapes
-B,C: same size, same shape,
mass & density C < mass & density B
Three identical weights are tied to each bob, and each is pulled completely
beneath the water. Which bob will
displace the greatest amount of water

A and B

All displace the


same amount of
water.

Pascal's

Bernoulli's

Archimedes'

Huygens'

Newton's

25N

500N

10000N

40000N

It increases.

It remains
constant.

It decreases.

It may increase or
decrease,
depending on the
shape of the rock.

The iron.

Both have the


same buoyant
force.

Cannot be
determined without
knowing their
densities.

8 Fluids

A hydraulic press allows large masses to be lifted with small forces as a result
of which principle?
A
A 500 N weight sits on the small piston of a hydraulic machine. The small
piston has an area of 2 cm2 . If the
large piston has an area of 40 cm2, how much weight can the large piston
support?
C

9 Fluids

As a rock sinks deeper and deeper into water of constant density, what
happens to the buoyant force on it?

7 Fluids

This is based on two principles. 1


Bernoullis principle says that when
speed increases
pressure drops. Second, continuity
says more area means less speed
based on A1v1 = A2v2
So the smallest area would have the
largest speed and therefore most
pressure drop.
5.fluids.PNG
Since A and B have the same mass
and density, they have the same
volume. C has the same
volume as A and B since its the same
shape as B. So all three objects have
the same
volume. When submerged, they will all
displace the same amount of water and
therefore all
have the same buoyant force acting on
them. Note: if the objects were floating
instead of
submerged than the heavier ones
would have larger buoyant forces.
Pascals principle of equal pressure
transfer in a fluid allows for hydraulic
lifts to function.

10 Fluids

50 cm^3 of wood is floating on water, and 50 cm^3 of iron is totally


submerged. Which has the greater buoyant
force on it?

11 Fluids

Salt water is more dense than fresh water. A ship floats in both fresh water
and salt water. Compared to the
fresh water, the amount of water displaced in the salt water is

more.

less.

the same.

Cannot be
determined from
the information
given.

12 Fluids

A liquid has a specific gravity of 0.357. What is its density?

357 kg/m3

643 kg/m3

1000 kg/m3

3570 kg/m3

Point A

Point B

Same at both A
and B

Cannot be
determined from
the information
given.

Pascals principle says P1 = P2 F1/A1 =


F2/A2 F2 = F1A2 / A1 = 500(40)/(2)
Buoyant force is equal to weight of
displaced fluid. Since the density is
constant and the volume
displaced is always the same, the
buoyant force stays constant
The wood is floating and is only partially
submerged. It does not displace a
weight of water
related to its entire volume. The iron
however is totally submerged and does
displace a
weight of water equal to its entire
volume. Since the iron displaces more
water, it has a
larger buoyant force acting on it.
For floating objects, the weight of the
displaced fluid equals the weight of the
object. For a more
dense fluid, less of that fluid needs to
be displaced to create a fluid weight
equal to the
weight of the object. Since the salt
water is more dense, it will not need as
much displaced.
Definition of specific gravity. s.g = x /
H20
This is based on two principles. 1
Bernoullis principle says that when
speed increases
pressure drops. Second, continuity
says more area means less speed
based on A1v1 = A2v2
So the smallest area would have the
largest speed and therefore most
pressure drop. Moving to more area ->
less speed -> more pressure

0.25 m/s

0.50 m/s

2 m/s

4 m/s

Flow continuity. A1v1 = A2v2 (0.02)^2


(1) = (0.01)^2(v2)

B1 < B2 = B3

Buoyant force is based on how much


weight of water is displaced. Since all
three are completely
submerged they all displace the same
amount of water so have equal buoyant
forces.

13 Fluids

14 Fluids

15 Fluids

Water flows through a pipe. The diameter of the pipe at point B is larger than
at point A. Where is the water
pressure greater?
B
Liquid flows through a 4 cm diameter pipe at 1.0 m/s. There is a 2 cm
diameter restriction in the line. What is
the velocity in this restriction?
D
A copper block is connected to a string and submerged in a container of
water.
Position 1: The copper is completely submerged, but just under the surface of
the water.
Position 2: The copper is completely submerged, mid-way between the water
surface and the bottom of the
container.
Position 3: The copper is completely submerged, but just above the bottom
surface of the container.
Assume that the water is incompressible. What is the ranking of the buoyant
forces (B) acting on the copper
blocks for these positions, from least to greater?
C

The wood.

B1 < B2 < B3

B3 < B2 < B1

B1 = B2 = B3

B3 < B1 = B2

16 Fluids

17 Fluids
18 Fluids

19 Fluids

20
21
22
23
24
25
26
27
28

Fluids
Fluids
Fluids
Fluids
Fluids
Fluids
Fluids
Fluids
Fluids

Two objects labeled K and L have equal mass but densities 0.95Do and Do,
respectively. Each of these objects
floats after being thrown into a deep swimming pool. Which is true about the
buoyant forces acting on these
objects?
A driveway is 22.0 m long and 5.0 m wide. If the atmospheric pressure is 1.0
x 10^5 Pa, How much force does
the atmosphere exert on the driveway?

The buoyant force


is greater on
Object K since it
has a lower
density and
displaces more
water.

The buoyant force


is greater on
Object K since it
has lower density
and lower density
objects always
float
higher in the
fluid.

The buoyant force


is greater on
Object L since it is
denser than K and
therefore heavier.

The buoyant
forces are equal
on the objects
since they have
equal mass.

For floating objects, the buoyant force


equals the weight of the objects. Since
each object has the
same weight, they must have the same
buoyant force to counteract that weight
and make
them float. IF the equal mass objects
sunk, then the one with the smaller
Without knowing
density would have
the specific gravity a larger volume and displace more
of the objects,
water so have a larger buoyant force.
nothing can be
But that is not the
determined.
case here.

9.09 x 10^8 N

1.1 x 10^-3 N

909 N

4545 N

1.1 x 10^7 N

kg/m^2

kg/(ms)

kg/s^2

kg/(ms^2)

(ms)/kg

2.010^4 m^3

3.010^-4
m^3

5.010^-4 m^3

8.010^4 m^3

1.010^-3m^3

Which of the following could be a correct unit for pressure?


D
A block is connected to a light string attached to the bottom of a large
container of water. The tension in the string is 3.0 N. The gravitational
force from the earth on the block is 5.0 N. What is the blocks volume?
A cube of unknown material and uniform density floats in a container of water
with 60% of its volume
submerged. If this same cube were placed in a container of oil with density
800 kg/m,what portion of the
cubes volume would be submerged while floating?

33%

50%

58%

67%

P = F / A 1x10^5 = F / (22*5)
P = F / A = ma / A = kg (m/s^2) / m^2 =
kg / (ms^2)

19.fluids.png

75%
21.fluids.png
22.fluids.png
23.fluids.png
24.fluids.png
25.fluids.png
27.fluids.png
28.fluids.png

29 Fluids

One cubic centimeter of iron (density ~7.8 g/cm and 1 cubic centimeter of
aluminum (density ~2.7 g/cm) are dropped into a pool. Which has the largest
buoyant force on it?
C

iron

aluminum

neither has a
both are the same bouyant force on it

30 Fluids

One kilogram of iron (density ~7.8 g/cm3) and 1 kilogram of aluminum


(density ~2.7 g/cm3 are dropped into a
pool. Which has the largest buoyant force on it?

iron

aluminum

neither has a
both are the same bouyant force on it

31 Fluids

Find the approximate minimum mass needed for a spherical ball with a 40 cm
radius to sink in a liquid of
density 1.4x103 kg/m
B

37.5kg

375kg

3750kg

37500kg

357000kg

Both object are more dense than water


and will sink in the pool. Since both
have the same
volume, they will displace the same
amount of water and will have the same
buoyant forces.
Again both samples sink. Also, both
samples have the same mass but
different densities. For the
same mass, a smaller density must
have a larger volume, and the larger
volume displaces
more water making a larger buoyant
force. So the smaller density with the
larger volume has
a larger buoyant force.
V of this ball is 4/3 r3 = 4/3 (0.4)3 =
0.2681 m3. For the ball to just sink, it is
on the verge of
floating, meaning the weight of the ball
equals the buoyant force of the fully
submerged ball.
mg = fl Vdisp g m (10) = 1400
(0.2681) (10) m = 375 kg
This object will float, so mobjg = Fb
objVobj g = ocean Vdisp g
(0.95x103)(V)(10) = (1.1x103)(x% V)
(10)
Gives x% = 0.86 but that is the amount
submerged, so the visible about would
87% be 1 0.86

33 Fluids

What vertical percentage of a 0.25 m deep sheet of ice, whose density is 0.95
x103
kg/m3, will be visible in an ocean whose density is 1.1x103 kg/m
A
The idea that the velocity of a fluid is high when pressure is low and that the
velocity of a fluid is low when the
pressure is high embodies a principle attributed to
E

Torricelli

Pascal

Galileo

Archimedes

Bernoulli

34 Fluids

The mass of a 1.3 m3 object with a specific gravity of 0.82 is

630kg

730kg

820kg

1100kg

1600kg

180N

250N

480N

1300N

4700N

Statement associated with Bernoullis


principle
s.g = obj / h20 0.82 = obj / 1000
obj = 820 then = m/V 820 = m /
1.3
The apparent weight is the air weight
the upwards buoyant force. The
buoyant force is given
by Fb = fl Vdisp g = 1.25x103 (0.375)
(10) = 4687.5 N.
The apparent weight is then (600)(10)
4687.5 = 1312.5 N

0.02V

0.11V

7V

49V

Using fluid continuity. A1v1 = A2v2


(7R)2v1 = (R)2V v1 = V / 49

32 Fluids

35 Fluids

36 Fluids

The apparent weight of a 600 kg object of volume 0.375 m3 submerged in a


liquid of density 1.25x103 kg/m3
D
A conduit of radius 7R carries a uniformly dense liquid to a spigot of radius R
at the same height, where it has a
velocity of V. What is its initial velocity
A

14%

34%

58%

71%

37 Fluids
38 Fluids

39 Fluids
40 Fluids

41 Fluids

42 Fluids

43 Fluids

44 Fluids

45 Fluids

46 Fluids

47 Fluids

1 Thermo

2 Thermo

The pressure in a pipe carrying a liquid with a density of and an initial


velocity v at the inlet is P, which is y
meters lower than its outlet, which has a velocity of 2v. In these terms, what is
the final pressure?
The units of specific gravity are
The buoyant force on an object is equal to the weight of the water displaced
by a submerged object. This is a
principle attributed to
If the gauge pressure of a device reads 2.026x105 N/m2, the absolute
pressure it is measuring is

B
E

kg/m3

g/m3

m/s2

N/m

The fluid flow is occurring in a situation


similar to the diagram for question #27.
Apply Bernoullis equation. P1 + gy1 +
v1
2 = P2 + gy2 + v2
2
P + 0 + v2 = P2 + gy + (2v)2
P2 = P + v2 4v2 gy = P
3/2 v2
gy
37.fluids.PNG
none of the above s.g is density / density and has no units

Torricelli

Pascal

Galileo

Archimedes

Bernoulli

1.013x10^5 N/m2

2.052x10^5 N/m2

2.026x10^5 N/m2

3.039x10^5 N/m2

experiences a
buoyant force
equal to L gV.

experiences a
buoyant force
equal to mB g

6.078x10^5 N/m2
does not
experience any
buoyant force,
because B> L

A block of mass m, density B , and volume V is completely submerged in a


liquid of density L. The density of
the block is greater than the density of the liquid. The block
C
A river gradually deepens, from a depth of 4 m to a depth of 8 m as shown.
The width, W, of the river does not
change. At the depth of 4 m, the rivers speed is 12 m/sec. Its velocity at the 8
m depth is
C

In the open manometer shown, water occupies a part of the left arm, from a
height of y1 to a height of y2
remainder of the left arm, the bottom of the tube, and the right arm to a height
of y
. The
3 are filled with mercury.
B
Water flows in a pipe of uniform cross-sectional area A. The pipe changes
height from y1 = 2 meters to y2 = 3 meters. Since the areas are the same, we
can say v1 = v2. Which of the following is true?
A
A vertical force of 30 N is applied uniformly to a flat button with a radius of 1
cm that is lying on a table. Which
of the following is the best order of magnitude estimate for the pressure
applied to the button?
E

A ball that can float on water has mass 5.00 kg and volume 2.50 x 10^-2 m3.
What is the magnitude of the
downward force that must be applied to the ball to hold it motionless and
completely submerged in freshwater
of density 1.00 x 10^3 kg/m3?
D
Water flows through the pipe shown. At the larger end, the pipe has diameter
D and the speed of the water is v1 . What is the speed of the water at the
smaller end, where the pipe has diameter d ?
E
The maximum efficiency of a heat engine that operates between
temperatures of 1500 K in the firing chamber
and 600 K in the exhaust chamber is most nearly
C

An ideal gas is made up of N diatomic molecules, each of mass M. All of the


following statements about this
gas are true EXCEPT:

experiences a
floats, because B buoyant force
> L
equal to B gV.

Definition of Archimedes principle


Pabs = Pg + Po = 2.026x10^5 + 1.01
x10^5 = 3.03x10^5 Pa

Definition of buoyant force

Using fluid continuity with W as river


width. A1v1 = A2v2 4(W)(12) = (8)(W)
12 m/sec
24 m/sec
6 m/sec
8 m/sec
16 m/sec
v2 v2 = 6 m/s
42.fluids.png
The relevant formula here is P = Po +
gh
Answer (a) is wrong, because at y1 on
both arms, the pressure is just the
atmospheric pressure.
The pressure in the right arm at y3 is
still just atmospheric, but on the left, it is
atmospheric plus
g(y1 y3). That rules out (a). The
pressure at the bottom of the tube is
everywhere the same
(Pascals principle), which rules out (c),
and at the same time, tells us (b) is
right. At y2, we can
say P = Pbottom Hggy2 on both
sides, so the pressure is equal. Answer
(d) is wrong because at
y3, the right arm is supporting only the
atmosphere, while the left arm is
the pressure at the
supporting the
bottom of the right the pressure at a the pressure at a atmosphere plus H20
the pressure at a the pressure at a arm is greater than height y3 is less in height y1 is less in pressure.
height y3 is the
height y2 is the
at the bottom of
the left arm than in the left arm than in gh. Finally, (e) is silly because both
same in both arms. same in both arms. the left arm.
the right arm.
the right arm.
arms at height y1 are at atmospheric
43.fluids.png
Apply Bernoullis equation. P1 + gy1 +
P1=P2 + g(y2
v1^2 = P2 + gy2 + v2^2P1 =
y1)
P1=P2
P1=0
P2=0
1>2
P2 + g(y2y1)
44.fluids.png

10 Pa

10^2 Pa

10^3 Pa

10^4 Pa

10^5 Pa

20.0 N

25.0 N

30.0 N

200 N

250 N

P = F / A = 30 / r2 use 3 for
since its an estimate 30 / (3*(.01)^2)
= 100000 Pa
From a force standpoint, for the object
to be completely submerged there
would be three forces
acting. Fb up, mg down and Fpush
down. Fb = Fpush + mg Fpush = Fb
mg
Fpush = ph20 Vdisp g mg
= (1000)(2.5x102)(10) (5)(10) = 200
N

v1

dv1/D

Dv1/d

d^2v1/D^2

D^2v1/d^2

Using fluid continuity. A1v1 = A2v2


(D/2)^2v1 = (d/2)^2v2 solve for v2

33%

40%

The temperature of
the gas is
proportional to the
average
translational kinetic All of the
energy of the
molecules have
molecules
the same speed.

60%

The molecules
make elastic
collisions with the
walls of the
container.

67%

The molecules
make elastic
collisions with
each other.

100%
The average
number of
collisions per unit
time that the
molecules make
with the walls of
the container
depends on the
temperature of the
gas.

ec=(TH-TC)/TH

While all collisions are elastic and


Kavg T, the molecules move with
a wide range of speeds
represented by the Maxwellian
distribution.

47.fluids.png

3 Thermo

For the process XY, U is greater than zero and

Q < 0 and W = 0

Q < 0 and W > 0

Q > 0 and W < 0

Q > 0 and W = 0

4 Thermo

For the process YZ, Q is greater than zero and


An ideal gas confined in a box initially has pressure p. If the absolute
temperature of the gas is doubled and the
volume of the box is quadrupled, the pressure is

W < 0 & U = 0

W = 0 & U < 0

W = 0 & U > 0

W > 0 & U = 0

p/8

p/4

p/2

universal
gravitational
constant

speed of light

mechanical
equivalent of heat

charge of an
electron

5 Thermo

6 Thermo

7 Thermo

James Joule did much to establish the value of the


An ideal gas in a closed container initially has volume V, pressure P. and
Kelvin temperature T. If the
temperature is changed to 3T, which of the following pairs of pressure and
volume values is possible?

For XY, the process is isobaric. Since


the gas is expanding, W < 0 and since
the temperature is increasing, U > 0
and U = Q + W so Q > 0 (it is also true
because process XY lies above an
adiabatic expansion from point X)
Q > 0 and W > 0
3.thermo.png
For YZ, the process is isochoric,
which means no work is done (W = 0)
and since the temperature is
increasing, U > 0
W > 0 & U > 0
4.thermo.png
PV T, or P T/V and if T 2 then
P 2 and if V 4 then P 4 so
2p
the net effect is P24
James Joule did experiments on
changing the temperature of water
specific heat
through various means,
capacity of helium including by doing work on it.

3P and V

P and V

P/3 and V

3P and 3V

The work done by


the gas is the
same for all three
paths.

The change in
The heat absorbed internal energy of
by the gas is the
the gas is the
same for all three same for all three
paths.
paths.

The expansion
along path IF is
adiabatic.

The expansion
along path IF is
isothermal.

PV T so to triple the
temperature, the product of P and
V must be tripled
Changes in internal energy are path
independent on a pV diagram as it
depends on the change in
temperature, which is based on the
beginning and end points of the path
and not the path taken
8.thermo.png

E/(2)

(2)E

2E

4E

Kavg T

A and 1/L
The pressure does
not change ( P =
0).

1/A and L
The internal
energy does not
change (U = 0).

1/A and 1/L


No heat flows into
or out of the
system (Q = 0)

(A) and L^2


No work is done
on or by the
system (W = 0)

H = kAT/L

P and V/3

10 Thermo

If three identical samples of an ideal gas are taken from initial state I to final
state F along the paths IAF, IF, and
IBF as shown in the pV-diagram above, which of the following must be true?
If the average kinetic energy of the molecules in an ideal gas at a
temperature of 300 K is E, the average kinetic
energy at a temperature of 600 K is
A metal rod of length L and cross-sectional area A connects two thermal
reservoirs of temperatures T1 and T2.
The amount of heat transferred through the rod per unit time is directly
proportional to

11 Thermo

Which of the following is always a characteristic of an adiabatic process?

A and L
The temperature
does not change
(T = 0).

12 Thermo

During which process is no work done on or by the gas?

AB

BC

CD

DE

EA

13 Thermo

16 Thermo

At which point is the gas at its highest temperature?


Gas in a chamber passes through the cycle ABCA as shown in the diagram
above. In the process AB, 12 joules of heat is added to the gas. In the
process BC, no heat is exchanged with the gas. For the complete cycle
ABCA, the work done by the gas is 8 joules. How much heat is added to or
removed from the gas during process CA?
If the gas in a container absorbs 275 joules of heat, has 125 joules of work
done on it, and then does 50 joules of work, what is the increase in the
internal energy of the gas?
In each cycle of a certain Carnot engine, 100 joules of heat is absorbed from
the high-temperature reservoir and 60 joules is exhausted to the lowtemperature reservoir. What is the efficiency of the engine?

17 Thermo

Which of the following is true of the mechanical work done on the gas?

18 Thermo

20 Thermo

Which of the following is true of the final temperature of this gas?


A
In a certain process, 400 J of heat is added to a system and the system
simultaneously does 100 J of work. The change in internal energy of the
system is
C
An ideal gas is initially in a state that corresponds to point 1 on the graph
above, where it has pressure p1, volume V1, and temperature T1. The gas
undergoes an isothermal process represented by the curve shown, which
takes it to a final state 3 at temperature T3. If T2 and T4 are the temperatures
the gas would have at points 2 and 4, respectively, which of the following
relationships is true?
B

21 Thermo

The absolute temperature of a sample of monatomic ideal gas is doubled at


constant volume. What effect, if any, does this have on the pressure and
density of the sample of gas?

8 Thermo

9 Thermo

14 Thermo

15 Thermo

19 Thermo

By definition
No work is done in an isochoric
process, or a process where V = 0 (a
vertical line on the pv graph)
The temperature at any point is
proportional to the product of P and V.
Point A at temperature
(T0) is at pressure volume (p0)(V0).
Point C is at 3(p0) 3(V0) = 9(T0) and
point D is at 2(p0) 4(V0) =8(T0)

12.thermo.png

12.thermo.png

20 J is removed.

4 J is removed.

4 J is added.

20 J is added.

For the entire cycle, U = 0 and W = 8


No heat is added J so Q = U W = +8 J (8 J added).
to or removed from This means Q(AB) + Q(BC) + Q(CA) =
the gas.
+8 J = +12 J + 0 J + Q(CAB) = +8 J
14.thermo.png

100 J

200 J

350 J

400 J

450 J

150%
167%
It is the same for
processes 2 and 3 It is the same for
and less for
all three
process 1.
processes.

It is greatest for
process 1.

It is greatest for
process 3.

67%
It is the same for
processes I and 2
and less for
process 3.

It is greatest for
process 1.

It is greatest for
process 2.

It is greatest for
process 3.

Q(H) = 100 J and Q(C) = 60 J; e=(W/Q


(H))=((Q(H)-Q(C))/Q(H))
Work is the area under the curve, the
line bounding the greatest area
indicates the most work
done
17.thermo.png
Temperature rises as you travel up and
to the right on a pV diagram. Since
processes 1, 2 and 3
It is the same for
It is the same for
are at the same volume, the highest
processes 1 and 2. processes 1 and 3. point is at the highest temperature
17.thermo.png

500 J

400 J

300 J

-100 J

-300 J

T(1)<T(3)

T(1)<T(2)

T(1) < T(4)

T(1) = T(2)

T(1) = T(4)

Q = +275 J; W = + 125 J + ( 50 J) =
+75 J; U = Q + W

40%

60%

Q = +400 J; W = 100 J; U = Q + W

Isothermal means the temperature is


constant. Points to the right or above
are at higher temperatures.
20.thermo.png
P T at constant volume. If T 2,
then P 2. Since the mass and
volume are unchanged, the
21.thermo.png
density is unchanged as well

22 Thermo

23 Thermo

24 Thermo

25 Thermo

26 Thermo

27 Thermo

28 Thermo

29 Thermo

30 Thermo

31
32
33
34
35
36
37
38
39
40
41

Thermo
Thermo
Thermo
Thermo
Thermo
Thermo
Thermo
Thermo
Thermo
Thermo
Thermo

Which of the following statements is NOT a correct assumption of the


classical model of an ideal gas?
D
A sample of an ideal gas is in a tank of constant volume. The sample absorbs
heat energy so that its temperature
changes from 300 K to 600 K. If v1 is the average speed of the gas molecules
before the absorption of heat and
v2 is their average speed after the absorption of heat, what is the ratio v2/v1? C
Two blocks of steel, the first of mass 1 kg and the second of mass 2 kg, are in
thermal equilibrium with a third
block of aluminum of mass 2 kg that has a temperature of 400 K. What are
the respective temperatures of the
first and second steel blocks?
C

An ideal gas may be taken from one state to another state with a different
pressure, volume, and temperature
along several different paths. Quantities that will always be the same for this
process, regardless of which path
is taken, include which of the following?
I. The change in internal energy of the gas
II. The heat exchanged between the gas and its surroundings
III. The work done by the gas
A
A square steel plate with sides of length 1.00 m has a hole in its center 0.100
m in diameter. If the entire plate is
heated to such a temperature that its sides become 1.01 m long, the diameter
of the hole will be
D

Which of the following will occur if the average speed of the gas molecules in
a closed rigid container is
increased?
C
In time t, an amount of heat Q flows through the solid door of area A and
thickness d represented above. The
temperatures on each side of the door are T2 and T1, respectively. Which of
the following changes would be
certain to decrease Q?
D
A gas with a fixed number of molecules does 32 J of work on its
surroundings, and 16 J of heat are transferred
from the gas to the surroundings. What happens to the internal energy of the
gas?
A

The only
appreciable forces
on the molecules
are those that
occur during
collisions.

The collisions
between
molecules are
inelastic.

If the collisions were inelastic, the gas


would change its temperature by virtue
of the collisions
with no change in pressure or volume.

400 K and 200 K

200 K and 400 K

400 K and 400 K

800 K and 400 K

I only

II only

I and III only

II and III only

0.090 m

0.099 m

0.100 m

0.101 m

The density of the


gas will decrease.

The density of the


gas will increase.

The pressure of
the gas will
increase.

The pressure of
the gas will
decrease.

related to average speed, v= (3RT/M)^


4 (1/2)
Being in thermal equilibrium means the
objects are at the same temperature.
Mass is irrelevant.
The question describes the zeroth law
None of the above of thermodynamics.
Changes in internal energy are path
independent on a pV diagram as it
depends on the change in
temperature, which is based on the
beginning and end points of the path
and not the path taken.
Different paths, with different areas
under them will do different amounts of
work and hence,
I, II, and III
different amounts of heat exchanged.
In linear expansion, every linear
dimension of an object changes by the
same fraction when
0.110 m
heated or cooled.
rigid container = constant volume. If
the speed increases, the temperature
The temperature of will increase, and if
the gas will
the temperature increases at constant
decrease.
volume, the pressure will increase.

Increasing A only

Increasing d and
Decreasing d only T2 T1 only

Decreasing A and
T2 T1 only

Increasing d, A,
and T2 T1

It increases by 16
J.

It increases by 48
J.

0.5

1 square root of 2

It decreases by 48 It decreases by 16 It remains the


J.
J.
same.

H= kA(Tf-Ti)/L

color of a metal
rod.

length of a liquid
column.

pressure of a gas electrical


at constant volume resistance.

mass of one mole


of gas at constant
pressure.

Q = 16 J; W = 32 J; U = Q + W
Mass is independent of the state of a
gas. (color will be addressed in a later
topic, think about a
yellow vs. blue flame or a red hot
piece of metal)

2/3 p

3/2 p

2p

3p

6p

P nT/V; if n 3 then P 3 and if


T 2 then P 2, the net effect is
P32

the molecules of
the water and
copper would have
equal average
speeds

the molecules of
the water and
copper would have
equal average
momenta

the molecules of
the water and
copper would have
equal average
kinetic energies

the water
molecules would
have twice the
average
momentum of the
copper molecules

the copper
molecules would
have twice the
average speed of
the water
molecules

both height and


both height and
both height and
width will remain
width will increase width will decrease unchanged

Which of the following could NOT be used to indicate a temperature change?


A change in:
E
A mass m of helium gas is in a container of constant volume V. It is initially at
pressure p and absolute (Kelvin)
temperature T. Additional helium is added, bringing the total mass of helium
gas to 3m. After this addition, the
temperature is found to be 2T. What is the gas pressure?
E

43 Thermo

A 200 gram sample of copper is submerged in 100 grams of water until both
the copper and water are at the same temperature. Which of the following
statements would be true?
A rectangular piece of metal 3 cm high by 6 cm wide has a hole cut in its
center 1cm high by 4 cm wide as shown in the diagram at right. As the metal
is warmed from 0oC to 1000C, what will happen to the dimensions of the
hole?

44 Thermo

A gas is enclosed in a cylindrical piston. When the gas is heated from 0oC to
100oC, the piston is allowed to move to maintain a constant pressure.
According to the Kinetic-Molecular Theory of Matter
A

42 Thermo

The volume of the


molecules is
negligible
compared with the The molecules
The molecules are volume occupied obey Newton's
in random motion. by the gas.
laws of motion.

the mass of the


gas will increase

the number of
molecules of gas
must increase

the size of the


individual
molecules has
increased

height will
height will increase
decrease while
while width will
width will increase decrease
the molecules
continue to strike
the average speed the sides of the
of the molecules
container with the
has increased
same energy

28.thermo.png

KavgT
In linear expansion, every linear
dimension of an object changes by the
same fraction when A
heated or cooled.

42.thermo.png

Kavg T

44.thermo.png

45 Thermo

Two containers are filled with gases at the same temperature. In the
container on the left is a gas of molar mass 2M, volume 2V, and number of
moles 2n. In the container on the right is a gas of molar mass M, volume V,
and moles n. Which is most nearly the ratio of the pressure of the gas on the
left to the pressure of the gas on the right?

46 Thermo

A thermally insulating container has a membrane separating the container


into two equal parts. In one part is a vacuum. In the other part is an ideal gas
of temperature T and internal energy U. The membrane is punctured and the
gas rushes into the region which was a vacuum. After the system has
returned to equilibrium, which of the following is NOT true for the gas?
A

47 Thermo
48 Thermo

49 Thermo

50 Thermo

51
52
53
54
55
56
57
58
59
60
61
62
63
64
65
66
67
68
69
70
71
72
73
74
75
76
77
78
79
80

Thermo
Thermo
Thermo
Thermo
Thermo
Thermo
Thermo
Thermo
Thermo
Thermo
Thermo
Thermo
Thermo
Thermo
Thermo
Thermo
Thermo
Thermo
Thermo
Thermo
Thermo
Thermo
Thermo
Thermo
Thermo
Thermo
Thermo
Thermo
Thermo
Thermo

A fan blows the air and gives it kinetic energy. An hour after the fan has been
turned off, what has happened to the kinetic energy of the air?
D
According to the kinetic theory of gases, when the absolute temperature of an
ideal gas doubles, the average kinetic energy of the molecules of the gas
E

When gas escapes from a pressurized cylinder, the stream of gas feels cool.
This is because
B
A monatomic ideal gas is used as the working substance for the Carnot cycle
shown in the figure. Processes A B and C D are isothermal, while
processes B C and D A are adiabatic. During process A B, there are
400 J of work done by the gas on the surroundings. How much heat is
expelled by the gas during process C D?
A
Two completely identical samples of the same ideal gas are in equal volume
containers with the same pressure and temperature in containers labeled A
and B. The gas in container A performs non-zero work W on the surroundings
during an isobaric (constant pressure) process before the pressure is reduced
isochorically (constant volume) to 12 its initial amount. The gas in container B
has its pressure reduced isochorically (constant volume) to 12 its initial value
and then the gas performs non-zero work W on the surroundings during an
isobaric (constant pressure) process.
After the processes are performed on the gases in containers A and B, which
is at the higher temperature?
E

1:01:00

8:01:00

2:01:00

There is no heat
The temperature of No work is done by exchanged by the
the gas is
the gas on the
gas with the
unchanged.
surroundings.
surroundings.

it disappears

it turns into sound


energy

it turns into
potential energy

4:01:00 P n/V at constant temperature


Since the container is insulated, no
heat is exchanged (C is true), since
there is no work done (no
force required to expand), choice B is
true. Since Q = 0 and W = 0, U and
T = 0 (A and E are
true). While entropy change does have
a heat component (and if Q = 0, the
change in entropy
There is no
The internal
may be incorrectly regarded as zero) it
entropy change of energy of the gas also has a volume component (how
the system.
is unchanged.
spread out the gas is)
This question is a bit of a paradox as
the energy form the fan giving the air
kinetic energy is
theoretically adding to the thermal
energy of the air, But as the air lowers
in temperature, this
energy will dissipate into the walls and
it turns into
it turns into thermal other outside areas of the room as
electrical energy
energy
thermal energy as well.
16:01:00

Kavg T
Gas escaping form a pressurized
cylinder is an example of an adiabatic
process. While the gas
rapidly does work (W < 0), U is
work is being done
pressurized
the gas inside the the moisture in the negative since heat does not have time
at the expense of of the convection cylinders are good cylinder is actually air condenses and to flow into the gas in a
thermal energy
inside the cylinder thermal insulators frozen
cools
rapid expansion.
quadruples

1600 J

The gas in
container A

doubles

stays the same

is cut in half

800 J

400 J

200 J

The gas in
container B

The value of the


work W is
necessary to
The gases have
answer this
equal temperature question.

is quartered

In a Carnot cycle
Q = and in process AB, U = 0 and
since WAB = 400 J, QAB = +400 J
100 J
and this is Qh
50.thermo.png
Since process A and B perform the
same amount of work, they must have
the same area under
The value of the
their respective lines. Since A does the
work W is
work at a higher pressure, it does not
necessary, along have to move as far
with both the initial to the right as process B, which
pressure and
performs the work at a lower
volume, in order to temperature. Since the end of
answer the
process B lies farther to the right, it is at
question.
the higher temperature.

81 Thermo
82 Thermo
83 Thermo

Timestamp

Untitled Question

S-ar putea să vă placă și